USMLE

180
The complete compilation of Hassan’s posts on July Tuberculosis pearls Tuberculosis TB treatment: - HIV negative: ERHZ or SRHZ for 2 months then RH for 4 months - Pregnant and lactating woman: PZA and Streptmycin CI. ERH (increased doses) for 4- 8weeks, then RH twice Qweek x 7months. If resistance is a concern: ERH x 9months. Add Vit B6 with INH use. Breatfeding is not contraindicated despite presence of small amounts of meds in milk. - HIV Positive: Same treatment with additional considerations: longer duration, Rifampin interaction with anti-HIV meds, directly-observed TB therapy should be used for all HIV pts, VitB6 mandatory with INH to reduce side effects. - Drug-resistant patient: Resistance only to INH = RZ with E or S x 6months or ER x 12 months. Others require expert intervention. - Extrapulmonary TB: 9months with same drugs if miliary, bone, meningeal, or joint TB. If bone: early surgical drainage and debridement of necrotic bone. Steriod therapy prevents cardiac constriction and neurologic complications from meningitis. - Latent TB: targeted testing to identify candidates, test for HIV, screen for prior TB Rx and current contraindications. Three regimens are considered: 1) Ideally: INH for 9months. Give Vit B6 if at risk to develop neuropathy (Pregnant, DM, HIV, alcoholic, uremia, Seizure disorder). 2) RZ x 2 months. 3) Rifampin x 4 months. Care must be taken if HIV patients on Nonnucleoside RT Inhibitors or Protease inhibitors. If Pregnant or lactating: INH QD or BID + Vit B6. - BCG Vaccine: recommended only on individual basis: e.g.: Healthcare worker with high % of multiresistant TB patients. CI if immune deficiency/impairement. - Steriod use in TB Rx: only in TB meningitis and TB pericarditis. PPD + is when: - >5mm: HIV+, Recent TB patient contact, CXR TB-like changes, Organ Transplant patients. - >10mm: Recent immigrant, IV drug user HIV-, TB lab personel, High risk medical personel, High risk medical conditions (Gastrectomy, GI bypass, DM, Silicosis, CRF, Leukemia, lymphoma, CA of HEENT or lung), kids<4yo or teens and adults exposed to TB patients. - >15mm: if no risk factors to TB. If PPD – then converts to <10mm => Repeat in 2 weeks (you cannot make a PPD- person become positive by repeated testing). If PPD is + => CXR: if abnormal => 3 AFB. TB prophylaxis now called latent TB: PPD + indicates 10% lifetime risk for active TB. - Exposed adult with PPD - => None

description

USMLE Study tips

Transcript of USMLE

The complete compilation of Hassan’s posts on July

Tuberculosis pearlsTuberculosis

TB treatment: - HIV negative: ERHZ or SRHZ for 2 months then RH for 4 months- Pregnant and lactating woman: PZA and Streptmycin CI. ERH (increased doses) for 4-8weeks, then RH twice Qweek x 7months. If resistance is a concern: ERH x 9months. Add Vit B6 with INH use. Breatfeding is not contraindicated despite presence of small amounts of meds in milk. - HIV Positive: Same treatment with additional considerations: longer duration, Rifampin interaction with anti-HIV meds, directly-observed TB therapy should be used for all HIV pts, VitB6 mandatory with INH to reduce side effects. - Drug-resistant patient: Resistance only to INH = RZ with E or S x 6months or ER x 12 months. Others require expert intervention. - Extrapulmonary TB: 9months with same drugs if miliary, bone, meningeal, or joint TB. If bone: early surgical drainage and debridement of necrotic bone. Steriod therapy prevents cardiac constriction and neurologic complications from meningitis. - Latent TB: targeted testing to identify candidates, test for HIV, screen for prior TB Rx and current contraindications. Three regimens are considered: 1) Ideally: INH for 9months. Give Vit B6 if at risk to develop neuropathy (Pregnant, DM, HIV, alcoholic, uremia, Seizure disorder). 2) RZ x 2 months. 3) Rifampin x 4 months. Care must be taken if HIV patients on Nonnucleoside RT Inhibitors or Protease inhibitors. If Pregnant or lactating: INH QD or BID + Vit B6. - BCG Vaccine: recommended only on individual basis: e.g.: Healthcare worker with high % of multiresistant TB patients. CI if immune deficiency/impairement. - Steriod use in TB Rx: only in TB meningitis and TB pericarditis.

PPD + is when: - >5mm: HIV+, Recent TB patient contact, CXR TB-like changes, Organ Transplant patients. - >10mm: Recent immigrant, IV drug user HIV-, TB lab personel, High risk medical personel, High risk medical conditions (Gastrectomy, GI bypass, DM, Silicosis, CRF, Leukemia, lymphoma, CA of HEENT or lung), kids<4yo or teens and adults exposed to TB patients. - >15mm: if no risk factors to TB. If PPD – then converts to <10mm => Repeat in 2 weeks (you cannot make a PPD- person become positive by repeated testing). If PPD is + => CXR: if abnormal => 3 AFB.

TB prophylaxis now called latent TB: PPD + indicates 10% lifetime risk for active TB.- Exposed adult with PPD - => None

- Exposed child<5yo with PPD - => INH x3months- PPD conversion but CXR - =>INH x6-12months- Pregnant woman + HIV+ with PPD+/Conversion/Exposure to TB but no active disease: INHx 6-12months + Vit B6

Side Effects of TB drugs: All are hepatotoxic except Streptomycin. - INH: Hepatitis – Peripheral neuropathy- Rifampin: colors body secretions, contact lenses. Hepatitis, Renal Failure – drug interaction with OCP/Coumadin/Digoxin/Oral hypoglycemics/HIV meds- PZA: hyperuricemia, hepatotoxic, - ETB: Optic neuritis (reversible)- Streptomycin: VIII damage, nephrotoxic.

Is there any correlation between lesion location and the develoment of post-stroke neuropsychiatric sequelae?

A 69-year-old patient is admitted to the neurology service following a stroke. During the next few days, the staff observes that the patient has developed the clinical picture of mania. Which area of the brain has most likely been affected by the stroke? A. Left hemispheric lesions including Broca's areaB. Left prefrontal cortexC. Midbrain lesionD. Right frontal lobeE. ThalamusAnswer D. Is there any correlation between lesion location and the develoment of post-stroke neuropsychiatric sequelae? Possibly. In the case of post-stroke mania, it appears to occur most often with right hemispheric lesions especially when they occur in the right orbito-frontal region or the right thalamus. Treatment of post-stroke mania: controlled studies have not yet been completed, although case studies have suggested that Lithium, Depakote and Carbamazepine, Clonidine, and neuroleptics, may each be effective in such an entity. Given that anticonvulsant mood stabilizers have shown to be more effective in secondary mania, and given the propensity of convulsions in post-stroke patients, mood stabilizing anticonvulsants may be the agents of choice in post-stroke mania. Notes:Controvesy exists in correlating locations with post-stroke depression. It is suggested that it may be related to left frontal cortex, and left frontal basal ganglia. Treatment involves SSRI's (first line), TCA's (risk of ortho hypotension, and cardiac conduction abnormalities), Stimulants (May stimulate during post stroke rehab and stroke is NOT a contraindication) and ECT. Reference: Primary Care Companion J Clin Psych.2003;5(2)http://www.psychiatrist.com/pcc/pccpdf/v05n02/v05n0205.pdfFragile X Syndrome

A- Background: Fragile X syndrome, also termed Martin-Bell syndrome or marker X syndrome, is the most common cause of inherited mental retardation and, after trisomy 21, is the second most common cause of genetically associated mental deficiencies.Two to four times as many females carry the gene abnormality as males, but only about one third of females carrying the abnormal gene show decreased intelligence. Males with the disorder are more likely to be sensitive to environmental factors. The pattern of inheritance most closely resembles X-linked dominant with variable penetrance. Occasionally, because the complex genetics of the disorder, a female will be affected severely.

B- Pathophysiology: The genetic defect is dynamic and lies at the distal end of the long arm of the X chromosome. Careful examination of the karyotype of affected individuals' lymphocytes, reveals a constriction followed by a thin strand of genetic material extending beyond the long arm of chromosome X. This constriction and thin strand produces the appearance of a fragile portion of the X chromosome, leading to the term fragile X. The underlying pathology is an unusual high number of repeats of triplets CGG. Unaffected individuals have 5 to 55 CGG repeats. A span of 65 to 230 repeats is known as a premutation, whereas more than 230 repeats is a full mutation. The number of repeats is unstable from generation to generation, making the pattern of inheritance difficult to predict.

• Males with a full mutation have fragile X syndrome.• Mothers of all males with fragile X syndrome have the premutation or fragile X syndrome themselves. • Males with fragile X syndrome pass a premutation to their daughters because sperm cells are mosaics. • Sons are unaffected because they receive the Y chromosome from their fathers.• Half of females with the full mutation on a single X chromosome are unaffected because of inactivation of the other X chromosome. The other half of females have fragile X syndrome, although with less severe mental retardation than males with the disorder. • Males with a premutation usually are unaffected to mildly affected and transmit the premutation to their daughters. The mutation is stable; thus, no increase in the CGG triplets exists, when it’s a male transmitting it. • Females with a premutation usually are unaffected to mildly affected. Unlike their male counterparts, the CGG triplets are unstable and increase in size during oogenesis. If the number of repeats exceeds 230 and the oocyte is fertilized, a male child will have fragile X syndrome, and a female child will have a 50% chance of having fragile X syndrome. The number of repeats is directly proportional to the risk of the disorder in an offspring.C- Clinically: Cognitive, behavioral, and neuropsychological difficulties predominate the clinical picture. These signs are especially important in alerting physicians, parents, and teachers to deficits exhibited by preschool and elementary school children—a time when the diagnosis of fragile X syndrome often is made or considered.

Problems include: mild-to-moderate autisticlike behavior (most notably, hand flapping and avoidance of eye contact), attention deficits, depressed affect, mental retardation with IQ typically 35-70, aggressive tendencies, deficiency in abstract thinking, developmental delays after reaching early milestones (especially speech and language delays), and decreasing IQ with increasing age. In addition, there are physical signs associated with fragile X syndrome; however, these signs are more obvious during adolescence or after puberty and rarely result in disabilities. In addition to the cognitive, behavioral, and neuropsychological findings, the organ systems most frequently involved are craniofacial, genital, and musculoskeletal. They manifest in adolescence as a long thin face with prominent ears, facial asymmetry, a head circumference higher than the 50th percentile, and a prominent forehead and jaw. In addition, the mouth has overcrowding and a high-arched palate. Macroorchidism is universal in adult males. Sometimes, scoliosis may be noted. D- Work-up- Cytogenetics (Karyotype) is not as sensitive as molecular testing (Southern Blot and PCR). - A comprehensive developmental evaluation by a speech/language therapist, physical therapist, and occupational therapist is recommended to assess weaknesses and to identify areas where improvement is needed most. As the patient matures, repeat evaluation may be necessary.E- Medical Care: o Workup and diagnosis can be done on an outpatient basis.o Routine care involves treating the medical problems that these patients experience commonly, including gastroesophageal reflux, sinusitis, and otitis media.o During infant and early childhood health care maintenance visits, focus examination on possible hip dislocations, hernias, and hypotonia.o Stimulants (eg, methylphenidate, dextroamphetamine, pemoline) have been used for attention deficits in the doses prescribed for patients with ADHD. Responses are variable.o Antiseizure and antireflux medications are useful for patients with these symptoms.

F- Complications: o Scoliosiso Mitral valve prolapse (most frequently encountered cardiac defect)

G- Special Concerns: • Prenatal screening: Because fragile X syndrome is underdiagnosed, has a high prevalence, and is inheritable, preconceptual and antenatal molecular genetic screening is encouraged for women as outlined below.References: - Emedicine: http://www.emedicine.com/ped/topic800.htm Updated March 2003ERCP or HIDA ScanHIDA scans have sensitivity (94%) and specificity (65-85%) for acute cholecystitis. They are sensitive (65%) and specific (6%) for chronic cholecystitis. On the other hand, ERCP provides both endoscopic and radiographic visualization of the biliary tract. It can be diagnostic and therapeutic by direct removal of common bile duct stones.

Ultrasound is 50-75% sensitive for choledocholithiasis. CT and HIDA scans are not better. Therefore, when a dilated common bile duct is found or elevated LFTs are present, suspicion should remain high for common bile duct stones, and an ERCP should be considered.Debate exists as to when an ERCP should be performed. In general, since cholecystitis is caused by obstruction of the ducts, the risk of common bile duct stones is approximately 10%.Some studies have classified people as low risk for common bile duct stones based on (1) lack of jaundice, (2) elevated transaminases, and (3) a common bile duct diameter of less than 8 mm. In this population, the risk of common bile duct stones may be as low as 1%. In patients with any of the risk factors, the rate of stones was 39%. Therefore, in general, people with any of the risk factors for common bile duct stones should undergo operative or ERCP evaluation of the common bile duct.Major complications of ERCP include pancreatitis and cholangitis. Reference:http://www.emedicine.com/emerg/topic98.htmIs a dog/cat's mouth cleaner than a human mouth?Animal and human bites carry a high risk of infectious complications. Human bites and in particular clenched-fist injuries as well as cat bites are highly prone to infection as are wounds that involve the hand or deep structures including joints, bones and tendons. The management of bite wounds consists of intensive irrigation with large volumes of normal saline and a cautions debridement of devitalized tissues. Generally, it appears prudent to leave the wounds open, however, in cases carrying a low risk of infection, a primary surgical closure might be appropriate. If a bite wound is infected, an antibiotic course with amoxycillin/clavulanic acid (first choice) or tetracyclines (second choice) for 10-14 days is recommended. In patients who present early after the injury, an antibiotic prophylaxis for 3-5 days is appropriate, particularly when the risk for the development of infection is high. Furthermore, a tetanus booster and in case of possible transfer of rabies, a rabies vaccination with immunoglobulins and inactivated virus preparation is recommended.Human bite wounds have long had a bad reputation for severe infection and frequent complication. However, recent data demonstrate that human bites occurring anywhere other than the hand present no more of a risk for infection than any other type of mammalian bite. The increased incidence of serious infections and complications associated with human bites to the hand warrants their consideration and management in three different categories: occlusional/simple, clenched fist injuries, and occlusional bites to the hand.

References: 1)Dog, cat, and human bitesSchweiz Rundsch Med Prax. 1998 May 20;87(21):716-8. 2)Dog, cat, and human bites: a review.J Am Acad Dermatol. 1995 Dec;33(6):1019-29. 3)Diagnosis and treatment of bites by cats, dogs and humansDtsch Med Wochenschr. 2003 May 9;128(19):1059-63. Q that remained unanswered about antiarrhythmics.

Here is my input. They are classifed into:Class I: Blocks Na ChannelsClass II: are the beta-blockersClass III: Block K channelsClass VI: Calcium channel blockers.Class V: Cardiac Glycosides. ClassI: * Ia = Phase 0 = Slow rhe rise of action potential = Make it longer thus more refractory. These agents are used only for chemical conversion. They alter the electrophysiologic mechanisms responsible for arrhythmia.Examples: Quinidine, Procainamide (Lupus, Fatal blood dyscrasias first 3 months!!), Disopyramide (Urinary retention!!), MoricizineNote: Can expect increased levels of procainamide metabolite NAPA in patients taking CIMETIDINE, RANIDITINE, beta-blockers, amiodarone, trimethoprim, and quinidine; Indications: SVT, VTach, Prevention of VFib, Symptomatic PVC Ib = shorten action potentialExamples: Lidocaine, Mexiletine(Leukopenia!!), PhenytoinIndications: VTach, Prevention of VFib, Symptomatic PVC's. Ic = Phase 0 = slow the rise of action potential/refractoriness but more than Ia. These agents are used only in patients with structurally normal hearts (ie, absence of coronary artery disease or cardiomyopathy).Examples: Flecainide, Propafenone (Weak CaBlock+BetaBlock)Indications: Life threatening VTach/VFib, Refractory SVT. Class II: Beta Blockers: These agents slow the sinus rate in addition to decreasing AV nodal conduction.Side effects: bronchospasm. Note: Esmolol (Brevibloc) -- Ideal for use in patients at risk of complications from beta-blockade. short half-life of 8 minNote: First line of treatment in Asthma secondary to beta blockers: ANTICHOLINERGICS/IPRATROPIUM (NOT BETA2 AGONISTS AS IN REGULAR ASTHMA). Class III: Blocking K channels results in widened QRS and longer QT interval. They pronlong action potential. Examples: Amiodarone (Pulm Fibrosis, Hyper/Hypothyroidism, Corneal and skin depostis, Neurotox!!!!!), Sotalol (noncardiac selective beta blocker), Dofetilide, Ibutilide, Bretylium. Class IV: Block Calicum from entering slow channels or voltage-sensitive areas of vascular smooth muscle and myocardium. They reduce rate of AV nodal conduction and control ventricular response in A Fib. Examples: Verapamil, Diltiazem, Nifedipine. Relevant Indication: DOC for longterm control of A Fib (better than Dig if long term) Class V:

Cardiac glycosides -- These drugs slow AV nodal conduction primarily by increasing vagal tone. They are used primarily in the setting of AF with CHF.Examples: Adenosine (Flushing, AV Block, S brady!!!), Digoxin (CI in idiopathic hypertrophic subaortic stenosis, constrictive pericarditis, Watch out for AV Block, GI problems, Visual changes, PAT with a 2:1 block!!!!)hypercalcemia and hypercalcemia predisposes patient to digitalis toxicity predispose patient to digitalis toxicity.

References: - CMDT p363- emedicinemost common brain t/m in children To let you know why it gets confusing, this question has to be classified by the location of the tumor: 1- Supratentorial (40%) - Astrocytoma (8-12%) - Glioblastoma (<5%) - Craniopharyngioma (5-8%) - Ependymoma (3-5%) - Choroid Plexus papilloma (2-3%) - Pituitary tumor (<1%) - Pineal tumor (2%) - Meningioma (<1%) 2- Infratentorial (60%) - Medulloblastoma (18-25%) - Cerebellar Astrocytoma (15-20%) - Brain stem glioma (8-10%) - Ependymoma (4-6%) - Schwannoma (<1%) - Meningioma (<1%)

Overall, in peds, the most common brain tumors are: - #1: Medulloblastoma- #2: Astrocytoma- #3: Ependymoma Reference: - Familypracticenotebook.comhttp://www.fpnotebook.com/HEM147.htmOld Question on Neuroblastoma Vs Wilms: CT or IVP?Facts: - Solid masses are ominous in a child as opposed to cystic ones, and may represent Wilms tumor, neuroblastoma, congenital mesoblastic nephroma, or other less common tumors, such as malignant rhabdoid tumors. - Wilms tumor arises from the kidney and most often is detected as a large asymptomatic abdominal mass. Neuroblastoma, which commonly arises from the adrenal gland, often presents with a mass and constitutional symptoms (eg, fever, weight loss). - Wilms tumor commonly displaces and compresses vessels. Neuroblastoma also

displaces and compresses vessels but more often is infiltrative. - Ultrasound with Doppler can reveal inferior vena cava (IVC) invasion. CT scan plays an important role in the management of Wilms tumor and neuroblastoma for staging (3). Neuroblastoma Fact: At initial presentation, ultrasound is useful for diagnosing intra-abdominal tumors and can display calcifications, but ultrasound interpretation requires skilled expertise. Falsely negative studies are misleading for the clinician unless an alternate imaging study such as a CT scan is done to make the diagnosis (1). Wilms Tumor Fact: Ultrasound or CT helps localize the mass, identifies associated genitourinary abnormalities, confirms function of the contralateral kidney, and indicates if there is extension to the inferior vena cava. CT scan is better at detecting subtle intra-abdominal abnormalities such as tumor spread, lymph node enlargement, vascularity (1). Additional Fact: In a recent study, radiologists were accurate at diagnosing Wilms' tumors using modern imaging methods (IVP was not included in it, nor in any theoretical review I found), however, care should be taken in children who are less than 1 year of age as a mesoblastic nephroma may have identical imaging characteristics (2). References: 1- Case Based Pediatrics For Medical Students and Residentshttp://www.hawaii.edu/medicine/pediatrics/pedtext/s12c03.html2- With advances in medical imaging can the radiologist reliably diagnose Wilms' tumours?Clin Radiol. 1999 May;54(5):321-7. 3- Radiographic Evaluation of the Pediatric Urinary Tracthttp://www.emedicine.com/ped/topic2751.htmthe next step in mgt in the below q....1.40 yr M , come with pain from 1 hr, severe, constant.he lies motionless, is diaphoretic, rapid shallow breathing. p/e- rigid abd, tenderness to palpaion, guarding+, rebound tenderness+a. plain x-rayb. emergency laparotomyc. r/o other causes of acute abd- amylase, c-xray, ecg2.in the 2 week of post-op for multiple gunshot wounds to the abdomen, patient becomes unresponsive and progressively disoriented. po2-65 on 40% o2.a. ABGb. c-xray 3.50 yr M, h/o smoking+, drinking+, progressive machanical dysphagia, wt loss+a. barium swallowb. endoscopy and biopsyAnswers: 1.emergency laparotomy - pt. presents with peritonitis, needs acute Mx,2.CXR, pt. most likely has ARDS -- occurs in pts after trauma, infections, massive transfusions, onset is acute several hrs. to several days post injury but could be variable, the most typical is several hrs. to several days after event, pt.has hypoxemia refcractory to oxygen, the PaO2/FiO2 ratio < 200 mmHg,65/0.4= 162.5 - characteristic for ARDS, pt. can have lethargy followed by obtundation, other symptoms are not noted here due to the altered mentation, like tachypnea, etc. -

CXR will show bilateral pulm. infiltrates and/or consolidation, CXR is fast to get...if CXR is -ve , or normal then it could be PE, so then get ABG. get ABG in either case after the CXR, see the A-a gradient , etc...correct me on this one if i am wrong 3.barrium swallow - usually the first Dx test in Pt w/ suspected esoph.CA especially if they have symptoms of dysphagia, can also Dx one of the most serious compl. - TE fitule,...then do EGD , then CT scan to do staging ,look for Mets Old Request about summary of treatment of AsthmaAsthma is defined primarily as an INFLAMMATORY DISEASE with mostly MAST CELL degranulation. Classify Asthma first:- Asthma variant cough: Cough without wheezing with atopic history. Treat as mild intermittent.

- Mild intermittent: * Pt asymptomatic* Diurnal symptoms 1x or 2x/week* Nocturnal symptoms 2x/month - Mild persistent: * Diurnal symptoms >2x/week / <1x/day* Nocturnal symptoms >2x/month* Exacerbations affect activity - Moderate persitent: * Symptoms daily* Night symptoms each week* Exacerbations affect activity >2x/week - Can last days - Severe persistent:* Continuing symptoms/exacb, mostly at night* Limited activity Treat the acute episode with Ox and Beta2 adrenergic short acting. Steroids are used in the exacerbations that needs admission. Long term use is the follwing: - Mild intermittent: * No long term Rx - Mild persistent: * Inhaled Steroids OR Inhaled Cromolyn - Moderate Persistent: * Inhaled steroids low/med dose AND Long acting beta adrenergic = SAlmeterol* OR!!!!!: Inhaled Cromolyn AND Salmeterol - Severe Persistent: * Inhaled Steroid high dose AND Salmeterol AND Steroids Tablets. * OR!!!!!: Inhaled Cromolyn AND Salmeterol AND PO Steroids. In clinical scenarios, you step up the long term Rx from one level to the other when: - Mild intermittent to moderate persistent WHEN Pt USES Beta2INHALER MORE THAN TWICE A WEEK - Mild persistent to Moderate persistent WHEN PT USES INHALER DAILY. - Moderate persistent to Severe persistent WHEN PT USES INHALER DAILY AND INCREASES DOSE (MORE THAN 2 PUFFS AT A TIME).

Note: Review treatment with Pt every 1 to 6 months and either step down or step up. This change will depend on the failure of other supporting measures (Technique of using inhaler, adherence, and environmental control). Note2: Beta2 adrenergics are first line in treatment of asthma. Ipratropium (anticholinergics) are first line of treatment of bronchospasm secondary to beta blockers, or in CI of beta adrenergics. Steroids Inhaled are first line in treatment of PERSISTENT ASTHMA. 2nd line of treatment out-pt after familure of beta2adrenergicsSteroids systemic (PO) are first line in treating exacerbation in-patient and in control of severe persistent. (GIve Calicum and Vit D in chronic use. Watch out for Adrnela insufficiency. Scenario of Pt undergoing accident or surgery and post-op clinical scnerio of adrenal insufficiency. H/o of persistent asthma. Or Pt with h/o of asthma who develops recurrent oral candidiasis). References:- Swanson- CMDTMercury poisoning

A 2 yo – peeling, erythematous rash – hands and feet. Mother says child has become ill tempered and refuses to walk. PE – irritable, pale child with photophobia. T = 99F, P=90, RR = 23. Tremor of the tongue evident. Further history reveals child’s elder brothers have been playing with household thermometers.What is the likely diagnosis?How do you confirm the diagnosis?

The culprit: what is mercury? Mercury is an elemental metal. At room temperature it is a silvery, odorless liquid which vaporizes easily. These properties have given it the common name quicksilver. Mercury's most well known use is in medical thermometers found in the home, but it is also used in electric switches, propellant and fluorescent lamps. Mercury use in diuretics, antiseptics, perservatives and pesticides has greatly declined becuase effective substitutes have been found. Mercury was previously used in paint, but no longer is. Mercury exists in 3 forms: (1) elemental mercury, (2) inorganic salts, and (3) organic compounds. Perhaps the most deadly form of mercury is methyl mercury. Only 2-10% of the ingested mercury is absorbed from the gut, and ingested elemental mercury is not absorbed at all; however, 90% of any methyl mercury ingested is absorbed into the bloodstream from the gastrointestinal tract. Mercury posoning by eating Fish:Japan, North Carolina, New Jersey, New york State, Puerto Rico, Amazon and virgin islands. Ritual Use of mercury: in haitian and carribean-american communities, particularly of spiritist faith such as Santeria. The ritual consists in the sprinkling of mercury about the home. Mercury is purveyed by some herbal medicine or botanical shops to consumers unaware of the dangers of the substance. This causes intoxication by mercury vapor. Some History:

Methyl mercury poisoning, better known as Minamata disease, is one of the most devastating forms of mercury exposure. It is named for Minamata Bay, a body of water in Japan where, in the early 1950s, the fish contained high concentrations of methyl mercury from the polluted waste of a nearby industrial plant. Local villagers ate the fish and began to exhibit signs of neurologic damage such as visual loss, extremity numbness, hearing loss, and ataxia. Babies exposed to the methyl mercury in utero were the most severely affected members of the village. Furthermore, because mercury was also discovered in the breast milk of the mothers, the babies’ exposure continued after birth. The setting: - When inhaled or absorbed through the skin, mercury accumulates in the liver, kidneys, brain and blood, causing both immediate and long term health effects.- Mercury taken into the body through air, water and food is absorbed in varying amounts depending on the route of intake. The major food sources of mercury are fish and shell fish.- Exposure to mercury also may occur by moving into a mercury-contaminated home, children playing with mercury, or exposing oneself through a hobby. Pathophysiology: Mercury is an element and cannot be broken down into harmless components. Because mercury binds to the body's ubiquitous sulfhydryl groups, toxicity involves multiple organ systems. Structural proteins, membranes, and enzymes are all disrupted. Methyl mercury exerts its most devastating effect on the central nervous system by causing psychiatric disturbances, ataxia, visual loss, hearing loss, and neuropathy. Methyl mercury is lipophilic and readily crosses the blood-brain and placentofetal barriers. Neurologic damage in the form of diffuse and widespread neuronal atrophy is most severe in patients exposed in utero. Necrosis of the proximal tubules is a common direct renal toxic effect. Unexplained renal abnormalities with neuropsychiatric disturbances should prompt the physician to consider Minamata disease or other forms of mercury poisoning. The effect:The most universal effect of mercury is damage to the nervous system. Mental instability, dizziness, numbness in the limbs and personality changes such as nervousness, increased excitability or insomnia may occur after exposure to mercury vapors. The phrase "mad as a hatter" originated from the often strange behavior of hat makers, caused by exposure to mercury vapor used in making felt hats. The history of mercury and its toxic effects date back to the fifteenth century B.C. Criminals sentenced to work in mercury mines had a life expectancy of three years. Can mercury affect some people more than others?Age: Children are especially susceptible to the adverse effects of mercury. In pregnant women, mercury easily crosses the placental barrier and concentrates in the fetus more readily than in the mother. Thus, a pregnant woman exposed to toxic levels of mercury may not exhibit any signs of mercury poisoning, but her child may be born with brain damage similar to cerebral palsy or autism. The scenario:Children develop the symptoms of mercury poisoning more quickly and severely than adults. Neurological symptoms are very similar to those seen in adults. Children also may

develop a bright red reash with sheets of peeling skin. Symptoms include:- Perioral and facial paresthesias- Extremity numbness- Dysarthria- Headache - Constriction of the visual fields- Difficulty in hearing- Memory loss- Ataxia - problems in walking Methyl mercury exerts its most devastating effect on the central nervous system by causing the following:Psychiatric disturbances, Ataxia, Visual loss, Hearing loss, and Neuropathy Clinical examination typically reveals the following:* Deficits in the visual field relative to confrontation* Ataxia* Tremor* Psychiatric disturbances such as anxiety* Seizures* Respiratory distress and dermatitis can occur acutely. Neurotoxicity is the most damaging syndrome.Severe poisoning eventually causes the patient to lie in a mute semirigid posture that is broken only by episodes of crying or primitive reflexive movements.Unexplained renal abnormalities with neuropsychiatric disturbances should prompt the physician to consider Minamata disease or other forms of mercury poisoning.The most damaging effect of ingested inorganic mercury (eg, mercuric chloride) is caustic gastroenteritis. Ingested elemental mercury is considered nontoxic because of its poor absorption in the gut; it is most dangerous as a vapor because it can cause acute lung injury and respiratory failure. Differential:Schizophrenia and Other Psychoses Substance Abuse: Cocaine Toxicity, Hallucinogens - PCP Autism: presents very many similarities (see 3) The Diagnosis:- Labs: blood, urine, and (sometimes) tissue analyses are required to confirm the diagnosis of mercury intoxication. Blood Analysis:Methyl mercury concentrates in red blood cells. Consequently, a direct determination of the blood mercury concentrations is essential.The normal range of mercury concentrations in whole blood is 0-10 mcg/L. Early signs and symptoms may occur with concentrations greater than 35 mcg/L.The severity of mercury poisoning is not always correlated with the blood concentration because of the redistribution of mercury in the tissues, specifically those of the central nervous system. Urinalysis

The detection of mercury in the urine demonstrates that exposure has occurred; however, it does not indicate the severity of mercury poisoning.Methyl mercury is primarily excreted through the feces; the urinary excretion of mercury is minimal.Chelated mercury is excreted primarily through the kidneys. Therefore, urinary assays are useful in monitoring chelation therapy. Histologic Findings: Necrosis of the proximal tubules is a common direct renal toxic effect. Medical Care: Medical Care: The general management measures in Minamata disease are the same as in those of any other toxicologic exposure. After initial assessment and stabilization of the patient’s condition, eliminate the patient's exposure to the source of the mercury. Provide general supportive measures, including monitoring, the performance of baseline laboratory studies, and the creation of a differential diagnosis. Once the neurologic consequences of Minamata disease appear, they are, unfortunately, irreversible. The goal of medical management in Minamata disease is to reduce the total body burden of mercury and minimize further damage. * Chelating agents (classified Category C = Safety for use during pregnancy has not been established): Because mercury binds to the body's ubiquitous cellular sulfhydryl groups, chelating agents should be administered early in treatment. These agents are thought to competitively bind the mercury by using its thiol groups. Currently, the best agent for the treatment of Minamata disease is 2,3-dimercaptosuccinic acid (DMSA). Its toxicity is low, and animal trials have shown that it is superior to older chelating agents such as dimercaprol (BAL) and d-penicillamine (DPCN). Even in cases of inorganic mercuric salt exposure, DMSA is preferred over DPCN. * Gastric Lavage: Because of the high propensity for neurologic impairment, patients with acute mercury ingestion should undergo gastric lavage with solutions containing proteins such as those from milk or egg whites. In addition, activated charcoal should be administered even though it does not absorb heavy metals well in general. Diet: Because of the high morbidity and mortality rates associated with methyl mercury poisoning, especially in utero, pregnant women and nursing mothers should avoid consuming larger fish (shark, swordfish, and large tuna steaks ) because their mercury concentrations tend to be higher than those in smaller fish. Outpatient follow-up:- Monitor Mercury levels for months- Monitor Renal Function- Neurological and Psych examinations Complications: Acute perioral and facial paresthesias, Respiratory distress and nonspecific dermatitis, Extremity numbness eventually along with headache, fatigue, tremor, Ataxia and dysarthria. Severe poisoning eventually causes the patient to lie in a mute semirigid posture that is broken only by episodes of crying or primitive reflexive movements.Babies exposed in utero are the most severely affected (low birth weight, seizure disorders, profound developmental delay, incomplete visual loss (including tunnel

vision), total blindness, and hearing loss).Long-term studies indicate that even prenatal exposure at low concentrations can cause subtle but detectable decrements in the areas of motor function, language, and memory.Children so affected may have long-term stigmata, including motor impairment, visual loss, hearing loss, developmental delay, and seizure disorders. Prevention: What should I do if I spill mercury in my house? If the mercury spills onto a surface, IT SHOULD NOT BE VACUUMED. Vacuuming causes mercury to vaporize and spread easily through the air, causing contamination. The correct way to dispose of small amounts of mercury is to pick it up with the sticky side of a piece of tape, place it in a sealed container and place it in the trash outside. Do not put mercury down the drain or dispose of it in the house, because it will continue to vaporize. Contact EPA when a spill of mercury would form a pool larger than a quarter. Medicolegal pitfalls: * Consider mercury intoxication in the differential diagnosis when unexplained neuropsychiatric disturbances are coupled with renal abnormalities.* Because of the high morbidity and mortality rates associated with methyl mercury poisoning, especially in utero, pregnant women and nursing mothers should avoid consuming larger fish because their mercury concentrations tend to be higher than those in smaller fish. Mercury intoxication after eating fish: What is mercury and how does it get into the environment?Mercury is a metal that occurs naturally at low levels in rock, soil and water throughout North Carolina. Mercury is also released into the air, water and land when fossil fuels (coal, oil and natural gas) are burned; when municipal solid waste or medical waste is incinerated; during forest fires; and during some manufacturing processes. Reference: 1) http://www.emedicine.com/ped/topic1461.htm2) http://www.epi.state.nc.us/epi/fish/mercuryhealthfacts.html3) http://www.whale.to/a/table_a.htm4) http://www.epa.state.oh.us/pic/facts/mercury.htmlWhat is the difference between Cushing Syndrome and Cushing Disease?

First of all, Answer to your question- Cushing disease = Pituitary adenoma ACTH secreting. It displays clinical Cushing Syndrome. - Cushing syndrome is everything else with clinical features except etiology is NOT pituitary. CUSHING SYNDROME/DISEASE...GENERAL REVIEW General Considerations: Cushing syndrome is caused by prolonged exposure to elevated levels of either endogenous or exogenous glucocorticoids. In an emergency situation, remembering that the most common cause of Cushing syndrome is the use of exogenous glucocorticoids is important. Exogenous steroids may cause suppression of the hypothalamic-pituitary-adrenal (HPA) axis that can last for as long as a year after exogenous steroid administration has ended. Etiology:

- Adrenal Hyperplasia * Pituitary adenoma = Cushing Disease* ACTH or CRH secreting tumor = Ectopic (Oat cell CA - CA of thymus - PAncreatic CA - Bronchial Adenoma)- Adrenal Neoplasia - Exogenous/ Iatrogenic causes = MOST COMMON. Pathophysiology:Endogenous glucocorticoid overproduction or hypercortisolism that is independent of adrenocorticotropic hormone (ACTH) usually is due to a primary adrenocortical neoplasm. ACTH-secreting neoplasms cause ACTH-dependent Cushing syndrome. 80% are due to Classic Cushing disease = an anterior pituitary tumor. Ectopic sources of ACTH make up the balance of ACTH-dependent Cushing syndrome cases. Ectopic non pituitary = an oat cell, small-cell lung carcinoma, or carcinoid tumor. Rarely ectopic corticotropin-releasing hormone (CRH) secretion. Sex: female-to-male ratio is 5:1 for Cushing syndrome due to an adrenal or pituitary tumor. Ectopic ACTH production is more frequent in men than in women, due to the increased incidence of lung tumors in this population. Age: The peak incidence of Cushing syndrome due to either an adrenal or pituitary adenoma occurs between ages 25 and 40 years. Ectopic ACTH production due to lung cancer occurs later in life. History: - Weight gain, especially in the face, supraclavicular region, upper back, and torso.- Changes in skin= purple stretch marks, easy bruising, and other signs of skin thinning.- Irregular menses and hirsutism- Progressive proximal muscle weakness, patients may have difficulty climbing stairs, getting out of a low chair, and raising their arms.- Psychological problems (depression, cognitive dysfunction, and emotional lability)- New onset or worsening of hypertension and diabetes mellitus, difficulty with wound healing, increased infections, osteopenia, and osteoporotic fractures- Patients with an ACTH-producing pituitary tumor (Cushing disease) may develop headaches, polyuria and nocturia, visual problems, or galactorrhea.- Mass effect on the anterior pituitary (hyposomatotropism, hypothyroidism, and hypogonadism) Physical: - Obesity, moon facies, buffalo hump, and supraclavicular fat pads. - Central obesity with increased adipose tissue in the mediastinum and peritoneum, increased visceral fat is evident on CT.- Skin, Facial plethora, Violaceous striae over the abdomen, buttocks, lower back, upper thighs, upper arms, and breasts. Ecchymoses may be present. Patients may have telangiectasias and purpura.Cutaneous atrophy with exposure of subcutaneous vasculature tissue and tenting of skin may be evident. - Steroid acne- Acanthosis nigricans, which is associated with insulin resistance and hyperinsulinism, may be present. The most common sites are axilla and areas of frequent rubbing, such as

over elbows, around the neck, and under the breasts. - Cardiovascular/renal: Hypertension, Volume expansion (edema from sodium and water retention). - Atherosclerotic heart disease is caused by lipid abnormalities, while diabetes mellitus and hypertension are caused by Cushing syndrome. - Gastroenterologic: Peptic ulceration (rare in endogenous hypercortisolism). - Endocrine: Hypothyroidism may occur from anterior pituitary tumors, which can interfere with proper thyroid-releasing hormone (TRH) and thyroid-stimulating hormone (TSH) function. - Galactorrhea may occur when anterior pituitary tumors compress the pituitary stalk, leading to elevated prolactin levels. - Other pituitary function may be interrupted without obvious clinical findings. Possibilities include polyuria and nocturia from diabetes insipidus. - With severe hypercortisolism, hypokalemic metabolic alkalosis may occur.- Osteoporosis = incident fractures and kyphosis, height loss, and axial skeletal bone pain. Avascular necrosis of the hip also is possible from glucocorticoid excess. - Adrenal crisisPatients with cushingoid features may present to the emergency department in adrenal crisis. This may occur in patients on steroids who stop taking their glucocorticoids or neglect to increase their steroids during an acute illness. It also may occur in patients who have recently undergone resection of an ACTH-producing or cortisol-producing tumor.Physical findings that occur in a patient in adrenal crisis include hypotension, abdominal pain, vomiting, and mental confusion (secondary to low serum sodium or hypotension). Other findings include hypoglycemia, hyperkalemia, hyponatremia, and metabolic acidosis. Lab Studies: - WBC>11,000/mm3- Hypokalemic (NA IS NORMAL) metabolic alkalosis may occur in patients with urinary free cortisol (UFC) levels higher than 1500 mcg/24-h.- 20% have Glucose intolerance/DM OVERVIEW OF EVALUATION OF PT WITH PRESUMED CUSHING SYNDROME:

1- CLINICAL SUSPICION2- SCREENING TEST = OVERNIGHT DXM SUPPRESSION TEST (EXCLUDES CUSHING SYNDROME WITH 985 OF CERTAINTY). 3- IF ABOVE ABONORMAL = 24H URINE FREE CORTISOL + CREATININE4- IF ABOVE ABNORMAL = CUSHING SYNDROMENEXT = HIGH DOSE DXM SUPPRESSION TEST5- IF SUPPRESSION <50% CONTROL: CUSHING DISEASE (PITUITARY ADENOMA)IF NO SUPPRESSION: ADRENAL NEOPLASIA VS ECTOPIC TUMOR6- NEXT = ACTH LEVELIF HIGH = ACTH PRODUCING TUMOR => CT CHESTIG LOW = ADRENAL NEOPLASIA => URNIARY 17KS - DHEA-S - ABDOMINAL CT: ADRNEAL ADENOMA VS ADRENAL CARCINOMA. DETAILED EXPLANATION:

Diagnosis of excess endogenous cortisol production requires the demonstration of inappropriately high serum cortisol levels or its urinary metabolites.Because acute illness activates the HPA axis, resulting in increases in ACTH and cortisol, the laboratory workup for Cushing syndrome should not be performed when subjects are acutely ill.Two common screening tests for Cushing syndrome are the 24-hour UFC test and the overnight (ON) 1-mg dexamethasone suppression test.* The 24-hour UFC test is an excellent indicator of overall daily cortisol production. Values higher than 3- to 4-times the upper limit of normal are very suggestive of Cushing syndrome, whereas values 1- to 3-times normal are consistent with either pseudo-Cushing or Cushing syndrome. Ensuring that the 24-hour collection for this test was adequate, by simultaneously measuring urinary creatinine excretion on the same urine sample, is important.* The ON 1-mg dexamethasone suppression test calls for ingestion of 1 mg of dexamethasone at 11 PM, with measurement of an 8-AM serum cortisol the next morning. In healthy individuals, the serum cortisol should be less than 2-3 mcg/dL. Cushing syndrome may be excluded with a cortisol level less than 1.8 mcg/dL. Medications that increase corticosteroid-binding globulin, such as estrogen and tamoxifen, may cause appropriate increases in cortisol levels. Finally, medications that facilitate the metabolism of dexamethasone, such as phenobarbital, phenytoin, and rifampin, may cause false-positive results with the dexamethasone suppression test. In many instances, additional studies must be performed to establish the diagnosis of excess cortisol production. The 48-hour low-dose dexamethasone suppression test (0.5 mg dexamethasone PO q6h for 8 doses) has been used for many years.In healthy individuals, 24-hour urinary 17-hydroxycorticosteroids are suppressed to 4 mg or less during the second day of dexamethasone ingestion. Unfortunately, the sensitivity and specificity of this test are only approximately 70%. A promising new method of detecting mild glucocorticoid excess combines the 48-hour low-dose dexamethasone suppression test with CRH stimulation. Ovine CRH (1 mcg/kg IV) is given 2 hours after the eighth dose of 0.5 mg dexamethasone.Serum cortisol is measured 15 minutes after ovine CRH administration. A cortisol level of greater than 1.4 mg/dL is very suggestive of Cushing syndrome.Other tests that may be useful to identify Cushing syndrome are as follows: In order to institute appropriate therapy, the cause of excess cortisol secretion must be determined. The logical first step involves establishing the differential diagnosis between an ACTH-dependent or ACTH-independent disorder. A plasma ACTH (measured by an immunoradiometric assay) of less than 5 pg/mL is suggestive of a primary adrenal tumor. An ACTH greater than 10-20 pg/mL is consistent with ACTH-dependent Cushing syndrome.The 8-mg ON dexamethasone suppression test and the 48-hour high-dose dexamethasone test may be useful when baseline ACTH levels are indeterminate. These studies also help in determining whether a patient who has ACTH-dependent disease has pituitary-dependent or ectopic ACTH disease.In the ON 8-mg dexamethasone suppression test, individuals ingest 8 mg dexamethasone orally at 11 PM, with measurement of an 8-AM cortisol the next day. A baseline 8-AM

cortisol measurement is required. Suppression of serum cortisol to less than 50% of baseline is suggestive of a pituitary source of ACTH rather than ectopic ACTH or primary adrenal disease. However, the diagnostic accuracy is only 70-80%. With the 48-hour high-dose dexamethasone suppression test, patients ingest 2 mg dexamethasone every 6 hours for 8 doses. A decrease in UFC of greater than 50% is suggestive of an anterior pituitary adenoma, rather than ectopic ACTH or a primary adrenal tumor. Unfortunately, the sensitivity of this test is only 80%, with a specificity of 70-80%. The more stringent criterion of a 90% decrease in UFC levels excludes the diagnosis of ectopic ACTH and has 100% specificity for anterior pituitary disease.If concern for adrenal carcinoma exists, measurement of 17-ketosteroid or other cortisol precursors (such as serum dehydroepiandrosterone sulfate [DHEAS]) is useful. Imaging Studies: An abdominal CT scan is recommended if a primary adrenal problem is suspected. CT-guided fine-needle aspiration then may have a role in management. If a pituitary source of excess ACTH is suspected, patients should undergo a contrast-enhanced magnetic resonance imaging (MRI) study of the pituitary. Chest and abdominal CT scans should be performed in patients with suspected ectopic ACTH production.Octreotide scintigraphy may be helpful in detecting ectopic ACTH tumors because neuroendocrine tumors typically have cell surface receptors for somatostatin. Differential Diagnosis: - Alcoholics: high cortisol levels with clinical cushing- Depression: High cortisol with no clinical features- Anorexia Nervosa: Same wasting as in Cushing with extraordinary high levels of 24h urine cortisol- Morbid obesity mimicks results of DXM suppression test but urine free cortisol is normal- Pts on HAART for HIV-1 develop partial lipodystrophy with thin extremities and central obesity, buffalo hump. Medical Care: OverviewTreatment of Cushing syndrome is directed by the primary cause of the syndrome. A culprit tumor should be removed if possible. - The treatment of choice for endogenous Cushing syndrome is surgical resection of the causative tumor. - The primary therapy for Cushing disease is transsphenoidal surgery. Pituitary radiation may be useful if surgery fails for Cushing disease. - The primary therapy for adrenal tumors is adrenalectomy. When surgery is not successful or cannot be used, as often occurs with ectopic ACTH or metastatic adrenal carcinoma, control of hypercortisolism may be attempted with medication. However, medication failures are common, and adrenalectomy may be indicated in ACTH-mediated Cushing syndrome. - The treatment for exogenous Cushing syndrome is gradual withdrawal of glucocorticoid. Cushing syndromeAgents that inhibit steroidogenesis, such as mitotane, ketoconazole, metyrapone,

aminoglutethimide, trilostane, and etomidate, have been used to cause medical adrenalectomy. These medications are used rarely and often are toxic at the doses required to reduce cortisol secretion. Thus, medical treatment should be initiated cautiously and, ideally, in conjunction with a specialist. Efficacy of these medical interventions can be assessed with serial measurements of 24-hour UFC. Patients receiving these medications may require glucocorticoid replacement to avoid adrenal insufficiency. Metyrapone and trilostane are agents that competitively inhibit a single steroidogenic enzyme. Ketoconazole and aminoglutethimide act at several sites. If enzymatic blockade is not complete, ACTH secretion overcomes the blockade so that hypercortisolism persists. Because ACTH production may persist or increase in patients with Cushing disease, radiation therapy of the pituitary often is required after unsuccessful initial therapy, either surgical or medical. These agents have higher efficacy when used in combination because they may act synergistically. Ketoconazole probably is the most popular and effective of these agents for long-term use and usually is the agent of choice. It acts on several of the P450 enzymes, including the first step in cortisol synthesis, cholesterol side-chain cleavage, and conversion of 11-deoxycortisol to cortisol. It also may inhibit ACTH secretion when used at therapeutic doses (200-400 mg bid-tid). Adverse effects of ketoconazole include headache, sedation, nausea, irregular menses, decreased libido, impotence, gynecomastia, and elevated liver function tests. The drug is contraindicated during pregnancy. Ketoconazole is ineffective in patients on H2 blockers or proton-pump inhibitors because gastric acidity is required for metabolism. If this agent is ineffective at controlling hypercortisolism, the dose may be maintained while another steroid enzyme inhibitor, typically metyrapone, is initiated. Metyrapone blocks 11-beta-hydroxylase activity (the final step in cortisol synthesis) and, at high doses, may inhibit ACTH secretion. Therapy is begun at 1 g/d divided into 4 doses and increased to a maximum dose of 4.5 g/d. Adverse effects are from increases in androgen and mineralocorticoid precursors, including hypertension, acne, and hirsutism. Aminoglutethimide is an anticonvulsant agent that blocks cholesterol side-chain cleavage to pregnenolone. It is a relatively weak adrenal enzyme inhibitor at doses that patients can tolerate. Aminoglutethimide typically is initiated at 250 mg twice daily, and increased to 2 g four times daily. - Adverse effects of aminoglutethimide include somnolence, headache, a generalized pruritic rash, hypothyroidism, and goiter. In rare cases, it may cause bone marrow suppression. Aminoglutethimide increases the metabolism of dexamethasone but not cortisol. Trilostane is not widely available and is not as well studied. Trilostane inhibits the conversion of pregnenolone to progesterone, which decreases the synthesis of cortisol, aldosterone, and androstenedione. It is not a first-choice agent because it is a weak inhibitor of steroidogenesis. In addition, trilostane interacts with some assays, causing a false elevation of cortisol measurements. Etomidate, an imidazole-derivative anesthetic agent, blocks 11-beta-hydroxylase. It is

used intravenously at 0.3 mg/kg/h. Its use is limited by the requirement for chronic administration by the intravenous route. Mitotane is an adrenolytic agent that acts by inhibiting 11-beta hydroxylase and cholesterol side-chain cleavage enzymes. This drug also leads to mitochondrial destruction and necrosis of adrenocortical cells in the zona fasciculata and reticularis. For this reason, it is used in treatment of adrenal cancer. Its survival benefit is unclear. It can be used in addition to radiation therapy for treatment of Cushing disease and in combination with metyrapone or aminoglutethimide for treatment of ectopic ACTH secretion. - Unfortunately, mitotane is expensive, and its utility is limited by adverse gastrointestinal and neurological effects, including nausea, diarrhea, dizziness, and ataxia. Other adverse effects include rash, arthralgias, and leukopenia. It is taken up by adipose tissues and persists in the circulation long after discontinuation. It is a potential teratogen and can cause abortion; therefore, it is relatively contraindicated in women interested in remaining fertile. Mifepristone (RU 486) is an antiprogestational agent, which, at high doses, competitively binds to the glucocorticoid and progesterone receptors. It currently is used only on an investigational basis for treatment of Cushing syndrome. Agents that decrease CRH or ACTH release have been studied for the treatment of Cushing disease. Such agents include bromocriptine, cyproheptadine, valproic acid, and octreotide. Currently, use of these agents is investigational. Surgical Care: - Cushing diseaseTreatment of choice for classic Cushing disease is transsphenoidal surgery by an experienced neurosurgeon. The goal of surgery is to remove the adenoma, preserving as much pituitary function as possible.Successful amelioration of hypercortisolism occurs in 60-80% of cases. Both open and laparoscopic techniques are possible. If unsuccessful, MRI-guided pituitary surgery, a new procedure, may be indicated. - Pituitary irradiation is employed when transsphenoidal surgery is not successful or not possible. The procedure is less successful than surgery in adults, with a 45% cure rate in adults and 85% cure rate in children. Late-onset adverse effects include hypopituitarism. - Bilateral adrenalectomy is an option if transsphenoidal surgery, pituitary irradiation, and medical therapy fail or if rapid normalization of cortisol levels is required. The patient then requires lifelong glucocorticoid and mineralocorticoid therapy. In individuals who undergo bilateral adrenalectomy, Nelson syndrome, ie, symptomatic enlargement of the pituitary gland and adenoma, may occur in one quarter to one half of adults not treated with pituitary irradiation and in as many as one quarter of patients pretreated with radiation therapy. - Ectopic adrenocorticotropic productionSurgical resection of the source of ACTH production may not always be possible. Medical therapy or bilateral adrenalectomy may be required.* Adrenal sourceAdenomas may be removed with unilateral adrenalectomy, often with a laparoscopic approach. Carcinomas should be resected for palliation.

Micronodular or macronodular hyperplasia causing Cushing syndrome may be treated effectively by bilateral adrenalectomy. Unilateral or subtotal adrenalectomy may lead to recurrence. - Hormone replacementPatients with endogenous Cushing syndrome who undergo resection of pituitary, adrenal, or ectopic tumors should receive stress doses of glucocorticoid in the intraoperative and immediate postoperative period. Typically, hydrocortisone at 200-300 mg is infused intravenously, either continuously or in boluses (60-100 mg every 8 h) starting prior to surgery and for the first 24 hours afterwards. If the patient does well, intravenous glucocorticoid replacement may be tapered over 1-2 days and replaced with an oral formulation. The rate of steroid taper may be slowed if severe preoperative hypercortisolism was present.In the event of pituitary destruction or bilateral adrenalectomy, lifelong steroid replacement is necessary. Complications:

OsteoporosisIncreased susceptibility to infectionsHirsutismDiabetes mellitusHypertensionRisk for adrenal crisisPanhypopituitarismDiabetes insipidus Medical/Legal Pitfalls:

Patients with Cushing syndrome due to exogenous steroid use are at risk for having an adrenal crisis if they do not receive stress doses of steroids during acute illnesses.Untreated adrenal crises can lead to death.High levels of endogenous or exogenous glucocorticoids may mask the abdominal symptoms associated with catastrophic abdominal events such as perforated bowel. Two catastrophic medical crises that occur in glucocorticoid excess states are perforated viscera and opportunistic fungal infections. References: 1) Emedicine:http://www.emedicine.com/MED/topic485.htm2) Kaplan notes3) CMDT p1126-1128

Gotta go..gotta go.. right now!!Diabetes insipidusHypercalcemia/Hypokalemia: mechanism by which they provoke DI?First of all, answer to your question:The actions of ADH are mediated through at least 2 receptors—V1 mediates

vasoconstriction, enhancement of corticotrophin release, and renal prostaglandin synthesis; V2 (Aquaporin) mediates the antidiuretic response. NDI arises from defective or absent receptor sites at the cortical collecting duct segment of the nephron or defective or absent aquaporin, the protein that transports water at the collecting duct. As a consequence of one of these defects, the ducts do not respond appropriately to vasopressin. Normally, vasopressin is transported in the blood to receptor sites on the basolateral surface of the collecting duct membrane. Through a G protein–adenylate cyclase coupling, activation of the vasopressin receptor increases cyclic adenosine monophosphate (AMP) production and stimulates protein kinase A, leading to increased recycling of the protein aquaporin in the plasma membrane. In the presence of vasopressin stimulus, exocytic insertion of aquaporin into the apical, or luminal, surface of the tubule cell occurs. Aquaporin enhances water entry into the cell from the lumen. Absence of the vasopressin receptor does not allow this process to take place, causing inhibition of water uptake and polyuria.Hypokalemia and hypercalcemia have been shown to suppress cortical Aquaporin 1 (AQP2) as well as to downregulate medullary AQP2, resulting in a reversible nephrogenic DI. Next: The review Diabetes InsipidusBackground: The word diabetes is derived from the Greek verb diabainein, which means to stand with legs apart, as in urination, or to go through. Insipidus comes from a Latin word meaning without taste. In contrast to diabetes mellitus (DM), which describes the excretion of sweet urine, diabetes insipidus (DI) describes the passing of tasteless urine because of its relatively low sodium content. Nephrogenic DI (NDI) reached North America in 1761, carried by Ulster Scots who arrived in Nova Scotia, Canada, on a ship named Hopewell. Scottish folklore reports the existence of the disease in Scotland before 1761. According to legend, a gypsy woman traveling with her thirsty son is denied water by a housewife. The gypsy woman curses the housewife, causing the housewife's sons to crave water while condemning her daughters to pass the curse on to future generations. General Considerations:- Central diabetes insipidus (DI) is characterized by decreased secretion of antidiuretic hormone (ADH), AKA arginine vasopressin (AVP), that results in polyuria and polydipsia by diminishing the patient's ability to concentrate urine. Diminished or absent ADH can be the result of a defect in one or more sites involving the hypothalamic osmoreceptors, supraoptic or paraventricular nuclei, or the supraopticohypophyseal tract. In contrast, lesions of the posterior pituitary rarely cause permanent DI because ADH is produced in the hypothalamus and still can be secreted into the circulation. - Nephrogenic DI is characterized by a decrease in the ability to concentrate urine due to a resistance to ADH action in the kidney. Nephrogenic DI can be observed in chronic renal insufficiency, lithium toxicity, hypercalcemia, hypokalemia, and tubulointerstitial disease. The rare hereditary form of nephrogenic DI is transmitted as an X-linked genetic defect of the V2 receptor gene. A rare autosomal variant is caused by mutation in the aqua porin

gene AQP2, a water-channel exclusively expressed in the collecting ducts of the kidney. Clinically:- The most common form of DI is that which follows trauma or surgery to the region of the pituitary and hypothalamus. It may exhibit 1 of 3 patterns—transient, permanent, or triphasic. The triphasic pattern is observed more often clinically.** First, a polyuric phase = fall in urine osmolality ** Second, an antidiuretic phase from release of stored hormone = urine osmolality rises.** The third phase can be permanent DI, when stores of ADH are exhausted Polyuria, polydipsia, and nocturia (from 3-18 liters) are the predominant symptoms. In infants, crying, irritability, growth retardation, hyperthermia, and weight loss may be the most apparent signs. In children, enuresis, anorexia, linear growth defects, and fatigability typically predominate. Patients with a nontraumatic onset typically have a much more indolent course. The daily urine volume is highly variable (3-20 L/d), and patient tolerance of dehydration also varies among individuals. Note: Pregnancy is associated with increased risk of DI. It's called VASOPRESSINASE-INDUCED DI. IT IS SEEN IN THIRD TRIMESTER AND PUERPERIUM. IT IS OFTEN ASSOCIATED WITH OLIGOHYDRAMNIOS, PRE-ECLAMPSIA, OR HEPATIC DYSFUNCTION. A circulating enzyme destroys native vasopressin; however synthetic desmopressin is unaffected, THEREFORE RESPONDING TO DESMOPRESSIN THERAPY AND SUBSIDING SPONTANEOUSLY THEREAFTER. Physical Examination: Normal or signs of dehydration Causes: - Central: traumatic/surgery, neoplastic or infiltrative of the hypothalamus or pituitary (adenomas, cranipharyngiomas, leukemia, sarcoid histiocytosis), Radiation therapy, HTN, and meningitis. 30% are idiopathic. - Mephrogenic: idiopathic, hypercalcemia, hypokalemia, Sickle Cell Disease, pyelonephritis, Sarcoidosis, Multiple Myeloma, or drugs (Lithium, demeclocycline, colchicine, foscarnet, methicillin). Workup: Perform testing with the patient maximally dehydrated as tolerated, ie, at a time when ADH release would be highest and urine would be most concentrated. Ruling out secondary causes, such as diabetes mellitus, also is important.1) USUALLY IN THE QUESTION STEM: *** 24h urine collection (volume, glucose, creatinine, urine specific gravity, urine Na, Uosm) ***Serum: electrolytes and glucose, simultaneous serum and urine osmolality, and ADH levels. A urine specific gravity of 1.005 or less and a urine osmolality less than 200 mOsm/kg is the hallmark of DI. Random plasma osmolality generally is greater than 287 mOsm/kg. 2) The water deprivation test = compares Uosm after dehydration versus Uosm after vasopressin. Details: All water intake is withheld and urine osmolality and body weight are measured hourly. When 2 sequential urine osmolalities vary by less than 30 mOsm or if the weight

decreases by more than 3%, 5 U of aqueous vasopressin is administered subcutaneously. A final urine specimen is obtained 60 minutes later for osmolality measurement.In healthy individuals, water deprivation leads to a urine osmolality that is 2-4 times greater than plasma osmolality. Administration of vasopressin results in less than 9% increment in urine osmolality. The time required to achieve maximal urine concentration ranges from 4-18 hours.In complete central DI, testing reveals minimal ADH levels and activity, with failure of the urine to be concentrated despite excessively concentrated serum. In response to exogenous vasopressin, urine osmolality increases by more than 50%.Patients with nephrogenic DI have a normal-to-elevated serum ADH level and failure of the kidney to respond to exogenous ADH during the water deprivation test. 3) Measuring Posm and Uosm at intervals and plotting them. Compare with relationship betwen Posm and Uosm in normal individuals. If on the right, it's DI. 4) Vasopressin challenge test: If injection of vasopressin normalizes response, diagnosis of Central DI is made. 5) Consider MRI of pituitary/hypothalamus/skull: T1-weighted images of the healthy posterior pituitary yield a hyperintense signal. In patients with central DI this signal is absent except in the rare familial form of central DI where the signal is still present. Medical Care: ** In an emergency, most patients can drink enough fluid to replace their urine losses. Replace losses with dextrose and water or IV fluid hyposmolar to the patient's serum. Avoid hyperglycemia, volume overload, and a correction of hypernatremia that is too rapid. A good rule of thumb is to reduce serum sodium by 0.5 mmol/L/h. Water deficit may be calculated based on the assumption that body water is approximately 60% of body weight in kilograms.In case of inadequate thirst, DESMOPRESSIN IS THE D.O.C. Generally, it can be administered 2-3 times per day. Patients may require hospitalization to establish fluid needs. Frequent electrolyte monitoring is recommended. Pharmaceutical therapy for DI includes subcutaneous, nasal, and oral preparations of vasopressin analogues, as well as chlorpropamide, carbamazepine, clofibrate, thiazides, and indomethacin (limited efficacy). Surgical Care: Postoperatively, administer the usual dose of desmopressin to patients with DI and administer (hypotonic) IV fluids to match urine output.After pituitary surgery, administer parenteral desmopressin every 12-24 hours, along with adequate fluid to match losses. Follow the specific gravity of the urine and administer the next dose of desmopressin when the specific gravity has fallen to less than 1.008-1.005 with an increase in urine output. When the patient can tolerate oral intake, thirst can become an adequate guide. Medical Treatment: - Intranasal Desmopressin acetate (DDAVP)= DOC for Central DI. It is also useful for DI associated with pregnancy or puerperium (category B = Usually safe but benefits must outweigh the risks). Drug is not given to patients with Liver Disease because of mild elevation of liver enzymes. - Nephrogenic DI may respond to combinations of indomethaci-hydrochlorothiazide or IDM-Desmopressin or IDM-amiloride.

- Some drugs can be used to stimulate ADH secretion: Chlorpropamide, Clofibrate, and Carbamezepine. Special Precautions:Patients with DI also must take special precautions, such as when traveling, to be prepared to treat vomiting or diarrhea and to avoid dehydration with exertion or hot weather. Differential: - Polyuria of Cushing Syndrome- Polyuria of Lithium Rx- Nocturnal Polyuria of Parkinson- Psychogenic polydypsia- CNS Sarcoidosis- IVF administration- R/O Diabetes Mellitus

References: 1- Early polyuria and urinary concentrating defect in potassium deprivation Am J Physiol Renal Physiol 279: F655-F663, 20002- Nephrogenic diabetes insipidusAnn Endocrinol (Paris). 1999 Dec;60(6):457-643- Decreased aquaporin-2 expression and apical plasma membrane delivery in kidney collecting ducts of polyuric hypercalcemic rats.Am Soc Nephrol. 1998 Dec;9(12):2181-934- CMDT p1075-10775- Kaplan Notes: 20026- Emedicinehttp://www.emedicine.com/med/topic543.htmhttp://author.emedicine.com/PED/topic580.htmClinically: Chronic Bronchitis Vs EmphysemaClassicly, COPD patients have been categorized into:- Pink puffers- Blue Bloaters Pink Puffers: Emphysema predminant:- Dyspnea is predominant, severe- Cough is rare, scant phlegm- Thin- weight loss - Barrel chest- Accessory muscles use, mouth almost closed to a puffing grimace to increase resistance to airway lost in the destruction of lung parenchyma. - Normal Hemoglobin, PaO2 normal, PaCO2 normal, O2Sat Normal. - CXR= Hyperinflation + Flat Diaphragm- TLC increased - DLco reduced- High V/Q areas (Dead space ventilation) Blue Bloaters: Chronic Bronchitis Predominant:- Chronic Cough is major complaint with mucopurrulent sputum- Frequent exacerbations due to chest infections. - Cyanotic and mouth wide open to breathe more air.

- Hemoglobin elevated with PaO2 reduced and PaCO2 elevated. Severe O2 desaturation. - CXR= Interstitial markings- TLC normal, DLco normal.- Low V/Q areas (increased perfusion)- Progressive cardiac/respiratory failure over time, with edema and weight gain- Patients may be obese (Obstructive sleep apnea might be associated)Because they share many of the same physical signs, COPD may be difficult to distinguish from CHF. One crude bedside test for distinguishing COPD from CHF is peak expiratory flow. If patients blow 150-200 mL or less, they are probably having a COPD exacerbation; higher flows indicate a probable CHF exacerbation. Pharmacological therapy for COPD includes a number of agents, enthusiasm for which has waxed and waned as our understanding of this disease has evolved. Table 1 lists agents that have been used in COPD for 1) alleviation of symptoms, for 2) treatment of acute exacerbations and 3) for the management of its long term consequences.Bronchodilators in COPDBronchodilator therapy in COPD is currently prescribed primarily for the relief of symptoms. There is no evidence that early regular use of these agents alters the progression of COPD. It is well established that both short acting beta-agonists and anticholinergic agents provide modest but significant relief for patients with COPD. Anticholinergics have been favored because they provide more consistent relief with fewer side effects (cardiovascular in particular) in this older population. For anticholinergic therapy, once a patient is found to suffer from daily symptoms, regular use of ipratropium bromide, the only currently available anticholinergic, is recommended on a regular basis, three or four times daily. Similarly, short acting beta-agonists, such as albuterol, pirbuterol or metaproterenol, are prescribed three to four times daily or before exercise. It is currently established that combined therapy with ipratopium and albuterol offer superior relief than mono-therapyBeta-2-agonistsSalmeterol xinafoate, a long acting beta-2-agonist, initially introduced for the treatment of asthma is now FDA-approved for COPD. A new agent, tiotropium bromide, not yet approved for clinical use in the U.S., combines anticholinergic safety with long duration of action.Theophylline compounds have lost their popularity because of their frequent GI and CNS side effects. Nevertheless, now given at relatively lower doses than previously prescribed, they offer an important bronchodilatator alternative, since they can be administered once or twice daily in oral form. Such a regimen may improve compliance. Additional potential effects that may benefit the COPD patient include their anti-inflammatory effect and their ability to improve respiratory muscle function.Anti-inflammatory TherapyThe role of corticosteroids in the management of COPD remains controversial. It is generally claimed that 15% of COPD patients can benefit from corticosteroid therapy. Corticosteroids are administered, both orally and parenterally, during acute exacerbation. Future DirectionsIn spite of clear-cut evidence that current pharmacotherapy (other than drugs associated with smoking cessation) alters the course of COPD, common sense and pathologic

evidence of persistent inflammatory effects in the airways of patients with COPD suggest that anti-inflammatory therapy should be helpful in the secondary prevention and management of COPD.Unlike the findings in asthma patients, the broncho-alveolar lavage (BAL) fluid in patients with COPD demonstrates that neutrophils (and not eosinophils) are the primary airway inflammatory cell involved in this process.Primary prevention, by smoking cessation, of lung and other cigarette related diseases remains a priority in our approach to COPD. Nevertheless, enhanced understanding of the natural history of this disease, its underlying pathology and the genetic causes for susceptibility promise to open many new avenues for the treatment of this disorder. References: 1) emedicine: http://www.emedicine.com/emerg/topic99.htm2) Cyberounds:http://www.cyberounds.com/conferences/pulmonary_medicine/3) CMDT: Please complete management p237-242HOW TO DIFFERENTIATE BTW ALZHEIMER AND HIVMr Smith goes to the doctor's office to collect his wife's test results. The lab tech says to him, "I'm sorry sir. There has been a bit of a mix-up and we've got a problem. When we sent the samples from your wife to the lab, the samples from another Mrs. Smith were sent as well, and now we are uncertain which one is your wife's. Frankly, it is either bad or terrible!!"."What do you mean?""well, one Mrs smith has tested positive for Alzheimer's and the other one for HIV". "That's terrible!!!. Can we do the tests over again?""Normally yes. But you have an HMO, and they won't pay for these expensive tests more than once"."Well, what am i supposed to do now?""Well, the HMO recommends that you DROP YOUR WIFE OFF IN THE MIDDLE OF TOWN. IF SHE FINDS HER WAY HOME, DON'T SLEEP WITH HER!!!”.erectile dysfunction in a diabetic patientA 62 yo man complains of impotence& decreased libido for 2yr.His PMH significant for NIDDM of 10yr,controlled by diet& oral agents.He’s been otherwise well.On Ph.E,the skin is thin& pale,testes r both small& soft.Lab:Serum testosterone:114 ng/dl(normal:270-1070ng/dl) HbA1c:8%LH:7 IU/L(normal:2-12 IU/L)FSH:5 IU/L(normal:1-8 IU/L)The most appropriate next step of management is:A:measurement of serum estradiolB:measurement of serum prolactinC:referral for penile prosthesisD:referral for Doppler studies of penile blood flowE:to start treatment with insulinF:to initiate administration of trazodoneG:to prescribe sildenafilH:to refer to a psychiatristAnswer is B.

The reported incidence of impotence in diabetic men at age 40 years is from 8-50%. The diabetes-related causes include: poor glycaemic control, autonomic neuropathy and atherosclerotic vascular disease.One is tempted to refer for Doppler studies of penile blood flow because of the long-standing diabetes where HbA1C is 8%. The cutoff point is 7.2% according to Swanson. However, Testosterone is important to erection. Testosterone is low in this patient along with normal gonadotrophins. This is suggestive of need for further assessment of hypothalamo-pituitary function. Prolactin level is the best option because of the effect of prolactin on the hypothalamic control of gonadotrophin. R/O Pituitary adenoma. References: - Swanson: DM- THE ASSESSMENT AND TREATMENT OF ERECTILE IMPOTENCEhttp://web.idirect.com/~ino/info18.htm

A 35 yo woman currently taking maintenance fluoxetine following recovery from a depressive episode 3mo ago complains of markedly decreased llibido.What’s the best initial intervention?A:Advise the woman that her libido will likely return when fluoxetine is discontinued as scheduled in 3moB:continue fluoxetine& add bupropionC:continue fluoxetine and add testosteroneD:discontinue fluoxetineE:discontinue the fluoxetine& start bupropionAnswer is E. Reference: 1- Improvement in fluoxetine-associated sexual dysfunction in patients switched to bupropion.J Clin Psychiatry. 1993 Dec;54(12):459-65.2- http://www.socialaudit.org.uk/4200ACAM.htmQ on viral hepatitisAn asymptoatic patient patient came with a h/o of exp to hepatitis C& serology came +ve.what shoud we do?Can we give interferon+ ribavarin? A patient came with a h/o exp to hepatitis A& serology came +ve.what shoud we doFirst, Answers to your questions:- Hepatitis A exposure with serology positive. No need for Immunoglobulin prophylaxis because it is then ineffective. It must be given right after exposure up to 2 weeks in the incubation period. No shown use if patient is Anti-HAV +. Similarly, patient already has serology positive, Vaccine is not recommended. Unless patient is symptomatic, only supportive measures are recommended for acute hepatitis. - Hepatitis C: Immunoglobulin Anti-C is no longer recommended. Treatment of Acute Hepatitis C involves Interferon 2b. The combination with Ribavirin results in increased response to treatment. - Hepatitis B: No antiviral treatment is established for Acute hepatitis B. Recommendations are for Immunoglobulins soon after exposure followed by Vaccine (see recommendations below).

Review of Treatment guidelines for Hepatitis A/B/C/D/E: Hepatitis A * Treatment for acute of hepatitis A virus infection Treatment for acute hepatitis caused by HAV is supportive in nature because no antiviral therapy is available for HAV infection. Hospitalization is needed for patients whose nausea and vomiting places them at risk for dehydration. Patients with acute liver failure require close monitoring to ensure that they do not develop FHF. * Prevention of hepatitis A virus infection The CDC now recommends vaccination against HAV for individuals traveling to endemic regions, and vaccination is recommended for any patient with chronic liver disease. A booster dose of the vaccine is recommended 6 months after initial vaccination. Whether HAV vaccine administration should be mandated in children (as is HBV vaccination) remains unclear. Administration of hepatitis A immune globulin is an alternative to vaccination against HAV infection. The dose is 0.02 mL/kg intramuscularly for individuals who anticipate spending fewer than 3 months in an endemic region. Travelers should receive 0.06 mL/kg intramuscularly every 4-6 months if they are planning to spend more than 3 months in a region where HAV is endemic. Postexposure prophylaxis with hepatitis A immune globulin is appropriate for household and intimate contacts of patients with HAV. It may be effective even when administered as late as 2 weeks after exposure. It also is recommended for contacts at daycare centers and institutions. Typical dosing of immune globulin is 0.02 mL/kg, given intramuscularly as a single dose. Prophylaxis is not necessary for casual contacts (office, factory, school, or hospital), for most elderly persons, who are very likely to be immune, or for those known to have anti-HAV in their serum. Hepatitis B: *Treatment of acute hepatitis B As with the treatment of acute hepatitis A, no well-established antiviral therapy is available for acute HBV infection. Supportive treatment recommendations are the same as for acute hepatitis A. Whether lamivudine and newer antiviral therapies have an impact on the natural history of severe cases of acute HBV infection remains unclear. * Treatment of chronic hepatitis B The goals of antiviral treatment of HBV infection are to reduce symptoms, if present; to inhibit viral replication, as marked by the loss of HBeAg and HBV DNA; and to prevent or delay progression of chronic hepatitis to cirrhosis or HCC. Antiviral therapy infrequently leads to viral eradication, as marked by the loss of HBsAg. Currently, no antiviral therapy is available for healthy carriers who do not have actively replicating virus. *Interferon alfa treatment for chronic hepatitis B Interferons have both antiviral and immunomodulatory effects. Treatment is typically for 16 weeks. Treatment with alfa-interferon is appropriate for many patients with chronic hepatitis B. Candidates for interferon therapy must have a clinical diagnosis of chronic HBV infection, with an elevated level of alanine aminotransferase (ALT). They must have evidence of active HBV replication, as marked by a positive HBeAg or a positive HBV DNA finding. The author recommends that liver biopsy be performed prior to therapy to confirm the diagnosis and document the severity of disease.

Adverse effects of interferon are common but lead to discontinuation of the drug in only 5-10% of patients. Adverse effects include flulike symptoms (eg, fatigue, fever, headache, myalgia, arthralgia), neuropsychiatric symptoms (eg, depression, irritability, somnolence), hematologic effects (eg, granulocytopenia, thrombocytopenia), and other miscellaneous effects (eg, pain at injection site, dyspepsia, alopecia, thyroid function abnormalities). Lamivudine for chronic hepatitis B Lamivudine is the negative enantiomer of 2’3'-dideoxy-3'-thiacytidine. It inhibits DNA polymerase–associated reverse transcriptase and can suppress HBV replication. Treatment with a dose of 100 mg orally once per day for 1 year results in loss of HBeAg in 32% of patients. Treatment also induces histologic improvement and a statistically significant reduction in the rate of development of hepatic fibrosis. The advantages of lamivudine over interferon include its ease of application, the virtual absence of adverse effects, and its efficacy in populations generally not responsive to interferon therapy. It also has been used successfully in patients with decompensated cirrhosis and in the treatment of recurrent hepatitis B following liver transplantation. However, 16-32% of patients who are considered to have responded to lamivudine experience a recurrence of HBV DNA positivity. This is explained by the development of a mutation at the YMDD locus in the HBV DNA polymerase gene. The long-term impact of such mutations is unknown. The possibility exists that combination therapy with nucleoside analogs, including lamivudine, will result in improved suppression of HBV replication. Such analogs include famciclovir, adefovir, and lobucavir. The latter 2 drugs are undergoing clinical trials for the treatment of both HIV disease and HBV infection and await FDA approval. * Hepatitis B virus vaccine: Pre-exposure prophylaxis:Plasma-derived and recombinant HBV vaccines use HBsAg to stimulate production of anti-HBs. The vaccines are highly effective, with a greater than 95% rate of seroconversion. Pregnancy is NOT a contraindication to vaccination. Vaccine administration is recommended for all infants and for adults at high risk of infection (eg, those on dialysis, health care workers). The recommended vaccination schedule for infants is an initial vaccination at the time of birth (ie, before hospital discharge), repeat vaccination at 1-2 months, and another repeat vaccination at 6-18 months. The recommended vaccination schedule for adults is an initial vaccination, a repeat vaccination at 1 month, and another repeat vaccination at 6 months. * Postexposure prophylaxis Hepatitis B immune globulin (HBIG) is derived from plasma. It provides passive immunization for individuals who describe recent exposure to a patient infected with HBV. Its contribution appears to be in reducing the frequency of clinical illness, not in preventing infection. HBIG also is administered following liver transplantation to those infected with HBV, in order to prevent HBV-induced damage to the liver allograft. Reccomendations are as follows:- Perinatal => HBIG immediately after birth + vaccination within the first 12 h, followed by the usual two other doses at appropriate times (90% effective) - Sexual contact with an acutely infected patient => HBIG within 14 days of exposure +

usual course of vaccination - Sexual contact with a chronic carrier => Vaccination - Household contact with an acutely infected patient => None - Household contact with an acutely infected person resulting in known exposure => HBIG +/- vaccination - Infant (<12 mo) primarily cared for by an acutely infected patient => HBIG +/- vaccination - Inadvertent percutaneous or permucosal exposure (e.g.: Needle Stick) => HBIG immediately after exposure + vaccination to begin within the first week after exposureWhen both HBIG and hepatitis B vaccine are recommended, they may be given at the same time but at separate sites. Hepatitis C: * Treatment of hepatitis C virus infection Antiviral therapy has a number of major goals. These include (1) to decrease viral replication or eradicate HCV, (2) to prevent progression of disease, (3) to decrease the incidence of cirrhosis, (4) to decrease the incidence of HCC, (5) to ameliorate symptoms such as fatigue and joint pain, and (6) to treat extrahepatic complications of HCV infection such as cryoglobulinemia or glomerulonephritis. Currently, Interferons are the backbone of antiviral strategies used against HCV. Future medications may target the enzymes responsible for HCV replication. They may have activity against viral helicases, proteases, and polymerases. Agents currently approved by the FDA for the treatment of HCV include (1) interferon alfa-2b (Intron, Schering, Kenilworth, NJ); (2) interferon alfa-2a (Roferon, Roche, Nutley, NJ); (3) consensus interferon (Infergen, Amgen, Thousand Oaks, Calif); and (4) ribavirin, which is used in combination with interferon alfa-2b (Rebetol and Rebetron, Schering, Kenilworth, NJ). Agents under study include (1) pegylated interferons alfa-2b and alfa-2a, (2) interferon-beta, (3) interferon-gamma, (4) natural interferon, (5) ursodeoxycholic acid, and (6) silymarin (milk thistle). * Treatment of acute hepatitis C Acute hepatitis C is detected infrequently. When it is identified, early therapy with interferon should be considered. A recent meta-analysis showed that 41% of patients treated with interferon had negative HCV RNA findings at the end of treatment, as opposed to only 4% of untreated controls. * Treatment of chronic hepatitis C Interferon alfa-2b, dosed at 3 million units subcutaneously 3 times per week, was approved by the FDA in 1991 for the treatment of chronic HCV infection.The combination of ribavirin, a nucleoside analog, with interferon alfa-2b has significantly improved patients' responses to treatment. *Postexposure prophylaxis for Hepatitis C:IG has been shown to be ineffective in preventing hepatitis C and is no longer recommended for postexposure prophylaxis in cases of perinatal, needle stick, or sexual exposure. * Preexposure Prophylaxis: Genotype and quasispecies viral heterogeneity, as well as rapid evasion of neutralizing

antibodies by this rapidly mutating virus, conspire to render HCV a difficult target for immunoprophylaxis with a vaccine.

Hepatitis D: *Treatment of hepatitis D Patients co-infected with HBV and HDV are less responsive to interferon therapy than patients infected with HBV alone. To date, lamivudine appears to be ineffective against HBV/HDV co-infection. * Prophylaxis for Hepatitis DInfection with hepatitis D can be prevented by vaccinating susceptible persons with hepatitis B vaccine Hepatitis E: *Treatment of hepatitis E The treatment of those infected with HEV is supportive in nature. Fulminant Hepatitis: In fulminant hepatitis, the goal of therapy is to support the patient by maintenance of fluid balance, support of circulation and respiration, control of bleeding, correction of hypoglycemia, and treatment of other complications of the comatose state in anticipation of liver regeneration and repair. Protein intake should be restricted, and oral lactulose or neomycin administered. Glucocorticoid therapy has been shown in controlled trials to be ineffective. Likewise, exchange transfusion, plasmapheresis, human cross-circulation, porcine liver cross-perfusion and hemoperfusion have not been proven to enhance survival. Meticulous intensive care is the one factor that does appear to improve survival. Orthotopic liver transplantation is resorted to with increasing frequency, with excellent results, in patients with fulminant hepatitis. References:- Emedicine http://www.emedicine.com/med/topic3180.htm- Harrisons 14th edition

Electroconvulsive TherapyAn acute response to unmodified ECT is bradyarrhythmia or even a cardiac asystole lasting for seconds followed by a transient tachycardia and increased blood pressure. The BP drops sharply during the initial vagal hypertonic phase and then rapidly increased upto 40% over baseline. BP peaks during the ictal period. The increase in systolic pressure is more than the diastolic pressure. Rate pressure product a product of heart rate and systolic BP is one measure of cardiovascular response during ECT. Use of atropine as premedication increases RPP response. Other agents used in ECT: Modified ECT use thiopentone sodium or methohexitol as anaesthetic, and succinyl choline as the muscle relaxant. Procedure: The treatment is carried out as follows: approximately 30 minutes before the scheduled treatment time, the patient may receive an injection of a medication (such as atropine) that keeps the pulse rate from decreasing too much during the convulsion. Next, the patient is placed on a cot and hooked up to a machine that automatically takes and

displays vital signs (temperature, pulse, respiration, and blood pressure) on a television-like monitor. A mild anesthetic is then injected into a vein, followed by a medication (such a Anectine) that relaxes all of the muscles in the body so that the seizure is mild, and the risk of broken bones is virtually eliminated. When the patient is both relaxed and asleep, an airway is placed in the mouth to aid with breathing. Electrodes are placed on the sides of the head in the temple areas. An electric current is passed through the brain by means of a machine specifically designed for this purpose. The usual dose of electricity is 70-150 volts for 0.1-0.5 seconds. In the first stage of the seizure (tonic phase), the muscles in the body that have not been paralyzed by medication contract for a period of five to 15 seconds. This is followed by the second stage (clonic phase) that is characterized by twitching movements, usually visible only in the toes or in a non-paralyzed arm or leg. These are caused by alternating contraction and relaxation of these same muscles. This stage lasts approximately 10-60 seconds. The entire procedure, from beginning to end, lasts about 30 minutes. Pre-care:Some medications, such as lithium (greater risk of cognitive side effects), benzodiazepines, and monoamine oxidase inhibitors, should be discontinued for some time before treatment. DO NOT DISCONTINUE ANTIEPILEPTICS USED TO TREAT EPILEPSY IN PT. SSRI's, Tricyclics, phenothiazines, are ok to continue Patients are instructed not to eat or drink for at least eight hours prior to the procedure in order to reduce the possibility of vomiting and choking. Aftercare After the treatment, patients are moved to a recovery area. Vital signs are recorded every five minutes until the patient is fully awake, which may take 15-30 minutes. Some initial confusion may be present but usually disappears in a matter of minutes. There may be complaints of headache, muscle pain, or back pain. Such discomfort is quickly relieved by mild medications such as aspirin. Risks Advanced medical technology has substantially reduced the complications associated with ECT. These include slow heart beat (bradycardia), rapid heart beat (tachycardia), memory loss, and confusion. Persons at high risk for ECT include those with recent heart attack, uncontrolled blood pressure, brain tumors, and previous spinal injuries. Normal results ECT often produces dramatic improvement in the signs and symptoms of major depression, especially in elderly individuals, sometimes during the first week of treatment. While it is estimated that 50% of these patients will experience a future return of symptoms, the prognosis for each episode of illness is good. Mania also often responds well to treatment. The picture is not as bright for schizophrenia, which is more difficult to treat and is characterized by frequent relapses. A few patients are placed on maintenance outpatient ECT. Special situations: - Patients with pacemakers: should be set to fixed mode. - Hypertension: Close control during procedure- Glaucoma: usually no problem. Consult ophthalmo. - Berry Aneurysm: No ECT until it is clipped.

- MI or CVA withint 3 months: Hyperoxygenate + Antihypertensives/antiarrhythmics PRN. - Cerebral Tumor: Ok to ECT if ICP normal. - COPD or respiratory limitations: Per anesthesiologist. Ok if can tolerate anesthesia. - Risk for retinal detachment: No ECT unless cleared by ophthalmo. - Pregnancy: Ok to ECT with close fetal monitoring only if high risk pregnancy. - Bone/Joint conditions (e.g.: joint prosthesis). Ok to ECT provided complete relaxation. - Deficiency of plasma pseudocholnesterase: leads to prolonged apnea after ECT. Guidelines are available. - Porphyrias: Avoid barbiturates during premedication. - Skull defect - Status post craniotomy: Avoid it when placing electrodes. - Dementia: Not a CI, but only if depression is a concomittent affective disorder will it respond to ECT. References: - ECT: http://home.iprolink.co.nz/~felicity/Ect_registrar_handout.pdfa 54 yo hospitalized woman who has severe recurrent MDD improves dramatically after her first 2Rx with bilateral ECT.after the 4th ECT she's disoriented to the date.The best choice for furthur Rx?a:administer 2more ECT and then initiate antidepressant medicationsb:discontinue ECT&treat with antidepressant medicationc:discontinue ECT until her cognitive status improves and then resume ECTd:initiate a mood stabilizing medication and continue ECTe:switch to unilateral ECT for 4 additional RxAnswer is E ECT: Bilateral is preferred where there is urgency or where the patient has a high seizure threshold and good responses are hard to acheive. Once the inital urgency has passed, consider changing to unilateral placement after a few ECT's, as this DOES protect against both immediate and longer term cognitive problems post-ECT. It should always be tried if a patient has marked post-ECT confusion or memory deficit. If a patient has not responded to unilateral ECT after 6 treatments, abandon it and use bilateral.If a patient is strongly left-dominant, better to use bilateral not unlateral ECT. Reference: http://home.iprolink.co.nz/~felicity/Ect_registrar_handout.pdf A 43 yo woman has depressive rumination,hypersomnia,hyperphagia and a subjective sense of heaviness in her limbs that have not responded to trials of fluoxetine& nortriptyline.The most appropriate next step of management of this case is initiate:A:desipramineB:methylphenidateC:sertalineD:tanylcypromineE:trazodoneAnswer is D. This is atypical depression due to features of hyperphagia, and leaden paralysis, unresponsive to tricyclics and SSRI's. Drug of choice is MAOI. Hydrazines (e.g.: phenelzine) and non Hydrazines (e.g.: Tranylcypromine) are to be considered in this

patient.An 85yo widow who lives alone presents with weight loss,decreased energy,insomnia and a lack of interest in her usual activities.Her PMH is positive for HTN,AF,urge incontinence and contact dermatitis.She’s taking oxybutynin that she refuses to discontinue bcoz of its effectiveness.With diagnosis of depression and excluding other possible diagnosis,which drug is the best choice for this pt?a:thioridazineb:amitriptylinec:imipramined:doxepine e:trazodonef:either d or eAnswer is: E. Trazodone It is a weak SSRI Antidepressant. The main side effect is Priapism (obvisouly not the main concern in this patient). The other side effect to bear in mind is arrythmias in pre-exisiting cardiac disease for which it is recommended to closely monitor patient on it. Patient has Afib. Patient is presented with a combination of Urinary incontinence and depression. From a test-taking strategy point of view, Doxepine, amitryptiline, and imipramine are all Tricyclics so they can't be considered because they have similar effects. And Thioridazine is not indicated. Trazodone is left. From a medical point of view, Urge incontinence is due to detrusor instability for which patients take anticholinergics OR Tricyclics. Patient is stabilized on anticholinergic Oxybutyrin. Adding a Tricyclic will potentiate the anticholinergic side effects and may result in urinary prooblems such as OVERFLOW URINARY INCONTINENCE. SSRI's are under investigation as a treatment for urinary incontinence as well with fewer side effects than tricyclics. Consequently, Trazodone is the right answer. References: - http://www.mentalhealth.com/drug/p30-d03.html#Head_4- http://www.healthandage.com/Home/gm=6!gid6=5013Review on Hypersensitivity PneumonitisHypersensitivity Pneumonitis A- General Considerations:Farmer’s lung is the most common type of hypersensitivity pneumonitis. Hypersensitivity pneumonitis, also known as extrinsic allergic alveolitis, is an immunologically mediated inflammatory disease of the lung involving the terminal airways. The condition is associated with intense or repeated exposure to inhaled biologic dusts. The classic presentation of farmer’s lung results from inhalational exposure to thermophilic Actinomyces species and occasionally from exposure to various Aspergillus species. These organisms flourish in areas of high humidity and prefer temperatures of 40-60° (e.g.: contaminated ventilation systems and in decaying compost, hay, and sugar cane or bagasse). Farming is currently ranked as one of the top three most hazardous occupations, along with construction and mining. B- Pathophysiology: HP is characterized by diffuse inflammation of lung parenchyma and airways in previously sensitized patients. Based on the length and intensity of exposure and

subsequent duration of illness, clinical presentations of HP are categorized as acute, subacute (intermittent), and chronic progressive.Strong evidence suggests the involvement of immune complex–induced tissue injury (type III hypersensitivity). The timing of development of symptoms after exposure supports this conclusion. Cell-mediated, delayed-type hypersensitivity (type IV hypersensitivity) also plays a major role in the pathogenesis of this syndrome.Death usually occurs 5 years after diagnosis. C- Clinically: ♣Acute farmer’s lungAcute farmer’s lung develops after large exposure to moldy hay or contaminated compost. Symptoms often spontaneously resolve within 12 hours to days if antigen exposure is eliminated or avoided.Acute farmer’s lung manifests as new onset of fever, chills, nonproductive cough, chest tightness, dyspnea, headache, and malaise.If the inhalational exposure is large, patients may develop acute respiratory failure. lung♣Subacute farmer’sSubacute farmer’s lung manifests as chronic cough, dyspnea, anorexia, and weight loss. Subacute disease is insidious in onset and may occur over weeks to months.Chronic farmer’s ♣lungChronic farmer’s lung results from prolonged and continuous exposure to the antigen. Patients may have irreversible lung damage. Patients may experience severe dyspnea at rest or with exertion. Physical: • Acute farmer’s lung: Fever, Nonproductive cough, Rales.• Subacute farmer’s lung: Normal examination findings between presentations, or chronic nonproductive cough• Chronic farmer’s lung: Bibasilar rales, Clubbing - More often observed in patients with chronic farmer’s lung with long-standing hypoxemia and parenchymal damage, and impaired exercise tolerance. D- Differential: • Allergic and Environmental Asthma • Mycobacterium Avium-Intracellulare (HIV Pos: fever, sweats, weight loss, fatigue, diarrhea, and shortness of breath, HIV Neg: pulmonary MAC infection include cough, sputum production, weight loss, fever, and hemoptysis). • Pneumonia • Pulmonary Fibrosis, Idiopathic • Rhinitis, Allergic • Sarcoidosis E- Labs and Imaging: - CXR: Normal findings between attacks. Pulmonary apices are usually spared. o Acute: Diffuse air-space consolidation is typical of acute farmer’s lung (with acute antigen exposure).o Subacute: Nodular or reticulonodular patterno Chronic: Linear radiodensities indicative of fibrosis. - High resolution CT Scan of chest: better than CXR. If normal, it eliminates the possibility of active acute or chronic framer’s lung. Abnormal findings are:

Honeycombing (for pulm fibrosis), and Ground Glass Appearance. - CBC: Leukocytosis with neutrophilia (NOT EOSINOPHILIA), High ESR, Positive IgG Precipitin - Pulmonary Function Tests: Restrictive - Bronchoscopy: useful to rule out other diagnoses, and may be useful also for transbronchial biopsy which would show peri-bronchovascular granuloma. F- Treatment: - Medical Care: Systemic corticosteroid administration and avoidance measures constitute the primary treatment for farmer’s lung. - Diet: No dietary restrictions are needed. - Activity: Patients may decrease activity because of cough and dyspnea on exertion. In a patient with acute farmer’s lung, pulmonary function improves once antigen exposure is eliminated. Between episodes of acute disease, activity may be unlimited. Note: Nonsteroidal anti-inflammatory drugs (NSAIDs) (eg, cromolyn, nedocromil) or systemic immune modulators are not indicated for treatment at this time. G- Complications: - Cor pulmonale- Hypoxemic respiratory failure- Pulmonary fibrosis- Death H- Medical/Legal Pitfalls: - Failure to diagnose farmer’s lung in children living on farms- Failure to review a patient’s work and occupational history with emphasis on progression or improvement of symptoms when the patient is away from the farm- Failure to recognize farmer’s lung when pulmonary function test and radiographic findings are normal between exacerbations- Failure to recognize the limits of transbronchial biopsy and serum precipitins: Interpretation of these tests is dependent on the size of the sample and proper testing with the correct antigen, respectively.- Making the common mistake of misdiagnosing acute farmer’s lung as viral/bacterial pneumonia or acute infection. I- Special Concerns: - History of recurrent pneumonia should prompt consideration of hypersensitivity pneumonitis.- A complete environmental and occupational history is the key to prompt diagnosis of farmer’s lung.- Periodic episodes of acute respiratory symptoms without obvious triggers should also clue the clinician to evaluate for farmer’s lung.- Consider recommending the wear of filtration masks. This practice is not the criterion standard and should be considered only as the last resort with the understanding that it may not provide complete protection from the antigen.- Encourage smoking cessation measures.- Krebs von den Lungen-6 (KL-6) has been recognized as a marker for the activity of diffuse interstitial lung diseases, and levels may be elevated in patients with farmer’s lung. Glossary:

Hypersensitivity pneumonitis includes: Bagassosis, Pigeon’s breeder’s lung, Cheeseworker’s lung, Grain Handler’s lung, Malt worker’s lung, Wheat weevil. Reference: Emedicine: - http://www.emedicine.com/MED/topic1103.htm updated january 2003- http://www.emedicine.com/med/topic771.htm updated March 2003

Review of Factor XIII deficiencyFactor XIII Deficiency A- General Considerations: It is a rare autosomal recessive disease usually associated with a severe bleeding diathesis. Although acquired FXIII deficiency has been described in association with hepatic failure, inflammatory bowel disease, and myeloid leukemia, the only significant association with bleeding in children is the inherited deficiency.The mortality and morbidity are primarily related to bleeding; intracranial hemorrhage can be life threatening. Women with FXIII deficiency have spontaneous abortion rates of almost 100%.Because the clinical bleeding is severe in most patients, the diagnosis is made at an early age, usually during infancy. B- Pathogenesis: Factor XIII is a plasma transglutaminase that catalyzes the final step in the coagulation cascade, cross-linking the loose fibrin polymer into a highly organized structure. In addition, FXIII covalently binds fibronectin, a2-plasmin inhibitor, and other molecules to the fibrin plug; this enhances adherence to the wound site, resistance to fibrinolysis, and wound healing. The FXIII zymogen circulates as a tetramer composed of 2 catalytic a subunits and 2 carrier b subunits (a2b2). The a subunits are synthesized in hepatocytes in the liver and megakaryocytes and monocyte precursors in the bone marrow; a2 dimers are present in circulating platelets and monocytes. The b subunits are synthesized in hepatocytes. Inherited FXIII deficiency is usually due to mutations in the gene encoding the catalytic a subunit, located on chromosome 6. C- Clinical Presentations: „X Bleeding from the stump of the umbilical cord within the first days to weeks of life is a characteristic sign that occurs in 80% of affected individuals; bleeding from this specific site is uncommon in other inherited hemostatic diseases except afibrinogenemia.„X CNS hemorrhage is frequent (25-30%) and may occur spontaneously or after minor trauma.„X Soft tissue bleeding and bruising are very common, as is bleeding into the mouth and gums during teething.„X Hemarthroses occur in 20% of cases„X Bleeding that is delayed (ie, 12-36 h) after trauma or surgery is pathognomonic of FXIII deficiency.„X Recurrent spontaneous abortions are very common in women with FXIII deficiencies who do not receive FXIII replacement.„X Wound healing is abnormal.

D- Differential and associated diseases„X Acquired FXIII deficiency can be caused by liver disease, inflammatory bowel disease, and disseminated intravascular coagulation. The development of autoantibodies to FXIII has been reported (Differentiate by mixing test. Plasma with inhibitor is still prolonged whereas that with true deficiency is corrected). „X Consider other congenital coagulation factor deficiencies, most notably dysfibrinogenemia and decreased fibrinogen levels. E- Work-up:- PTT and PT are normal in FXIII deficiency- Measurement of clot stability is the most common screening test. The patient's plasma is incubated with thrombin and calcium for a sufficient period to allow formation of a stable clot; the formed clot is suspended in 5 mol/L of urea or 1% monochloroacetic acid. In the presence of FXIII, the clot is stable for more than 24 hours; in its absence, the clot dissolves in minutes to hours- Functional and immunological assays available to confirm Ds F- Medical Care: „Ï Treatment of bleeding: Plasma, and cryoprecipitate have been used for replacement of FXIII but the treatment of Choice is plasma-derived Factor XIII concentrate. „Ï Prophylaxis: Prophylactic therapy with FXIII concentrate 10-20 U/kg every 4-6 weeks provides adequate plasma levels in most patients. „Ï Prophylaxis in pregnant patients: Half-life is shorter. Consequently, treating pregnant patients requires more frequent prophylaxis (every 3 wk). In addition, a booster dose is recommended during labor to decrease the risk of bleeding in the mother.„Ï Prior to surgery: patients should receive FXIII concentrate immediately before surgery to ensure optimal hemostasis and wound healing.„Ï Patients requiring prophylaxis should be vaccinated for Hep A and B„Ï Medical/legal pitfall: failure to investigate family members for FXIII deficiency, and failure to implement prophylaxis as the treatment of choice. Reference: Emedicine: http://www.emedicine.com/ped/topic3040.htm Updated November 2002Emergency ContraceptionEmergency Contraception Background: Unprotected sex will result in pregnancy about 8% of the time it occurs. According to different surveys, EC is well-studied and safe, and has the potential to reduce unplanned pregnancy by at least 75% (and, derivatively, induced abortions). Emergency contraception is indicated in any situation in which a woman has had unprotected intercourse but does not wish to become pregnant. This can include failed contraception, intercourse with no contraception for any reason or forced intercourse. The Methods: There are two currently well accepted methods of emergency contraception. The hormonal method consists of various formulations of estrogen and progestins or progestins alone; the other is the emergency insertion of the copper-containing IUD (intrauterine device). The latter can be used up to five days after unprotected coitus and is highly effective. It has the advantage of working as an ongoing contraceptive for up to ten years. Its use is limited by the requirement for special training for insertion, the initial expense and the fact that some women are not candidates for IUDs.

1- The Progestin-only Method Levonorgestrel (LNG) = Plan B® has been shown to be equivalent to the Yuzpe method. Treating more than 4000 women in 10 countries, the World Health Organization (WHO) changed the dose of levonorgestrel (Plan B) from 2 doses of 0.75 mg taken 12 hours apart to a single dose of 1.5 mg. They also extended the treatment period from 3 days (72 hours) to 5 days (120 hours). 2- The Yuzpe Method = Preven® 100 mcg of ethinyl estradiol and 1 mg of norgestrel (2 pills immediately, followed in 12 hours by 2 more pills), will reduce the rate of pregnancy from 8% to 3.2%. This medication is administered in two doses started within 72 hours after unprotected intercourse. The Preven® formulation is equivalent to the Yuzpe regimen but it contains levonorgestrel, which has twice the potency of norgestrel, so only half the milligrams, or 0.5 mg per dose, is needed. Note that this progestin dose is 1.5 times the amount of levonorgestrel than is in the Yuzpe regimen using levonorgestrel. Plan B® is the progestin-only formulation. The side effects: There are significant side effects to hormonal EC, most prominently nausea and vomiting, particularly with the Yuzpe method. The main concern is whether the vomiting will interfere with the absorption of the medication. The timing: It is very important to start EC as early as possible in the 72-hour window after intercourse. But it is also very important that the second dose be 12 hours later. The screening prior to prescribing: three questions to which answer should be YES. - Have you had sex within the last 72h?- When was the first day of your LMP (should be less than 4 weeks ago)?- Were your periods normal in time and length?Comparison of both methods:A large multicenter trial published in 1998 definitively showed that the progestin-only regimen was more effective than the Yuzpe. When used correctly the progestin prevented 89% and Yuzpe prevented 76% expected pregnancies Contraindications: Same as with any OCP, but rates so low that WHO only cites Current Pregnancy and hypersensitivity to any of the components as contraindications. H/O thromboembolism and migraine are two relative contraindications. Undiagnosed vaginal bleeding is a CI of the Progestin-only method. Resumption of menses: 72% of patients resumed menses early or within three days of their expected date. If menses have not resumed 21 days after the administration of emergency contraception, a pregnancy test should be done. If the woman is found to be pregnant, there should be no adverse effect on the pregnancy from taking EC. Birth control pills have frequently been taken in early pregnancy without adverse effects on the fetus, and the FDA required manufacturers to remove warnings about increased risk to the fetus several years ago. Additional advice: WHO now makes the following recommendations:o One immediate dose of 1.5 mg levonorgestrel (Plan B)o Five-day cut-off time for treatment, rather than 3 dayso Ongoing contraception started at time of emergency contraception Although the Preven kit comes with a pregnancy test, it need not be used.o A woman must be pregnant for several days before the test will turn positive. If a woman uses the test and it turns out negative, she may feel that she took the pills when she didn't need to.o Even if a woman were to be pregnant and take the pills, it would not cause spontaneous

abortion, and no harm would come to the fetus. Women should be cautioned to avoid unprotected coitus until their menses occur, because if EC delayed ovulation, ovulation may yet occur for that cycle. They should abstain from intercourse or use reliable back-up contraception until their next menstrual period. Oral contraception can be started immediately after EC. A follow up visit after successful EC is always advisable. It presents an opportunity to review contraceptive needs and insure maintenance or initiation of effective contraception for women who wish it. It is also a chance to educate and institute protection against STD’s.

Although EC is not a good method of contraception, it can be repeated without risk. Providing a prescription to be filled and kept at home is a wise precaution and will result in more immediate use and a higher success rate. Remember, both products have a long shelf life: Preven®, four years, and Plan B®, three years. Other considerations: Mifeprex®, RU486, the "Abortion Pill" and EC Mifepristone is a very effective emergency contraceptive. In this capacity, it is not causing an abortion. It is not interrupting a pregnancy; it is preventing a pregnancy. Its efficacy as an emergency contraceptive is primarily due to its effect of delaying ovulation and possibly delaying maturation of the endometrium. Mifepristone is available in the U.S. only for medical abortions. Its availability is restricted to health facilities doing medical abortions under strict protocols, so off label use for emergency contraception would be difficult. Mifepristone also blocks corticosteroid receptors, so adrenal insufficiency and chronic treatment with corticosteroids are listed as contraindications in the insert. Porphyria, hemorrhagic disorders and anticoagulant medications are also listed. References: - Cyberounds: http://www.cyberounds.com/conferences/womens_health/ Updated March 2003. - Emedicine: http://www.emedicine.com/aaem/topic498.htm Updated May 2003Approach to a rape victimApproach to a rape victim 1- Verify occurence of sexual assault. In doing so, avoid interpreting the victim's calmness or composure as evidence that a sexual assault did or did not occur. (Note: False accusations of sexual assault are estimated to occur at the low rate of 2%—similar to the rate of false accusations for other violent crimes.) 2- History Taking: Unrushed and sensitive manner. Use direct questioning to disclose forced oral or anal penetration by victim. Ask about sexual history before rape. PMH-contraceptive history-Gynec History. Approach victims calmly. Showing your outrage at the crime may cause victims even more trauma. Ask victims whether they would prefer a male or female physician. 3) Physical Examination: Injuries requiring immediate attention should take precedence. Anal examination including proctoscopy should be performed if there is history of forced anal penetration with documentation of found injuries. 4) A full STD screen at presentation as research suggests a significant incidence of acquisition of STD's prior to rape event. Initial investigations:

- N. Gonorrhea and Chlamydia Trachomatis cultures from specimens collected from any sites of penetration or attempted penetration. In addition, FDA-approved nucleic acid amplification tests (as a substitute for culture). - Wet mount and culture of a vaginal swab specimen for Yeasts and Trichomonas Vaginalis. - Collection of a serum sample for immediate evaluation for HIV, hepatitis B, and syphilis. 5) Treatment: Postexposure hepatitis B vaccination, without HBIG, should adequately protect against HBV. Hepatitis B vaccine should be administered to sexual assault victims at the time of the initial examination if they have not been previously vaccinated. Follow-up doses of vaccine should be administered 1--2 and 4--6 months after the first dose. An empiric antimicrobial regimen for chlamydia, gonorrhea, trichomonas, and BV may be administered. Recommended Regimen

-------------------------------------------------------------------------------- Ceftriaxone 125 mg IM in a single dosePLUSMetronidazole 2 g orally in a single dosePLUSAzithromycin 1 g orally in a single doseORDoxycycline 100 mg orally twice a day for 7 days.

--------------------------------------------------------------------------------If patient pregnant or breast feeding: Amoxicillin 3g STAT + Probenecid ag STAT + Erythromycin 500mg BID x14days. - Hep B Vaccine up to three months after assault 3 doses. 0-1-6 months. - Offer Post exposure Prophyaxis for HIV. If considered PEP should be given no later than 72h after event. - Offer Pregnancy prevention using Levonorgestrel (Ovral) 0.75mg x1 dose only (recent guideline) or x2doses 12h apart no later than 73h after event. IUD copper-contraception can also be considered with ATB coverage. 6) Offer ongoing counseling. 7) Treat sexual partners if pt found to have STD. Special considerations:Sexual assault of a child: Specimen collection for culture for N. gonorrhoeae from the pharynx and anus in both boys and girls, the vagina in girls, and the urethra in boys. Cervical specimens are not recommended for pre-pubertal girls. For boys with a urethral discharge, a meatal specimen discharge is an adequate substitute for an intraurethral swab specimen. Only standard culture systems for the isolation of N. gonorrhoeae should be used. All presumptive isolates of N. gonorrhoeae should be confirmed by at least two tests that involve different principles (i.e., biochemical, enzyme substrate, serologic, or DNA probe methods). Isolates and specimens should be retained or preserved in case additional or

repeated testing is needed. Gram stains are inadequate to evaluate pre-pubertal children for gonorrhea and should not be used to diagnose or exclude gonorrhea. Cultures for C. trachomatis from specimens collected from the anus in both boys and girls and from the vagina in girls. Culture and wet mount of a vaginal swab specimen for T. vaginalis infection and BV. Collection of a serum sample to be evaluated immediately, preserved for subsequent analysis, and used as a baseline for comparison with follow-up serologic tests. Sera should be tested immediately for antibodies to sexually transmitted agents. Agents for which suitable tests are available include T. pallidum, HIV, and HbsAg. To treat or not treat? Presumptive Treatment The risk of a child acquiring an STD as a result of sexual abuse or assault has not been determined. Presumptive treatment for children who have been sexually assaulted or abused is not recommended because a) the prevalence of most STDs is low following abuse/assault, b) pre-pubertal girls appear to be at lower risk for ascending infection than adolescent or adult women, and c) regular follow-up of children usually can be ensured. However, some children or their parent(s) or guardian(s) may be concerned about the possibility of infection with an STD, even if the risk is perceived to be low by the health-care provider. Such concerns may be an appropriate indication for presumptive treatment in some settings and may be considered after all specimens for diagnostic tests relevant to the investigation have been collected. Reference: - CDC 2002 Guidelines: http://www.cdc.gov/std/treatment/8-2002TG.htm#AssaultSTDs- National guidelines for response to sexual assaluthttp://www.mssvd.org.uk/PDF/CEG2001/sexassault%2006%2001.PDFQuestionQuestion about Pt with picture of skin lesion showing nodule. Biopsy reveals Cheese-like material. What is it?Epidermal Inclusion Cys Background:Also called Sebaceous cyst although not of sebaceous origin. Epidermoid Cyst is more favored. Epidermoid cysts result from the proliferation of epidermal cells within a circumscribed space of the dermis. Inflammation is in part mediated by the horny material contained in epidermoid cysts. Extracts of this material have been shown to be chemotactic for polymorphonucleocytes. Epidermoid cysts are slow growing and usually asymptomatic, but they may become inflamed or secondarily infected, resulting in pain and tenderness. Rarely, malignancies, including basal cell carcinoma, Bowen disease, SCC, and even mycosis fungoides, have developed in epidermoid cysts. History: Discharge of a foul-smelling cheeselike material is a common complaint.Less frequently, the cysts can become inflamed or infected, resulting in pain and tenderness.In the uncommon event of malignancy, rapid growth, friability, and bleeding have been reported. Physical: Epidermoid cysts appear as firm, round, mobile, flesh-colored to yellow or

white subcutaneous nodules of variable size. A central pore or punctum is an inconsistent finding that may tether the cyst to the overlying epidermis and from which a thick cheesy material can sometimes be expressed.Epidermoid cysts of the genitals are common in the general population and may appear as a mass in the breast, the vulva, the clitoris, the penis, the scrotum, or the perineum. The ocular and oral mucosae can also be affected, and cysts have been reported on the palpebral conjunctivae, on the lips, on the buccal mucosa, under the tongue, and even on the uvula. FNA: Fine needle aspiration has been used to help diagnose epidermoid cysts. Smears of aspirated material can be stained with Wright-Giemsa, and they demonstrate nucleated squames and wavy keratin material. Occasionally, Squamous Cell carcinoma (MC cancer in this entity). Treatment:- Abstinence- May be injected with triamcinolone if uninfected. - I&D if infected followed by antistaph ATB PO- Excision in toto = definitive treatment preventing recurrence by excising Keratin producing the lining of the cyst. - The treatment of epidermoid cysts on the terminal phalanx is more complicated and may consist of curettage or chemical cautery followed by packing with bone chips. Medical/Legal Pitfalls:The major pitfall in managing epidermoid cysts is failure to diagnose an associated malignancy. Therefore, removal is recommended for any cyst behaving in an unusual way (eg, rapid growth). In turn, all excised cysts should be sent for pathologic analysis. Reference: emedicine: http://www.emedicine.com/derm/topic860.htmUpdated Feb 2003 Microbiology (if infected) and US Question34 yo female patient presenting with heavy menstrual blood loss and elarged uterus. Physical examination reveals a large uterus with irregular consistency. You suspect uterine fibroids. Which work up is essential in determining the severity and therefore the treatment indication of this patient?

From the hx,the most likely diagnosis is either Subserosal fibroid or Intramural fibroid(as it is given as enlarged uterus).the most common presentation of submucosal fibroid is abnor.vaginal bleeding.(enlarged uterus is very unlikely)Reg investigation,Hysteroscopy will detect only the submucosal fibroids whereas USG can detect all types of fibroids.Abd sono- detect intramural/subserosal (as in this case)& vaginal sono - detect submucosal.So, USG is the best initial non-invasive investigation to do in this case.Biostats questionWith IQ test mean of 100 and SD 15.What will be percentile of IQ if person IQ is 115 ?50, 68, 84, 95 ,99 th ??This question has been discussed in length before. I was wrong in my reasoning just like Salil. Here's the exaplanation.

What we know from the bell-shaped curve is between +1SD and -1SD: the area under the curve consists in 68% of the population. But that's the trick. It is not what is asked in the question. The question asks about the percentile. Meaning, it is a directional question. How much percent is between 0 and +1sd. We already know that between 0 and the dome of the curve is 50 % because it's the mean. So the hump of the curve corresponds to the 50th percentile. Now from the hump of the curve to +1sd corresponds to half the area under the curve we talked about in the beggining. This area situated in the interval -1SD and +1sd which is 68%. Half of that area is 34% which is between the mean and +1sd. Now let's join the two. We had calculated between 0 and the mean as being the 50th percentile. If you add to it the new found number of between the mean (hump of the curve) and +1sd which is 34%, it gives you 84%. The interpretation is a number AT +1sd is at the 84th percentile. Read the explanation after drawing the bell shaped curve you'll understand it better. Similarly, the area between -2SD and +2SD is 95%. Half of it which is the area between the mean and +2SD is 47.5%. Therefore the percentile located AT +2sd is 50+47.5 = 97.5%. It means of our population 97.5% are located below the +2SD point on the curve. Mechanism by which Indomethacin helps in the closure of PDAPatency of PDA is an active state maintained by the action of prostaglandins. Indomethacin either postnatally or sometimes prenatally as PDA prophylaxis inhibits Cyclo-oxygenase thereby inhibiting the production of Prostaglandins, and leading to closure of PDA in as many as 86% of the patients. Sometimes, it requires surgical intervention either because of failure to close or reopening after many treatments with Indomethacin. Usually one dose is enough. It has also been shown to increase pulmonary blood flow, therefy reducing the tendency to Pulmonary hypertension, consequence of long-standing PDA.Migraine and Pregnancy26yo G1P0 with irregular menses and an LMP approximately 10 weeks ago is reffered to your clinic secondary to a positive pregnancy test. Her PMH is negative for anything other than migraine headaches that she suffered from regularly since high school. What treatment would be best for her migraine during pregnancy?In general, migrain HA normally improves in pregnancy. But if persist, tx w/acetaminophen and antiemetics. (If severe use codene ormeperidine). Avoid ergotamine.Pseudotumor Cerebri and PregnancyI recall having read a question in which a pregnant woman came in with signs indicative of pseudotumor cerebri. I wondered about the treatment in pregnancy. Here's what I found. Medical literature search: few articles describe two scenarios, one in which a patient has pseudotumor and gets pregnant or the patients is pregnant and develops signs of increased intracranial pressures. The patients who already had the established diagnosis were treated medically, however, if the ICP became resistent to therapy (one study followed progressive visual loss) a shunt was placed. 3/4 of patients responded to shunting the other 1/4 required optic nerve sheath decompression. In all instances healthy babies were delivered. Another study concluded that psudotumor 1. does not increase the risk of relapses 2. does not mean a worsening prognosis and 3. does NOT affect the

developing child. The second scenario, in which elevated ICP occur during pregnacy is rare. The article listed 2 case studies , in both the baby was delivered vaginally without complications.

Role of Thiazide in DI- Thiazide diuretics combined with mild salt restriction is the most effective therapy for nephrogenic DI (can reduce polyuria as much as 50-70%). Thiazides (e.g. HCTZ 50-100 mg/d) induce mild salt depletion (block Na reabsorption in cortical diluting site) which results in volume contraction. The latter results subsequently in decreased GFR, increased isotonic proximal fluid absorption and thus decreased delivery of fluid to the collecting duct. Salt restriction augments this effect. Potassium should be given as needed to prevent hypokalemia. Main limitation is symptomatic volume depletion. Volume depletion may decrease lithium excretion, increasing risk of lithium toxicity. - Amiloride: Agent of choice in lithium-induced nephrogenic DI since it may block lithium uptake in distal tubules and collecting ducts. Allows lithium use to be continued. May also have thiazide-like action since it causes a negative Na balance. Must monitor for hyperkalemia and cannot use in renal insufficiency. May be combined with thiazide for additive antidiuretic effect and to balance potassium. clinical Scenario:JB a 57yo woman has been having recent epidoses of nocturnal enuresis. Upon further evaluation, she reveals that she has been thirsty of late, and drinking lots of water. She therefore started experiencing frequent urination. BP: normal. PE: no peripheral edema. Lytes: Hypernatremia. JB's got h/o of DM2 controlled by diet and exercise, as well as chlorpropamide 100mg/d. Next step: Work up is done preferably with pt maximally dehydrated as tolerated (i.e.: at time when ADH should be highest and urine most concentrated). 1) 24h urine collection: volume, creatinine, Urine Specific Gravity, Urine Na, Uosm)2) Serum: Electrolytes and Glucose. ADH levels. In DI, hallmarok is: Urine Sp. Grav=1.005 - Uosm < 200mOsm/kg 3)Differentiate bwteen Central and Nephrogenic: Water Deprivation Test: Compares Uosm after dehydration Vs Uosm after Vasopressin. In Central, Uosm improves after administration of ADH. 1st step in management: 1)In ER: Hydrate IV dextrose and water or IV hyperosmolar fluid. Avoid hyperglycemia, volume overload, and rapid correction of hypernatremia. 2) Central DI: DOC = Intranasal DDAVPAssess for drugs that potentiate effect of DDAVP: Chlorpropamide (patient IS DIABETIC AND ON CHLORPROPAMIDE), Carbamezepine, thiazides, clofibrate, and indomethacin. 2) Nephrogenic DI: Combination Indomethacin + HCTZ or Indomethacin-Desmopressin or Indomethacin-Amiloride, the latter being DOC for Lithium-indeuced Nephrogenic DI. 3) DI and pregnancy: Pregnancy increases risk for DI = Vasopressinase-induced DI. See in 3rd Trimester and puerperium. Often associated with oligohydramnios, pre-eclampsia, or hepatic dysfunction. The mechanism = circulating desmopressinase which destroys

endogenous ADH but not synthetic. Rx = DDAVP. 4) DI in children: mostly ENURESIS - LINEAR GROWTH RETARDATION Note: Risk factors for DI:- Hypokalemia - Hypercalcemia- Lithium, Demeclocycline(ATB tetracycline used for SIADH), Colchicine, Foscarnet, Methicillin). - Diabetes Mellitus- Pituitary surgery- Pregnancy- Sickle Cell Disease- Multiple Myeloma Reference: - Endocrinology UCLA http://www.endocrinology.med.ucla.edu/diabetes_insipidus.htm- Emedicine- CMDT- Kaplan notes 2002QuestionA 24 yo man known to be HIV+,currentlt receiving no treatment,develops the acute onset of petechiae and oropharyngeal bleeding.Plt count:2000,No RBC fragmentation is noted on exam of peripheral blood smear.Rx?a:DDAVPb:plt transfusionc:aminocaproic acidd:IV gammaglobuline:FFPIV immunoglobulin This is HIV-induced ITP. The quickest response would be with IV immunoglobulin which is indicated in severe thrombocytopenia. Long-term treatment options would include spelectomy (increased risk for encapsulated organisms infections), AZT (80-90% susteained positive response), and steroids (40-80%). A 54 yo woman presents with abrupt decline in urine output& RF.Her PMH is positive only for hysterectomy 1mo earlier& chronic migraine headaches controlled with methysergide.Lab:Na:130,K:6.2,Cl:99,HCO3-:16,glucose:101,Ca:7.9,P:6.3,Albumin:4,Cr:3.2U/A:PH:1.010,protein&Hgb:trace,sediment:unremarkable Urine output over the past 12hr:60ccNext step of Mx?Stop offending drug and do CT. Side effect of this drug: Retroperitoneal fibrosis. Urinary tract obstruction since it is abrupt onset? It is also a vasoconstrictor, but the abrupt onset doesn't make it sound like ATN secondary to vasoconstriction. Also U.Sp.G=1.010. this pt has a presentation typical of retroperitoneal fibrosis:clear association with methysergideCT generally confirms the Dx by showing medial deviation&extrinsic compression of the ureters,so diagnostic test of choiceRenal transplant rejection Vs Cyclosp. nephrotoxicity

Renal transplant patient on immunosuppressants among which cyclosporine. 4 weeks later: rapid rise of creatinine, fever, edema, and oligouria. Is it graft rejection or cyclosporine nephrotoxicity? To differentiate the two: - cyclosporine toxicity: >6 weeks use, afebrile, Cyclosporine trough level>200ng/ml, gradual rise in creatinine, US: unchanged graft corss-sectional area, manometry: capsular pressure is<40mmHg, Biopsy: arteriolopathy (intimal thickening, hyalinosis, endothelial vaculoization). Rsponds to decreasing cyclosporine. - Transplant rejection: <4weeks, oligouria after initial function, rapid rise in creatinine, fever and weight gain, Cyclosporine trough level <150ng/ml, Biopsy: endovaculitis (intimal arteritis, necrosis and sclerosis), US: increased graft cross-sectional area, MRI: loss of distinct cortico-medullary junction + swelling + density similar to that of Psoas and fat tissue, Manometry intracapsular pressure >40mmHg. Responds to increased steroids or antilymphocyte globulin. Reference: - http://www.stadtlander.com/content/transplant/Generic_Cyclo_PI.pdfQuestionA 68 yo white man is reffered to u for evaluation of renal failure.Lab:Na:135,K:4.2,Cl:109,HCO3-:24,glucose:101,Ca:10.9,P:4.3,Albumin:4,Hct:29%Cr clearance:55ml/min,urine protein:6.2g/dayNext step of Mx?this pt has multiple myeloma and his renal failure most likely is related to overproduction of Ig light chains.the pt has anemia,he has significant proteinuria WITH NORMAL serum albumin so nephrotic syndrome excluded!and the presence of LMW protein that's readily filterated at glomerulus such as Ig light chain,could explain the proteinuriafirst step of Mx:immunofixation electrophoresis of the serum&urineconfirmation by BMA&biopsy&bone surveya 65 year old postmenopausal lady presents with bone pain,her bone density test [dexa scan[ shows t score of 0.9. she is a smoker, alcoholic, with sedentary lifestyle, obese.past history of dvt. what is best for her1.hrt2. bisphosphanates3. raloxifene4. calcitoninAnswer is 2. Bisphosphonates. According to CMDT the agents to use first are bisphosphonates. This patient has no history or risk factor mentioned for breast cancer, therefore protecting her against it is not necessarily warranted. Her t score is 0.9. Normal t scores are above -1. So she's still not extremely depleted. The risk she needs to be protected against is pathological fractures. The evidence for reducing fracture risk is currently considered insufficient for most agents, including calcitonin, estrogen(HRT), and etidronate. Raloxifene significantly reduces the risk of vertebral fractures by approximately 30% but does not reduce the risk of hip or other nonvertebral fractures. The evidence of antifracture efficacy is most convincing for the 2 bisphosphonates alendronate and risedronate. According to CMDT Alendronate is the first agent to consider. Bisphosphonates have the mechanism of action of inhibiting

osteoclast activity this stopping calcium mobilization from the bones. If the patient were to have mentioned any risk factor for breast cancer, then Raloxifene would be first choice. Varicella Vaccine Facts6 year old girl's mother is afraid the baby may get chicken pox becoz her friend got it.baby was not vaccinated .what will u tell the mother?Answer: If age is 6yo = give vaccine. If age is 6months old = DO NOT VACCINATE. - Vaccinate ANY child who is not immunocompromised and is more than one year old, with no DEFINITE prior history of chickenpox or varicella vaccine. This baby's age is probably 6 months as opposed to six years old (probably a typo). DO NOT VACCINATE BABIES LESS THAN 1 YEAR. There is a possibility of rash 4-6 weeks after vaccine, and it has been shown that this rash is more severe in immunocompromised patients. Babies less than one year are perhaps considered immunocompromised?? At any rate it is not recommended IN THE USA. - HIV brother, leukemic grandmother, pregnant women, same patient with mild sore throat or diarreha, in the household ARE NOT CONTRAINDICATIONS TO THE VACCINE WHEN IT'S DUE. - It MAY BE GIVEN AT THE SAME TIME AS MMR. - PPD TEST MAY BE DONE AT THE SAME TIME AS VARICELLA VACCINE. - The varicella vaccine should not be given for at least 5 months after receipt of an immune globulin preparation or a blood product (except washed red blood cells). Immune globulin products should not be given for at least 2 to 3 weeks following the varicella vaccine. If, however, they are given at the same time, the vaccine should be repeated in 5 months. - The vaccine is not 100% protective. 15% of patients with get a mild rash. - Son exposed to chickenpox at school. Should he get vaccine if not immunized before? Yes within 3-5 days of exposure. - Chickenpox developped after vaccine:1% of vaccinated kida will have what is called "breakthrough disease." This is a mild case of chickenpox caused by the wild-type virus at least 6 weeks after vaccination. There are usually less than 50 lesions and no severe symptoms. This illness should be treated as regular chickenpox. - Son had rash at 4 months of age. MD said that it's "probably" chickenpox. Now, son is due for vaccination. Should he be vaccinated? Yes. Since it's not a DEFINITE history of chickenpox. - Son is 8yo on inhaled steroids for asthma. Never vaccinated for chickenpox. Shoul dhe get vaccine?YES. Inhaled steroids are not a ptoblem. Vaccine is contraindicated if ORAL STEROIDS. - Pregnant woman 12 weeks. Was never vaccinated for chickenpox. Asks you if she should be concerned and get the vaccine now?NO. Most adults are immune. and Vaccine is contraindicated in pregnancy. - Pregnant woman had a sever case of shingles. She asks you if her 1 yo son STILL has to get the vaccine or he's probably immune by now because of the exposure. NO. He still has to get the vaccine. Reference:

http://www.vaccinecheck.com/vacinfo_3-7.jsp?vac=3-7What is Saturday night syndrome?Two aspects are to consider for this syndrome: 1- Saturday night SYNDROME: USMLE Takers winding up forming a sorry newsgroup of DOCTORS without a date on a Saturday night. Prognosis: short-term = very high likelyhood of sarcasm and irritability. Long-term = roughly 67% will end up with a date in the next three years. The rest was...already married. 2- Saturday night PALSY: Radial Neuropathy or Saturday Night Palsy or Wrist Drop: Basic Anatomy Stuff: The radial nerve contains axons contributed by the fifth and sixth cervical roots (C5-6). It forms the POSTERIOR CORD of the brachial plexus.

PresentationThe patient has injured his upper arm, usually by sleeping with his arm over the back of a chair. Another presentation is a patient using crutches. Sometimes happens after humerus fracture since radial nerve trails down around humeral groove. Patient now presents holding the affected hand and wrist with his good hand, complaining of decreased or absent sensation on the radial and dorsal side of his hand and wrist, and of inability to extend his wrist, thumb and finger joints. With the hand supinated (palm up) and the extensors aided by gravity, hand function may appear normal, but when the hand is pronated (palm down) the wrist and hand will drop. What to do:Look for associated injuries. This sort of nerve injury may be associated with cervical spine fracture, injury to the brachial plexus in the axilla, or fracture of the humerus. If there is complete paralysis or complete anesthesia, arrange for additional neurological evaluation and treatment right away. Incomplete lesions may be satisfactorily referred for followup evaluation and physical therapy. Construct a splint, extending from proximal forearm to just beyond the metacarpophalyngeal joint (leaving the thumb free) which holds the wrist in 90 degree extension. This and a sling will help protect the hand, also preventing edema and distortion of tendons, ligaments, and joint capsules which can result in loss of hand function after strength returns. Explain to the patient the nature of his nerve injury, the slow rate of regeneration, the importance of splinting and physical therapy for preservation of eventual function, and arrange for followup. What not to do:Do not be misled by the patient's ability to extend the inter phalangeal joints of the fingers, which may be accomplished by the ulnar-innervated interosseus muscles. DiscussionThis neuropathy is produced by compression of the radial nerve as it spirals around the humerus. Most commonly it occurs when a person falls asleep, intoxicated, held up by his arm thrown over the back of a chair. Less severe forms may befall the swain who keeps his arm on his date's chair back for an entire double feature, ignoring the growing

pain and paresis. If the injury to the radial nerve is at the elbow or just below, there may be sparing of the wrist radial extensors as well as the radial nerve autonomous sensation. The deficient groups will be the wrist ulnar extensors as well as the metacarpophalyngeal extensors. A high radial palsy in the axilla (e.g., from leaning on crutches) will involve all of the radial nerve innervations, including the triceps. Questions:which of the following when given in excess would most likely cause pseudomotor cerebri?a.vit A b.vit Bc.vit Cd.vit De.zinc In an adolescent presenting with ptyriasis rosea,which f the following tests would be an appropriate blood test to order?a. VDRLB. complete blood countc.hepatitis A IgMd.FANAe.glucose1- Vit A (Typo: pseudoTUMOR cerebri)2- VDRL: Quick review on Pityriasis Rosea Presentation: Child with Herald patch 1-2 cm, salmon colored with darker periphery (primary plaque). 1-2 weeks later generalized rash which lasts approximately 6 weeks. The rash has a Christmas Tree appearance on the back. PALMS AND SOLES NOT INVOLVED (as opposed to roseola of secondary syphilis). Cause: seasolnal predilection, occurs in clusters among contacts, no organism involved. Labs: RPR or VDRL to R/O secondary syphilis. (Sensitivity for primary syphilis: RPR higher 86% (vs 78% for VDRL)=>use RPR first. Secondary syphilis, both have 100% sensitivity and 98% specificity, so you can use either one of them). Both are nontreponemal. So if positive, confirm with treponemal FTA-ABS for ex. Treatment: Self limited. No need to exclude child from school. Supportive measures: Water, sweat, and soap may cause irritation and should be avoided early in the disease. Topical zinc oxide and calamine lotion are useful for pruritus. Reference: - emedicine - Lab Diagnosis of syphilishttp://www.hc-sc.gc.ca/pphb-dgspsp/std-mts/csg-ldcm/lab_e.html

Trauma in Pregnancy..High Yield..Q#1Many people came back from the exam with the common feedback that they have seen people that day flying through the windshield of their cars, and challenging cases of pregnant wome, so I thought I'd share what I had come across a while ago. This has been updated as of february 2003. Here goes.

You are the only emergency physician in a well-equipped emergency department. It is an hour before handover in the morning. You are a little tired after a busy night. EMS calls to say they are bringing in three patients from a rollover car wreck. Two of the patients -- an elderly couple -- appear critically injured. They both require intubation, but EMS is unable to perform this because of 'laryngeal spasm.' The third patient -- a restrained rear seat passenger -- is a 22-year-old female, complaining of some low back pain and left upper quadrant abdominal pain. When the patients arrive, within minutes of each other, you quickly examine the 22-year-old. She has no airway compromise, but has a respiration rate of 22 secondary to the pain on the left side of the chest inferiorly. She is awake and aware. Her pulse rate is 92 and her blood pressure is 110/75 mm Hg. She is complaining bitterly of low back pain. She says that she was struck on the left side when the car hit the first pole and is tender in the left upper quadrant of the abdomen. You order intravenous fluids, bloods, urinalysis, and an x-ray of the cervical spine, chest and pelvis, together with lumbar spine views. You also order oral contrast for the CT of the abdomen and pelvis to be given as soon as the lumbar spine views are completed. You quickly head off to deal with the other two victims. OK this is an easy one given the title. What did you forget to order specifically?Trauma in pregnancy..Q#2A pregnancy test! Moreover, of course, it is positive. A quickly ordered quantitative bHCG shows that she is between 6 and 8 weeks pregnant. Talking with the patient reveals that her last period was about 7 weeks earlier. She has never been pregnant before. So, given that she has had the x-rays ordered (cervical spine, chest, pelvis, lumbar spine) and has completed the CT of the abdomen and pelvis, (all of which were reported as 'normal'), what would you tell her now?Trauma in pregnancy ...Q#3In general, the fetus will be exposed to about 50-100 millirads (mrad) during the course of the 9-month pregnancy. Radiological exposures to the fetus can be divided into a 'low dose' group and a 'high dose' group. Examinations in the 'low dose' group include cervical spine, thoracic spine, chest and all extremities. Each of these examinations, correctly performed, should result in less that 1 mrad exposure to the fetus. 'High dose' examinations include lumbar spine, pelvis and hip. The lumbar spine is the highest exposure, with a range between about 200 and 1200 mrad. The pelvis and hip x-rays produce about 200 to 400 mrad each. In addition, intravenous pyelograms (IVP), urethrocystograms and the 'KUB' (or 'flat plate abdomen') are considered 'high dose'. An IVP causes 500-800 mrad exposure, while a KUB is between 200 and 500 mrad. A urethrocystogram produces about 1500 mrad. Estimating exact exposure of any individual fetus is very difficult, as it can vary according to the equipment used, the technique, maternal size and the age of the fetus. With modern radiographic machines, coned views aimed more than 10 cm away from the fetus result in very low exposure to the fetus. Computerized tomography (CT) can produce more significant exposure levels. CTs of the head result in about 50 mrad of exposure, while chest CT produces about 1000 mrad (significantly less if shielded). CTs of the upper abdomen can produce up to 3000 mrad of exposure, while, understandably, CT of the pelvis produces a much higher exposure

level of between 3000 and 9000 mrad. Thus, our patient has had some low risk examinations -- chest and cervical spine x-rays, and some significant exposures from the pelvis and lumbar spine x-rays, the abdominal CT and, especially, the pelvic CT. Total dose could be between about 5000 and 13,000 mrad, depending upon exact technique, equipment, number of CT cuts and size of patient.

Now, given this information, what is considered a 'dangerous' dose of radiation, and what are the risks?Trauma in pregnancy..Q#4Information about radiation exposure of fetuses has been gathered from observational studies of radiation exposures from the use of nuclear weapons in Japan and from unintentional exposures to fetuses as a part of medical therapy. Additional information has been extrapolated from animal studies. The highest risk for fetal viability is in the earliest stages of pregnancy, especially within the first two weeks, while malformations are greatest if radiation exposure occurs between 2 and 8 weeks gestational age. This is the period of maximum organogenesis in the developing fetus. Abnormalities described include prenatal or neonatal death, multiple types of congenital abnormalities, severe mental retardation and either temporary or permanent growth retardation. In addition, there may be an increased risk of carcinogenesis, sterility and germ cell mutations. In the Japanese cohort, the major congenital abnormality described has been microcephaly. In general, exposures of less that 5000 mrad have negligible effects.5 Between 5000 and 10,000 mrad there appears to be a slight increase in the incidence of childhood cancers. At doses above 15,000 mrad, there is a greater risk. Figures quoted indicate a 6% chance of mental retardation and about a 2-3% chance of developing childhood cancers. Another article1 quotes a risk of 1 adverse event/1000/1000 mrad of exposure (i.e, for every 1000 infants exposed to 1000 mrad, there is one adverse event). Thus, our patient probably did receive a dose that puts her fetus at a slightly increased risk for childhood cancer and at very low risk for fetal malformations. I would counsel the patient about the slightly increased risk of fetal abnormality, suggest she see an obstetrician as soon as possible -- even when in the emergency department if she wished -- and arrange this for her if she lived in the catchment area of the hospital. I would also caution her that the radiation dose is cumulative and she should be sure to mention that she has already had exposure should she require any further studies during the course of this pregnancy. The take home message here should be that although we should take every precaution to reduce fetal exposure to radiation -- through shielding, minimizing numbers of x-rays or using alternative investigative techniques -- we should not deprive the mother of adequate investigation in the setting of trauma because of the theoretical risks from radiation. A recent article addresses how often this scenario actually happens.2 Bochicchio and colleagues studied trauma admissions over a four-year period and they found that 114(2.9%) of 3976 women admitted were pregnant. Of these women, 13(11%) were incidental pregnancies. Mean gestational age was lower in this group, as one might expect, at about 7 weeks. The estimated mean initial radiation exposure of all patients was 4500 mrad. The cumulative radiation exposure exceeded 5000 mrad in about 85% of patients. In this article, fetal loss was significantly higher in the incidental pregnancy

group when compared with the 'known' pregnancy group. After being given information about the risks of the pregnancy and discussing it with an obstetrician in the department, our first patient elected to leave against medical advice to return to her home out of state. She was lost to follow up. Epidemiology of Trauma in Pregnancy Trauma is the leading cause of non-obstetric morbidity and mortality in the pregnant patient in the United States. It is estimated to occur in about 7% of pregnancies. During pregnancy, the commonest cause of trauma in the United States is motor vehicle crashes, accounting for up to 70% of blunt abdominal trauma. The next leading cause is falls, followed by direct assault. The incidence of domestic violence in pregnancy is alarming and emergency physicians should be attuned to this as a possibility if they see a pregnant patient who has apparently been assaulted. Domestic violence is thought to account for as much as 30% of all trauma seen in pregnancy. Very few women will admit to physical abuse and thus the physician must maintain a high index of suspicion for this problem. Direct questioning, with the partner out of the room, will often elicit the truth. Penetrating injury is less common than blunt injury. In America, gunshot wounds are the most frequently encountered cause of penetrating injury. Fetal mortality rates in penetrating trauma are as high as 70%. Complications peculiar to the pregnant patient as a result of trauma include preterm labor, placental abruption, fetal-maternal hemorrhage and fetal demise. A frequently asked question by pregnant patients is 'Should I wear my seat belt, doctor?' What is your response?

Trauma in pregnancy...Q#5Your answer should be "YES." Unfortunately, up to one-half of all pregnant women do not wear their seat belt, or wear it incorrectly. Unbelted pregnant women are 4 times more likely to have a fetal demise and about 2.5 times more likely to give birth within 2 days of the injury, when compared with belted pregnant females. Correct placement of the belt is to wear the lap portion of the belt across the pelvis, below the pregnant abdomen, with the shoulder belt placed over the mid point of the clavicle, between the breasts, and to the side of the pregnant abdomen. Instruction by health care workers in the wearing and the correct positioning of belts will result in a significant increase in the correct wearing of seat belts by patients (83% vs. 65% for those not instructedTrauma in pregnancy...Q#6: Second CaseThe patient is a 27-year-old female who states that she is 28 weeks pregnant with her second child. While driving to work, she has been involved in a road crash where she was struck head-on by a truck that crossed the mid-line. Because her primary care health worker had counseled her, she was wearing her seatbelt correctly. However, there was significant damage to the patient's vehicle and she describes a momentary loss of consciousness. The patient is currently complaining of shortness of breath and has a cramping abdominal pain. She feels a little light headed. She is lying flat on a stretcher immobilized in a neck collar. She has one intravenous line in place and has oxygen running via a facemask. She is able to protect her airway. Her respiratory rate is 24 and shallow. Her blood pressure is 100/60 mm Hg and the oxygen saturation is 99%.

Given that her airway is protected and her breathing probably adequate currently, what ONE measure could immediately help her and why?Trauma in pregnancy...Q#7The one immediate step that should be taken is that a wedge of some kind -- a rolled towel or other object -- should be placed under the right hip, moving the uterus over to the left side slightly. An angle of about 15° is usually recommended. This removes the uterus from the inferior vena cava and facilitates venous return. Compression of the vena cava can result in a significant reduction in venous return, enough to appear that the patient is hypovolemic. The result of this movement is that the blood pressure is now 118/68 mm Hg. Is this blood pressure 'normal'? And if it is normal, does this indicate we don't have to worry about occult blood loss here?Trauma in pregnancy...Q#8During pregnancy, a number of physiological changes occur. Among them, the blood pressure initially falls in the first trimester, rises slightly in the second, then rises again in the third trimester almost back to pre-pregnancy levels. The pulse rate gradually rises through pregnancy, with a baseline of 85 by the third trimester. The blood pressure recorded on our patient could be considered 'normal'. However, the presence of a normal blood pressure does not preclude the possibility of a significant bleed. Blood volume gradually increases through pregnancy, to levels about 50% or more above pre-pregnancy levels. The result of this is that clinical signs of significant hypovolemia may be delayed because of the increased reserve. Pregnant women suffering significant trauma should have two large bore IV lines established as a routine, even if they appear hemodynamically stable. What fluid should be run through these lines, and why?Trauma in pregnancy...Q#9In the pregnant patient, the use of lactated Ringer's solution is preferred. This is because the fluid is more physiologic and less acidotic that normal saline. Saline appears more likely to cause a significant acidosis and should be avoided. As is usual in the management of trauma victims, consideration should be given to warming the fluids. The patient is now complaining of a cramping feeling in her abdomen. What complication is a possibility now?Trauma in pregnancy...Q#10The presence of a cramping sensation should raise the possibility of the development of a placental abruption. This is a feared complication of trauma in pregnancy. It results from the relatively inelastic placenta separating from the elastic uterine wall when the latter is distorted because of trauma. It can occur after relatively minor trauma. It is the reason why careful observation is required for pregnant women after trauma. Unfortunately, ultrasound is not always reliable at spotting this problem. What monitoring modality is useful?Trauma in pregnancy...Q#11Patients need to have at least four hours of external fetal monitoring if they are beyond 20 weeks gestation. The absence of any uterine activity over the four-hour period immediately following the trauma virtually excludes the possibility of an abruption. The presence of 8 or more contractions in an hour is highly suggestive of the diagnosis of abruption. Management then should be by an experienced obstetrician and will depend

upon fetal gestational age. Between 3 and 7 contractions in an hour require further observation and many would recommend 24 hours of continuous monitoring. Assessment of the fetal heart tones are also vital and should be performed regularly while in the emergency department. What is the normal fetal heart rate? What is the fetal response to stress?Trauma in pregnancy...Q#12The normal fetal heart rate is between 120 and 160 beats per minute. The normal fetal response to stress is bradycardia and the commonest cause of this is hypoxia. If correction of maternal hypoperfusion, hypoventilation or hypothermia does not improve the heart rate, then abruption or another uterine abnormality is likely. You are very suspicious that your patient has an abruption. From what complication of the abruption is the mother at risk? You also find out that the mother is Rh negative. The patient has no vaginal bleeding, so does this matter?Trauma in pregnancy...Q#13Abruption is a risk factor for the development of disseminated intravascular coagulation, secondary to the passage of placental products into the maternal circulation. We are also told that the patient is Rh negative. The patient has abruption, and there is up to a 30% rate of fetal maternal hemorrhage, and she must be given Rh immune globulin. The usual dose is 300 micrograms intramuscularly. As little as 0.5 ml of fetal blood is required to sensitize 70% of Rh negative women. Some authorities use the Kleihauer-Betke test to estimate the degree of fetal blood loss into the maternal circulation. This test is, however, not always available in the emergency department and is, thus, rarely useful in the emergency management of patients. Patients who have an abruption have fetal distress obvious in at least 60% of cases on arrival in the emergency department. In these cases, immediate intervention is required. In the remainder, the abruption may be small enough to allow continuation of the pregnancy, but the mother must be observed very closely. If there is a significant abruption, and the fetus is over 32 weeks, many authorities would proceed to delivery. Let us assume that examination of the abdomen in our patient now revealed a boggy-feeling uterus and you are sure you can easily palpate a foot through the abdominal wall. What devastating complication has now arisen?Trauma in pregnancy...Q#14The presence of a boggy uterus and easily palpable fetal parts are signs of a uterine rupture. Although this complication is very rare, it is devastating in that there is profuse bleeding and rapid deterioration of mother and fetus. Immediate delivery is paramount to save either life. The technique of emergency department caesarian section is a little beyond this presentation, but it is indicated if there are signs of fetal life and resuscitative efforts on the mother have been unsuccessful, with no pulse or blood pressure, for four minutes. The technique involves a midline incision from the pubis to the umbilicus. This incision is extended down through all layers to the uterus. The uterus is opened in the lower section with a small vertical incision from a scalpel. A scissors is then inserted into the incision and, using the fingers as a guide so as to keep the points away from the fetus, extended up to the top of the uterus. The baby is delivered through the incision and the cord clamped. If the placenta is encountered on the anterior wall, it is cut through. The management of pregnant trauma victims can be challenging and stressful for the

staff. The immediate resuscitation should follow well-established advanced trauma life support (ATLS) guidelines. An organized approach, using the skills of the emergency department personnel, together with obstetric, pediatric and trauma surgical colleagues will result in the best outcome for our patient. Neurological localization Case 1a 35yo black male is seen in clinic with a 3 month history of weakness and muscle cramps, first felt in the left arm, but then progressing to both legs. His voice is not as loud as it used to be, and the tone has changed. Sometimes, he complains having trouble eating and swallowing on one side of his mouth, and he noticed he can do so if he moves it with his finger. Has has no sensory loss. Reflexes are brisk, including a jaw jerk. His toes are equivocal. Fasciculations are present in the tongue at rest, and all four proximal extremities. One year later, he has difficulty swallowing, is short of breath, and appears emaciated. Can you localize the lesion? Ans: ALSNeurlogical localization Case 2A 55 year old normotensive man has an episode of sudden loss of vision in his right eye. This persists for 15 minutes and then rapidly resolves. He has a completely normal neurological and ophthalmological examination. What has happened to this patient and what is the appropriate management?DDx: TIA Labs: duplex (ateriogram if duplex not dx'ic) then tx w/aspirin and antiplateletsNeurological localization Case 3A 60 year old man has a 15 minute episode in which he cannot express himself. He becomes increasingly frustrated with this and suddenly stops talking. He remains unable to express himself and unwilling to talk for a period of 18 hours. One day later he comes to the emergency room for evaluation and at this time he is entirely neurologically normal and his speech is normal. What distribution is it and what subsequent evaluation should be carried out?Broca's TIA. CT will most likely be normal. Arteriogram after that.Neurological localization Case 426 yo female graduate student was conducting a philosphy seminar when she suddenly started stuttering and became incoherent. She seemed confused, and her mouth was twisted. One arm hung limply, and she walked unsteadily. She had a past hisotry of untreated rhematic heart disease, and is taking OCP.What is the most likely localization, hemisphere, arterial supply and next step in management?strokelocalisation-frontal lobehemisphere-dominant usually leftarterial supply-middle cerebral arterymanagement-give thrombolytic if not contraindicated.start heparin .stop ocsNeurlogical localization Case 568 yo white female presents with inability to walk. Upon further questioning, you find that this has progressed over a month or two, and is not associated with back pain. On exam, she is slightly inattentive, and sometimes inappropriate. Language is intact. No CN deficits, and good strength of the UE's. Her legs are diffusely weak, 3 to 4 over 5,

proximally and distally. Sensory exam reveals questionable mild loss of light touch and pain sensation distally to LA's, with no demarcated level. Reflexes are brisk in the legs, and she has bilateral Babinski. Where is the lesion? Alteration of mental status = cortical. If it were medullary, you would find a sensory level, as well as problem with GiI and continence. Territory: Ant Cerb. Art. Problem chronic = Brain Tumor. Neurlogical localization Case 6A 40 year old male hypertensive, non insulin dependent diabetic awoke with weakness of the left side of the body and was admitted to the hospital. He gave a history of a transient episode of mild weakness of the right side of the body occurring a week before that had resolved after 3-4 hours for which he did not seek medical attention. He gave a history of nocturia, thirst and weight loss for the prior one year, and had recently been diagnosed with diabetes. There was a family history of diabetes His medications included hydrochlorthiazide and glyburide. On examination, 2 hours after awakening, the blood pressure was 170/90, pulse 110, temperature 36.5 C. General examination was unremarkable. On neurological examination he was awake, anxious, and poorly attentive. He was oriented to person and place but was unsure of the month and day. Speech was fluent with good comprehension, repetition and naming. He was able to follow three step appendicular commands. On cranial nerve examination the fundi were benign and there was a right sided facial palsy sparing the forehead. On motor examination weakness was detected in the right upper and lower extremities, with 4/5 strength in the deltoid, triceps, wrist and finger extensors, hip flexors, knee flexors and foot dorsiflexors. Cerebellar testing was impaired by weakness on the right side but otherwise normal. Sensory examination was normal. DTRs were brisk throughout, slightly brisker on the right side. The right plantar reflex was extensor and the left was flexor. Where is the lesion most likely?Ans: Lesion is at the pons at the level of VII. Ipsilateral facial paralysis (fibers don't cross midline), and ipsilateral hemiplegia (before pyramidal decussation).Neurlogical localization Case 770 yo male presenting with dull aching pain in bothcalves after moderate exercise. The symptoms started a few months ago, typically developing after the patient walked 300 to 400 yards. Symptoms were releived after a few minutes rest or when the patient sat down and stopped forward. In addition to the pain, the patient has experienced numbness in his thighs. He has had no sphincter disturbance and had low back pain for many years. PE: loss of lumbar lordosis and reduced flexion and extension of lumbar spine; tone, power, and coordination in lower limbs are normal. Refelxes in LE symmetrical but reduced compared to upper limbs. Plantar reflexes are flexor; peripheral arterial pulses both at rest and after exercise are normal. Ans: The reason i posted this Q is that it didn't make sense to me. It has features of CONUS MEDULLARY SYNDROME: it is BILATERAL (Cauda Equina is unilat), DTR's are reduced but PRESENT (In Cauda Equina, they are absent), but then it SHOULD HAVE EARLY SPHINCTER PROBLEMS AND URINARY INCONTINENCE AS WELL AS IMPOTENCE. Cauda Equina Syndrome has the Saddle Sensory Deficit but LATE SPHINCTER CHANGES, UNILATERAL

SYMPTOMS, AND ABSENT DTR's. Because of affected DTR's I would choose Cauda Equina Syndrome, and that's the correct answer, but I wanted to make the distinction in the explanation for more info. Neurological localization Case 8A 65 yo white male develops sudden severe headache and rightsided hemiplegia. On PE: patient is a known hypertensive and takes his meds irregularly. He now has both fecal and urinary incontinence. BP210/180, no fever, right sided hemiplegia with fundoscopic examination revealing papilledema in addition to hypertensive retinopathy; right sided babinski; eyes deviated toward the left; no meningeal signs present. Where is the lesion most likely?Ans: Hypertension...most commonly affects Putamen because of rupture of penetrating arteries = This is putamenal infarct.Neurlogical localization Case 915 yo male brought to you by his parents for a follow-up evaluation of herpes simplex encephalitis that he had a few months ago. The patient feels fine, but the parents report a recent change in behavior from markedly agressive to extremely placid. The patient upon further questioning reveals he like to taste things and is gaining weight because he is eating too much. Despite that he doesn't seem to have a problem getting dates since he noticed an increase in his sexual activities, but reassures you he's using condoms. There is no prior history of psychiatric illness. PE; normal. Your diagnosis and localization of the lesion?Ans: kluver bucy syndrome involving anterior temporal lobe-amygdaloid nucleus.there is hyperphagia,docility,hypersexuality and sometimes visual agnosia.Neurlogical localization Case 1050yo male patient presents to the ER with uncontrollable wide movements with his left arm. He throws it in all directions and claims he can't stop. He has been diagnoses with diabetes and HTN but has taken his medications intermittently. He is a chronic smoker. PE assesses the wild flailing movements of the left arm, but remainder of exam is normal. Labs: elevated blood glucose of 190. Diagnosis? Where is the lesion?Ans: hemiballismus-vascular lesion of the subthalmic nucleus-contraletral ballistic movemenets of one or both extremetiesQuestionwhat are the differential diagnosis would you consider even if you suspect the patient has saturday night syndrome. Please give me 3 or 4. thanksAmong the problems associated with the 3 major nerves in the upper extremity, radial nerve entrapment is the least common. Carpal tunnel syndrome (median nerve compression at the wrist) and cubital tunnel syndrome (ulnar nerve compression at the elbow) are much more frequent. In regards to Radial Nerve Syndromes, I didn't find many of them. In fact they all relate to the radial nerve:- Radial Nerve Palsy: Most common cause is Fx of the humerus (holstein lewis fracture). Other cause is compression (in its course inside the triceps muscle or teres latissmus muscle). Nerve injury secondary to compression or traction depends on intensity and duration

- Radial tunnel syndrome: result of overuse. - Posterior Interosseous Nerve Syndrome: Compression is thought to occur after takeoff of the branches to the radial wrist extensors. Other possible etiologies for posterior interosseous nerve dysfunction include trauma (Monteggia fractures), synovitis (rheumatoid), tumors, and iatrogenic injuries. - Wartenberg syndrome, is essentially entrapment of the superficial sensory branch of the radial nerve. In patients with de Quervain tendovaginitis, secondary irritation of the Radial sensory nerve is frequent. Other common causes include postsurgical injury, external compression, and trauma. Reference:emedicinehttp://www.emedicine.com/orthoped/topic549.htmNeurlogical localization Case 11A 55 year-old restaurant manager complained to her physician that she had become increasingly unsteady on her feet to the point that now she "couldn't walk straight" and several times over the past few months had been embarrassed in public by people who assumed that she was intoxicated. She also said that she felt self-conscious that her face was "crooked." In addition, over the past five years her hearing had become "difficult," and she suffered from "ringing" in her left ear. An examination revealed the following: greatly reduced hearing acuity in the left ear; horizontal nystagmus; loss of the corneal reflex in the left eye;; reduced sensation on the left side of the face; weakness in biting down on the left side; drooping of the left corner of the mouth; a dry, red left eye, but normal right eye; a broad-based, weaving gait. Ans: This is a large acoustic neuroma in the cerebello-pontine angle on the left side of the brainstem which has begun to compress the cerebellum and brainstem. This kind of tumor generally arises from the Schwann cells on the vestibular division of the VIIIth nerve and enlarges (often very slowly, perhaps over 10 - 20 years!) to put increasing pressure on the facial and trigeminal nerves, then on cerebellar peduncles, cerebellum and underlying brainstem nuclei. The first symptoms detected are usually auditory and involve a progressive loss of hearing, sometimes preceded by a period of tinnitus. One of the most reliable symptoms is impairment of the corneal reflex, since the trigeminal afferent axons of that reflex arc (unlike the other cranial nerves in the region) are particularly sensitive to the mechanical stresses imposed by the tumor. Other tumors of the cerebellopontine angle follow a different clinical sequence. The symptoms above resulted from the following: 1- greatly reduced hearing acuity in the left ear: left cochlear (VIII) nerve2- horizontal nystagmus: left vestibular (VIII) nerve3- loss of the corneal reflex in the left eye: left sensory V and left motor VII, 4- reduced sensation on the left side of the face: sensory root of the left trigeminal (V) nerve, 5- weakness in biting down on the left side: motor root of the left trigeminal (V) nerve, 6- drooping of the left corner of the mouth: left facial (VII) nerve,

7- a dry, red left eye, but normal right eye: left facial (VII) nerve, 8- a broad-based, weaving gait: cerebellar peduncles and cerebellum and/or vestibular (VIII) nerve. (Could be either side, but since other symptoms are on the left, the left side should be the strongest suspect.)2 cases of gallbladders disease and pregnancyCase 135 weeks pregnant with third child. After almost 2 years of recurrent gallbladder episodes (whereby severe pain was experienced but no visible sign of gallstones was detected in ultrasounds), pt had an ulstrasound that confirmed she has at least one very large stone (too large to pass, according to the tech). Knowing that this stone will likely not go away -- and therefore continue to present pain and possibly more problems -- would it be a reasonable consideration to a) have it removed before the baby is due (and what sort of risks are involved?) b) request a cesarean section so that the gallbladder might be removed at the same time as the delivery? Case 216 weeks pregnant, and has had an abdominal ultrasound which shows no stones or sludge. Pt has been on a low fat diet for about a week, and still experiencing low level pain in GB area as well as solar plexus and under right shoulder blade. The diet has, however, helped with indigestion and nausea. 1) what are the indications for cholecystectomy in this patient if at all? 2) what if things get very bad in 3rd trimester, will they still operate???Ans: Case 1: Lap Chole is performed ideally in second trimester. Indications are intractable pain refractory to pain medication and diet. Third trimerster cholecysitis with intractable pain may require OPEN CHOLE since gravid uterus may interfere with procedure. C-Section may not be done since it is not an indication. Case 2: No surgical indications in this patient, since pain is low level and can still be taken care of by analgesics. If it continues to third trimester like Case 1, OPEN CHOLE is to consider. Compiled data about age of repair for cryptorchidismMedical Articles review showed the following:- Any infant with undescended testes should be monitored for 6 months. Most would descend by then.- If by 6 months they haven't descended, REFER to UROLOGIST. Most commonly orchiopexy would be done at the end of first year of life. I have found articles saying after 6 months of age, but there is data suggesting they may still spontaneously descend until first year. After first year of life, they hardly ever will on their own. - Trial of hCG is indicated for some cases of BILATERAL undescended testes. Success rate <10%. Not indicated for unilateral. - By second year, irreversible changes might have already occured especially if intra-abdominal. - Orchipexy after that is still indicated IF NOT TO RESTORE FERTLITY, AT LEAST TO MAKE TESTICLES READILY AVAILABLE FOR EXAMINATION. Orchipexy does NOT decrease risk for Testicular cancer. - Men who had their orchipexy done before 11 yo, will have virtually not a significantly elevated risk for testicular cancer. For those who didn't, they have a 32 fold increase of

that risk compared to general population. - Patients who have had orchipexy before may not have a clue what was done to them as children, because their mothers rarely tell them, or they rarely ask. And if they do know, they rarely bother to tell. ALWAYS INQUIRE SPECIFICALLY. - Cryptorchidism DOES NOT CAUSE TESTICULAR CANCER. IT MERELY INDICATES A TESTICULE IS AT RISK. - ANY BOY WITH BILATERAL NON-PALPABLE TESTES SHOULD BE CONSIDERED FOR A DIFFERENTIAL OF CONGENITAL ADRENAL HYPERPLASIA UNTIL PROVEN OTHERWISE. Odd HIV caseWith the rise of syphilis as an STD in HIV populations, syphilis is a hot topic. The diagnosis of syphilis may be more complicated in HIV-infected patients because of false-negative and false-positive serologic results for T. pallidum and atypical clinical presentations in the presence of HIV infection. Facts: - VDRL and RPR titers are higher than in HIV neg population. - CNS disease can occur during any stage of syphilis. Clinical evidence of neurologic involvement warrants CSF examination. - Serologic tests for syphilis are the cornerstone in diagnosing untreated syphilis infection--even in HIV-infected patients. - A negative RPR or VDRL test result may not rule out syphilis in patients with HIV infection. - A patient with suspicious lesions but negative serologic results, positive findings on dark-field examination or DFA stain can be diagnostic. Obtain dark-field examination or direct fluorescent antibody (DFA) staining of exudate from suspicious lesions of primary syphilis, or selected secondary lesions, such as condylomata. - Case Scenario: HIV patient with secondary syphilis was treated, then developed symptomatic hepatitis and subclinical reticulonodular pulmonary infiltrates. => Jarisch-Herxheimer reaction. In this patient, it included a flare of hepatitis symptoms, resembling cholangitis. Avoid trap of pneumonitis or cholangitis until seeing outcome of specific antitreponemal therapy. - Before treating, carefully examine HIV-infected patients with syphilis for clinical evidence of neurologic involvement (eg, optic and auditory symptoms, cranial nerve palsies). If no clinical evidence of neurologic involvement, the same treatment regimen as for patients without HIV infection is recommended: 2.4 million units of benzathine penicillin G administered intramuscularly at a single session. - In penicillin-sensitive patients, confirm allergy, then desensitize. If patient is not pregnant and refuses desensitization, consider doxycycline (100 mg orally 2 times a day for 2 weeks). No data are available, however, on the efficacy of tetracyclines in treating syphilis in HIV-infected patients. If compliance and close follow-up cannot be ensured, then desensitization to penicillin and management in consultation with an infectious disease expert. - For HIV-infected patients diagnosed with neurosyphilis (including ocular or auditory syphilis), aqueous crystalline penicillin G is the treatment of choice (12 to 24 million units intravenously per day, eg, 2 to 4 million units every 4 hours for 14 days). Desensitize penicillin-sensitive patients to penicillin.

- If intravenous administration is impossible, then aqueous procaine penicillin G is another option (2.4 million units intramuscularly daily plus probenecid 500 mg by mouth 4 times daily for 14 days). - Because of the possibility of recent clinical relapse following syphilis therapy in HIV-infected patients, close follow-up: reexamine at 1 to 2 weeks and reteste with a quantitative nontreponemal test at 3, 6, 9, and 12 months after treatment for primary, secondary, and early latent syphilis and at 6, 12, 18, and 24 months after treatment for late latent syphilis or syphilis of unknown duration. Reference:- http://hivinsite.ucsf.edu/InSite?page=kb-05-01-04#S7X What is the anterior and central cord syndrome?Central cord syndrome is the most common of incomplete Spinal Cord injury. Risk Factor: Cervical Spondylosis in older patients Mechanism: Hyperextension Trauma (minor bleeds in the tissue) Symptoms: Sensory deficit with level, various degrees of motor involvement with paresis more pronounced in UE than LE. Hands are especially involved. DTR's below level are absent at first but return with level of spasticity once over Spinal shock. Bladder involvement common. Other: Neurogenic bladder (retention), Pain of neuro etiology, Spasticity. Management: Admit to Neuro ICU, Maintain a level of mild hypertension for spinal perfusion, Gabapentin for neuro pain, PT/OT/Speech, May be tube feeding in acute phase because of adynamic ileus. Prognosis: 97% completely recover if less than 50yo. More somber prognosis is after that. The anterior cord syndrome anatomically involves the anterior portion of the cord. Disc protrusion, hyperflexion, or vascular injuries may play a role. Clinically, paralysis below the injury with variable impairment of pain and temperature sensation is present. Difference with Central Cord Syndrome:- Mechanism of injury is different- Position and vibration modalities (posterior columns) are preserved.- Motor symptoms are different between upper and lower extremities in Central. Action: Since diagnosis is pretty much narrowed down. Start Steroids: Solumedrol 30 mg/kg over 15 minutes then followed after 45 minutes by an infusion at 5.4 mg / kg / hr is the typical regimen employed... Methylprednisolone is thought to impact the biochemical cascade of injury that progresses after the initial injury for some hours. Next: MRI: shows narrowing of the white column of CSF and impingement of the darker appearing spinal cord.Orbital cellulitis and Cavernous Sinus ThrombosisHow to differentiate between Cavernous sinus thrombosis and orbital cellulitis? - Cavernous Sinus Cellulitis: Cavernous sinus contains internal carotid, sympathetic fibers, III, IV, V1 and V2, and VI. 1. Patients generally have sinusitis or a midface infection (most commonly a furuncle) for 5-10 days2. Headache, fever, and malaise typically precede the development of ocular findings.

3. orbital pain and fullness accompanied by periorbital edema and visual disturbances.4. Increased RETROBULBAR PRESSURE: Exophthalmus and Ophthalmoplegia5. Increased INTRAOCULAR PRESSURE: sluggish pupillary response and decreased visual acuity6. CRANIAL NERVE PALSIES7. Without effective therapy, signs appear in the contralateral eye by spreading through the communicating veins to the contralateral cavernous sinus. This is pathognomonic for CST. The patient rapidly develops mental status changes from CNS involvement and/or sepsis. Death follows shortly thereafter.8. Mainstay of therapy: Braod Spectrum Antibiotics (Staph Aureus Most common) - Heparin therapy - Steroids (especially if progressed to pituitary insufficency to prevent adrenal crisis). - Orbital Cellulitis: Orbital infections develop via direct inoculation, extension from adjacent structures, and hematogenous spread. 1. Sinusitis (60% patients, ethmoid most commonly), dacryocysitis, dentition, complication of periorbital infection2. Classification: Group I - Preseptal (periorbital) cellulitis = inflammatory edema of the eyelids and periorbital skin with no involvement of the orbit. Lids cannot be opened because of edema. Group II - Orbital cellulitis = CT scan is not sensitive for diagnosing this entity; therefore, clinical examination guides therapy. + Lids cannot be opened because of paralysis secondary to III involvement. + Fever and leukocytosis+ Orbital signs Group III - Subperiosteal abscess = collections of purulent material between the orbital bony wall and periosteum. This diagnosis is confirmed by CT scan, but it can be suspected based on physical examination + Orbital signs (see above)+ Limitations of ocular motility = pain in globe movement toward the abcess.+ Directional proptosis = Globe is looking away from the abcess. Group IV - Orbital abscess = collections of pus within the orbital soft tissue. Diagnosis is confirmed by CT scan, but the physical signs or papilledema on funduscopic examination, are suggestive. + Severe unilateral ptosis+ Severe ophthalmoplegia (ie, palsy of the pupillary and extraocular muscles)+ CN V1 (forehead) anesthesia Group V - Cavernous sinus thrombosis+ Bilateral symptoms: + Ophthalmoplegia, proptosis+ Corneal hypesthesia with increased intraocular pressure Treatment: - Admit all children because children are deficient in IgG2 and are predisposed to bacteremia.- Preseptal: D/C only if adult with PO ATBx and close follow-up

- Orbital: Admit with IV ATBx +/- Surgical intervention if necessary (i.e.: compromised vision)Oxacillin or nafcillin can be used with the addition of ampicillin and sulbactam in children to cover H influenzae. Patients who are allergic to penicillin can use vancomycin, clindamycin, or chloramphenicol.Alternatively, a cephalosporin (eg, cefuroxime, cefoxitin, cefotetan) can be used alone.QuestionPx comes in from a MVA. He is breathing well but obviously has mult bruises over the chest with the impression of a mercedes steering wheel (or yugo, you add in your favorite car). Palpation of the chest exquisitely tender over the sternum at a point where there is a gritty feeling of bone grating on bone, elicited by palpation. Trauma "ABCDE" has been done. Your immediate DDx is a sternal fracture. NBS?Problems to be considered: Aortic DissectionCardiac contusion or tamponadeFlail chestPulmonary contusionThoracic spine injury To do: - CXR as procedure of choice to substantiate Sternal fracture, and r/o pneumothorax/Aortic disruption. - ECG is next to r/o myocardial contusion: dysrhythmias, conduction disturbances, or ST segment changes. To give: Adequate analgesia is treatment of choice as taping or splinting is contraindicated (risk of atelectasis and pulm insufficiency). No admission necessary if no associated conditions. To pay attention to: If pregnant: shield abdomen and pelvis with lead apron before CXR. NSAID's contraindicated, use meperidne or other Category B opiates. what test is used for document osmotic diarreahea secondary to lactase deficinecy?1- Lactose elimination diet2- Lactose Hydrogen breath test: 90% sensitivereview of ethics by ValiumRULES OF ETHICS

HOW TO DEAL WITH A DYING PATIENT: 1) Tell the patient EVERYTHING. There is no excuse for not doing so. If you know, the patient knows.2) DO NOT GIVE FALSE HOPE3) Allow the person to talk about his feelings4) Kept he patient involved in social activities5) Avoid social isolation

GENERAL RULES: 1) “Substituted judgment”: when a patient cannot make a decision, the decision is made based on what is thought WOULD BE that person’s choice. The decision is finally made

by who is most likely to represent the patient’s own wishes (not necessarily who is closest next of kin). 2) “Best interest standard”: trying to determine what a never-competent patient would have wanted is practically impossible. When you are not clear about the patient’s wishes, you should make the decision as a dispassionate, rational observer: do what a rational person would do. It is not your personal preference is. DO WHAT MOST PEOPLE WOULD WANT in this circumstance. “What would a jury of 12 people do if they knew what I know?” Who makes the decision is not really important: anybody using the best interest standard should arrive to the same decision. You must set aside your personal preferences: like strong religious beliefs (that is considered irrelevant) As a general rule, parents cannot withhold treatment from their children. Yet, in Infant Doe’s case, they did. In this case, the best interest standard rule was applied. 3) Patients decide over their own bodies: The patient ALWAYS MAKES THE DECISION. This was decided over the ROE vs WADE case in 1973, the case that made abortion legal. The issue will never be over abortion in the USMLE, but it illustrates the principle that governs medicine in the US: the patient always decides, and the only thing a doctor can do is lay out the possibilities. SPECIFIC RULES: Rule #1: Competent patients have the right to refuse medical treatment, no matter what. Rule #2: Assume that the patient is competent unless clear behavioral evidence indicates otherwise. -Drunk, schizophrenia, Alzheimer’s: these are all medical dg. DIAGNOSIS SAYS NOTHING ABOUT THE LEGAL COMPETENCE OF A PERSON!!! Competency can ONLY be decided by a COURT OF LAW: it is not a medical dg, it is not a blood alcohol level! Clear behavioral evidence of incompetence: Attempted suicide Patient is grossly and evidently psychotic and dysfunctionalPatient’s physical or mental state prevents communication However, when in doubt, assume competency! Rule #3: Decision-making should occur in clinical setting if possible, without going to court. Normally, the USMLE will want YOU to make the decision: try to avoid the answer that says “go to court”, unless it is clearly stated that the guardian (ex: parent of a sick child) is NOT acting in the patient’s best interest. And that is only if the case is not an emergency: if it can wait going to court. Rule #4: When surrogates make decisions for a patient, they should use the following criteria and in this order: 1) Patient expressing wishes in the past: what historically did the patient say in the past? (wish for organ donation expressed to relative, for example)2) What would the patient want? : Substituted judgment3) Best interest standard: what would most people want Rule #5: If patient is incompetent, physician may rely on advance directives Directives that a patient can leave for his doctor before becoming incompetent:

- Can be oral directive from patient to his doctor: does NOT necessarily have to be a written document. - Can be living will: expression in writing, notarized, by the patient. - Health powered attorney: person that was named by the patient to represent him. TREAT THIS PERSON AS THE PATIENT HIMSELF, IN TERMS OF DECISIONS: this person is the VOICE of the patient: a health powered attorney BEATS ALL OTHER CHOICES ON THE USMLE (it is the patient talking to you).

Rule #6: Feeding tube is a medical treatment and can be withdrawn at the patient's request. A competent patient has the right to refuse hydration and nutrition. Period. In the case of anorexia nervosa: if the patient is a minor: not legally competent. If NOT a minor: go to court. Refusing food and water: may seem close to euthanasia, but on the exam this is accepted. Rule #7: Do nothing to actively assist the patient to die sooner. Do not ACTIVELY do anything (as opposed to number 6) Rule #8: the physician decides when the patient is dead.Futile treatment: means a treatment that is not AND WILL NOT improve anything. Still, if the patient or the family want the treatment to continue: it is not YOUR decision, it is the patient’s or the patient’s family.In case of clear cortical death: even if the family is hoping for a special doctor to arrive, for a special treatment to come: CALL THE DEATH. Rule #9: Never abandon a patient: even if they can’t pay you, even if you don’t like the patient. If you simply CANNOT continue to be the doctor to this patient: you need to arrange that he will have care and make sure that they are getting it. Never, ever threat to abandon your patient (not even if you are doing it to make sure they follow treatment). Rule #10: Always obtain informed consent: before you do ANYTHING!!!Informed consent can be oral. The patient can revoke written consent orally, at any momentOf the patient signs “consent” without reading it: it is NOT INFORMED CONSENT Informed consent: means that the patient understands:1) Nature of the procedure2) Purpose or rationale3) Benefits of treatment or procedure4) Risks5) Availability of other alternatives “GAG CLAUSES”: you work for an institution that tells you not to discuss certain procedures or possibilities. THEY ARE ILLEGAL. Exceptions to informed consent:1) Emergency situation2) Waiver by the patient: the patient says it’s OK not to know what is going to happen (exploratory surgery, drug undergoing trial to know side effects)DO NOT ASSUME YOU HAVE A WAIVER UNLESS THE USMLE TELLS YOU.3) Patient is incompetent4) Therapeutic privilege: doctors have the right and obligation to deprive the patient of

their autonomy in the interest of the patient and other people. Ex: patient on PCP, violent and dangerous: put him on restraints!

Rule #11: Special rules apply with children Rule #12: Parents cannot withhold life- or limb-saving treatment from their children Rule #13: For the purposes of the USMLE, issues governed by laws that vary widely across states cannot be tested Rule #14: Good Samaritan Laws limit liability when physicians help at accidents Rule #15: Confidentiality is absolute Rule #16: Patients should be given the chance to state DNR (Do Not Resuscitate) orders, and physicians should follow them Rule #17: Committed mentally ill patients retain their rights Rule #18: Detain patients to protect them or others. Rule #19: Remove from patient contact health care professionals who pose risk to patients Rule #20: Focus on what is the best ethical conduct, not simply the letter of the lawI looked it up: rule # 8 says this: - if there are no more treatment options (if the patient is cortically dead), and the family insists in treatment?: if there are no options and there is nothing the physician can do, it is his duty to stop the treatment. (The USMLE wants you to be able to make decisions when the patient is DEAD) - if the physician thinks tratment is futile and the patient won't improve, but the patient (or surrogate) insists on continued treatment: then treatment must continue. rule # 17: Commited mentally ill adults legally are entitled to the following:- they must have treatment available- they can refuse treatment- they can command a jury trial to determine sanity They lose only the civil liberty to come and gothey retain their competence for everything UNLESS A COURT OF LAW DECIDES they are incompetent. The underlying rule here is that no matter what the psychiatric diagnosis is, treat the patient as you would any other competent person (unless they show signs of clear incompetence, stated on #2) rule #20: Focus on what is most ethical. USMLE wants you to pick the answer where there are no doubts that it is the most ethical thing to do. In other words, don't worry about being fired, sued or that your hospital may go to shreds if you do the "right thing". ACT NOT AS A LAWYER WOULD, BUT AS MOTHER THERESA WOULD.

There is also something interesting that they pointed out: what do you do if you find out a collegue or fellow resident is having a substance abuse problem? Who do you talk to? RULES: - talk to the collegue and REMOVE him from patient care- if there is a direct employer or supervisor (like your residency program director) : TELL THE SUPERVISOR. Failure to do so will endanger patients, and will ALWAYS be the

worong answer on the USMLE. What they meant was: the best way to get someone to treatment is if their employer forces them to: if they are afraid to lose their job. So don;t waste time talking to the person or the family or anyone: go to the supervisor.Exacerbation of COPD on ABG’sHow can U interprete Acute exabe of COPD with Blood gas analysis esp. when cases like Previous COPD patient with COPD who comes with acute exabe. are given ?Ans: When interpreting ABG's, we don't always look for ph, bicarb and pCO2 only. All of you mentioned increasing pCO2. It's true, but how about decreasing pO2? Think about it. After all, Home O2 indications guidlines for Medicare are based on O2Sat or pO2 and those of Medicaid REQUIRE ABG's where pO2 would be documented, and they won't authorize Home O2 for a patient with ONLY O2 Sat. Ex: pH = 7.38, pCO2 = 43, pO2 = 68mmhg, HCO3 = 28 On exacerbation:pCO2 will go up pCO2 = 53pO2 will go down pO2 = 55O2 Sat= 84%HCO3- Will go up to compensate but not too much, because the exacerbation is acute so it'll be probably 30. Similarly, when talking about other Acid-Base disorders, you look at other parameters depending on the clinical scenario. If diarreha or vomiting, electrolytes come in handy. If intoxication, anion gap is important. And so on and so forth. I posted some ABG's for your exercise. Let me know if you need more explanation

ABG exercises#1---pH -- pCO2-- pO2 -- Na --HCO3- --Cl- -- K+A)-7.14 -- 70 -- 52 -- 140 -- 22 -- 110 -- 5.6B)-7.22 -- 24 -- 98 -- 136 -- 12 -- 106 -- 4.8C)-7.26 -- 34 -- 95 -- 144 -- 15 -- 108 --4.9D)- 7.56 --20 --114 --138 -- 25 -- 112 -- 3.2E)- 7.10 -- 24 -- 56 -- 140 -- 12 -- 110 -- 3.0F) -7.29 --30 -- 104-- 138 -- 15 --111 --5.4G) -7.50 -- 44 -- 98 -- 148 -- 34 -- 115 -- 4.5 1) 68 yo woman despondent about her disfiguring rheumatoid arthritis attempts suicide by ingesting a number of pills for a medication she has at home 2) 21 yo college senior is seen the morning of her finals complaining of palpitations, anxiety, and tingling in hre hands 3) an 18 yo man is left in the ER entrance and is found comatose 4) a 17yo high school student leaves summer camp and refuses to take his insulin for the first week of camp 5) a 29yo man with AIDS is brought to the ER lethargic and extremely tachypneic. His family reports that he has been this way for 2 days. BP 60/40 and RR 40 6) 58yo African American man is treated with diuretics for HTN 7) a 34 yo IDDM patient is seen on his annual exam. U/A = 3+ proteinuria and serum creatinine 2.1 mg/dl

#2: ---pH -- pCO2-- pO2 -- Na --HCO3- --Cl- -- K+A)-7.32 -- 49 -- 57 -- 140 -- 30 -- 108 -- 4.8B)-7.50 -- 28 -- 78 -- 138 -- 25 -- 108 -- 3.6C)-7.44 -- 40 -- 70 -- 140 -- 24 -- 108 --3.5D)- 7.30 --32 --96 --138 -- 16 -- 114 -- 3.0E)- 7.52 -- 43 -- 96 -- 138 -- 16 -- 114 -- 3.0F) -7.26 --28 -- 94-- 138 -- 10 --110 --5.0 1- A homeless man appears intoxicated and complains of blurry vision. His serum alcohol level is zero. 2- A 23yo man with asthma with severe tachypnea and use of accessory muscles of respiration does not improve after 60min in ER. 3- A 68yo with 40 year history of smoking and chronic cough. 4- a 25yo develops severe watery diarrhea for 2 days while on vacation in MExico. 5- A 19yo asthmatic presents to the ER with wheezing and tachypnea with RR 26 6- a 30yo develops severe vomiting 6 hours after a family picnic answers: ---pH -- pCO2-- pO2 -- Na --HCO3- --Cl- -- K+A)-7.14 -- 70 -- 52 -- 140 -- 22 -- 110 -- 5.6B)-7.22 -- 24 -- 98 -- 136 -- 12 -- 106 -- 4.8C)-7.26 -- 34 -- 95 -- 144 -- 15 -- 108 --4.9D)- 7.56 --20 --114 --138 -- 25 -- 112 -- 3.2E)- 7.10 -- 24 -- 56 -- 140 -- 12 -- 110 -- 3.0F) -7.29 --30 -- 104-- 138 -- 15 --111 --5.4G) -7.50 -- 44 -- 98 -- 148 -- 34 -- 115 -- 4.5 1) 68 yo woman despondent about her disfiguring rheumatoid arthritis attempts suicide by ingesting a number of pills for a medication she has at home B)-7.22 -- 24 -- 98 -- 136 -- 12 -- 106 -- 4.8 Salicylate IntoxicationpH= AcidpCO2=decreased (If it were respiratory, it would have been increased. Therefore, it's not respiratory. It is compensation)HCO3- = decreased (makes sense) => Metabolic Acidosis(Na+K+)-(Cl+HCO3)= Anion Gap: 22.8 = Increased Anion Gap Anion Gap etiologies: MUDPILES: Methanol, Uremia, DKA, Paraldehyde, INH/Iron, Lactic Acidosis, Ethanol, Salicylates. 2) 21 yo college senior is seen the morning of her finals complaining of palpitations, anxiety, and tingling in her hands D)- 7.56 --20 --114 --138 -- 25 -- 112 -- 3.2 Panicking, Hyperventiliating = Resp AlkalosispH: AlkalosisPatient with no underlying pulm disease = No hypoxiapO2 expected to be normal to high pCO2 Low: Patient probably hyperventilating HCO3-: normal since no time for compensation. 3) an 18 yo man is left in the ER entrance and is found comatose

Probably intoxication since he was "left in the ER entrance". Comatose, think narcotics. A)-7.14 -- 70 -- 52 -- 140 -- 22 -- 110 -- 5.6Low respiratory drive : Hypoxia, and hypercarbia = Profound respiratory acidosis with hyperkalemia in response to the deep acidosis. 4) a 17yo high school student leaves summer camp and refuses to take his insulin for the first week of campDKA: Anion Gap Metabolic Acidosis B)-7.22 -- 24 -- 98 -- 136 -- 12 -- 106 -- 4.8 pH= AcidpCO2=decreased (If it were respiratory, it would have been increased. Therefore, it's not respiratory. It is compensation)HCO3- = decreased (makes sense) => Metabolic Acidosis(Na+K+)-(Cl+HCO3)= Anion Gap: 22.8 = Increased Anion Gap Anion Gap etiologies: MUDPILES: Methanol, Uremia, DKA, Paraldehyde, INH/Iron, Lactic Acidosis, Ethanol, Salicylates. Difference between DKA and Salicylate acidosis:- Bedside Blood sugar/Glucosuria- In Salicylates: Respiratory Alkalosis FOLLOWED by Metabloic Acidosis (mixed later on, ph will be NORMAL with Low pCO2 and High HCO3-). 5) a 29yo man with AIDS is brought to the ER lethargic and extremely tachypneic. His family reports that he has been this way for 2 days. BP 60/40 and RR 40E)- 7.10 -- 24 -- 56 -- 140 -- 12 -- 110 -- 3.0 Patient has Septic Shock. Metabolic acidosis with hypotension. In response to shock, there is inadequate perfusion with resultant secretion of stress hormones. Increase in glucocorticoids along with catecholamines causes Hypokalemia among other changes. 6) 58yo African American man is treated with diuretics for HTNG) -7.50 -- 44 -- 98 -- 148 -- 34 -- 115 -- 4.5 HTN in african american. DOC for African Americans is Thiazide diuretic. Thiazide induced metabolic acidosis. They enhance NaCl excretion in the distant tubule, which stimulates aldosterone secretion. The latter enhances reabsorption and increases hydrogen and to some extent K secretion. Cl is high. pCO2 is high to compensate. 7) a 34 yo IDDM patient is seen on his annual exam. U/A = 3+ proteinuria and serum creatinine 2.1 mg/dlF) -7.29 --30 -- 104-- 138 -- 15 --111 --5.4 Hyperkalemia, Hypercholremia, Low Bicarb. Uremic Metabolic Acidosis. Borderline Anion Gap. The acidosis occurring in uremic patients is due to failure of excretion of acid anions (particularly phosphate and sulphate) because of the decreased number of nephrons. There is a major decrease in the number of tubule cells which can produce ammonia and this contributes to uremic acidosis. ---------------------------------------------------------pH -- pCO2-- pO2 -- Na --HCO3- --Cl- -- K+A)-7.32 -- 49 -- 57 -- 140 -- 30 -- 108 -- 4.8B)-7.50 -- 28 -- 78 -- 138 -- 25 -- 108 -- 3.6C)-7.44 -- 40 -- 70 -- 140 -- 24 -- 108 --3.5

D)- 7.30 --32 --96 --138 -- 16 -- 114 -- 3.0E)- 7.52 -- 43 -- 96 -- 138 -- 16 -- 114 -- 3.0F) -7.26 --28 -- 94-- 138 -- 10 --110 --5.0 1- A homeless man appears intoxicated and complains of blurry vision. His serum alcohol level is zero. F) -7.26 --28 -- 94-- 138 -- 10 --110 --5.0 Methanol Intoxication Anion Gap Metabolic Acidosis. Explained before. Antidote: ethanol. 2- A 23yo man with asthma with severe tachypnea and use of accessory muscles of respiration does not improve after 60min in ER. C)-7.44 -- 40 -- 70 -- 140 -- 24 -- 108 --3.5Status Athmaticus. We just talked about this a few posts down. pCO2 normal, pH alkaline but normalizing, pCO2 decreasing because of the hypoxia. HCO3-: normal since no time for compensation. 3- A 68yo with 40 year history of smoking and chronic cough. A)-7.32 -- 49 -- 57 -- 140 -- 30 -- 108 -- 4.8This guy is retaining his CO2 = pCO2 highResult= pH lowBUT pO2 is very low as well. for COPD patients, pO2 usually is around 50-60. Below 55, order Home O2. 4- a 25yo develops severe watery diarrhea for 2 days while on vacation in MExico. In response to diarrhea, there is loss of Bicarb. D)- 7.30 --32 --96 --138 -- 16 -- 114 -- 3.0It's a metabolic acidosis but with NORMAL anion gap. Chloride is high. pCO2 is low to compensate. pO2 is normal. 5- A 19yo asthmatic presents to the ER with wheezing and tachypnea with RR 26This time, it's Athma exacerbation Early response. Hyperventilation = Respiratory Alkalosis. B)-7.50 -- 28 -- 78 -- 138 -- 25 -- 108 -- 3.6pO2 is low, but not critically low, as you would find for a COPD. Bicarb again normal because no time to compensate. Potassium low to try to get as many hydrogen ions as possible in the extracellular compartment and compensate for the alkalosis. 6- a 30yo develops severe vomiting 6 hours after a family picnic E)- 7.52 -- 43 -- 96 -- 138 -- 16 -- 114 -- 3.0 Gastric secretions are rich in HCl. The secretion of HCl by the stomach usually stimulates bicarbonate secretion by the pancreas once HCl reaches the duodenum. Ordinarily, these substances are neutralized, and no net gain or loss of hydrogen ions or bicarbonate occurs. When HCl is lost by vomiting or NG suction, pancreatic secretions are not stimulated and a net gain of bicarbonate into the systemic circulation occurs, generating a metabolic alkalosis. Volume depletion maintains alkalosis. In this case, the hypokalemia is secondary to the alkalosis itself and to renal loss of potassium ions from the stimulation of aldosterone secretion. GI questiona 32 yr old woman with no past medical history comes with difficulty swallowing foods(esp. solids for one year)problems have not worsened at all.wt is stable.exam is

unremarkable.what is the next step in evaluation of this pt?Ans: First test to order: Barium swallow Differential in dysphagia/Young patient/No med Hx:- Mucosal or Shatzki ring: Follow UGI series with endoscopy both diagnostic and curative with attempt to dilate the ring.- Esophageal web: plummer-vinson syndrome...but physical exam is normal and PV comes with long standing iron-deficiency anemia and glossitis. Nonetheless, it's a differential. Follow Barium with CBC, and endoscopy (r/o esophagel cancer associated with Plummer-vinson).- Esophagel Stricture following lye ingestion: but there is no history of major depression with suicidal thoughts and ingestion of caustic agents such as lye. Most likely diagnosis: Shatzki ringa 27 yr old woman presents with complaint of diarrhea for 14 months. she reports no constitutional symptoms. her examination is normal. her lab results show iron deficiency and low phosphorus. you consider diagnosis of malabsorption syndrome . what is the best next step in this case.Ans: once u suspect malabsorption then the next step would be to see whether it is with steatorrhea or without steatorrhea this done by sudan stain or fecal fat estimation.if there is increased fat in the stool i.e., more than 14gr per dl. then it is steatorrhea,common causes celiac disease and pancreatic insuffiency.next you do d-xylose test where you will have a normal test in pancreatic insuffiency and abnormal in celiac .the former is confirmed by trypsin level and secretin test and the latter by a biopsy.if there is no steatorrhea then you should excludelactase def. and pernicious anemia.a 46 yr old man is brought to emergency department after vomiting Bright red blood twice .what is the first step in management of this pt. after stabilizing pt what do you do next. given that this pt has no prior history of gi bleeding before, what type of anemia would you likely find while he is being evaluated in ER.Next step to upper GI bleed = Endoscopy...but first things first:- ABC's : 2 large bore IV's + Fluid resuscitation (RL or NS) - Type and cross match pRBC's x 2U in case needed later.- Keep NPO + PPI - CBC, EKG to look for infarct/ischemia- Endoscopy (Barium studie are CI in the setting of an acute UGI bleed as it wil interfere with subsequent endoscopy or surgery if needed). CBC= Because it's acute : Normocytic Normochromic Anemia Differential:- Esophagus: Esophageal varices, esophagitis, esophageal cancer, and MAllory weiss tear- Stomach: Gastric Ulcer, Gastritis, Gastric Cancer- Duodenum: Duodenal ulcer, Aortoenteric fistula (if previous aortic graft)- Coagulopathy (drugs, renal disease, liver disease). a pt comes with new onset epigastric pain and dyspepsia. what clinical markers would make you consider a gastroenterologist referral.pt does not have these alarming clinical markers. what qs do you ask pt. that would help you consider your next step.

pt is not taking any medication. what is the first test to do.Ans: Clinical Markers: Weight loss, Dysphagia, sensation of fullness post prandial, supraclavicular lymphadenopathy, oral thrush, Question to ask for next step:- Is patient on any medications (NSAID's, Antibiotics,..)- Sexually active/unprotected sex First test to do:- Endoscopy42 yr old woman has diarrhea for 6 months. she has stopped taking milk, but no improvement. diarrha occurs throughout the day with no blood or pus. she has PUD for last many yrs that has been resistant to medical treatment. labs are normal except for a mild hypercalcemia. what is your next step in evaluation.Ans: Clues: PUD for last many yrs that has been resistant to medical treatment, diarrhea, hypercalcemia Zollinger-Ellison syndrome Labs: 1. check gastrin levels 2. IV secretion test 3. CTFirst line of treatment: PPI. Surgery follows if single gastrinoma that hadn't spread to adjacent structures. Follow-up tests to screen for MEN IStatus AsthmaticusPt with classic scenario of Status Asthmaticus comes to ER tired, using accessory muscles. On PE: chest silent. ABG's pH normal and PCO2 borderline. Whatis responsible for his respiratory "problem"? - Bronchoconstriction- Muscle Fatigue Also others mentioned high yield: Mechanism of Asthma? Others: Management of Status Asthmaticus?A- Mechanism:Although BRONCHOCONSTRICTION and BRONCHIAL HYPERACTIVTY are components of asthma, as of 1997 asthma is DEFINED AS CHRONIC INFLAMMATION. Event 1: Beggining of response of bronchial wall to the antigenic stimulationEvent 2: EARLY PHASE RESPONSE: Mast Cell degranulation of preformed mediatorsEvent 3: LATE PHASE RESPONSE = RELEASE OF SECONDARY MEDIATORS: Histamine, Chemotactic factors, Proteolytic Enzymes, HeparinEvent 4: Smooth muscle bronchoconstrictionEvent 4: Recruitment of other inflammatory cells: Neutrophils, Eosinophils, Mononuclear cellsEvent 6: Release of cytokines, vasoactive factors, Arachidonic acid metabolitesEvent 7: Activation of epithelial and endothelial cells enhancing inflammatory responseEvent 8: Release of IL3 thru Il10 and IL13, TNF, IFNGamma. Result: + Increased bronchial hyper-responsiveness to stimuli+ Reversible airflow obstruction by:Bronchoconstriction - Mucous Plug Formation - Edema B- Answer to the Q

the answer is --> muscle fatigue, a normalizing CO2 in acute exacerbation, most likley indicates fatigue of ventilatory muscles and impending resp. failure...the pt. is getting worse by the min. and could be / is crashing... the ref. is in the first aid and also i looked it up in Cecil's and it is there as well...

my little sidenote:... it simply comes down to the pt. is fatigued to have to keep trying to breathe as he is very bronchoconstricted , over a period of time...as to bronchoconstriction ...how much more bronchocostricted can a pt. get...he is already probably quiet constricted...so muscle fatigue,..

as status asthmaticus ensues , the pt. is bronchoconstricted,...when a pt. is bronchoconstricted, he/she does not have enough air coming into thier lungs,...therefore their ventilation is decreasing...to conpensate for that pt. breaths faster- i.e. pt. is hyperventilating...as pt. is hyperventilating,...CO2 decreases as they are blowing off CO2 and obviousely they are also hypoxic as they are not getting enough oxygen due to the bronchocostriction... as time goes on and the pt. is breathing at this rate, pt. gets fatigued...not able to sustain this kind of ventilation, so pt. decreases breathing efforts and decreases ventilation, as pt. decrases ventilation, pt. is not blowing off as much CO2, so he/she is retaining it, therefore CO2 levels begin to NORMALISE , as they were low before ( when pt. was hyperventilating),..thus you will have normalization of the Ph as well, bcz pt. was in resp. alkalosis b4 , and now is normalizing as CO2 is increasing,...as well as you will have "quiet chest" bcz pt.'s breathing effort is decreased,..so you will not hear wheezes as much...that is a RED FLAG, as that means the pt. is getting worse...do not get full into thinking pt. is having his bronchocontsriction resolving...that is the whole pt. they are STILL bronchoconstricted but muscle fatigued to keep breathing...this mech. is strictly moreof a mechanical mech. , not a drive mech ( meaning brain stem stimulation problem as it might be in COPD if you give them too much oxygen)... here is quote from Cecil's essentials of medicine, " the ABG analysis in pt's w/ mild attacks or early in the course of a severe attack shows hypoxemia ( a widened A-a gradiant) and hyperventilation ( a decreased PaCO2). With INCREASING SEVERITY or RESPIRATORY MUSCLE FATIGUE , the PCO2 RETURNS to NORMAL and ultimately begins to rise.A rising PaCo2 in a pt. w/ asthma is an ominous sign and may portend a medical emergency. these pt's require continued direct observation and monitoring, and may require mechanical ventilation..." Cecil'sMore on the subject:The 4 stages of blood gas progression in persons with status asthmaticus are as follows: + First Stage: hyperventilation to maintain normal PO2.+ Second stage: hyperventilation accompanied by hypoxemia.+ Third stage: Moderate = Unobstructed airways (called Fast Compartment) normally compensate. As the obstructed compartement (called the slow compartment) increases, pCO2 increases resulting in false-normal value for PCO2 and pH. This is one of the indications of admission to the intensive care unit (ICU), and, probably, intubation with mechanical ventilation.

+ Fourth Stage: Severe : Slow compartment expands further and therefore removal of pCO2 decreases. pCO2 rising as well as Low PO2 (due to increased airway obstruction and atelectasis) will result in hypoxemic respiratory acidosis. This is an even more dangerous sign that mandates intubation and ventilatory support. C- Management: Patient sitting up- Pulse Ox Monitoring, ABG's, Potassium monitoring (as hypokalemia is a frequent side effect of the anti-asthma medication)- FEV1 baseline and monitoring- Hypoxemia is most common cause of death. Therefore OXYGEN IS PRIMARY TREATMENT OF PATIENT. Warm humidified 100% via a non rebreather mask. - Start 2 IV lines. One for CORTICOSTEROIDS- COMBINATION OF IPRATROPIUM AND ALBUTEROL nebulizer- THEOPHYLLINE +/- increased diaphragm function and CNS breathing stimulation. MONITOR LEVELS, NOT INDICATIED IN GER-INDUCED ASTHMA AS IT MAY EXACERBATE. - If PNT: CXR and Thoracocentesis (Other cause of silent chest)- If GER-INDUCED, ANTI-REFLUX MEDS/H2 RECEPTOR BLOCKERS- INTUBATE IF: Apnea, Decreased consciousness, Continued increase in pCO2 or hypoxemia despite treatment - Caution: if ASTHMA AND NASAL POPYPS: DO NOT GIVE ASPIRIN. Sources: - Swanson- Emedicine- CMDTApproach to milk-like discharge32 yr old woman has noticed milklike discharge from her breasts the past one month. exam otherwise is normal. how should you approach this caseDo pregnancy test first. Complete approach is: First of all, assess careful history taking and physical examination:- Gynecological history: menses, infertility, h/o of excessive nipple stimulation- History of headache and visual disturbances- History of prior irradiation to the head, h/o encephalitis, h/o of head injury- History of medication intake which would provoke hyperprolactinemia- History of illicit drugs: canabis, opioids, amphetamines, #1: Assess if it is GALACTORRHEA (microscopic examination). - If non galactorrhea: consider intraductal breast cancer, fibrocystic breast disease, etc.- If Galactorrhea (Fat globules in discharge) => Labs #2: Labs: - hCG Level- PRL level- TSH level- BUN and Creatinine +> High TSH and Low T4: Thyroid hormone replacement therapy+> Normal TSH, High PRL: MRI of BRain+> Normal TSH and Normal PRL: Regular menses?

....If regular menses: Reassurance/Observe, D/c offending drug if applicable

....If Irregular menses: MRI of BrainIF PROLACTINOMA: BROMOCRIPTINE, CABERGOLINE (DOC OF CHOICE IF NOT WISHING TO CONCEIVE), SURGERY, OR RADIATION THERAPY. Reference: http://www.aafp.org/afp/20010501/1763.html AnaphylaxisUsually a typical case of anaphylaxis...Management includes the following: - IM or SQ Acqueous Epinephrine (Repeat Q15-30min PRN)- IVF Rapid infusion (LR, Saline, Volume expanders)- Vasopressors if remains hypotensive- Inhaled Terbutaline or Albuterol if severe bronchospasm- Antihistamine as an adjuvant. MONITOR FOR 24h because of Late-phase Anaphylaxis (Recrudescence 6-12h later after initial improvement). 1) What if patient is on chronic beta blocker treatment for some reason (Refractoriness to adrenergic drugs)? What changes in the management? 2) What if the patient is on chronic ACE Inhibitor Treatment (More Severe Hypotension)? What changes in the management? Ans: 1) Give more higher doses of adrenergics plus Glucagon 2) Higher doses of pressorsReview: ATOPIC Dermatitis Vs Allergic D/Irritant DI was always confused about the four of them. I looked them up for my own review so here goes. A- ATOPIC DERMATITIS (ATOPIC IS DIFFERENT FROM ALLERGIC):

1) Hallmarks: PRURITUS. h/o of dry skin, h/o of Atopy, ERUPTION IS SYMETRIC, ON BENDS OF ELBOWS AND KNEES. 2)Other: .......Acute (Itchy Erosions + serous exudate, Vesicles on erythematous base).......Subacute (Scaling excoriated plaques)....... Chronic (lichenification/Pigmentary changes with excoriated papules) 2)In infants: cheeks, forehead, sometimes diaper, flexor areas of extremities.Prevention is mainstay of treatment. B- ALLERGIC CONTACT DERMATITIS: 1)Hallmarks: 1-3 days after exposure (TYPE IV HYPERSENSITIVITY), Not THAT itchy, Red, followed by vesicle/blotchy eruption weeping and crusting, ASYMETRIC, around area of contact (e.g.: Poison Ivy, Poison Oak, Dyes, Nickel, etc.). 2)Variants: .......IRRITANT CONTACT DERMATITIS: Mostly Red and scaly. Rarely with weeping and crusting (Contact with soaps, solvents, acids, dyes, etc.).......CONTACT URTICARIA: TYPE I HYPERSENSITIVITY!!!!=Wheal and Flare at contact area.......PHOTODERMATITIS: Sunexposed areas C- Main Differential for both: 1)Impetigo or Secondary infection (i.e.:impetiginization). Gram stain will r/o it out.

Impetigo = honey crusted lesions2)Seborrheic Dermatitis: Dry scales and underlying erythema. Distribution: Scalp, Central face, presternal, interscapular, umbilicus, and body folds. References: CMDT, Kaplan Center notes, EmedicineQuestionAn 8-month-old infant is brought to your office by his mother for assessment of a diaper rash. His mother has tried cornstarch, talcum powder, vitamin E cream, zinc oxide, and three different prescribed corticosteroid creams from three different physicians as remedies. She tells you that she went to three doctors because the first two said, "Oh, don't worry dear, just a little diaper rash, it will go away; don't worry your pretty little head about it." On examination, the infant has an intensely erythematous diaper dermatitis that has a scalloped border and a sharply demarcated edge. There are numerous "satellite lesions" present on the lower abdomen and thighs. What is the most likely diagnosis in this infant? A- atopic dermatitisB- allergic contact dermatitisC- seborrheic dermatitisD- infectious eczematoid dermatitisE- candidal diaper dermatitisAns: E - Candidal Diaper DermatitisCompartment syndrome as a reperfusion injury: MechanismReperfusion syndrome is till being investigated as to where do the free radicals exactly come from. Some speculate xanthine oxidase, and allopurinol has been shown to reduce reperfusion effects experimentally. Some say leukocytes through the NADPH oxidase. Anyways, the result is the release of those toxic agents built up during the short ischemic episode creates increased permeability locally and significant edema which leads to increased pressure in the concerned compartment.One of the interesting questions to study about compartment syndrome is the abdominal one. 1 day after a major gut surgery for bypass, or trauma, where ischemia took place, oliguria starts which is refractive to fluid resuscitation. In fact, the more fluid you give, the more dangerous it is. Take back to the OR, and you'll find leakage of fluid (significant amounts) in the peritoneum. Drain, and DON'T CLOSE. Close using a "plastic-like" medium called silex I believ or something. PAtient will improve dramatically. You can take back to OR 2 days later and close abdomen correctly. PAtient would be discharged a couple of weeks later if everything goes well. QuestionA 34-year-old male is brought to the Emergency Department by paramedics after having collapsed in a marathon. Apparently, at approximately the eighteenth mile he fell to the ground in an unconscious state. On physical examination, his blood pressure is 90/60 mm Hg. His pulse is 128 bpm. The patient's temperature is 41° C. Which of the following statements about this patient is (are) true? A- this patient has heatstrokeB- hepatic and renal abnormalities are common in this condition

C- treatment should be directed at lowering the core temperature as quickly as possibleD- all of the above statements are trueE- none of the above statements is trueAns: D- all of the above statements are trueQuick Note on TPN - High YieldComplications of TPN: Central Vein nutritional support occur in up to 50% of patients. They can be divided into: CATHETER RELATED, AND METABOLIC. CATHETER RELATED: - Pneumothorax/Hemothorax- Arterial LAceration- Air Emboli ++++++- Brachial Plexus Injury- Catheter Thrombosis +++/ Catheter-related Sepsis +++Patient with TPN using indwelling catheter, who develops fever without apparent source => Change line immediately, and culture tip of old one. METABOLIC: Exam related are in my opinion- Hyperglycemia: Caused by too rapid an infusion of dextrose, or use of steroids. Reduce infusion rate, add insulin if needed. - Hyperchloremic nonketotic dehydration: reduce chloride. - Azotemia (Creatinine normal): Reduce protein- Acalculous Cholecystitis: mostly from biliary stasis. Give fat orally if possible- Zinc Deficiency: +++++ PATIENT DEVELOPS RASH, CHANGE OF TASTE, AND HAIR LOSS. Check for possible diarreha or bowel fitula. Increase Zinc intake.- Magnesium: muscle weakness and tremor, Decreased DTR and respiration. - Other: Copper, Selenium (cardiomyopathy) Source: CMDThttp://www.gray-ink.com/quillen/gi.htmlCases of Gunshot wounds and Facts of managementA- ABDOMINAL GUNSHOT WOUND19yo gan member is shot in the abdomen with a .38 caliber revolver. The entry wound is in the epigastrium, to the left of the midline. The bullet is lodged in the psaos muscle on the right. He is hemodynamically stable, and the abdomen is moderately tender. Which of the ffg is most appropraite next step in management? A- Close ObservationB- Emergency USC- CT abdomenD- Diagnostic Peritoneal LavageE- Exploratory Laparotomy Ans: E. The abdomen is full of important strudtures that should not be penetrated, solid organs that can bleed, and hollow viscera that will spill fluids in to the peritoneum. The rule of abdominal gunshot wounds is simple: They belong to the OR before any sign of peritonitis starts. B- GUNSHOT WOUNDS TO THE EXTREMITIES: A 25 yo man is shot with a .22 caliber reveolver. The entrance wound is in the anterior, lateral aspect of his thigh, and the bullet is seen on X-Ray films to be embedded in the

muscles posterolateral to the femur. The ER departement MD cleans the wound thoroughly. Which of the following is the most appropriate next step in management? A- Tetanus prophylaxisB- Doppler StudiesC- ArteriogramD- Surgical Explration of the femoral vesselsE- Surgical Removal of the embedded bullet. Ans: A. Tetanus prophylaxis is first. In wounds of the extremities, the main concern is the possibility of major vascular injuries. They can be evaluated by Doppler, arteriogram, or surgical exploration. But one should know the femoral artery is located anteromedial in the upper thigh, and eventually becomes central when it becomes the popliteal. It is NOT located in the lateral aspect where the bullet is located here. Removing the bullet, although obligatory in movies, is not necessary if it's not threatening to erode some vital structure. A 25 yo man is shot with a .22 caliber revolver. The entrance wound is in the anteromedial aspect of the upper thigh, 5 cm below the groin crease. The exit wound is in the posterolateral aspect of the thigh, half way between the great trochanter and the knee. He has palpable pulses in the dorsum of his foot and in the posterior tibial artery behind the malleolus. The popliteal pulse is reported normal by one examiner, but canno be felt by another. There is no hematoma under the entrance wound, and blood is oozing from both wounds, but not at an alarming rate. He is hemodynamically stable. Neurologic examination of the leg is normal. X-ray films show the femur to be intact. In addition to local wound care and the appropriate tetanus prophylaxis, which of the following is the most appropriate next step in management?A- Discharge homeB- Digital exploration of the wounds in the EDC- Admit to observe for development of complicationsD- ArteriogramE- Formal surgical exploration of the area in the OR Ans: D. Arteriogram. Anatomic proximity to major vessels is the main criterion to suspect vascular injury in gunshot wounds of the extremities. Although absent pulses and an expanding hematoma make such injury virtually certain (and dictate the need for surgical exploration), the presence of normal pulses and the absence of a hematoma does NOT rule out vascular injury. Only an arteriogram can provide the necessary reassurance.

C- GUNSHOT WOUNDS TO THE NECK: A young man is short in the upper part of the neck with a .22 caliber revolver. Inspection of thew entrance and exit wounds indicates that the trajectory of the bullet is all above the levl of the angle of the mandible, but below the skull. He is fully conscious, and neurlogically stable. A steady trickle of blood flows from both wounds, and it does not seem to responds to local pressure. He is again hemodynamically stable. Which of the following is the most appropraite next step in diagnosis? A- Continued Clinical observationB- Barium SwallowC- ArteriogramD- Endoscopy

E- Surgical exploration Ans: C. In gunshot wounds og the upper part of the neck, the main concern is the possibility of significant vascular injuries. The area is too high to involve the aerodigestive tract, and it is also rather difficult to explore surgically. Arteriograms offer the best way to assess the extent of the injuries, and also provides a way for embolization of major arteries that might be bleeding significatnly. A 25-year-old African American man arrived at the emergency room approximately 30 minutes after sustaining a single gunshot wound to the left posterior cervical region. His chief complaints were neck pain and the inability to move any extremity. Systolic blood pressure before his arrival was reported to be 90 mm Hg (palpatory). Neurologic examination showed intact cranial nerve (II through XII) functions, with flaccid paralysis of all four extremities except for bilateral forearm flexion, which was possible against gravity. Sensory examination was intact for pin prick and light touch throughout. Deep tendon reflexes were absent bilaterally. Rectal sphincter tone was decreased, with preserved bulbocavernosus reflex. The bullet entrance wound was in the left posterior cervical region, above and medial to the left scapula. The exit wound was midline in the anterior aspect of the neck. A bullet fragment was palpable in the left anterior aspect of the neck, lateral to the cricoid cartilage. A large left anterior neck hematoma was present. Both carotid pulses were palpable, and there were no carotid bruits. What is the next step in management? Ans: Surgery. Findings on the initial workup were consistent with a zone II neck injury (see below), resulting in hypotension and neck hematoma as well as cervical spine injury with an incomplete motor deficit at the C5 level. This is an absolute indication for neck exploration. He was taken immediately to the operating room. Surgical exploration of the left anterior cervical region revealed an extensive hematoma and a lacerated left external jugular vein, which was ligated. The hyoid bone was fractured and repaired. D- GUNSHOT WOUNDS TO THE CHEST: A 27 yo man is shot point blank with a .22 caliber revolver. The entrance wound is in the anterior chest wall, just to the left of the sternal border, at the level of the 4th intercostal space. There is no exit wound. He is diaphoretic, cold, shivering, and anxious, and is asking for a blanket and a drink of water. His blood pressure is 65/40 mmHg, and his pulse is 145/min, and barely perceptible. He has large, distended veins in his neck and forehead. He is breathing adequately and has bilateral breath sounds. He is neurlogically intact. Which of the folloing is the most likely diagnosis?A- Extrinsic cardiogenic shock due to pericardial tamponadeB- Extrinsic Cardiogenic shock due to tension pneumothoraxC- Hemorrhagic shockD- Intrinsic cardiogenic shock due to Myocardial injuryE- Vasomotor Shock Ans: A. Obviously, patient is in shock and the distended veins identify the type as cardiogenic. Given the location of the injury, pericardial tamponade is the obvious mechanism. A 19yo gang member is shot once with a .38 caliber revolver. The entry wound in the left mid-clavicular line, two inches below the nipple. The bullet is lodged in the left

paraspinal muscles. He is hemodynamically stable, but he is drunk and combative. PE is diffult to do. What is the next step in management? (no choices)

Ans: CXR (chest tube if needed) which is the usual for a chest injury, in addition to Exploratory laparotomy which is the usual for an abdominal injury. The point is to remind of the boundaries of the abodmen. Although it sounds like a chest wound, it is also abdominal. The belly begins AT the nipple line. The chest does NOT END AT THE NIPPLE line though. Belly and chest are stacked up and separated by a dome. E- SOME FACTS AND REVIEW 1- The Wounding Capacity of a BulletIt is related to the following factors: Kinetic Energy, Angle at impact, Bullet type, and Tissue Density. The consequences are generally: +> Laceration and Crushing: projectile displacing the tissues in its track. They are recognized as the primary wounding mechanism produced by handguns. +> Shock Waves: compression of tissues that lay ahead of the bullet. Produced by high velocity projectiles (not from handguns). +> Cavitation (permanent and temporary): also from even higher velocity projectiles. When a missile enters the body, the kinetic energy imparted on the surrounding tissues forces them forward and radially producing a temporary cavity or temporary displacement of tissues. The temporary cavity may be considerably larger than the diameter of the bullet, and rarely lasts longer than a few milliseconds before collapsing into the permanent cavity or wound (bullet) track. 2- Classification of Gunshot Wounds + Penetrating : Missile is retained in tissue, Entry wound is typically small and ragged + Perforating: Missiles pass completely through the target, Entry wound is comparable to size of missile, and Exit wound is often considerably larger + Avulsive: Small entrance comparable to missile size Exit wound is usually gaping with large amount of tissue loss. 3- Treatment of gunshot injuriesPrimary - resuscitative efforts as well as establishment of airway and restoration of hemodynamics Secondary - depends on the location. NECK TRAUMA: A significant number of patients of neck trauma die at the scene and others on the way to hospital. Patients arriving shocked and bleeding are resuscitated and usually undergo urgent surgery with repair or ligation of the bleeding vessels. Progressive neurological deficit calls for rapid evaluation and management. Stable patients, on the other hand, can be subjected to whatever diagnostic modalities are required to diagnose the extent of injury accurately. Neck wounds are classified as zones I, II and III. +> Zone I injuries are those that are below the cricoid cartilag+> zone II are above the cricoid cartelage but below the angle of the mandible+> zone III are above the angle of the mandible extending to the base of the skull. Most institutions explore zone II injuries routinely. In zone I and III more definite evidence of injury is required, which can be obtained by endoscopy as well as

angiography before exploration is embarked upon because of the difficulty in gaining access in zones I and III. GUNSHOT WOUND TO THE SPINE: Gunshot wounds (GSW) are the 3rd most common cause of traumatic spinal cord injuries in the U.S. civilian population. The injury from a gunshot wound could be either direct or indirect. Indirect injury results from shock waves or secondary fragments damaging the neural elements. Direct injury is a consequence of the projectile crossing the spinal cord and/or canal causing compression, contusion, or laceration of the spinal cord/ nerve roots, with or without laceration of the dura.Overall most studies in the literature recommend a conservative (non-surgical) approach to GSWs to the spine, as surgery has not been shown to improve much the neurological status, but some authorities beleive that removing the bullet still gives the best chance to recovery. Indeed, it is still highly controversial. The firm indications for surgical intervention are usually:+> Progressive neurological deficits+> Persistent cerebrospinal (CSF) leaks+> Incomplete neurological deficits with radiographic evidence of neural compression (especially in the cervical spine and cauda equina). Reference: www.rcsed.ac.uk/journal/vol43_2/4320019.htm www.uic.edu/depts/doms/rounds-6.html http://www.medschool.lsumc.edu/Nsurgery/GSWSp.htmlhttp://www.medscape.com/viewarticle/410823_4Cases are from Kaplan Q book and Kaplan Surgery NotesHIV Associated NephropathyWith HIV being such a hot topic on the USMLE, I post high-yield facts I encounter. Please refer to the post: ODD HIV case posted on July18 for HIV and Syphilis association. HIV associated Nephropathy Pearls (HIVAN) - Mostly African Americans (So and so, that in the absence of renal biopsy, and if the patient is white, think of other diagnoses) - Patient may present with Uremic Encephalopathy with confusion, new-onset seizure (Management of seizure with subsequent treatment using phenytoin), asterixis etc. - Labs: Uremia, massive proteinuria in the nephrotic range with little to ne edema - Order Ultrasound: which in contrast with usual shrunken small kidneys of End-Stage Renal Disease, in HIVAN, kidneys are ENLARGED!! - Confirm with Renal Biopsy: FOCAL SEGMENTAL GLOMERULAR SCLEROSIS is the most typical one. Management: Antiretroviral Therapy is the most important feature. It not only slows the progression but may in fact reverse it. Hemodialysis should also be started. Of note: Encephalopathy is a sign of rapidly changing nephro status either deteriorating ir improving (encephalopathy on first hemodialysis = Dialysis dysequilibrium Syndrome). ACE-I also have been used successfully to further slow progression. Corticosteroids have also been used but with caution because of the advanced stage of AIDS already for the patient.

Differential: Hep B and Hep C associated nephropathy, Heroin Associated Nephropathy. Reference: - Medscape (posted from the AIDS reader)New Onset Seizures as an Initial Presentation of End-Stage Renal Failure in Patients With HIV/AIDSToyin F. Olatinwo, MD; Ross G. Hewitt, MD8/2002Mechanical Ventilation Lecture and Case ScenariosInadequate teaching is not confined to medical or nursing schools, but extends (sadly) to mainstream textbooks, where chapters on mechanical ventilation seem written more for the author's colleagues than for novices trying to learn the subject. Please find in the following my little rehashing of what I read about the subject. **************** MECHANICAL VENTILATION ************** The science of mechanical ventilation is to optimize pulmonary gas exchange; the art is to achieve this without damaging the lungs. FIRST LET'S DIFFERENTIATE:- Mechanical ventilation: the ventilator is active and the patient passive- Assisted Ventilation: the patient initiates and may or may not participate in the breath. Note: For general purposes, both will be referred to as Mechanical Ventilation throughout the chapter. Mechanical Ventilation could be throughout intubation (invasive) or a tight-fit mask (non-invasive). WHAT IS A VOLUME VENTILATOR?term used because you set a volume and the machine delivers that volume, at whatever pressure necessary (up to a limit). The ventilator blows that volume at a certain FLOW. Flow can be: - Volume targeted and pressure variable: set the parameter of the volume. It will be delivered with increasing pressure until the set volume is given. The problem is: barotrauma. - Volume variable and pressure targeted: set the parameter of the inspiratory pressure, tidal volume will flow within that range. The problem is: since delivery of a set volume is not guaranteed, gas exchange can vary, making dangerous hypercapnia or alkalosis possible. Indicated for conditions where risk of barotrauma can be instantly life-threatening. Example: ARDS. THE MECHANICAL RESPIRATORY CYCLE :- Air is pushed in under POSITIVE PRESSURE, to a degree far greater than the patient could deliver on his or her own; air delivered under positive pressure is physiologically distinct from spontaneous breathing, where air enters the lungs by virtue of a slight negative airway pressure. The concept of POSITIVE PRESSURE is that a baseline pressure is applied throughout the cycle to maintain alveolar recruitment. - Exhalation is passive, utilizing the recoil nature of the chest to let air be exhaled; passive exhalation is physiologically the same as during spontaneous breathing. WHAT'S THE DIFFERENCE BETWEEN CPAP AND PEEP?They virtually refer to the same thing. PEEP is the positive pressure you set for the end of expiration. CPAP refers to when inspiration starts, the pressure starts from the PEEP and on up. Continuing Positive pressure throughout the entire cycle instead of intermittently

at the end of expiration is called Contiued Positive Airway Pressure CPAP. WHEN DO YOU DECIDE THAT A PATIENT SHOULD BE INTUBATED? 1. Loss of gag/cough reflex e.g. head injury with GCS <8 (to prevent massive aspiration).

2. Airway obstruction: acute laryngeal edema – e.g. inhalation burn, Ludwig’s angina, epiglottitis. 3. Anticipated loss of control of the airway: anticipated laryngeal edema– e.g. neck trauma, acute stridor etc. Either indication must be based SOLELY on the clinical examination, although ABG's are often helpful to assure that mechanical ventilation is not necessary. WHEN DO YOU INDICATED MECHANICAL VENTILATION IN ADDITION TO INTUBATION? Take home message: - Apnea- Impairment of alveolar ventilation (assessed by PaCO2>50mmHg) and/or oxygenation (assessed by PaO2<50mmHg) are the only physiologic reasons for instituting mechanical ventilation. IN CHRONIC LUNG DISEASE, HIGH pCO2 IS SOMETIMES ACCEPTABLE BUT NOT CRITICALLY LOW pH (ph<7.1 is an indication to mechanical ventilation). Again, rule of thumb, correlate with clinical setting. Treat the patient not the numbers. Although mechanical ventilation can lead to better cardiac, renal, or cerebral function, the basic goal for its use must be to improve the PaO2 and/or the PaCO2 or TO REDUCE THE FiO2 OR THE MECHANICAL WORK needed to maintain blood gas values at an acceptable level. Official Criterias are: 1- Apnea 2- Impaired alveolar ventilation (as assessed by PaCO2) when accompanied by one or more of the following: a. Depressed mental status b. Increasing fatigue c. Reduced PaO2 that cannot otherwise be correctedd. Severely deranged pH that cannot otherwise be corrected (below 7.1 is considered an indication for mechanical ventilation)e. Compromise of upper airways (e.g., by secretions) 3- Low PaO2 (e.g., less than 60 mm Hg): a. that cannot be improved with an FIO2 less than 0.50, and b. that is causing symptoms or seriously impairing bodily function CLINICAL-BASED PROBLEM: WHICH (ONE OR MORE) OF THE FFG CASES SHOULD BE INTUBATED AND MECHANICALLY VENTILATED BASED ON THE ABOVE? a. A 50-year-old man is comatose from drug overdose. PaCO2 is 51 mm Hg, PaO2 is 76 mm Hg, and pH is 7.31 while breathing room air. b. A 29-year-old man is alert but in respiratory distress; he is breathing 42 times/min. PaCO2 is 38 mm Hg. pH is 7.42, and PaO2 is 47 mm Hg while breathing 60% oxygen through a face mask. c. A 61-year-old woman who has severe emphysema is alert but is in moderate

respiratory distress; her respiratory rate is 24/min. PaO2 is 75 mm Hg while breathing nasal oxygen at 2 L/min, PaCO2 is 59 mm Hg, and the pH is 7.37. Her chest x-ray is clear. d. A 29-year-old woman is suffering from diabetic ketoacidosis. Her pH is 7.10, PaCO2 is 26 mm Hg and PaO2 is 110 mm Hg while breathing room air. e. A 31-year-old drug addict responds briefly to the administration of Narcan (a narcotic antagonist) by opening her eyes and crying out and then lapses back into a state of semi-stupor. PaCO2 is 31 mm Hg. pH is 7.38, and PaO2 is 89 mm Hg while breathing nasal oxygen at 3 L/min. WHEN FACED WITH A BORDERLINE ABG AND POSSIBLE MECHANICAL VENTILATION, HOW DO YOU EVALUATE? It is essential to deduce what part of the respiratory apparatus is malfunctioning. 1- Is it failure to ventilate (is the PCO2 > 50mmHg), or failure to oxygenate (is the PO2 <50mmHg)? Remember that a low O2 is much more significant than a high PCO2, but is frequently easier to treat. 2- In essence the problem is one or more of the following: * The chest cage is not effective in guaranteeing adequate minute ventilation. * Air is not able to pass effectively from the upper to the lower airway – increased airway resistance. * Gas is unable to pass effectively from alveoli to capillaries – due to some obstruction in the interstitial space. * Ventilation is being wasted – alveoli are being ventilated but not perfused: dead space ventilation or more air than the blood can utilize (high ventilation/perfusion (V/Q) ratio). * Blood flow is inadequately utilized and blood is passing through the lungs without coming into contact with aerated alveoli: perfused but not ventilated – shunt or ventilation falls behind blood flow (low V/Q ratio). HOW DO WE INITIATE MECHANICAL VENTILATION?The ventilation strategy is determined by whether the patient has failure to ventilate or failure to oxygenate. The first problem is managed by increasing the patients minute ventilation, the second by recruiting collapsed lung units and controlling mean airway pressure. Every patient who is intubated is in need of a rest, and usually patients are started on controlled modes (see below Modes of ventilation).- If failure to ventilate or protect the airway was the problem, controlled volume ventilation is used, to correct the respiratory acidosis, being careful not to damage the lung (be mindfull of the pressures generated). - If failure to oxygenate is the problem, usually controlled pressure modes of ventilation are used, and carefully titrate the CPAP and the pressure control levels to set targets. While the choice of control mode is probably irrelevant (assist control (AC) or intermittent mandatory ventilation (IMV)), it is important that the patient’s spontaneous breaths are supported, which means adding pressure support to (S)IMV. HOW ARE MECHANICAL VENTILATORS CLASSIFIED?1) How the ventilator knows how much flow to deliver = CONTROL = Volume Controlled , Pressure Controlled, or Dual Controlled. 2) We determined how much flow and at what pressure. Now how long does it stay there? = CYCLING: how the ventilator switches from inspiration to expiration:

Time cycled, Flow cycled, OR Volume cycled. 3) What causes the ventilator to cycle to inspiration? = TRIGGERING = Ventilators may be time triggered, pressure triggered or flow triggered (see next note). One type of ventilators to be familiar with is: FLOW-BY = FLOW-TRIGGERED RESPIRATOR (The patient's own breath triggers the breath to be delivered at set standards of volume and pressure). 4) We determined the volume, pressure, time, trigger, Now the questions is how is that breath going to be delivered to the alveoli? = BREATHS = Mandatory (controlled = which is determined by the respiratory rate), Assisted (as in assist control, synchronized intermittent mandatory ventilation, pressure support), or Spontaneous (patient sucks up his/her own breath). 5) Very much linked to the precedent, ventilators got smarter now with MODES OF VENTILATION: +> CMV = Controlled Mandatory Ventilation. dOESN'T allow spontaneous breathing. Many anesthesia ventilators operate in this way.+> AC: Assist-Control = Allows the trigger of the breath, and the patient to make own effort but the flow/volume/pressure are controlled breaths.+> IMV: Intermittent Mandatory Ventilation = Patient initiate own breath and sucks up air, but also breath controlled by ventilator is delivered. The problem? "stacked breaths" where there is build-up of high-pressures and therefore alveolar stretching and damage. The solution? Breaths may also be synchronized to prevent "stacking". +> High Frequency Ventilation = where mean airway pressure is maintain constant and hundreds of tiny breaths are delivered per minute. WHY ARE THEY SO MANY DIFFERENT WAYS TO VENTILATE A PATIENT? - It all started in mid-1950's with the polio epidemic. Patients suffering with this virus die from asphyxia - respiratory muscle paralysis and failure to ventilate. Medical students were assigned to manually ventilate paralysis victims until restoration of neuromuscular activity occurred. "old ventilators" called "iron lungs" used to provide negative pressure about the rib-cage allowing sucking up air. But they were of little value since the disease was not inability to ventilate but that to oxygenate. - Then came the Pressure Controlled Ventilators. - During the 1970s and 1980s ventilators were developed which allowed patients breathe spontaneously, initially with assisted breaths (assist control ventilation) and subsequently with spontaneous breathing limbs – (synchronized) intermittent mandatory ventilation (SIMV). The latter was the first mode to allow partial ventilatory support and thus gradual liberation from the ventilator.- During the 1990s widespread concern developed about ventilator induced lung injury. Accumulating evidence revealed that larger tidal volume, low PEEP, ventilation strategies were damaging the lungs. This has led to the development of lung protective ventilator strategies (renewed interest in plateau pressure limitation and increasing mean airway pressures).- Dual modes, combining pressure limitation with guaranteed tidal volume, have been developed. Physicians are now demanding more control over gas flow than before - hence the development of active exhalation valves, dynamic inspiration valves, rise time control, automatic tube compensation and, of course, waveform analysis.- Modern ventilators deliver enhanced patient interactivity using better triggering sensors,

and more comfortable spontaneous breathing - even in inverse ratio ventilation. HOW DO YOU GO BY MECHANICAL VENTILATION SETTING....PRACTICALLY? A- Oxygenation. Arterial oxygen content should be maintained at 60 mm Hg or higher, or saturations at 90% or higher. Generally, initiate mechanical ventilation with an FiO2 of 100%, then taper 10% every 10 to 15 minutes to find the lowest FiO2 necessary to maintain adequate oxygenation. An FiO2 of greater than 60% for over 24 hours has been associated with lung injury. PEEP may be added to decrease the A-a gradient, allowing a lower FiO2 while maintaining oxygenation. For all practical purposes, a patient who is intubated mainly for hypercapnia will usually be adequately oxygenated with an FIO2 under 0.40. A patient intubated because of severe hypoxemia or during cardiopulmonary resuscitation may need an initial FIO2 of 1.00. Blood gas measurements should be obtained in the first half hour after treatment, and adjustments made to keep the PaO2 between 60 and 90 mm Hg at the lowest FIO2 possible. B- Respiratory rate and Ventilation (Measured by minute ventilation = tidal volume x respiratory rate, and is reflected in the PCO2). Increases in minute ventilation will cause a decrease of PCO2. Goals of ventilations should be to maintain a pH (as determined by PCO2 and underlying diseases) of 7.3-7.4. The respiratory rate is set by using a dial on the machine. - For controlled ventilation, the rate equals the total number of ventilator breaths the patient will receive. - For assist control ventilation, the rate represents the minimal number of breaths; depending on the inspiratory sensitivity (also set by the machine), the patient may initiate more than the minimal amount. - For intermittent mandatory ventilation, the respiratory rate is also the total number of ventilator breaths per minute; however, between the machine breaths, the patient may breathe spontaneously.

C- Permissive hypercapnia. In certain situations (e.g., ARDS) it may be permissible to allow the PCO2 to rise (permissive hypercapnia) to decrease injury from ventilation as long as the patient maintains hemodynamic stability and oxygenation. This has been shown to decrease mortality in some cases (e.g., ARDS). D- Minute ventilation is the product of tidal volume and rate; it is approximately 5 to 10 L/min or 100 ml/kg/min. E- Tidal volume (Vt) and Inspiratory Pressure limit. Initial volume is 8 to 10 ml/kg. A large Vt improves gas exchange and prevents atelectasis. However, it may decrease venous return higher volumes may increase risk of barotrauma. A smaller Vt may be required if PEEP is added. For patient for whic it is expected to have some sort of airway obstruction, the inspiratory pressure limit will protect patient from further complications. Example: setting the tidal volume for delivery of 700 cc might achieve a peak airway pressure of 30 cm H2O; a pressure limit of 50 cm H2O can be set at the same time. If, for example, the endotracheal tube slips into the patient's right main stem bronchus, the machine will attempt to deliver 700 cc to just one lung (half the previous lung volume), and the peak inspiratory pressure will acutely rise. Conceivably the elevated airway

pressure could rupture the right lung or cause other damage. Instead, however, when 50 cm H2O airway pressure is reached, the machine stops inspiration and an alarm sounds, perhaps after delivering only 400 cc. With this warning, the therapist or nurse can quickly investigate the problem. The alarm will sound each time airway pressure reaches the preset inspiratory pressure limit. F- Inspiratory time and flow. The peak inspiratory flow rate determines how fast each breath will be delivered to the patient and is therefore a determinant of inspiratory time. Adjust inspired flow rate to maintain a ratio of inhalation time (I:E ratio) to exhalation time of 1 to 1.5 in most patients. and is usually achieved with a peak inspiratory flow rate between 40 and 70 L/min.Patients with airway obstruction (asthma, COPD) may require additional time for exhalation. This can be accomplished by decreasing inspiratory time, or by decreasing respiratory rate. G- Positive end-expiratory pressure (PEEP) may increase compliance and decrease the work of breathing by preventing atelectasis, and thereby decreasing shunting. It is usually begun at 3 to 5 cm H2O and increased in small increments. High levels may result in decreased venous return and severe hemodynamic compromise. Other negative consequences include overventilation, barotraumas, and elevated intracranial pressure. Cardiac output should be measured if there is an indication of problems because it may increase or decrease with increased PEEP. H- Peak airway pressure reflects the pressure required to overcome airway resistance and is the peak pressure during the inspiratory cycle. The alarm limit should be set 10 cm H2O above this. If the peak inspiratory pressure increases, you need to consider obstruction in the ET tube, bronchospasm, decreased lung compliance, or a pneumothorax from barotrauma. * Sedation and neuromuscular paralysis allow the patient to rest, decrease anxiety, and ensure better compliance with the ventilator. However, periodic interruption of sedation (if tolerated) reduces the total number of days on a ventilator. Initial therapy includes midazolam, diazepam, lorazepam and propofol. Dosages should be titrated to desired effect, with monitoring of hemodynamic and respiratory status. * Neuromuscular paralysis is occasionally necessary if sedation fails, but patients should still be sedated. Monitoring alarms must be functioning because ventilator malfunction is rapidly fatal if the patient is paralyzed. Immediate, short-term paralysis (3 to 7 minutes) can be achieved with succinylcholine 1 mg/kg IV. For long- term paralysis use non-depolarizing agents such as pancuronium, vecuronium, or cis-atracurium. If repeated dosing or continuous drips are necessary, consider nerve-stimulation testing to avoid over-medication. Prolonged use of these agents, especially in continuous infusions, is associated with prolonged (days to months) muscle weakness and ventilatory dependence. Use of nerve-stimulators can decrease the dose of paralyzing agents while maintaining adequate control. If necessary, a neostigmine-atropine combination can be used to reverse the non-depolarizing agents. CLINICAL SCENARIO: A 60year old patient is in the hospital for treatment of a myocardial infarction. During the night she suffers acute pulmonary edema and requires cardiopulmonary resuscitation. Before the patient is intubated and mechanical ventilation is begun, her blood gas measurements show pH of 7.06, PaCO2 of 61 mm Hg, and PaO2 of 50 mm Hg while

breathing 100% oxygen delivered by manual ventilation with an Ambu bag. The patient's estimated body weight is 50 kg (110 Lbs). What initial ventilator settings would you choose for the following: a. FIO2 b. Tidal volume c. Inspiratory pressure limit d. Respiratory rate e. Peak inspiratory flow rate f. Would you provide PEEP? Answer:Inability to Oxygenate secondary to Alveolar exchange problem (V/Q mismatch: high perfusion low ventilatin)a. FIO2: 100%b. Tidal volume: 8-10cc/kg. W=55Kg. Choose higher and readjust per subsequent ABG's. Vt= 500cc/minc. Inspiratory pressure limit: empirically 40cmH2O since patient has pulomnary edema and is liekly to have airway obstruction and high airway pressure. d. Respiratory rate: Start with an empirical rate of 10-14/min. e. Peak inspiratory flow rat: 40-70 L/min to acheive I:E ratio of 1 to 1.5. f. Would you provide PEEP? Yes. Alveoles are full of fluid and will tend to collapse. To ensure continued alveolar recruitment, PEEP must be delivered. Lung compliance is adequate but FiO2 is 100% at first so, PEEPis 15-20 cmH2O and readjust once you decrease FiO2. A 72yearold man with severe chronic obstructive pulmonary disease is in the intensive care unit. His pH is 7.24, PaCO2 is 84 mm Hg, and PaO2 is 58 mm Hg while breathing 28% oxygen through a Venturi mask. His chest xray suggests severe emphysema. Despite optimal drug therapy, his blood gas measurements cannot be improved, and he is almost unarousable. To prevent respiratory arrest, he is intubated and given mechanical ventilation. His estimated body weight is 70 kg (150 lbs). What initial ventilator settings would you choose for the following: a. FIO2 b. Tidal volume c Inspiratory pressure limit d. Respiratory rate e. Peak inspiratory flow rate f. Would you provide PEEP?

Answer:a. FIO2: It is an oxygenation problem. FiO2 should be modest. 40% is a number to start with. b. Tidal volume: Modest as well, enough to correct acidosis 500 - 600cc/min with a RR of 10-14l/minc Inspiratory pressure limit: 50 is a good limit. d. Respiratory rate: 10-14 and readjuste. Peak inspiratory flow rate 60-70 L/min since it's a COPD guy and should be allowed more time for expiration.

f. Would you provide PEEP? Lung compliance is increased. 15-20 cmH2O still stands for this patient as well, to allow proper compliance to ventilatory oxygenation. A comatose 20year old patient is brought to the emergency room following an overdose of sleeping pills. Because of very shallow respirations and cyanosis, the patient is intubated before his blood gas results are known. Initial ventilator settings include a tidal volume (VT) of 700 cc, a respiratory rate (RR) of 12/min, and an FIO2 of 0.50. The patient has no spontaneous breathing. Blood gas results obtained (1) before intubation and (2) 20 minutes later show the following: pH---PaCO2---PaO2 FIO2 VT RR (1) 7.10 79 38 Room air 0 0 (2) 7.25 56 117 50% oxygen 700 12 Following the second blood gas analysis, would you change the FIO2, the tidal volume, or the respiratory rate'? If so, what settings would you choose? Decrease the FiO2 by 10% and redraw ABG's. Patient shows sings of improvement. No reason to change other settings.

WHAT ARE THE COMPLICATIONS OF MECHANICAL VENTILATION? A- VENTILATOR-ASSOCIATED PNEUMONIA: Continuous subglottic aspiration of secretions reduces the incidence of nosocomial pneumonia. A semirecumbent position in bed also will minimize the risk of ventilator-associated pneumonia. A bacteriologic diagnosis should be aggressively pursued in ventilator-associated pneumonia and will reduce mortality. B- Stress ulcer prophylaxis. Sucralfate, H2 blockers, and proton-pump inhibitors have all been shown to be effective. However, sucralfate may be associated with a lower rate of ventilator associated pneumonia. C- DVT prophylaxis. Heparin 5000 U SQ Q12h or LMW heparins (enox-aparin 40 mg SQ QD or 30 mg SQ Q12h) are preferred, unless contraindicated (coagulopathy, thrombocytopenia, active bleeding, recent or future surgery). Compression stockings and intermittent pneumatic devices (TEDS and Kendals) are also effective. WHEN DO YOU WEAN THE PATIENT OFF OF MECHANICAL VENTILATION? Guidelines for weaning from mechanical ventilation: - An awake, alert patient. - PO2 >60, with an FiO2 <50%. - PCO2 acceptable and a pH in normal range. - PEEP <8 cm H2O. - Minute ventilation less than 10 L/min. - Patient is able to generate maximum voluntary ventilation without retractions. - Patient is able to generate a peak negative inspiratory pressure of at least 20 cm H2O. HOW DO YOU ACHEIVE WEANING OF MECHANICAL VENTILATION?The most effective method of weaning to discontinuation is spontaneous breathing trials (SBT). Otherwise:1- Pressure-support method:Switch from an assisted mode of breathing to pressure support, setting pressures to generate Vt similar to the assisted volumes with a ventilation rate less than 20. Gradually decrease the inspiratory pressure until 8-10 cm H20 above expiratory pressure. If patient

can maintain adequate volumes with a ventilation rate of less than 20 for 30-60 minutes, consider extubation. 2- T-tube method:(A T-tube allows the patient to breathe through an endotracheal tube without assistance from the ventilator.) Have the patient use a T-tube with humidified oxygen. If the patient tolerates this for 1 to 4 hours without deterioration, discontinue mechanical ventilation. If the patient fails the attempt, resume mechanical ventilation and consider IMV method (below) for weaning. 3- IMV method. Gradually decrease the number of assisted respirations in 1 or 2 breath increments over 30- to 90-minute intervals. Monitor ABGs and vital signs. When an assisted rate of <4 breaths/min is achieved, consider a brief T-tube trial. If the patient remains stable, discontinue mechanical ventilation. If the trial fails, increase assisted rate until patient stabilizes. Repeat attempt the following day with a more gradual decrease in the rate of assisted breaths. HOW DO YOU KNOW WHEN TO D/C WEANING TRIALS AND RESUME MECHANICAL VENTILATION? When: - pH <7.3, PCO2 >50, PO2 <60. - The patient becomes anxious, fatigued, demonstrates increasing respiratory distress, or develops significant arrhythmias or hemodynamic deterioration. Clinical Scenario: A decision is made to wean a 67 year old man from the ventilator. Before weaning is begun, the machine is in the assist control (AC) mode. The patient is initiating 16 breaths/min and is receiving 700 cc/breath. He is switched to IMV at a rate of 12/min and within a half hour is noted to be in respiratory distress with a total respiratory rate (machine initiated plus spontaneous) of 20/min. Blood gas measurements obtained before and after the change to IMV are shown below. How would you explain the changes? Should you D/C the weaning? Assist control 16 (700 cc) 0 7.45 38 78 0.40 IMV 12 (700 cc) 8 7.39 47 65 0.40 Ans: Patient was in Assist Control with a controlled rate of 16 and a tidal volume of 700cc. Switching to IMV caused both hypercapnia and acidosis (inability to ventilate)=> Increase minute ventilation, and hypoxia (inability to oxygenate)=> Give PEEP and increase FiO2. No need to D/C weaning yet at this time, since the ABG's are not yet critical. This decision should be supported by more thant those numbers: What are the consequences of Weaning?- The single most traumatic event for the patient is conversion from positive pressure to negative pressure ventilation. - To extubated a patient, they need to be awake, able to cough and protect their airway.- Although the ventilator only appears to support on organ system, the lungs, this is not in fact the case. For a patient to self ventilate, many body systems must be functioning: the cardiopulmonary apparatus, the central nervous system, the nerves that supply the diaphragm (including the neuromuscular junctions), the muscles themselves. Moreover the patient must be willing to breath and maintain their own functional residual capacity (not if there is diaphragmatic splinting due to pain). There must be room in the abdomen

for the diaphragm and lungs to move into. There must be adequate hemoglobin to deliver oxygen to the tissues. - A reintubation rate of 10% is acceptable. Patients deserve a trial of extubation, and many will do well in spite of poor mechanics (you must use clinical judgment). CLINICAL SCENARIOS: I) A 22 year old male found collapsed in the street, pinpoint pupils, respiratory rate of 5 and a PCO2 of 70 mmHg, PO2 60mmHg Hints: Is it ventilatory failure or oxygenation failure (is the PCO2 > 50mmHg, is the PO2 <50mmHg). Remember that a low O2 is much more significant than a high PCO2, but is frequently easier to treat. If it is ventilatory failure, where is the injury – in the brain (the medulla), in the spinal cord, in the peripheral nerves, at the neuromuscular junction, in the muscle itself or in the chest cage? If the problem is oxygenation failure, where is the injury: is it in the blood supply, at the alveolar-capillary interface or in the upper, middle or lower airways? II- A 47 year old male with a two week history of upper respiratory tract infection is admitted with a history of bilateral lower limb weakness and shortness of breath. His forced vital capacity is 1 liter and his pCO2 is 70mmHg and pO2 60mmHg. III - A 74 year old female is admitted unconscious, GCS 3, Cheyne Stokes breathing pattern, in atrial fibrillation, BP 170/100mmHg, PCO2 70mmHg, PO2 60mmHg. IV- A 35 year old male with a history of asthma complains of acute severe left sided chest pain, and becomes acutely dyspneic: PCO2 is 47mmHg, PO2 49mmHg. V- A 54 year old female, 36 hours post total abdominal hysterectomy, becomes confused and hypotensive – PO2 is 45mmHg, PCO2 is 29mmHg. VI- A 16 year old female presents with a two month history of severe fatigue, exercise intolerance, shortness of breath and weight loss. She is admitted through the ER in extremis, with a PCO2 of 70mmHg and a PO2 of 50mmHg. VII- A 73 year old male is discharged from the intensive care unit, following a three week admission for sepsis following a perforated appendix. He has been short of breath all evening. Now he is severely distressed and his chest is moving up and down in a seesaw manner. This patient had a collapsed right lung on chest x-ray. VIII- A 67 year old male is admitted with an acute asthmatic attack. 4 hours after admission you are called because he is hypoxemic, finds it difficult to get air in, his chest is hyperinflated and he is becoming hypercarbic.

ANSWERS: I) This man has ventilatory failure, as you can see from his high CO2. He is also somewhat hypoxemic, which is not surprising, as CO2 will displace O2 from the alveolus when it builds up (we know this from the alveolar gas equation: PAO2 = PiO2 – PaCO2/R). The combination of meiosis and bradypnea immediately suggests narcosis, which can be reversed, at least temporarily, with naloxone. The mechanism of his respiratory failure is thus loss of respiratory drive due to opioids reducing the sensitivity of the respiratory center to carbon dioxide. Intubate and ventilate in controlled mode. Start with RR 10-14, and a tidal volume 7-10cc/kg. FiO2 of 100% with CPAP, then taper 10% every 10 to 15 minutes to find the lowest FiO2 necessary to maintain adequate oxygenation. Inspiratory flow rate; generally set between 60 and 100 l/min (the faster the

flow rate, the quicker inspiration and the longer the patient has to exhale). II- Patient has high pCO2 = Inability to Ventilate. This patient has ventilatory failure, as evidenced by his inability to clear carbon dioxide. His diagnosis turns out to be Guillain-Barre syndrome, which is characterized by motor, sensory and autonomic neural demyelination and thus neuropathy, which usually eventually reverses. The low FVC is a sign of poor physiological reserve, and this patient requires controlled mechanical ventilation. Oxygenation is OK, so FiO2 should be modest and readjusted until stabilization of O2 Sat and ABG's. Tidal volume 8-10cc/kg. Add PEEP to allow lower FiO2. III- This patient is failing to ventilate and failing to protect her airway. A comatose patient with this breathing pattern is a brain stem stroke until otherwise proven. The cause is either a bleed (hypertension) or an embolus (atrial fibrillation). Mechanical ventilation in this circumstance is invariably futile, yet indicated if patient is full code. IV- This patient is acutely hypoxemic secondary to a barotrauma. He is not adequately clearing carbon dioxide, which should be much lower in view of the degree of hypoxemia. This suggests that there is a shunt present. The major concern here is that the patient has had an acute pneumothorax and there is a loss of hypoxic pulmonary vasoconstriction. He requires urgent placement of a chest tube. Intubation is not necessary at this stage. Pneumothoraces are relatively common in young asthmatics. The main problem is an inability to ventilate (high pCO2) with an associated problem: inability to oxygenate (high pO2) due to shunt. V- This is, in many ways the opposite to case 4. The patient is hypotensive, hypoxemic and hypocarbic after pelvic surgery. The most likely diagnosis is a massive pulmonary embolism from the pelvic veins. The problem is failure of oxygenation due to a massive amount of wasted ventilation (dead space ventilation), due to obstruction of blood flow. In view of her hypoxemia, she should be intubated in IMV mode since patient can breathe on her own, along with Heparin therapy and other measures for PE management. VI- This patient has a ventilatory abnormality. She presents with a combination of fatigue, and the inability to clear CO2, suggesting muscle weakness. The diagnosis is Myasthenia Gravis, which is characterized by anti-acetylcholine receptor antibodies, and thus effective neuromuscular blockade. Mechanical Ventilation on IMV mode with increased minute ventilation, increased FiO2, PEEP. VII- The combination of recent discharge from ICU, which indicates potentially a recent extubation, and paradoxical breathing, is strongly suggestive of upper or middle airway obstruction. The cause may be in the oropharynx (tongue or dentures obstructing breathing), the larynx (laryngeal edema or stenosis) or below the larynx. One thing to strongly consider is accumulation of secretions or inspissation of mucus, due to ineffective bronchial toilet (the patient probably has a very poor cough). Intubate patient for pulmonary toilet and removal of impacted secretions. VIII- Failure to Ventilate due to outflow obstruction: the patient is attempting to ventilate at high lung volumes where the lungs are least compliant.This patient has severe outflow obstruction and gas trapping. Due to the high resistance to ventilation, air is slow to exit the lungs, and the patient feels uncomfortable, he attempts to actively exhale and this causes dynamic airways collapse, causing further airway closure. Some airways may remain closed during the entire ventilatory cycle, and oxygen is not replenished – and there is a ventilation-perfusion mismatch. This man is

exhibiting signs of acute gas trapping (auto PEEP) and hypercarbia, indicating worrisome loss of physiological reserve. He needs to be intubated and PEEP applied to his airway in excess of the auto-peep generated: he should be treated with pressure support ventilation: this mode provides limitless flow to match the patient's demands.

References:- http://www.ccmtutorials.com/rs/mv/- http://www.ccmtutorials.com/scenarios/intvent/index.htm- http://www.vh.org/adult/provider/familymedicine/FPHandbook/Chapter04/03-4.html- http://www.mtsinai.org/pulmonary/books/physiology/chap10a.htm#introductionQuestion30 yrs old hiv + man with fever, cough, cxr middle lobe infiltration & rusty sputum . what is the cause ?a. chlamydia pnemoniaeb.mycoplasma pneumoniaec.strep pneumoniaed.pcpe. aspergillusAns: Rusty sputum = Strep Pneumonia. Small vessels rupturing and combined with phlegm gives the rusty look. Patient is HIV+ Not AIDS patient. When the patient has less than 200 od CD4, we start talking about PCP. Otherwise they still have the same epidemiology as HIV- people. 20 Y/O GIRL WHO IS HIV + DELIVERED A BABY.WHEN DO WE GIVE AZT PROPHYLAXIS?1.BABY NOT TESTED FOR HIV ANTIBODIES OR VIRUS(PCR)2.BABY NOT TESTED FOR HIV ANTIBODIES AND CAME +VE PCR -VE.3. BABY NOT TESTED FOR HIV ANTIBODIES AND CAME +VE PCR NOT DONE ?THANKSAns: Upon checking more specific references: http://www.hivdent.org/pediatrics/reduce/ch4.html My answer is: All three should have prophylaxis irrespectice of PCR. Here's the reasoning: Using DNA PCR, 25% to 30% of infected infants may be identified at birth and the remaining 70% to 75% of infected infants can be identified by one month of age. According to the guidelines, the evaluation of the infants infection status should begin within 48 hours of birth, with repeated evaluations at one to two weeks and at one, two, and six months (CDC, 1998c).Q: For the child,should AZT be given for 6 weeks or 6 months.In kaplan CD it says 6 months.Please clarify.For the mother is it oral or IV AZT.?A: It is 6wks of AZT irrespective of PCR(antibodies anyway can be from the mother and not of any diagnostic value),to be followed by PCP prophylaxisQ: to immunize a preterm baby,,should we administer vaccines based on the birth date? for example in a preterm 8 month(33-34 wk)baby,,should we give the first DTaP at 2

month after birth or 3 months after birth?A: Immunize as he/she were term. Reference: - Kaplan notes 2002why maternal hyperpara causes hypocalcemia in neonate??First answer to your question: Apparently the elevated maternal calcium levels suppress fetal production of parathyroid hormone until delivery occurs. Following delivery, the suppressed neonatal parathyroid hormone and the abrupt halt of maternal calcium produce hypocalcemia.12 The mechanism is analogous to the problem of neonatal hypoglycemia in newborns of mothers with diabetes. Quick Review on Hyperparathyroidism: What is it?Elevated Parathormone. The effects of parathyroid hormone on serum calcium are mediated by :- Increasing renal tubular resorption of calcium- Increasing calcium absorption from the intestines (via vitamin D)- Increasing release of calcium from bone. A negative feedback mechanism normally decreases production of parathyroid hormone as the ionized serum calcium level increases. What's a normal Calcium level?4.5-5.5 mEq/L9-11mg/dLor 2.23-2.57 mmol/LDisease states with altered protein binding require a correction factor to determine the ionized calcium level from the total calcium level. A low measured calcium with low albumin may be corrected by adding 0.8 mg per dL (0.2 mmol per L) of calcium for each gram per dL that the albumin is below 4 g per dL (1 mmol per L). Alternatively, the serum ionized calcium level can be measured directly. Normal values are: 2.2-2.5 mEq/L, 4.25-5.25 mg/dL, or 1.15-1.30 mmol/L. What causes it? Common causes of hypercalcemia 1- Hyperparathyroidism, including ectopic hyperparathyroidism 2- Ectopic PTH: Humoral hypercalcemia of malignancy via parathyroid-related protein (especially malignancy of the lung, kidney, ovary, head and neck, and esophagus) 3- Renal failure 4- Malignancy via direct bone destruction (especially myeloma, lymphoma and metastatic breast cancer) 5- Thiazide diuretics Uncommon causes of hypercalcemia 1- Immobilization 2- Lithium use 3- Vitamin D toxicity 4- Hyperthyroidism 5- Milk alkali syndrome 6- Multiple endocrine adenomatosis syndromes

7- Granulomatous diseases, especially sarcoidosis, via increased levels of 1,25(OH)2D3 8- Familial hypocalciuric hypercalcemia How do you diagnose it? ALGORITHM: * ELEVATED CALICUM LEVEL = 1st step: RECHECK CALCIUM* If still elevated, look for FAMILY HISTORY 1- Fam/h positive: URINE CALICUM LEVEL + If urine Calicum is low: FAMILIAL HYPOCALCIURIC HYPERCALCEMIA+ If urine Calcium is high: CHECK FOR MEDICATION INTAKE: THIAZIDE, MILK-ALKALI (ANTACIDS), LITHIUM. If positive => STOP MEDS AND RECHECK CALCIUM. 2- If no meds => CHEM PANEL: Hyperparathyroidism suggested by the following: - Calcium elevated but still less than 14.5mg/dl (normal 9-11)- Chloride > 102mEq/dl- Low phosphate- Chloride/Phosphate ratio > 33 is suggestive- Mild acidosis If Hyperparathyroidism suspected on electrolytes => PTH LEVEL. 3- If PTH level elevated: PRIMARY HYPERPARATHYROIDISM 4- If PTH level low or normal: Check 1,25(OH)2D3 and PTH-related peptide + if High 1,25(OH)2D3 and low PTH-related peptide: GRANULOMATOUS DISEASE+ if Low 1,25(OH)2D3 and high PTH-related peptide: HUMORAL HYPERCALCEMIA OF MALIGNANCY. How does it manifest in an individual?- Depression- Different ailments: myalgis, tingling- Constipation, Nausea - Cardiovascular complications - Osteoporosis- Polyuria and nephrolithiasis- Parathyroid storm or crisis: Nausea and vomiting, together with polyuria and profound dehydration, Hypercalcemia and cardiac arrhythmias, CHF and HTN. What if it happens in pregnancy? Maternal hyperparathyroidism can lead to profound hypocalcemia and tetany in the newborn.This may be the first manifestation of maternal hyperparathyroidism. Untreated or medically treated hyperparathyroidism in pregnancy carries a higher rate of morbidity for both fetus and mother. The optimal time for maternal neck exploration is during the second trimester. This timing avoids the period of organogenesis in the first trimester and the risk of preterm labor that is present in the third trimester. How do you treat it? NON SURGICAL TREATMENT: Intravenous hydration !!!Intravenous hydration !!!Intravenous hydration is the most critical treatment for a patient with an acute presentation of hyperparathyroidism.

Intravenous hydration is the most critical treatment for a patient with an acute presentation of hyperparathyroidism. The addition of furosemide (Lasix) will increase urinary calcium loss. Administration of pamidronate (Aredia) inhibits bone resorption and lowers serum calcium levels. Destruction of parathyroid glands with alcohol injected under ultrasound guidance has been successful. Multiple injections may be required, and paralysis of the vocal cords, sometimes permanent, has occurred But...what if it a post-menopausal woman? won't it aggravate her osteoporosis if you try to decrease serum calcium levels? Postmenopausal women, the largest group of patients with hyperparathyroidism, present clinicians with the challenge of trying to decrease serum calcium levels while also trying to prevent osteoporosis. In the absence of absolute contraindications, hormone replacement therapy with estrogen is indicated in these women. A recent trial demonstrated the protective effect of estrogen on bone in women with otherwise untreated hyperparathyroidism. SURGICAL TREATMENT:In some studies, preoperative localization of adenomas decreased the time required for surgery and lowered the incidence of complications. Localization of the pathology by computed tomographic (CT) scan, magnetic resonance imaging (MRI), ultrasonography or radionuclide scans has been used with varying degrees of success. Recently, parathyroid localization with technetium-99m sestamibi has been shown to have high sensitivity and specificity for single adenomas. This technique of localization also has resulted in the discovery of parathyroid tumors in the mediastinal area. Larger adenomas are easier to localize than smaller lesions. An externally palpable parathyroid gland should be considered malignant until proved otherwise. Thyroid nodules can be difficult to differentiate from parathyroid pathology on some imaging studies. It has been suggested that the best way to localize an abnormal parathyroid gland is to let a good parathyroid surgeon look for it. In fact, this approach is successful about 95 percent of the time. What is/are the potential complication(s) of this surgery?Parathyroid surgery has the potential complication of damage to the recurrent laryngeal nerve. Transient hypocalcemia in the immediate postoperative period also is not unusual. Such hypocalcemia responds to calcium supplementation and usually resolves spontaneously. A precipitous drop in the calcium level, accompanied by tetany and seizures, has been called the "hungry bone syndrome". Are there other concerns in the follow-up of a patient? - Avoid dehydration- Avoid immobilization- Treat Hypertension- Use thiazide and loop diuretics cautiously- Avoid high calcium diet and calcium-containing antacids- Replacement of estrogen in postmenopausal women (in absence of contraindications) Reference: American Academy of Family PhysiciansSyphilis and PCN allergymale w/SY and allergies to Pen. Tx?

a. Azithromycin POb. Pen IMc. Cephalexin POd. ciprofloxacin POe. Desensitize, then tx w/penf. doxycycline POEffectiveness of doxycycline has not been formally established in Syphilis. Desenssitize in allergy is first step. IF PATIENT REFUSES, THEN DOXY 200mg PO x14 days.

Breast Feeding FactsContra-indications to breastfeeding:- when a mother is HIV+- when a mother uses street drugs- when a mother is an alcoholic- when a mother has HTLV-1 virus - when a baby has galactosemia YES IT IS OK TO BREATFEED WHEN:- a mother is getting vaccine for RUBELLA - HepB- Influenza- a mother has breat infections- a mother needs a mammogram- a mother is pregnant- a mother has breast augmentation implant- a baby is jaundiced- a baby has diarrhea- a baby is over 2 years- a baby who has PKU (along with supervised phenyl-alanine free formula) Special Considerations: - Active Hepatitis A: Breastfeed when over acute state and after 24h treatment- Active TB: Wait until Treatment established, at least 2 weeks- Active Hepatitis B: wait until baby is immunized- Maternal Chickenpox within 6 days of delivery or postpartum: Isolate mother and neonate. Give neonate Zoster Immunoglobulin. Only 1/2 of nenonates born to mothers who developed disease 5-15 days prior to delivery will develop the disease. If no lesions develop by time mother is noninfectious, send them home together and start BF. - Babies may be breastfed until 3 hours before their own surgery- Mothers postoperatively may breastfeed if they can hold the baby unassisted. At that point, enough of the sedative is out of the body and BF is safe. How to differentiate between Cavernous sinus thrombosis and orbital cellulitis? - Cavernous Sinus Cellulitis: Cavernous sinus contains internal carotid, sympathetic fibers, III, IV, V1 and V2, and VI. 1. Patients generally have sinusitis or a midface infection (most commonly a furuncle) for 5-10 days2. Headache, fever, and malaise typically precede the development of ocular findings. 3. orbital pain and fullness accompanied by periorbital edema and visual disturbances.4. Increased RETROBULBAR PRESSURE: Exophthalmus and Ophthalmoplegia5. Increased INTRAOCULAR PRESSURE: sluggish pupillary response and decreased

visual acuity6. CRANIAL NERVE PALSIES7. Without effective therapy, signs appear in the contralateral eye by spreading through the communicating veins to the contralateral cavernous sinus. This is pathognomonic for CST. The patient rapidly develops mental status changes from CNS involvement and/or sepsis. Death follows shortly thereafter.8. Mainstay of therapy: Braod Spectrum Antibiotics (Staph Aureus Most common) - Heparin therapy - Steroids (especially if progressed to pituitary insufficency to prevent adrenal crisis). - Orbital Cellulitis: Orbital infections develop via direct inoculation, extension from adjacent structures, and hematogenous spread. 1. Sinusitis (60% patients, ethmoid most commonly), dacryocysitis, dentition, complication of periorbital infection2. Classification: Group I - Preseptal (periorbital) cellulitis = inflammatory edema of the eyelids and periorbital skin with no involvement of the orbit. Lids cannot be opened because of edema. Group II - Orbital cellulitis = CT scan is not sensitive for diagnosing this entity; therefore, clinical examination guides therapy. + Lids cannot be opened because of paralysis secondary to III involvement. + Fever and leukocytosis+ Orbital signs Group III - Subperiosteal abscess = collections of purulent material between the orbital bony wall and periosteum. This diagnosis is confirmed by CT scan, but it can be suspected based on physical examination + Orbital signs (see above)+ Limitations of ocular motility = pain in globe movement toward the abcess.+ Directional proptosis = Globe is looking away from the abcess. Group IV - Orbital abscess = collections of pus within the orbital soft tissue. Diagnosis is confirmed by CT scan, but the physical signs or papilledema on funduscopic examination, are suggestive. + Severe unilateral ptosis+ Severe ophthalmoplegia (ie, palsy of the pupillary and extraocular muscles)+ CN V1 (forehead) anesthesia Group V - Cavernous sinus thrombosis+ Bilateral symptoms: + Ophthalmoplegia, proptosis+ Corneal hypesthesia with increased intraocular pressure Treatment: - Admit all children because children are deficient in IgG2 and are predisposed to bacteremia.- Preseptal: D/C only if adult with PO ATBx and close follow-up- Orbital: Admit with IV ATBx +/- Surgical intervention if necessary (i.e.: compromised vision)Oxacillin or nafcillin can be used with the addition of ampicillin and sulbactam in

children to cover H influenzae. Patients who are allergic to penicillin can use vancomycin, clindamycin, or chloramphenicol.Alternatively, a cephalosporin (eg, cefuroxime, cefoxitin, cefotetan) can be used alone.Central Cord SyndromeCentral cord syndrome is the most common of incomplete Spinal Cord injury. Risk Factor: Cervical Spondylosis in older patients Mechanism: Hyperextension Trauma (minor bleeds in the tissue) Symptoms: Sensory deficit with level, various degrees of motor involvement with paresis more pronounced in UE than LE. Hands are especially involved. DTR's below level are absent at first but return with level of spasticity once over Spinal shock. Bladder involvement common. Other: Neurogenic bladder (retention), Pain of neuro etiology, Spasticity. Management: Admit to Neuro ICU, Maintain a level of mild hypertension for spinal perfusion, Gabapentin for neuro pain, PT/OT/Speech, May be tube feeding in acute phase because of adynamic ileus. Prognosis: 97% completely recover if less than 50yo. More somber prognosis is after that. Anterior Cord SyndromeThe anterior cord syndrome anatomically involves the anterior portion of the cord. Disc protrusion, hyperflexion, or vascular injuries may play a role. Clinically, paralysis below the injury with variable impairment of pain and temperature sensation is present. Difference with Central Cord Syndrome:- Mechanism of injury is different- Position and vibration modalities (posterior columns) are preserved.- Motor symptoms are different between upper and lower extremities in Central. Action: Since diagnosis is pretty much narrowed down. Start Steroids: Solumedrol 30 mg/kg over 15 minutes then followed after 45 minutes by an infusion at 5.4 mg / kg / hr is the typical regimen employed... Methylprednisolone is thought to impact the biochemical cascade of injury that progresses after the initial injury for some hours. Next: MRI: shows narrowing of the white column of CSF and impingement of the darker appearing spinal cord.fever w/o a focusFever in a 2months old child(>1 month),well appear(not ill),WBC=10000(5000<WBC<15000).Guys,what should we do next?If he/she is less than 1 m of age or is ill we admit in hospital,if WBC is >15000 we give Abx(kaplan),,but what about this case?Thanx in advanceANSWER: 2 MONTHS, NO TOXIC MANIFESTATIONSSepsis work up as UTI's are a frequent cause of fever in children under 3 months of age, accounting for 15% to 20% of cases. If clear, no need to admit. Give IM Rocephin x1 (not accepted by ALL authorities by recommended), and observe outpatient. The decision to observe a low-risk febrile infant at home, with or without administration of a parenteral antibiotic, should be made only after carefully assessing the caregiver and

making sure that a responsible physician will be available to provide follow-up evaluation. GUILDELINES ARE AS FOLLOWS: - Fever with toxic signs: admit for workup- fever without toxic signs: 1) 0-7days: admit2) 7-28days: may follow as outpatient if:- caregiver reliable and taught what to expcet and when to bring patient back- patient belongs to low-risk grooup. See below. 3) 28 to 90days: allow to be treted as outpatient ONLY if low-risk group. 4) over 90days: NO NEED TO ADMIT IF TEMP IS LESS THAN 39/102.2 AND NO TOXIC SYMPTOMS. *** Toxic signs are: ***Clinical appearance consistent with the sepsis syndrome (ie, lethargy, poor perfusion, marked hypoventilation or hyperventilation, or cyanosis).

*** Low-risk criteria for infants with fever without source *** 1) Clinical criteriaBorn at term (gestation >37 wk)Previously healthyNo toxic manifestationsNo focal bacterial infection (except otitis media) on examination 2) Laboratory criteriaWBC count 5,000-15,000/mm3, <1,500 band cells/mm3Normal urinalysis (<5 WBC/hpf) or negative Gram's stainWhen diarrhea is present, <5 WBC/hpf REFERENCE: POST GRADUATE MEDICINE ONLINEhttp://www.postgradmed.com/issues/2000/02_00/park.htmQ & A about DonepezilThe foundation for the concept of Choline Esterase Inhibitors in AD treatment rests on the cholinergic hypothesis for the disease proposed over 20 years ago. The ChEIs prolong the actions of the neurotransmitter acetylcholine (ACh) at the synaptic cleft, stimulating presynaptic and postsynaptic muscarinic and nicotinic receptors, which are decreased in patients with AD. Q was about mechanism of action of Donepezil: A: The ChEIs are classified according to their duration of enzyme inhibition as short-acting, intermediate-acting, or long-acting inhibitors. - Tacrine and donepezil are short-acting agents that bind to AChE by hydrogen bonding and are hydrolyzed within minutes by the body's water. Another term for this mechanism of action is reversible inhibition. - Rivastigmine fits into the enzyme's active site in a similar fashion to ACh, which results in the prolonged inhibition of AChE. Like tacrine and donepezil, rivastigmine is hydrolyzed, but unlike these two agents, it can inhibit enzyme activity up to 10 hours. Due to this longer duration of action, rivastigmine is classified as an intermediate-acting (pseudo-irreversible) agent.

- An example of a long-acting ChEI is the organophosphate compound metrifonate. Metrifonate is a prodrug that is converted to the active agent dichlorvos (DDVP). On binding to AChE, DDVP forms an extremely stable complex with a half-life of enzyme regeneration of 15 days. Reference:Medcape: Rivastigmine, a New-Generation Cholinesterase Inhibitor...Article 2000ABG QuestionA 55 year old insulin dependent diabetic woman was brought to Casualty by ambulance. She was semi-comatose and had been ill for several days. Past history of left ventricular failure. Current medication was digoxin and a thiazide diuretic. Results include: K+ 2.7, glucose 67 mmols/l, anion gap 34 mmol/l Arterial Blood Gases pH 7.41

pCO2 32 mmHg

pO2 82 mmHg

HCO3 19 mmol/lANSWER:#1: Acid-Base disorder: IT's a mixed disorder. The disorder usually starts with alkalosis and then acidosis ensues. The result is a "normal" pH. COMPENSATION NEVER "NORMALIZES" pH. It's always close to normal. #2: PAtient has been sick for a while, May be had an infection which precipitated DKA. She has hyperglycemia (67. Normally it's below 6), which corroborates it. The anion gap is high, so we can concude that it is metabolic acidosis. #3: No creatinine level is given, no sodium level, and no calcium level. One must rule out if patient is having some degree of renal failure due to long-standing diabetes which would aggravate the acidosis. #4: The patient is on thiazide diuretic. The most common side effects of thiazide diuretic is hypochloremic hypokalemic metabolic alkalosis. In addition, dehydration probably increased plasma concentration of Thiazide, aggravated by the possible renal failure the patient might have been experiencing, which would explain the profound hypokalemia instead of the usual hyperkalemia you would have in a simple DKA episode. #5: Why is it important to make those speculations? Stopping the offending drug and replacing it with another diuretic would probably correct some of the electrolyte imbalances. In addition, management of DKA differs in patients with renal failure. In the presence of renal failure administration of large amounts of fluid is unnecessary and generally is contra indicated; Overhydration is a concern in adults with compromised renal or cardiac functions and in elderly with incipient congestive heart failure. Consequently, hemodynamic assessments must be made in an intensive care setting in order to administer adequate quantities of fluid while avoiding overhydration that may complicate the condition. Hemodialysis may be needed in some occasions to treat acidosis in this situation. Another example of mixed disorders is salycilate intoxication whereby the first change is respiratory alkalosis, and the next is metabolic acidosis as the ASA accumulates. The

result is a normal pH. Correct me if I am wrong. Transplantation factsTYPES OF TRANSPLANTATIONS: + ORTHOTOPIC: organ graft transferred to an anatomically normal recipient site e.g.: heart transplant. + HETEROTOPIC: organ transplant to an anatomically abnormal recipient site e.g: kidney transplant to iliac fossa of recipient. + AUTOGRAFT: transfer of one's own tissue to another location of the body e.g: bone graft+ SYNEGENEIC GRAFT: graft between identical twins.+ ALLOGRAFT: graft between genetically DISSIMILAR members of the same species. + XENOGRAFT: graft between different species e.g.: porcine heart valves. GRAFT REJECTION REACTIONS: Allografts may be rejected through either a cell-mediated or a humoral immune reaction of the recipient against transplantation (histocompatibility) antigens present on the membranes of the donor's cells . * HVGR = HOST-VS-GRAFT REJECTION: 1) Acute Rejection (4-60 DAYS POSTOP): Main mechanism is LYMPHOCYTE-MEDIATED IMMUNE REACTION. It is a HYPERSENSITIVITY REACTION TYPE IV culminating in graft enlargement and tenderness. Usually, vascular integrity is maintained, although the arterial endothelium appears to be a primary target of the HVGR.Cell-mediated rejection may be reversed in many cases by intensifying immunosuppressive therapy. After successful reversal of an acute rejection episode, severely damaged elements of the graft heal by fibrosis and the remainder of the graft appears normal. After resolution of acute rejection, the allograft commonly survives for prolonged periods, even though immunosuppressive drug dosages are reduced to very low levels. This process of graft adaptation is most likely explained by the loss of highly immunogenic passenger leukocytes, including dendritic cells (see below), and maybe by the development of donor-specific suppression of the recipient's immune response. 2) Chronic Rejection: Clinically less dramatic, and can be suspected on the basis of low-grade fever with signs of graft insufficiency (e.g: decreased clearance of creatinine for kidney transplant). There is no therapy for chronic rejection. The pathological basis is anti-vascular antibodies that cause extensive proliferation of the arterial endothelium, which may gradually occlude the vessel lumen, resulting in ischemia and fibrinoid necrosis of the graft. 3) Hyperacute rejection: Within hours or even minutes (intraoperatively). It is irreversible. The reaction manifests as an Arthus Reaction (Type III hypersensitivity). The role of humoral antibody in hyperacute graft rejection is evident when the recipient has been presensitized (by pregnancy, blood transfusion, or previous transplant) to HLA in the graft. Histologically, this rejection reaction is characterized by small vessel thrombosis, and graft infarction is unresponsive to known immunosuppressive therapies. Liver grafts seem to be less susceptible to such antibody-mediated hyperacute rejection. Pretransplantation evaluation usually includes a lymphocytotoxic test between recipient serum and donor lymphocytes in the presence of complement, te detect presensitization

in the recipient. * GVHR = GRAFT VS HOST REJECTION: It is a major obstacle encountered in Bone Marrow transplants, It occurs from the grafting of an immunocompetent donor tissue containing EFFECTOR T CELLS, which respond to -allo-antigens expressed on host cells. Occasionally seen in intestinal grafts (lymphoid tissue). Major target organs are: SKIN (Dermatitis), GASTROINTESTINAL TRACT (Diarrhea), and LIVER. Therapy may be prolonged and usually consists of some combination of Steroids, cyclosporine, and low-dose azathioprime. IMMUNOSUPPRESSANTS AND COMMON ADVERSE EFFECTS:- CYCLOPHOSPHAMIDE: Hemorrhagic cystitis, infertility, and alopecia- PREDNISONE: adverse effects on growth in children, osteoporosis, redistribution of fat tissue, sugar imbalance, etc.- AZATHIOPRINE: Bone Marrow depression, and hepatitis.- CYCLOSPORINE: Nephrotoxicity (ie.: renal artery vasoconstriction), gum hypertrophy, hirsutism, and B-cell lymphoproliferative disorders secondary to reactivation of EBV. - TACROLIMUS: is an immunosuppressive drug for liver transplant recipients. Its adverse effects are similar to cyclosporine, although gum hypertrophy and hirsutism are less prominent. It also may induce diabetes. SPECIFIC ORGAN TRANPLANTATIONS: I)KIDNEY TRANSPLANTATION ISSUES: - Despite prophylaxis with immunosuppressants begun just before or at the time of transplantation, most recipients undergo one or more acute rejection episodes in the early posttransplant period. Rejection is suggested by deterioration of renal function, hypertension, weight gain, tenderness and swelling of the graft, fever, and appearance in the urine sediment of protein, lymphocytes, and renal tubular cells. If the diagnosis is unclear, percutaneous needle biopsy is performed for histopathologic evaluation of tissue. In cyclosporine-treated recipients, drug-induced nephrotoxicity is sometimes difficult to differentiate from rejection, even with biopsy. Intensified immunosuppressive therapy usually reverses rejection. If it cannot be reversed, immunosuppressive therapy is tapered, and the patient returns to hemodialysis to await a subsequent transplant. Nephrectomy of the transplanted kidney is necessary if hematuria, graft tenderness, or fever results from the rejection response with withdrawal of immunosuppressants. Complications: Immunosuppressant toxicity, Acute Graft Rejection in 3 to 4 months (but they return to normal health and function with increased suppression and reversal), Chronic Graft Rejection, and risk of Epithelial Carcinoma and Lymphoma (in which case, do not stop immunosuppressants). II) LIVER TRANSPLANT: The advent of cyclosporine has permitted early reduction of corticosteroid dosage, resulting in better postoperative healing and greater resistance to overwhelming infection.

End-stage chronic hepatitis and biliary cirrhosis are the most frequent indications for liver transplantation in adults, as are biliary atresia and inborn metabolic deficiencies in children. Cadaveric donors of livers must be of previously healthy persons who are size- and ABO-matched to the recipient. Livers are stored in cold solutions generally for 8 to 16 h after

removal. Some grafts stored for > 24 h are successfully transplanted, but the incidence of graft nonfunction increases with prolonged storage. Surprisingly, liver allografts are less aggressively rejected than other organ allografts. However, when either fulminant acute rejection or chronic rejection is refractory to immunosuppressive therapy, retransplantation is the treatment. The vanishing bile duct syndrome, characterized by intrahepatic cholestasis with preserved hepatocellular function, is a pattern of chronic rejection.Rejection is suspected by development of hepatomegaly, light-colored bile (seen in T-tube drainage) or stools, and complaints of anorexia, right-sided pain, and fever. Jaundice and elevated serum levels of hepatic enzymes are corroborative findings. Needle biopsy can provide pathologic confirmation. Suspected rejection episodes are treated with IV corticosteroids, antithymocyte globulin (ATG), or monoclonal antibodies. III) CARDIAC TRANSPLANTATION: Rejection onset may be heralded by fever, malaise, tachycardia, hypotension, and heart failure that is predominantly right-sided. Arrhythmias may occur in more severe rejection episodes. In milder cases, rejection may be suggested by biopsy findings only. With the use of cyclosporine, routine protocol transvenous endomyocardial biopsy has been used increasingly to diagnose rejection, because other signs and symptoms are often absent and rejection may be detected before function of the graft deteriorates. Rejection is treated with corticosteroid and ATG or OKT3, if necessary. Mild rejection by histologic criteria without detectable clinical sequelae requires no treatment. IV) BONE MARROW TRANSPLANT: Early complications include rejection by the host of the marrow graft, acute GVHD, and infections. Later complications include chronic GVHD, prolonged immunodeficiency, and disease recurrence.- GVHR: Symptoms and signs of acute GVHD are fever; exfoliative dermatitis; hepatitis with hyperbilirubinemia; vomiting; diarrhea and abdominal pain, which may progress to an ileus; and weight loss. - Infection (See next post: Infections in Transplant patients): Following the preparative regimen for BMT, the WBC count can take 2 to 3 wk to recover. During this time, patients are very susceptible to infections. Acyclovir prophylaxis has dramatically decreased the risk of herpes simplex infections during this time. Even after engraftment, patients continue to be immunocompromised and at risk for infections because of the drugs used to treat GVHD. A worrisome late infection is cytomegalovirus interstitial pneumonitis, which generally occurs 40 to 60 days after transplantation. Patients present with tachypnea, dyspnea, hypoxemia, and a chest x-ray with bilateral pulmonary infiltrates. The mortality rate of cytomegalovirus interstitial pneumonitis was 80 to 90%, but treatment with ganciclovir and passive immunity with immunoglobulin has decreased the mortality rate to about 25 to 40%. Patients are also at risk of developing pneumocystis pneumonia, but prophylactic use of trimethoprim-sulfamethoxazole has dramatically decreased the incidence of this infection.

References: - Merck Manual- Immunology references (Compiled previous paper I had I don't recall from which references)

- Harrison 14th editionClinical Case in Lung TransplantI- 29-year-old woman with cystic fibrosis who underwent bilateral single lung transplantation on April 7, 1998. Both she and the donor were cytomegalovirus-(CMV) positive. Postoperatively, the patient received 1 dose of CMV hyperimmune globulin (CMVIG) and was started on a standard immunosuppressive regimen with tacrolimus, azathioprine, and prednisone. She was discharged and placed on prophylactic oral ganciclovir for 3 months. Two weeks after transplantation, the patient underwent bronchoscopy because of decreasing pulmonary function on spirometry. Bronchoscopy showed no signs of rejection, but significant mucus plugging was present. Biopsies were negative for CMV. Four weeks later, lung cultures showed mucoid Pseudomonas, which was treated with antibiotics. Repeat bronchoscopies done over the next 3 months were all negative for rejection and CMV. At 6 months after transplantation and 3 months after the last dose of oral prophylactic ganciclovir, the patient again reported shortness of breath (SOB). She underwent transbronchial biopsy, which disclosed CMV infection. Which one of the following agents is considered the drug of first choice in treating acute CMV infection in lung transplant recipients? A- AyclovirB- GanciclovirC- FaqmciclovirD- FoscarnetE- Cidofovir 2- After completing her course of antiviral therapy, the patient was placed on oral ganciclovir, 1 g TID, as prophylaxis. Three weeks later, she had a bronchial stent placed, after which she developed fevers and was found to have respiratory syncytial virus (RSV) on bronchoalveolar lavage (BAL). The RSV infection was treated with ribavirin, but because of her history of CMV, the patient was also treated with ganciclovir IV for 14 days. The patient did well for the next 10 weeks, until she again developed dyspnea. A BAL was positive for CMV, despite continuing prophylaxis with oral ganciclovir. In this situation, which of the following alternatives would be most appropriate for treating this infection?

A. Increased dosage of oral ganciclovir to 2 g TID B. Ganciclovir IV, 10 mg/kg/day x 14 days C. Foscarnet IV, 120 mg/kg/day x 14 days D. Ganciclovir IV plus foscarnet IV 3- Because of suspicions that her CMV strain may have developed resistance to ganciclovir, the patient was admitted to the hospital for treatment with a combination of foscarnet IV and ganciclovir IV. After four days of in-house monitoring for electrolyte disturbances (which did not occur), the patient was discharged home to complete a 14-day course of antiviral therapy. One week after antiviral therapy finished, follow-up bronchoscopy was again positive for CMV. The patient received a repeat course of foscarnet IV and ganciclovir IV for 14 days.

Which one of the following adverse effects associated with foscarnet is the most common serious toxicity?

A. Electrolyte disturbance B. Seizures C. Nephrotoxicity D. Anemia E. Nausea, vomiting, and diarrhea 4- All of the following tests have been used for rapid, early detection of CMV infection -- except for which one?

A. Shell vial assay with immunofluorescent staining B. Serologic monitoring of IgM/IgG antibody levels C. CMV pp65 antigenemia D. PCR for CMV DNA E. Digene hybrid capture CMV DNA assay

Answers: 1- Ganciclovir is the drug of choice for treating CMV infection. Most episodes of CMV pneumonia respond to a 2- to 3-week course of therapy.Patients who have frequent relapses on ganciclovir may have their treatment augmented with IV CMV hyperimmune globulin (CMVIG).Foscarnet and the new agent Cidofovir are effective therapies for treating CMV infection -- including infection due to ganciclovir-resistant strains. However, these drugs are associated with a significant risk for nephrotoxicity and other adverse events that limit their overall use. 2- Answer is D. In this situation, the recurrence of CMV infection, despite ongoing ganciclovir prophylaxis, suggests the possibility of ganciclovir resistance. Foscarnet is approved for the treatment of CMV infections and represents an appropriate second-line agent for use in patients in whom ganciclovir treatment has failed. However, studies in AIDS patients with relapsing CMV retinitis have shown that combination therapy, using both ganciclovir and foscarnet, is superior to monotherapy with either agent alone. 3- Answer is C. The most common serious toxicity associated with foscarnet is nephrotoxicity, which may occur in more than 25% of patients. Adequate hydration and careful monitoring of renal function are critical in reducing the likelihood of this effect. Saline loading, dose modification, and avoidance of other nephrotoxic agents may also help to reduce the occurrence of renal impairment. Other common abnormalities associated with foscarnet include changes in serum electrolyte concentrations, including phosphorus and particularly calcium. Less common side effects include seizures, anemia, neutropenia, fever, nausea, vomiting, diarrhea, and headache. 4- The correct answer is: BThe appearance of anti-CMV IgM antibodies is the primary humoral response to CMV, but this response is delayed in immunosuppressed persons and often appears after the onset of disease. In addition, in organ-transplant recipients, IgM antibodies may not be appropriately replaced by IgG antibodies. Thus, serologic monitoring of antibody levels

is inadequate in screening for CMV infection. References: Interactive Grand Rounds in ImmunologyClinical Case of Cyclosporine Adverse Effects1- 43-year-old woman with a history of congestive heart failure secondary to a nonischemic dilated cardiomyopathy as well as hereditary protein S deficiency. Because of progressive heart failure, she was evaluated for cardiac transplantation. A donor heart became available 15 days after the patient was admitted, and orthotopic heart transplantation was performed without complications.The patient was extubated on the first postoperative day and weaned off vasopressors. Initial immunosuppression included intravenous azathioprine and methylprednisolone. Maintenance immunosuppression consisted of oral cyclosporine, azathioprine, and prednisone. Approximately 1 week after transplantation, diltiazem was added to the treatment regimen after the patient developed new-onset hypertension.The patient made satisfactory progress and was transferred out of the intensive care unit on the sixth postoperative day. Discharge medications included prednisone, azathioprine, cyclosporine, diltiazem and warfarin. Approximately 2 months after transplantation, the patient reported the development of "swollen gums" that tended to bleed when she brushed her teeth. There was no history of significant dental problems before transplantation. Oral examination showed generalized gingival enlargement involving the mandible and maxilla. Mild hemorrhage of the gingival tissue occurred on probing. Cyclosporine-induced gingival hyperplasia was suspected. Which one of the following strategies has been shown to prevent the development of gingival hyperplasia? A. Meticulous dental hygiene, with twice-daily brushing and daily flossing B. Cyclosporine dosage reduction C. Change antihypertensive medication from a calcium channel blocker to ACE inhibitor or other class of antihypertensive D. Subgingival scaling before initiation of cyclosporine E. None of the above is effectively preventive. 2- Which of the following methods is commonly used to treat advancing gingival hyperplasia?

A. Short-term dosage reduction of cyclosporine B. Short-term azithromycin C. Surgical reduction (gingivectomy) D. Discontinuation of cyclosporine 3- At this time, the patient also reported concern about aberrant hair growth involving her face, cheeks, and forehead regions, which had begun appearing 2 months previously. Darker, thicker hair was noted on her hands and arms, extending onto her shoulders. Which of the following statements about cyclosporine-induced hypertrichosis is/are correct? A. Clinical manifestations of cyclosporine-induced hypertrichosis usually begin to appear approximately 4 to 6 months after initiation of cyclosporine therapy. B. Cyclosporine-induced hypertrichosis affects primarily females, especially children and

adolescents. C. Hypertrichosis results from an increased production of 17-hydroxyprogesterone and DHEA sulfate, by hair follicles stimulated by cyclosporine. D. Higher dosages and serum levels of cyclosporine are associated with an increased incidence and severity of hypertrichosis. E. All of the above statements are correct. 4- Besides cyclosporine, which one of the patient's current medications is associated with the induction of gingival hyperplasia?

A. Prednisone B. Azathioprine C. Diltiazem D. Warfarin

1- Answer : E2- Answer : C3- Answer : D4- Answer : CTransplantation facts #2 - 13 Clinical Cases I- BONE MARROW TRANSPLANT: FIRST MONTH: Give Prophylaxis TMP-SMX or CIPROFLOXACIN for GN bacteremia prevention. WBC recover in 2-4 weeks. SECOND MONTH: CMV Disease is a major concern. Occurs between 2 weeks and 4 months. Treatment with Ganciclovir and Immunotherapy. 6 MONTHS AND BEYOND: ENCAPSULATED ORGANISMS (Pneumococcus and H. Flu) AND VZV. Patient should still be on TMP-SMX up to one year. It also prevents T. Gondii. If Allergic, give Dapsone + Pyrimethamine. OTHER: - Herpes Simplex: 6 weeks in seropositive host or donor. HSV-1 = Esophagitis. HSV-2 = Anogenital Disease. Prevent by Acyclovir in seropositives. - CMV: Typically 30-60 days. CMV infection may cause interstitial pneumonia, bone marrow suppression, or graft failure. Risk only if donor is seropositive and recipient negative. Prophylaxis includes ORAL GANCICLOVIR. Treatment includes combination of IV GANCICLOVIR AND CMV IG. - Varicella Zoster Virus: Reactivation after several months (can be as early as 1 month). Treat with HIGH DOSES OF ACYCLOVIR. Low doses of Acyclovir PO have been traditionally given for prevention, but they also inhibit development of VZV specific immunity => high risk when treatment stopped. - EBV: 1-3 months. B-cell lymphoproliferative disease can be fatal (high fever, cervical lymphadenopathy). II- SOLID ORGAN TRANSPLANT: The organisms are different from BMT recipients because solid organ recipients do not go through a period of neutropenia. However, they do have LONGER IMMUNOSUPPRESSION, AND WOUND INFECTIONS (Due to surgery). - <1 month: EXTRACELLULAR BACTERIA FROM SURGICAL WOUNDS OR ANASTOMOTIC SITES (Staph, Strep, E.Coli, GN Organisms).

- 2 months (TO YEARS): EBV LYMPHOPROLIFERATIVE DISEASE = lung and heart transplants are most likely to develop EBV-induced B cell proliferation. - 1-6 months: CMV infection organ-specific+ glomerulopathy in kidney transplant+ bronchiolitis obliterans in lung transplant + premature atherosclerosis in heart transplant + vanishing bile duct syndrome in liver transplant (progressive cholestasis with normal LFT's). - >6 months: infections characteristic of patients with defects in cell-mediated immunity e.g., infections with Listeria monocytogenes, Nocardia, various fungi, and other intracellular parasite. III- VACCINATION OF TRANSPLANT PATIENTS: BMT Patients: AFTER TRANSPLANTATION- Pneumovax, H. Flu Vaccine, Diphtheria, Tetanus, Neisseria Meningitidis and Inactivated Polio at 12 months, and then 12 months thereafter. Household contacts should get IPV as well.- Influenza every year- MMR at 24 months. SOLID ORGAN TRANSPLANT: BEFORE TRANSPLANTATION:- Pneumovax before. Repeat every 6 years. - INFLUENZA Every year, N. Meningitidis, H. Flu Vaccine- No MMR before or after due to immunosuppression. If contact with measles => Immune Globulin Post-Exposure Prophylaxis (PEP). - No Varicella Vaccine. If contact with chickenpox => VZV IG ASAP (Within 96h). IV- TRAVEL VACCINATIONS FOR IMMUNOCOMPROMISED PATIENTS: ANYTHING APPROPRIATE TO THE AREA OTHER THAN LIVE VACCINES: 1- YES:- Japanese Encephalitis- Hepatitis A, Hepatitis B- Meningococcal Vaccine- IPV- Typhoid immunization (not the live one). - INFLUENZA 2- NO: - Live Typhoid- Live Yellow Fever- MMR- Varicella V- TIMELINE AND COMMON CASE SCENARIOS AFTER TRANSPLANT: 1) UTI: <1 MONTH: E.COLI, KLEBSIELLA, PSEUDOMONAS, ENTEROCOCCUS, CANDIDA1-6MONTHS: CMV LUNG INFECTION: * By Organism: <1 MONTH: LEGIONELLA, OTHER

1-6 MONTHS: CMV PNEUMONITIS, PCP, ASPERGILLUS, LEGIONELLA> 6 MONTHS: NOCARDIA, ASPERGILLUS, MUCOR * By CXR findings: - Localized infiltrates: Legionella- Nodular Infiltrates: Fungi, Nocardia, Recurrent tumor- Diffuse Infiltrates: CMV, Chlamydia, PCP, Toxo, Mycobacterium, CHF, Radiation Pneumonitis, Drug-induced, Diffuse Alveolar Hemorrhage. CNS: 1-6 MONTHS: LISTERIA MONOCYTOGENES, CMV, TOXOPLASMOSIS>6MONTHS: CMV RETINITIS, LISTERIA M., CRYPTOCOCCUS, ASPERGILLUS, NOCARDIA VI- ANTIBIOTIC PROPHYLAXIS:

+ TRAVEL IN FUNGAL RISK AREAS (Coccidio, histoplasm) => Amphotericin B or imidazoles+ LATENT VIRUSES: ACYCLOVIR AFTER BMT for HSV and VZV, GANCICLOVIR IN CMV SEROPOSITIVE DONOR OR RECIPIENT+ LATENT FUNGI/PARASITES: TMP/SMX or DAPSONE/PYRIMETHAMINE x 1 year for PCP and T. Gondii. + HISTORY OF EXPOSURE TO TB: INH IF RECENT POSITIVE CXR. + in BMT: -> Pre-op: Fluconazole x 1week before procedure, Acyclovir x1 day prior-> Post-op: Bactrim x 1 year, Ganciclovir x 100days VII: ALGORITHM OF DIAGNOSIS AND TREATMENT OF FEBRILE NEUTROPENIC PATIENT: - INITIAL EVAL: Skin Lesions, IV Catheter, Peri-rectal- LABS: Granulocye count<500- BCx - CXR - (Sputum/Urine/Skin biopsy accordingly) NEXT: Initial Broad-based ATBx (GN and GP aerobes)FOLLOW-UP: 1- If no obvious infection site and patient febrile: AMPHOTERICIN B2- If no obvious infection site and patient afebrile: Con't ATBx.3- If obvious infectious site found: CON'T BROAD-BASED SENSITIVITY-ORIENTED ATBx. CLINICAL CASES:

Case #1: Q309: U visit a 25 yo male pt who just received BM transplantation,which statement is correct about immunization in this pt?a:influenza vaccine at 4mo following transplant and annually thereafterb:MMR at 24mo following transplant c:varicella at 6mo following transplantd:HBV at 12 mo following transplante:Td at 6mo following transplant Case #2: Q 300A 31yo man with diabetic nephropathy undergoes an uneventful renal transplant from his sister.his immunosuppressive regimen includes azathioprine,steroids&cyclosporine.on

postoperative day3 the pt is doing well,but u notice on his routine lab tets that his WBC is 2000.What's the most appropriate next step of management?a:start gancyclovirb:start broad-spectrum Antibioticsc:Administer filgrastimd:Administer FK50e:decrease cyclosporinef:increase cyclosporineg:decrease azathioprineh:increase azathioprinei:withhold stroidsj:administer stroid boostk:obtain renal US Case #3: Q366: A 30 yo man on vacation has had symptoms of URI for 1 wk&persistent headache for 3days.He received a cadaveric renal allograft 1 yr ago&is now taking prednisone,azathioprine&cyclosporine.T:37.8,but Ph.E shows no abnormalities.The allograft is not enlarged or tender,there’s no nuchal rigidity&there r no focal neurologic findings.Serum Cr:1.5mg/dl,WBC:5000.Which test should u do next to evaluate the pt’s condition?A:CT scan of the headB:LPC:xray of skull sinusesD:U/AE:CxrayF:serum cyclosporine measurement Case #4L Q373: A 32 yo man received an allogenic renal transplant 50 days ago&has been taking TMP-SMZ.he now has a fever,malaise&evidence of hepatitis followed by progressive dyspnea&hypoxemia with a diffuse interstitial pulmonary infiltrate developing over 5-7days.What’s the most likely diagnosis? Case #5: More prevalent in small bowel transplant than other transplants ,related to the presence large amount of lymphoid tissue. a. acute rejection.b.hyper acute rejection.c.chronic rejection.d.graft versus host disease.e.acute tubular necrosis. Case #6: Q445: A 37 yo man develops arthralgias,fever,urticaria 2wk after a heart transplant.The immunosuppressive agent most responsible is:a:azathioprineb:cyclosporinec:prednisoned:antitymocyte globulin Case #7: Renal transplant patient on immunosuppressants among which cyclosporine. 4 weeks later: rapid rise of creatinine, fever, edema, and oligouria. Is it graft rejection or cyclosporine nephrotoxicity? Case #8: 60 year old man, 6 months after renal transplantation comes with cough, SOB,

fever 100. Sputum is positive for Gomori coloration. Best step in treatment:a. gancyclovirb. trimetoprim IV c. Amphotericined. Imipemene. Acyclovir Case #9: The patient is a 28-year-old woman who 7 years previously underwent bilateral lung transplantation as treatment for cystic fibrosis. The patient was CMV negative, and the donor was CMV positive. Both were negative for hepatitis antibodies. Postoperatively, the patient was started on standard immunosuppressive therapy with intravenous cyclosporine, azathioprine, and prednisone. At 2 weeks after transplantation, routine biopsy surveillance showed the development of acute mild rejection, which was treated with intravenous methylprednisolone. Other postoperative complications included diabetes mellitus, for which the patient remains under treatment, as well as repeated episodes of acute rejection and lung infection. The patient continued to manifest reduced FEV1. Which one of the following histologic descriptions best characterizes acute mild rejection? A. Infrequent perivascular mononuclear infiltrates B. Frequent perivascular infiltrates around arterioles and venules C. Frequent perivascular infiltrates around arterioles and venules, with occasional foci of parenchymal or vascular necrosis D. Dense perivascular lymphocytic infiltrates with extension into alveolar septae Case #10: , Approximately 3 months after successful treatment of her acute rejection episode with Methylprednisone, the patient’s lung function began to deteriorate. Crackles were heard in the right lung base. Histologic examination of a transbronchial biopsy specimen showed severe, grade A3/4 rejection. For repeated recurrences of rejection, treatment could include any of the following options, except for which one? A. Intravenous methylprednisolone alone B. Methylprednisolone plus antilymphocyte globulin C. Methylprednisolone plus plasmapheresis with immunoglobulin replacement D. Conversion from cyclosporine to tacrolimus Case #11: The patient was treated with methylprednisolone and antilymphocyte globulin. Her acute rejection episode resolved with this therapy, but she was left with reduced lung function (FEV1 reduced by approximately 25% compared to baseline). Review of the biopsy specimen revealed findings suggestive of chronic rejection as well as acute rejection. Which of the following histologic findings, are indicative of chronic rejection?

A. Submucosal lymphocytic infiltrate with submucosal granulation tissue B. Submucosal fibrosis of terminal bronchioles, with intimal thickening and sclerosis of small airways; perivascular mononuclear infiltrate C. Intraluminal plugs of granulation tissue in small airways, extending to alveolar ducts and alveoli D. Lymphocytic bronchiolitis with CMV inclusion bodies

Case #12: Although several nonspecific clinical signs and symptoms may be suggestive of bronchiolitis obliterans, which one of the following is most strongly associated with the diagnosis? A. Nonproductive cough B. Progressive dyspnea C. Fatigue/reduced exercise tolerance D. Deteriorating expiratory airflow E. Crackles or wheezes at lung bases Case #13: A 45-year-old white male was transplanted for Child C alcoholic liver cirrhosis with portal thrombosis.Orthotopic liver transplantation (LTX) was performed. Cold ischemia time was 10 hours and a peroperative thrombectomy had to be done.The usual postoperative rise of aminotransferases (ALT and AST) was observed with a peak of 2,760 IU on POD2. At that time echo-Doppler of the portal vein and hepatic artery was normal.Patient developed fever (38°C) on POD4.Chest X-ray was non contributive. Blood, urine, abdominal fluid, throat, bile and transcutaneous portions of the catheters were sent to the bacteriological laboratory for direct examination and culture. Vascular catheters were exchanged and antibiotics (Ciprofloxacine-Ciproxine(r) and Vancomycine-Vancocin(r)) were instituted. Fever subsided whereas bacteriological work-up was negative. On POD7, bilirubin rose to 11.7 mg/dL together with alkaline phosphatase: 320 IU/L (N < 250 IU/L) whereas aminotransferases continued to drop. Next step in diagnosis?

A. Doppler ultrasonography (Doppler US) of portal vein and hepatic artery exclusivelyB. T-tube cholangiography exclusivelyC. Liver biopsy exclusivelyD. All of these procedures (Doppler US, T-tube cholangiography, liver biopsy) step by step if the previous test does not give a definitive diagnosis

ANSWERS TO CLINICAL SCENARIOS: Case #1: Answer is D: HBV at 12 mo following transplant Pts who recieved BMT,should be revaccinated,the schedule for this pts:1-influenza vaccine>6mo following transplant&annual therafter2-inactivated vaccines like Td,IPV,HBV,PPV after 12mo3-MMR after 24mo,if he's immunocompotent4-varicella not recommended CASE #2: the answer is g: Broad antibiotics and filgatrim are started only if the count falls below 500. Firthermore, the major side effect of azathioprine is BM toxicity,both WBC&plts should be monitored in the immediate posttransplant period.the pt’s decrease in WBCs is secondary to azathio toxicity and the most appropriate step is to decrease its dose. Case #3: the ANSWER is:B: doing LP to rule out cryptococal meningoencephalitisChief complaint is heacache Case #4: CMV Pneumonitis.

Case #5: d. graft versus host disease. hyper acute rejection occurs when there is serum incompatibility. the antibodise that are already formed are resposible.acute rejection can be prevented by immunosuppressants.acute tubular necrosis is a normal phenomenon after kidney transplant, that occurs due to storage and handling etc. it resolves in few days or weeks. chronic rejection occurs after few years. it is chronic deterioration in organ function. it is neither preventable nor treatable. Case #6: d:antitymocyte globulin serum sickness Case #7: Cyclosporine ToxicityTo differentiate the two: - cyclosporine toxicity: >6 weeks use, afebrile, Cyclosporine trough level>200ng/ml, gradual rise in creatinine, US: unchanged graft corss-sectional area, manometry: capsular pressure is<40mmHg, Biopsy: arteriolopathy (intimal thickening, hyalinosis, endothelial vaculoization). Rsponds to decreasing cyclosporine. - Transplant rejection: <4weeks, oligouria after initial function, rapid rise in creatinine, fever and weight gain, Cyclosporine trough level <150ng/ml, Biopsy: endovaculitis (intimal arteritis, necrosis and sclerosis), US: increased graft cross-sectional area, MRI: loss of distinct cortico-medullary junction + swelling + density similar to that of Psoas and fat tissue, Manometry intracapsular pressure >40mmHg. Responds to increased steroids or antilymphocyte globulin. Case #8: PCP pneumonitis. IV Trimethoprim-Sulfamethoxazole. Gomori offers best contrast for Pneumocystis Carinii (dark brown-black). Case #9: Answer is B. acute mild rejection (grade A2) is characterized by the presence of frequent perivascular mononuclear infiltrates around the arterioles and venules. This pattern of infiltrates must be differentiated from airway inflammation (B), which is characterized by lymphocytic bronchitis or bronchiolitis and may be related to infection or other causes. Early acute rejection is a common occurrence, affecting a majority of transplant patients. Typical therapy consists of intravenous methylprednisolone in three successive daily doses, which in this patient was successful in resolving her rejection episode. Case #10: The correct answer is A. When corticosteroids alone fail to stem the decline in lung function, the next option is usually to add treatment with a cytolytic drug, such as OKT3 monoclonal antibody, antithymocyte globulin, or antilymphocyte globulin, to halt acute rejection or to treat potential bronchiolitis obliterans. Case #11: The correct answer is B. Chronic allograft rejection is characterized histologically by the findings of bronchiolitis obliterans. The hallmark sign is the proliferation of submucosal or intraluminal fibrous tissue in the terminal bronchioles. Patchy narrowing or obliteration of the small airways follows, leading to the clinical manifestation of progressive expiratory airflow obstruction. If inflammatory cells are present (characterized by a submucosal mononuclear infiltrate of lymphocytes, plasma cells, and monocytes), the lesion is considered "active"; if inflammatory cells are absent, the lesion is "inactive". Case #12: The correct answer is D. The FEV1 score, reflecting declining expiratory airflow, has been shown to be a reliable and consistent indicator of impaired graft

function.Because histologic proof of bronchiolitis obliterans cannot always be obtained, the International Society for Heart and Lung Transplantation in 1993 proposed using spirometric data to reach the diagnosis. The diagnosis of bronchiolitis obliterans syndrome (as opposed to bronchiolitis obliterans, which is based on biopsy results) is defined as a deterioration in FEV1 of more than 20% compared to the baseline FEV1, the baseline score is an average of the two highest postoperative measurements taken 3 to 6 weeks apart. The decline in FEV1 must be present for at least 1 month and cannot be due to other factors such as infection, acute rejection, or bronchial anastomatic complications.

Case #13: The Algorithm in the presence of abnormal liver tests in a liver transplanted patient is "All of these three procedures, step by step if the previous test does not give a definitive diagnosis": #1: Liver and Abdomen US as well as Doppler US (normal: excluding hepatic artery and portal vein thrombosis), T#2: T-tube cholangiography (Normal, excluding bile leak) and then #3: CTscan, Liver Biopsy and microbiological examinations. Differential diagnosis between hepatitis and rejection of the liver allograft * HepatitisInterval > 2 months Lobular necro-inflammation Steatosis, abundant councilman bodies (HCV) Ground glass hepatocytes, positive immunoperoxidase (HBV) Fibrous and inflammatory septa No bile duct loss * Acute rejection Interval < 2 months Triade:Activated lymphocytes, eosinophils, PMN> 50% bile duct lesionsEndothelialitis * Chronic rejection Interval > 2 months > 50% loss of bile ducts Arteriopathy ± centrolobular necrosis/fibrosis Portal fibrosis

References: - Liver dysfunction in a patient having liver transplanthttp://www.ulb.ac.be/erasme/edu/gastrocd/Case35/C35r.htm- Harrison: Infections in Transplant Patients- Merck Manual: Transplantation- USMLE.net Forum: Previous postsRemembered Question on TransplantationLet's say you have a patient who recently received a renal transplant. After a short time you notice a decline in renal function. A biopsy of the kindey reveals rejection. What do you do?

a. increase the doses of the immunosuppresive drugsb. wait for a weekc. remove the transplant surgicallyANSWER: increase the dose of immunosuppressive drugs. It usually takes care of the usual acute rejections in the post op period. Sometimes, in solid organ transplant, they use infusion Methylprednisone alone.FiO2 simply putInspiratory Fraction of Oxygen: That's how much percent of Oxygen is delivered in the air breathed by the patient in the mechanical ventilation. FiO2 in standard atmosphere composition = .21 (21%). That is if you are to calculate A-a gradient using FiO2 and you find out you don't have it, but the patient is breathing room air, it is by default: 21%.If mechanical ventilation, FiO2 has to be reset according to ABG's and different parameters discussed previously in my review: MEchanical Ventilation LEcture and Case Scenarios.When to d/c asprin or antiplatelets prior to surgery?asp we d/c befor i wk of surgery and heparin 4 h b4 goingto surgery its half lif is 90 minutes and it doesnt cause bad efefct during surgery like bleeding andocp we stop imonth b4 surgery bcz it cause dvt and smoke cessation should b done at least 6 wk.to...8 wk b4 surgery. and coumadin should b stop 48 h b4 surgery..thats what we do in common practice refrence would b crush and nail the board ....ifu dont have that book dont go for exam ..excellent and easy book i will recomond to all...by the way hasan u r doing great job and valium u tooo...this forum helped me alot so i will do som ething...i will post every day some qs or facts like u and valium do hasan bcz thats why this forum is.there...wish u all the best hasan and valium..and so many others really nice ppl who post qs everyday ..goood luck to u and valium and every body else..just follow valium advice dont read every book just finish what u have and read it coverto cover and it will b more than enough....try to do recall qs bcz 70% come back to every one...believe me its a fact...Bovis and colon CA?Ans: bacteremia with streptococcus bovis makes it mandatory to rule out the CA colon. The bacteremia with this agent is almost pathognomonic for the latterGeneral Considerations: Streptococcus bovis is the main human pathogen among nonenterococcal group D streptococci. S bovis infection is a well-documented cause of infective endocarditis. The portal of entry for S bovis bacteremia is the GI tract. A strong association exists between S bovis bacteremia with or without endocarditis and underlying malignancy or premalignant lesions of the colon. The organism also has been isolated more frequently from the stools of patients with such malignancies. Some authors have found a similar relationship between bacteremia (or endocarditis) and liver diseases or nonmalignant diseases of the colon. Whether S bovis plays a causative role in colon cancers or is only a marker of the disease is unclear. In any event, every patient with S bovis bacteremia with or without endocarditis should be examined for a GI tract malignancy. Work-up: - Blood cultures (BCs) are the most important tests. -- Gram Stain: Gram-positive cocci

in pairs or chains. - The barium enema should be performed on patients with S bovis bacteremia or endocarditis.- Liver ultrasound and CT scan should be performed in cases of associated hepatobiliary disease. Treatment: The antimicrobial therapy for both Streptococcus viridans and S bovis endocarditis is identical. i.e.: intravenous penicillin G for 4 weeks. Ceftriaxone is an alternative to penicillin and is administered once a day, which may facilitate outpatient therapy. Vancomycin is useful for patients with a history of major penicillin hypersensitivity (immunoglobulin E [IgE]-mediated). However, 1 strain of S bovis has been isolated that is resistant to vancomycin. Resistance mechanism (Van A) is identical to the one identified in most vancomycin-resistant enterococci (VRE). Complications for S bovis infection are similar to infective endocarditis caused by viridans streptococci. Reference: - CMDT- Emedicine- Medline

Genetic Counseling TURNERS SYYDROME,DOWNS SYNDROME, IN A PREGANANT LADY GIVING BIRTH TO A CHILD WITH THE DEFECT WHEN SHE ALREADY HAS A CHILD WITH THE DISORDER?TURNER SYNDROME: Can Turner Syndrome be inherited? NO. Like any chromosomal syndrome, if parents show a normal karyotype it is not inherited. So, we can say it has happened by chance, it is not anything we can avoid and it is not the result of something that has happened during the pregnancy. Then why does Turner Syndrome Happen?There are two theories that try to explain this chromosomal anomaly (the loss of one of the sexual chromosomes): A.- The “MEIOTIC” theory says that during the formation of the ovule or sperms (gametogenesis) some of them could have suffered an error and for this reason they carry a sexual chromosome less. If the ovule or the sperm have suffered this chromosomal loss, the person formed from the fertilisation will carry this chromosomal error. B.- The “MITOTIC” theory assures that the loss of one of the chromosomes is not produced in the gametes (ovule or sperm) but it is originated later, during the first period of the embryo growth (in the first gestation weeks). The most recent investigations support this last theory not the first. Are there possibilities to have another baby with Turner Syndrome? As previously commented, Turner syndrome is not inherited, therefore, al mothers have the same possibility of having a Turner girl; it is one of every 5000. The fact of having a Turner girl yet doesn’t increase the probability of having another one; they are two independent facts, unless any of the parents suffer from an alteration in his/her sexual

chromosomes, if this happened they should consult their doctor. In addition, Risk doesn't increase by maternal age. Furthermore, 75% will result in spontaneous abortion. Of those who escape, 50% will have a full monosomy, but 40% will have a mosaic of 45X and 46XX. DOWN SYNDROME: Are some people more likely than others to have a baby with Down syndrome? A young mother who has one child with Down syndrome has about a 2 in 100 chance of having another. An older mother will continue to have about the same chance for her age.If the mother has been diagnosed with a chromosome abnormality, she has an increased risk of having a baby with Down syndrome. In addition, The risk of Down syndrome increases with the age of the mother. The chance goes up quickly when the mother is over 34 years. It also goes up when the father is over 60 years of age. Female with ah/o PCOD and dysmennorhoea wishes to get pregnant . which drug answers her both infertlity and dysmenorrohea problem?Treatment with an insulin-sensitizing agent such as metformin (Glucophage), in a dosage of 500 mg two to three times daily, has been shown to:- improve insulin sensitivity - decrease serum LH - Decrease free testosterone levels. - restore menstrual cyclicity in 68 to 95 percent of patients treated for as short a time as four to six months. Although insulin-sensitizing agents show promise in the treatment of polycystic ovary syndrome, there are no studies of adequate power or design to allow them to be recommended as standard therapy, especially in women with normal glucose function. However, the mainstay of treatment for infertility in PCO is Clomiphene Citrate. Ovulation is successful in approximately 75 percent of women treated with clomiphene, but subsequent pregnancy rates are only 30 to 40 percent. An alternative: Women who do not respond to clomiphene or are unable to conceive with clomiphene therapy may be treated with human menopausal gonadotropins such as follitropin alpha (Gonal-F). This therapy has achieved pregnancy rates of 58 to 82 percent, but the risks from ovarian hyperstimulation and multiple pregnancies remain of concern. In fact one of Clomiphene Citrate's side effects is Dysmenorrhea. Pergonal stimulates follicular maturation directly and therefore has no effect on dysmenorrhea either as far as I understand. OCP's in PCO inhibit the enogenous secretion of FSH and LH, and restore the cyclicity of menses thereby taking care of the dysmenorrhea. OCP does have effect on hirsutism by decreasing testosterone and increasing sex harmone binding globulin production in liver.Reference: American Association of Family Practice AAFPhttp://www.aafp.org/afp/20000901/1079.htmlSmoking Cessation effects: Does it parallel non-smokers?- The most important longitudinal (prospective) study of lung function in smokers who have early COPD is the Lung Health Study (JAMA: 272:1497, 1994): Smoking cessation

for more than a year leads a slight improvement (i.e.: INCREASE) in lung function during the first year, then a gradual decline thereafter. The FEV1 DECLINE then was equal to that of non smokers (20-30 mL/year). So by 5 years, the FEV1 should be DECLINING!!!. THE RATIO SHOULD NOT BE DECLINING SINCE THE DECLINE IS IN BOTH FVC AND FEV1. Other studies revealed the following: - Quit before 40yo => at 65yo, no detectable difference in lung function compared with non-smokers- Quit between 40 and 60 => at 65yo, Lung function was equal to non-smokers older that then 2.5 to 7.5 years (i.e.: they have the lungs of a 67.5 to 82.5 yo). - Quit after 60 yo => at 65yo, lung function is equal to that of older patients of 5.5 to 15 years. Morale of this: the earlier you quit the better.

- Quitting decreases the overall risk of death (all causes combined) by 50% in 15 years as compared to continuing smokers. - The risk of heart attack is decreases by 50% within 24 hours of quitting smoking, due to reversal of nicotine's effects on inducing platelet aggregation and vasospasm. After 2 years of abstinence from tobacco, the risk of heart disease is decreased to about the level of never smokers. - The risk of having a stroke or a brain aneurysm go down by 30-50% in quitters. - The risk of lung cancer is decreased by 80-90% after 15+ years of abstinence but never reaches levels of non-smokers. - Within three months of quitting, the smokers' cough disappears in most people. Reference: - Post GRaduate MEdicine onlinehttp://www.postgradmed.com/issues/1998/12_98/hays.htm- JAMA: +>Higgins MW, Enright PL, Kronmal RA, et al. Smoking and lung function in elderly men and women: the Cardiovascular Health Study. JAMA 1993;269(21):2741-8 +>Anthonisen NR, Connett JE, Kiley JP, et al. Effects of smoking intervention and the use of an inhaled anticholinergic bronchodilator on the rate of decline of FEV1: the Lung Health Study. JAMA 1994;272(19):1497-505 Postinfarction IschemiaClinical Scenario: A 68-year-old Asian man with a previous cardiac history, 9 days status-post inferior wall MI, presented with the chief complaint of chest pain 6 times a night since MI.The patient's chest pain was pressing and radiating; it started in the chest and spread to the throat, jaw, lateral nose, and under each eye. Episodes lasted 5-15 minutes, hourly between midnight and 6 am. Previously active, the man now spent most of his time in bed. His medical history was significant for coronary artery bypass graft surgery twice, 5 and 12 years ago. He denied any arrhythmias or congestive heart failure. The review of systems was notable only for skin allergies. There were no indications of diabetes.His medications included nitroglycerin, lansoprazole, atorvastatin, and atenelol.BP: 114/69 mm Hg. His skin showed evidence of excoriation, a few ecchymoses, and

cherry angiomata. On his chest was a well-healed sternotomy scar with a purple area at the inferior aspect. Lungs and heart were normal. Lower extremities were cool with venous stasis. EKG: changed from previously, reflecting an acute MI. Cardiac enzymes confirmed this. Hypercholesterolemia was also noted. Angiography demonstrated multiple vessel occlusion and good motility. POST INFARCTION ISCHEMIA - What is it? Anginal pain at rest, which develops within 2 weeks after Acute Myocardial Infarction episode. - Incidence: 30% of patients. - Who'll get it?: Especially if pt had angina prior to the MI or non-ST Seg Elev MI. - Why is it important?: increased short- and long-term mortality. - How does it happen?: inadequate blood flow throuh a recanalized vessel /OR/ reocculsion. Trials of Calcium channel blockers for prevention didn't show results suggesting pathophysiology is different from that of Pre-MI angina. - How it is managed?: Nitrates, BEta-blockers, Aspirin, Heparin, and platelet-glycoprotein IIb/IIIa antagonists. - What if medical therapy contraindicated? Then take patient to Cath lab and do PTCA or CABG. - Does it happen in the same territory?: There are two forms of post-infarction angina: ischemia at a distance (angina with new electrocardiographic changes distant from the acute infarct) and ischemia in the infarct zone (angina with new electrocardiographic changes limited to the leads originally involved by the acute infarct). - What's the morbidity and mortality of this condition? It is associated with risk for infarct extension, repeat infarction, or sudden death. Despite numerous therapies including intra-aortic balloon angioplasty, this condition remains associated with a poor prognosis. References: - CMDT- New England Journal Of Medicine

CATCH UP IMMUNIZATION SCHEDULECHILD ABOUT TO ENTER SCHOOL. REQUIRED VACCINATIONS? - 4 doses of DPT (the last before 4 yo)- Three doses of Polio- Two doses of MMR OLDER CHILD NOT IMMUNIZED. INQUIRING ABOUT CATCH UP IMMUNIZATION SCHEDULE +>CHILDREN 4 MONTHS TO 6YO:- Chickenpox (Varicella): at any age if they have never had chickenpox. People who do not get the vaccine until 13 years of age or older should get two doses, 4-8 weeks apart. - DTaP: a five-dose series. 4 weeks should elapse between #1/#2/#3 then 6 months period elapse between 3rd/4th and 4th/5th dose. If 4th dose given after 4 yo, 5th dose is NOT necessary.

- Measles, Mumps and Rubella: Children should get two doses of MMR vaccine anytime provided 4 weeks have elsapsed between the two. - Polio: 4 doses at 4 weeks interval each. If third one is given after 4 yo, no need for a 4th dose!!!! - Hepatitis B: 3 doses with 4 weeks between #1 and #2 and 8 weeks between #2 and #3 - Hemophilus Influenzae: 4 doses with 4 weeks interval if started before 12 months old. Stop giving it after ANY dose given >15 months old. If 1st dose given after 15 months old, only ONE dose is needed!!!! If second dose given after 15 months old, stop there (Total 2)!!!. and so on. +>CHLIDREN OF 7YO TILL 18 YEARS: - Td: 3 doses with 4 weeks interval between #1 and #2, and 6 months interval between #2 and #3. Every ten years thereafter. - Polio: IPV 3 doses 4 weeks apart. - Hep B: 3 doses 4 weeks between #1 and #2 and 8 weeks between #2 and #3. - MMR: 2 doses 4 weeks apart. - Varicella: 2 doses 4 weeks apart. ADULT NOT VACCINATED AS A CHILD: - Influenza: 6 months or older. One shot/year for designated population - Pneumococcal: If before 65yo, repeast seconds shot 5 years later. After 65yo, only one is enough. Who needs it? Adults age 65 and up, Anyone who has chronic heart, lung, or liver disease, diabetes, sickle-cell disease, alcoholism, or compromised immunity or who has had spleen removed. - Meningococcus: 2 years or older. One dose for those with increased risk (travel to meningitis belt, dorms, endemic areas, healthcare personel) - Hepatitis B: Hepatitis B 3 doses; at least 4 weeks between 1st and 2nd doses and at least 8 weeks between 2nd and 3rd.Who needs it? At risk populations: All adolescents ages 11-19. • Sexual partners of and people who live with hepatitis B carriers. • People who have multiple sex partners or abuse injectable drugs. • Anyone on kidney dialysis. • Health-care workers. • Clients and workers in institutions for the mentally impaired. • Prisoners. • Travelers to endemic areas. - Hepatitis A: 2 years or older. 2 doses at least 6 months apart. Who needs it? Travelers to endemic areas. • Anyone with chronic liver disease or clotting-factor disorders. • Illicit-drug users. • Male homosexuals with multiple partners. • Food handlers in high-incidence communities. - Tetanus/Diphteria: If receiving for the first time, patients should get a series of three shots: the second and third will follow two and six months (respectively) after the first. Patients should get boosters every 10 years. - MMR: 1 dose. Who needs it? Adults born after 1957 (patient is >47years old now), who've never been immunized, should get one shot. College students and health care workers are advised to have a second dose (at least 4 weeks between doses). - Varicella: 2 doses; 4-8 weeks between doses. Who needs it? Anyone who wasn't vaccinated with it before. - Lyme Disease: 3 doses; 1 month between 1st and 2nd doses, 11 months between 2nd and 3rd doses. Who needs it? Anyone who frequently spends time in tick-infested habitat in endemic areas should consider the vaccine.

- Polio Vaccine: 3 doses of IPV: The first dose at any time, The second dose 1 to 2 months later, The third dose 6 to 12 months after the second. Who needs it? people traveling to areas of the world where polio is common, laboratory workers who might handle polio virus, and health care workers treating patients who could have polio ANSWER TO PREVIOUS QUESTIONS FROM EXAM:

So for example a 6 yo who wasn't immunized before and mother brings her in now for that should get:- DtaP: 4 doses- Varicella: 1dose (2 if she were 13yo)- MMR: 2 doses- Hemophilus Influenza: 1 dose- Polio: 3 doses (because >4yo)- Hepatitis B: 3 doses An 35yo adult who wants to catch up: - Td: 3 doses then every ten years get booster- MMR: 1 dose. - Varicella: 2 doses- Polio: 3 doses if indicated. - Influenza: once a year if indicated- Pneumococcal: Once if indicated. Repeat once after 65yo. - Meningococcus: 1 dose if indicated- Hepatitis B: 3 doses if indicated- Hepatitis A: 2 doses if indicated References: - www.CDC.giv/nip- http://www.aafp.org/adultimmunizations.xml- Polio Vaccine Facts http://www.parentsplace.com/health/vaccines/articles/0,10335,166606_222430,00.htmlTeratogenic effect of IsotretinoinWHAT KIND OF TERATOGENIC EFFECT DOES ISOTRETIONON CAUSE? Answer:- Behavioral Teratogen- Craniofacial anomalies: Cleft lip and Cleft Palate, maxillary and mandibular hypoplasia, Flat nose bridge, Fusion of teeth - Cardiac and CNS abnormalities - Thymus developmental abnormalitiesisoretinoin also increases triglycerides in the body .QuestionsTransient sensiry symptom electric shock-like precipitated by neck flexion is a recognized feature of:A- MeniereB- PresbycusisC- Multiple Sclerosis

D- AminoglycosidesE- Acoustic NeuromaThis is Lhermitte sign beleive it or not. It's Multiple Sclerosis. :) It's one of the remembered MCQ'sPatient with classic retrosternal chest pain of GERD. PE: thick skin of finger and toe pads. Ds? Ans: SclerodermaYoung man come to clinic with anal discharge, anal itching, soreness, bleeding, and sometimes painful bowel movements. Has multiple sex partners.Ans: Gonorrhea or the rectum. In a child with sickle cell disease1.immunization for pnemococci &H.inf.2.penicillin prophylaxis till 5yrs age.Do we have to give both or any specific indications for each?ThanksAns: I would say both according to my review. (source Medscape) "Penicillin prophylaxis of patients with sickle cell anemia significantly decreases the risk of septicemia and death due to encapsulated micro-organisms and is recommended for all children starting at 2 months of age.... since infants may not be protected by maternal antibodies beyond that time...it was agreed that a 14-valent pneumococcal vaccine would be administered to all children at 1 year and 2 years of age.". Reference: the Prophylactic Penicillin Study (PROPS)multicenter, double-blind, placebo-controlled trial. Gaston MH, Verter JI, Woods G, et al. Prophylaxis with oral penicillin in children with sickle cell anemia: a randomized trial. N Engl J Med 1986;25:1593-99. Falletta JM, Woods GM, Verter JI. Discontinuing penicillin prophylaxis in children with sickle cell anemia. J Pediatr 1995;127:685-90. -> delerium and schizo have what in common? waxing and waningmemory impairmenthallucinationfamily hx of psychopathologysocial withdrawalANS: Hallucinations. They're auditory in schizophrenia, and visual mostly in delirium.

Why does hyperlipidemia cause pseudohyponatremiaHyponatremia associated with normal plasma osmolality can occur when there is a reducation in the fraction of plasma that is water. In normal subjects, the plasma water is approximately 93% with the reminaing 7% made of Lipids and proteins. Thus, a normal plasma sodium concentration of 142mmol/L actually represents a concentration in the physiologically important plasma water of 154 mmol/L. However, the plasma fraction may fall below 80% in patients with marked hyperlipidemia (as with lactesent serum in uncontrolled diabetes mellitus), or hyperproteinemia (as in multiple myeloma). In these settings, the plasma water sodium concentration and plasma osmolality are unchanged, but the measured sodium concentration in the total plasma volume will be reduced (since the specimen contains less plasma water).

Reference: Systematic approach to hyponatremiahttp://www.acta-clinica.kbsm.hr/Acta2002/ACTA2002_2/05RATK~1.pdf

Coombs test mystery REVEALEDYou take the patient's blood specimen and centrifuge it. Separate:- Serum- RBC's #1: DIRECT COOMBS: Take the RBC's and expose them to KNOWN ANTIBODIES. Wherever there is coagulation, THAT'S THE ANTIGEN COATING THOSE RBC's. e.g.: ABO TYPING. Patient's RBC's exposed to anti-A, anti-B, and anti-AB. e.g.: RHESUS TYPING. Patient's RBC's exposed to anti-D. e.g.: DIRECT COOMBS USED TO KNOW BABY'S RHESUS TYPE IF MOTHER IS RHESUS NEGATIVE. #2: INDIRECT COOMBS: Take the patient's SERUM and add to it KNOWN RBC's. Wherever there is coagulation, THAT'S THE ANTIBODY CIRCULATING IN THE PATIENT'S SERUM. e.g: Mother's serum to see if there are CIRCULATING ANTI-D ANTIBODIES.e.g.: BABY'S SERUM to see if there are CIRCULATING ANTI-D ANTIBODIES therefore COATING BABY's RBC's. QUICK REVIEW: - TYPE AND RHESUS DETERMINED WITH DIRECT COOMBS TEST- ANTIBODY SCREEN DETERMINED BY INDIRECT COOMBS TEST.Pulmonary Capillary Wedge Pressure REVEALEDPulmonary Capillary Wedge Pressure The measurement of pulmonary capillary wedge pressure (PCWP) provides an indirect measure of left atrial pressure and is particularly useful in the diagnosis of left ventricular failure and mitral valve disease. The measurement is made as follows. A balloon-tipped, multi-lumen catheter (Swan-Ganz catheter) is advanced from a peripheral vein into the right atrium, the right ventricle, and then positioned within a branch of the pulmonary artery. There is one opening (port) at the tip of the catheter (distal to the balloon) and a second port several centimeters proximal to the balloon. These ports are connected to pressure transducers. When properly positioned in a branch of the pulmonary artery, the distal port measures pulmonary artery pressure (~ 30/15 mmHg) and the proximal port measures right atrial pressure (~ 0-2 mmHg). The balloon is then inflated with air using a syringe (the balloon volume is about 1 ml) and this occludes the branch of the pulmonary artery. When this occurs, the pressure in the distal port rapidly falls, and after about 10 seconds, reaches a stable lower value that is very similar to left atrial pressure (normally about 8-10 mmHg). The balloon is then deflated. The recorded pressure during balloon inflation is similar to left atrial pressure because the occluded vessel, along with its distal branches which eventually form the pulmonary veins, acts as a long catheter which measures the blood pressures within the pulmonary veins (this pressure is virtually the same as mean left atrial pressure). A PCWP exceeding 15 mmHg suggests - mitral stenosis

- mitral insufficiency- severe aortic stenosis- aortic regurgitation- ventricular failure When the PCWP exceeds 20 mmHg, the transmission of this pressure back into the pulmonary vasculature increases pulmonary capillary hydrostatic pressure which can lead to pulmonary congestion and edema. Other indications: - evaluating blood volume status when fluids are administered during hypotensive shock (maintain PCWP at 12-14)- differentiating cardiogenic from non-cardiogenic (Septic shock - ARDS, etc.) = > PCWP in caardiogenic will be elevated in the presence of pulmonary edema, but it would be normal or low in non-cardiogenic shock. Nerve SectionClean wound cut w/laceration and incomplete section of the nerve, management?a. suture of wound immediatelyb. leave the wound openTo answer your Q first:Surgical exploration immediately with suture of the nerve and the wound. SUMMARY AND USMLE TAKE HOME MESSAGE:OPEN LACERATION: SURGICAL EXPLORATION IMMEDIATELY, AND SUTURE OF THE NERVE IF INDICATED. CLOSED INJURY: USUALLY NOT EMERGENT REPAIR OF THE NERVE UNLESS SURGERY INDICATED FOR OTHER PURPOSES (E.G.: VASCULAR). Review of Emergency Nerve SurgeryIn this evaluation we have to distinguish two cases: • Open lesion • Closed lesion A- OPEN LESION: RULE OF THUMB: Surgical exploration is imperative in every case of open lesion. The wound exposure may be of various types:• Punctiform or minimum• Extended• With sub-amputation Moreover, the lesion may be:• SIMPLE: = clean-cut lips and involvement limited to => the risk is to merely practice a skin suture, not being able to recognizee the nervous lesion, with the obvious severe medical legal consequences. The operation must be done under general or peripheral anaesthesia, preferably without curarizing the patient, in order to permit, during the operation, a normal excitability of the motor endplate. Nerve dissection must be limited to a few centimetres in order not to damage its vascular contributions. If there is a paralysis the nerve is usually interrupted and nerve suture must be done under operative microscope. • COMPLICATED = with contused lips, poli-tissutal involvement (nerve, bone, vessel, skin), and contamination => In these cases the same principles mentioned above are valid . If the lesion is clean, a suture will be practised, otherwise if the nerve is frayed for a

tract, or there is a loss of nervous substance and it is not possible to practice a direct suture, nervous grafts will be necessary (Secondary repair). B- CLOSED LESION: usually it is not necessary to explore the nerve in emergency. Some exceptions exist to this principle, when associated lesions (usually vascular and/or bone lesions) must be treated. ADJUVENT THERAPY: Rehabilitation treatment is absolutely necessary in these lesions to prevent and treat stiffness using even orthopaedic devices for hand or fingers. Electrotherapy of the denervated muscles is also suggested. The recovery of the nerve lesions is usually slow but, if the treatment has been correct, it can be fairly good. Reference: http://www-cdu.dc.med.unipi.it/eates4/Invitedspeaker/Trauma/vigasio.htmfactor V Leiden mutationGeneral Considerations:Factor V Leiden is a genetically acquired trait that can result in a thrombophilic (hypercoaguable) state resulting in the phenomenon of activated protein C resistance (APCR). The mutation refers to the specific G-to-A substitution at nucleotide level in the gene for factor V that predicts a single amino acid replacement (Arg) to (Gln) the cleavage sites for Protein C in the factor Va molecule.The result is Factor V Leiden is inactivated at a rate approximately ten times slower than normal factor V and persists longer in the circulation, resulting in increased thrombin generation and a mild hypercoagulable state. Mechanism of action: The function of protein C is to inactivate factor Va and factor VIIIa. The first step in this process is the activation of thrombomodulin by thrombin. Subsequently, protein C combines with thrombomodulin in order to produce activated Protein C. Activated protein C then combines with protein S on the surface of a platelet. Activated protein C can then degrade factor Va and factor VIIIa. When one has factor V Leiden, the factor Va is resistant to the normal effects of activated protein C, thus leading to a thrombophilic state by having increased activity of factor V in the blood. Factor Va is an essential cofactor for the factor Xa-catalyzed activation of prothrombin to the clotting enzyme thrombin. Having too much of it will generate too much thrombin and a hypercoagulable state. Epidemiology: Patients are mostly northern european: Swedish, greek, etc. Risk for DVT in case of mutation: 1 in 1000If heterozygote allele: autosomal dominantIf homozygote allele: autosomal recessive Diagnosis: Suspect it when you have: - A first venous thrombosis at less than 50 years of age - A first unprovoked venous thrombosis at any age Recurrent venous thrombosis - Venous thrombosis at unusual sites (such as cerebral, mesenteric, portal and hepatic veins)

- Venous thrombosis during pregnancy, the puerperium, or in association with oral contraceptives or hormone replacement therapy - A first venous thrombosis and a strong family history of venous thrombosis The diagnosis of factor V Leiden thrombophilia is made either using a coagulation screening test or by DNA analysis of the F5 gene, which encodes the factor V protein. Labs: - Prolonged PT. PT correct by mizing test (adding deprothrombinized rabbit serum). - Bleeding times and clotting times are consistently prolonged Important note: The presence of a factor V Leiden allele is not a major risk factor for arterial thrombosis. Management: - Evaluate for other inherited or acquired thrombophilic disorders- Usual management of DVT (Coumadin f/u is 3 to 6 months out patient). Prophylaxis: - No h/o DVT: no prophylaxis needed- Heterozygotes with only first episode: Coumadin hemorrhagic risk is 1-2%/year and is greater than <1%/yr DVT risk. No prophylaxis needed. - Prophylaxis in high risk settings: surgery, pregnancy, and prolonged immobilization.- Pregnancy: no consensus yet. (Enoxaparin prophylaxis throughout pregnancy?) Reference: - Gene Reviewshttp://www.geneclinics.org/profiles/factor-v-leiden/details.html- National Library of medicinehttp://www3.ncbi.nlm.nih.gov/htbin-post/Omim/dispmim?227400Dementia Qs1- Old woman with rapidly developing decreased intellectual abilities with startle myoclonus. 2- A 57-year-old man is evaluated because of progressive memory problems and language disturbance. Several months ago, he started noticingincreasing difficulty with driving, turning left instead of right and getting lost in his own neighborhood. His family states that he would repeatedlyask the same question and started forgetting easily, particularly recent events. They also noted waxing and waning difficulties with speech, anataxic gait, uncontrollable trembling, and a tendency to doze off easily. There is no family history of psychiatric illness, epilepsy, or neurologicalor neurodegenerative disorders. 3- A 45-year-old man is admitted to the hospital for the evaluation of diplopia, weakness of his lower extremities, and gait difficulties. During theinterview, the patient reveals that has been forcing himself to vomit after almost every meal over the last 6 weeks. He denies associated eyepain and discomfort, headache, or dysphagia. He has never experienced such symptoms in the past. Examination of the patient demonstratesslight disorientation, vertical nystagmus worse on downgaze, diffuse weakness of the lower extremities, bilateral dysmetria, and hypothermia.Laboratory analyses reveal mild dehydration and hypokalemia.

4- A 68-year-old woman presents to her primary care physician complaining of clumsiness and urinary incontinence. The daughter tells thephysician that her mother "walks oddly" and has been falling with increasing frequency. She says that her mother's symptoms began a year ortwo before, and gradually became worse, starting with a changing gait, followed by urinary urgency and incontinence. More recently, she saysher mother has had difficulty remembering things and has "ruined her credit rating" because she forgets to pay her bills. Her medical historyincludes gall bladder surgery 10 years ago and hysterectomy 15 years ago for abnormal bleeding. Family history is negative for strokes,aneurysms, or intracranial bleeds. She quit smoking fifteen years ago, and does not drink alcohoI. On physical examination, the physiciannotes that the patient has an ataxic gait. She does not have a tremor, and on neurologic examination, she is found to have normal strength andmuscle tone. MRI reveals enlargement of the ventricles without cortical atrophy 5- A 56-year-old man is brought to the psychiatrist with a three-year history of progressive speech difficulties associated with altered socialbehavior. The family states that he became aggressive, talked inappropriately to strangers, and showed insensitivity, contrary to his pastconsideration to others. He has had problems with speech production, using some phrases repeatedly, and showing a decreased vocabulary.He started to eat a great deaI, and has developed obsessional cravings for sweets. As a result, the patient has gained 40 pounds over thepast year. His neighbors caught him stealing things from their back yard. The patient repeats the examiner's words and imitates theexaminer's gestures. Testing reveals difficulties in naming common objects or pictures. 6- A 72-year-old man has impaired concentration and an inability to recollect names and appointments. These memory problems have becomeincreasingly worse over a period of months and begin to interfere with his social and financial activities. Subsequently, the patient becomesdepressed, and his wife persuades him to see a physician. A neurologic examination confirms the presence of moderately severe short-termmemory loss associated with disturbances in language, such as difficulty in naming familiar objects and verbal comprehension. MRI of thehead reveals diffuse cerebral atrophy without focal lesions or tumors.ANSWERS:#1: CJD. #2: CJD again. CJD is a degenerative disorder of the central nervous system that is caused by accumulation of abnormally folded protein (PrPsc) particles termed prions. Normal prion protein is termed PrPc (cellular), while an abnormal, pathogenic isoform of the prion protein is designated PrPsc. It usually presents in late middle age (50-75 yr), with rapidly progressive dementia, ataxia, dysarthria, myoclonic fasciculations, somnolence, and eventually death, usually following pneumonia, within a year of onset. MRI typically shows bilateral areas of increased intensity, predominantly in the caudate and putamen. On light microscopy, the pathologic hallmarks of CJD are spongiform degeneration, astrogliosis and the lack of an inflammatory response. Spongiform changes

occur in the putamen, caudate nucleus, cerebral cortex, thalamus, and cerebellum. #3: This is Wernicke encephalopathy caused by a nutritional deficiency of thiamine. Classic symptoms and signs include "Wernicke's triad": acute mental confusion, ataxia, and ophthalmoplegia, although not all the patients present with all of these. A common misconception about Wernicke encephalopathy is that it is seen exclusively in alcoholics. Prolonged vomiting and malnutrition, eating disorders, hunger strikes, prolonged intravenous feeding, and malabsorption syndrome can also be a potential cause of thiamine deficiency. Mental confusion is characterized by impaired awareness, spatial disorientation, and inability to concentrate. Ocular abnormalities are the hallmarks of this disease. Horizontal or vertical nystagmus and paralysis of lateral rectus muscles are common. The wide-based ataxic gait results from cerebellar dysfunction, either alone or in combination with vestibular dysfunction. #4: This patient has the classic triad of normal pressure hydrocephalus, consisting of urinary incontinence, mental status changes, and gait disturbance. NPH is a disease usually found in older adults. Most cases are idiopathic, and are due to impaired cerebrospinal fluid absorption. Subarachnoid hemorrhage or meningitis are risk factors for the future development of NPH. The CT makes the diagnosis likely because of the enlarged ventricles. #5: Pick Disease. Pick disease is a rare form of neurodegenerative disorder characterized by a distinct progressive dementing process. Presenile onset (under 65 years old), initial personality change, progressive language dysfunction, hyperorality (overeating with obsessional craving for certain types of food), and disinhibition are the key features of this disease. Compared to Alzheimer disease, memory loss and impairment of intellect occur at later stages of the disease. The symptoms of Pick disease occur because the frontal and temporal lobes are affected. Circumscribed ('knife-like") lobar atrophy is the hallmark of Pick disease, and sometimes can be so severe that the postmortem brain weight can be as low as 800 g. The atrophy affects the anterior temporal and frontal lobes, the orbital frontal lobe, and the medial temporal lobe, but spares the posterior part of the superior temporal gyrus and the pre- and postcentral gyri. These lobes are important for language skills, impulse control, energy and enthusiasm, problem solving, and maintaining socially appropriate behavior. In most cases of Pick disease, the cause cannot yet be determined. However, there is a strong genetic component in certain families. A mutation on chromosome 17 has been identified, which leads to production of an abnormal tau protein. #6: This patient has Alzheimer disease (AD), the most common cause of dementia in the Western world. His symptoms are typical of early AD and the diffuse cerebral atrophy is also characteristic; MRI scans may later show hippocampal atrophy. The common form of this disease typically affects people over age 60. In the early stage of AD, patients experience recent memory loss, personality changes, language difficulties (especially word finding), emotional lability, and diminished judgement. Some patients are aware of their difficulties, causing frustration and anxiety; others are seemingly unaware of their symptoms. Mild to moderate depression is common in early stages. Surprisingly, sociability may not be affected at this early stage. The intermediate stage is characterized by a worsening memory, both recent and remote. Various apraxias are common; patients may have difficulties with sequential motor tasks, leading to difficulties with dressing, eating, bathing, and toilet functions. Their sense of time and place are lost, leading to a

host of behavioral problems, including wandering and becoming lost. Sometimes they can be quite aggressive, agitated, and uncooperative, which can alternate with being socially withdrawn and passive. They can still ambulate, but are at risk for accidents resulting from their confusion. In end-stage AD, patients are unable to walk or perform tasks of daily living and their recent and remote memory is gone. Death generally results from malnutrition, secondary infections, and heart disease. The typical duration of the disease is 8 to 10 years.Cat Scratch Disease Vs Cat bite This is an old MCQ from Orbit #360: A 6yo girl was scratched on the left hand by a cat. 10days later she had a tender,left axillary node,T:37.8,a red left eye without any pain&discharge&a left preauricular lymphadenopathy.Aspiration of the axillary node recovered pus,from which no organism was grown on routine culture.She developed erythematous nodules and plaque over both shins 1 wk later.Which of the following statements regarding the most likely diagnosis,is not correct?A:A scratch by a kitten is more likely to cause the disease than a scratch by an adult catB:the presentation of skin nodules is a self-limited condition&corticostroids r not recommendedC:boys r affected more often than girlsD:Warthin-Starry silver stain useful to show the organismsE:azithromycin decreases the duration of disease in 50% of patients if prescribed during the first 30 daysF:Incision and drainage of nonsuppurative nodes should be avoided because chronic draining sinuses may result FIRST THE ANSWER TO THE MCQ, THEN I WILL DUPLICATE THE REVIEW OF CAT SCRATCH DISEASE. the answer is:E:azithromycin decreases the duration of disease in 50% of patients if prescribed during the first 30 days A case of CSD with erythema nodusom(which is self-limited&corticostroids have no effect)&Parinaud's occuloglandular syndrome:the most common atypical presentation,noted in 2–17% of patients,which is unilateral conjunctivitis followed by preauricular lymphadenopathy.Direct eye inoculation as a result of rubbing with the hands after cat contact is the presumed mode of spread. A conjunctival granuloma may be found at the inoculation site. The involved eye is usually not painful and has little or no discharge, but it may be quite red and swollen.A small prospective study of azithromycin shows decrease in initial lymph node volume in 50% of patients during the first 30 days, but after 30 days there was no difference in lymph node volume. No other clinical benefit was found. It is clear that for the majority of patients, the disease is self-limited, with resolution occurring over weeks to months, and that treatment affords minimal, if any, clinical benefit.Suppurative lymph nodes that become tense and extremely painful should be drained by needle aspiration, which may need to be repeated. Incision and drainage of nonsuppurative nodes should be avoided because chronic draining sinuses may result. ------------------------------------------------------------------------------------------------------------------------------------------------------------------------------------CAT SCRATCH DISEASE REVIEW:

Answer is E. Please Read following: We are trying hard to keep up with all the name changes for the culprit organism of CSD! As if any of them are easy to pronounce! We’ve gone from Rochalimaea to Afipia and now to Bartonella henselae! CSD is typically a benign and self-limited illness lasting 6 to 12 weeks in the absence of antibiotic therapy. - Regional lymphadenopathy (axillary, head and neck, inguinal) is the predominant clinical feature of CSD; affected nodes are often tender and occasionally suppurate. - Between 25% and 60% of patients report a primary cutaneous inoculation lesion (0.5- to 1-cm papule or pustule) at the site of a cat scratch or bite. The skin lesions typically develop 3 to 10 days after injury and precede the onset of lymphadenopathy by 1 to 2 weeks.- Low-grade fever and malaise accompany lymphadenopathy in up to 50% of patients; headache, anorexia, weight loss, nausea and vomiting, sore throat, and splenomegaly may develop. - In addition, short-lived, non-specific maculopapular eruptions, erythema nodosum, figurate erythemas, and thrombocytopenic purpura have been observed.- Unusual manifestations of CSD, which occur in up to 14% of patients, include Perinaud's oculoglandular syndrome (6%), encephalopathy (2%), hepatic granulomas (0.3%), osteomyelitis (0.3%), and pulmonary disease (0.2%) (4,5,8). In general, these complications resolve without sequelae. Perinaud's oculoglandular syndrome is manifested by conjunctival granuloma, periauricular lymphadenopathy, and nonsuppurative conjunctivitis. Encephalopathy, manifested as fever and coma that progress to convulsions, may last for days to weeks; cerebrospinal fluid is unremarkable. Optic neuritis with transient blindness may also occur.Bartonella henselae is now regarded as the etiologic agent of CSD. This agent has requirements necessary for growth in vitro, including enriched, non-selective blood agar incubated over a prolonged period in a CO2 atmosphere. The bacterium is most frequently isolated in young, male kittens that are not ill and require no treatment, but that serve as passive vectors in transmitting the disease to humans. Fleas and ticks have also been associated with the transmission of CSD, although more evidence is required to establish ticks as vectors. Transmission of CSD occurs from a bite, scratch, or petting, as a result of direct contact with the cat’s saliva. The saliva is deposited on an infected cat’s fur and claws from self-grooming. Historically the diagnosis of CSD is confirmed by three of the four following criteria: 1) history of cat contact and the presence of a primary lesion, 2) development of lymphadenopathy approximately 2 weeks after the primary inoculation, with the exclusion of other causes of lymphadenopathy, 3) a positive CSD skin test, and 4) histopathologic findings on lymph node biopsy specimens showing a granulomatous process with stellate necrosis and pleomorphic bacilli visualized by Warthin-Starry silver staining. A basic laboratory evaluation of CSD should also include a CBC with differential to rule out other causative conditions and a TB skin test to rule out tuberculosis. The erythrocyte sedimentation rate is usually elevated during the initial stages of lymphadenopathy, and the CBC is normal or shows mild leukocytosis.

Currently available for diagnostic purposes are the serum immunofluorescent-antibody (IFA) test for B. henselae, which has 91% positive predictive value, a 96% specificity, and an 84% sensitivity, and the enzyme immunoassay (EIA) test, which is thought to be the most accurate. Although the CSD skin test has a high degree of specificity and has been used for over 40 years to confirm a diagnosis of CSD, the test is not standardized, is not available commercially, and is no longer recommended. Differential: It can include virtually all known causes of lymphadenopathy. As a general rule the diagnosis is favored by chronicity, unilateral occurrence, tenderness and characteristic sites of involvement, such as the axillary, epitrochlear and preauricular nodes. Cervical, femoral, inguinal and generalized lymph node involvement is less specific for CSD and necessitates more care in differential diagnosis. The most common diagnoses in a series of patients with adenopathy and negative CSD skin test were pyogenic lymphadenitis or abscess, benign of malignant neoplasms, and cervical adenitis caused by mycobacteria. Tularemia, toxoplasmosis, plague, and Kawasaki disease must be considered because of the need for specific therapy. Complications: The most serious complication of CSD is involvement of the CNS in the form of encephalitis or encephalopathy. High fever and convulsions develop within 6 weeks of the onset of lymphadenopathy followed by alteration in the level of consciousness, headache and muscle weakness. The CSF is normal or shows minimal pleocytosis or elevated protein content. EEGs reveal diffuse slowing or focal abnormalities. Recovery has occurred without residua in most cases. Neuroretinitis one of the most frequently reported neurologic syndromes, has been serologically and microbiologically linked to B. henselae infection recently. The syndrome of Leber’s idiopathic stellate neuroretinitis is characterized by visual loss, stellate macular exudates, and optic disc edema. It has been associated with CSD in both children and adults. Osteolytic bone lesions far from the inoculated site have been noted in several well-documented cases, suggesting hematogenous spread. Granulomatous hepatitis is another newly recognized systemic manifestation of CSD, and it may present as fever of unknown origin with or without lymphadenopathy. Other rare complications include erythema multiforme, thrombocytopenic purpura, mesenteric lymphadenitis, pneumonia, arthralgia, neuroretinitis, iritis, urethritis, thyroiditis and non-traumatic atlanto-axial dislocation (Grisel syndrome). Treatment: CSD is almost uniformly a self-limiting illness, regardless of antibiotic treatment. For the majority of patients with CSD, management should include conservative symptomatic care and observation. Complete resolution of lymphadenopathy usually occurs after 2 to 6 months. Patients with severe CSD (encephalopathy, neuroretinitis) may have a shortened course and thus benefit from antibiotic therapy, but choice of antibiotics is unclear.In AIDS patients and in other people who have suppressed immune systems (chemo, Diabetes, ESRD..), cat scratch disease is not benign, and treatment with antibiotics is recommended.In a retrospective study of uncontrolled data, Margileth (1992) suggested the efficacy of oral rifamin (87%), ciprofloxacin (84%) TMP/SMX (58%), and intramuscular gentamicin

(73%) for improving clinical symptoms. In the same 1992 study, oral rifampin was prescribed using 10 to 20 mg/kg two to three times daily for 7 to 14 days, oral TMP/SMX using 6 to 8 mg/kg two to three times daily for 7 days, and oral ciprofloxacin using 20 to 30 mg/kg twice daily for 7 to 14 days. Intramuscular gentamicin was prescribed for severely ill patients using 5 mg/kg in divided doses every 8 hours for 72 hours. Collipp (1992) reported improvement in 101 children with CSD by treating with 20 mg/kg of TMP/SMX twice per day for 7 days. Paradoxically, children who are immunocompromised appear to respond quite well to common antibiotics, such as erythromycin and doxycycline, prescribed over several weeks or months. but failures also have been reported after treatment with gentamicin and TMS-SMX. In addition, in vitro and in vivo antibiotic susceptibilities to Bartonella species often do not correlate and cannot be used to guide antibiotic recommendations. Warm moist compresses to affected nodes may decrease swelling and tenderness. Isolation of the affected individual is not required, and there is no evidence that CSD is transmitted by humans. Although the complication of CSD can be serious (central nervous involvement, hematological disorders, boney lesions, and abscesses), they are relatively rare, and most patients with CSD recover fully with no permanent sequalae in a variable period of time from 2 to 4 months. Relapsing Bartonella is rare, and one episode of CSD appears to confer lifelong immunity in children and young adults. Aspiration may be helpful for those cases in which suppuration occurs. Incision and drainage carries the reported risk of sinus tract formation and formal node excision may be preferable.Families become concerned about the transmission of CSD to other family members by infected cats. Cats seldom infect more than one family member, even though others may have been scratched by the same cat. Periods of transmission are thought to be limited (possibly 2 to 3 weeks), and the cats do not appear ill in any way. Disposal of the offending cat is not recommended, but using good judgment when handling cats and kittens is essential. Young cats and kittens may be most frequently implicated in CSD transmission because they are held more often than are older cats and are less experienced in getting away. Consistent with the 1994 CDC guidelines, parents and cat owners should consider the following recommendations: (a) wash hands after petting or playing with cats; (b) treat animals for fleas and ticks; (c) wash cat scratches, cuts, or bites immediately; (d) never allow cats to lick open wounds; (e) declaw cats if possible; (f) handle cats and all animals gently; (g) instruct children to avoid contact with stray animals; (h) supervise young children, especially toddlers, when handling cats; and (i) ensure routine veterinary care for all pets. Reference: - CDC- Medline- Grand Rounds Archives:http://www.bcm.tmc.edu/oto/grand/12592.html

------------------------------------------------------------------------------------------------------------------------------------------------- Second MCQ is also an old one Q404:

A 20 yo man comes to ur office with a dog bite to his left thigh received after he and a friend taunted a neighbor's dog.He reports that the bite occurred about 36 hrs ago and only came to your office after coworkers informed him that dog bites frequently become infected.His temperature is 37,BP:110/70 mm Hg,HR:63,RR:13.On Ph.E you notice a shallow abrasion on his left thigh, which is mildly tender.There is no surrounding edema or erythema. The most appropriate management of this patient is to:a:administer 5 doses of Rabies vaccines within 28daysb:administer RIG&4 doses of Rabis vaccine within 28daysc:prescribe amoxicillin-clavulanated:prescribe clindamycine:prescribe penicillinf:administer RIG& penicilling:administer 5 doses of Rabies vaccines within 28days& amoxi-clavulanate h: administer 5 doses of Rabies vaccines within 28days& clindamycini:provide local wound care without antibiotic & vaccination ANSWER: I.provide local wound care without ABX & vaccination., pt. has small , shallow abrasion, no edema,no erythema- ABX not necessary,the neighbor's dog is a domestic dog, most likely vaccinated, if not sure you can check w/ public health dept. before vaccinating the pt, most likely dog is not rabid. :provide local wound care without antibiotic & vaccination Only 5-20% of dog bites become infected.This pt does not seem to have any systemic signs of infection and his wound doesn't appear to be infected.Puncture wounds become infected more often than abrasion and this pt doesn't have any skin punctures.Local wound care including debridement,cleaning,and irrigation are essential.Rabies vaccination is not indicated now.Vaccination for rabies should be considered only when you have a strong suspicion of rabies.In this case, the dog was a household pet, which probably indicates that it was vaccinated in the past.Also, the dog did not seem rabid (the dog bite occurred only after being provoked). If the dog spontaneously bit the patient,suspicion might be higher.If rabies is suspected, an attempt to contact the dog owner is a reasonable first step to determine the pet's vaccination status. Local wound care remains important.Antibiotics would be indicated for pts with signs of local or systemic infection.Hand bites should also be treated with antibiotics since infection can be devastating to a patient.Amoxicillin-clavulanic acid and clindamycin would both be acceptable antibiotics since they have broad spectrums covering likely infectious organisms such as Pasteurella multocida, Staphylococcus, Streptococci, and anaerobes.Penicillin is a well studied choice for prophylaxis of hand bites but concerns about narrow spectrum of activity have caused many physicians to use alternative antibiotics.

ADDENDUM: IS A DOG/CAT'S MOUTH CLEANER THAN A HUMAN MOUTH? Animal and human bites carry a high risk of infectious complications. Human bites and in particular clenched-fist injuries as well as cat bites are highly prone to infection as are wounds that involve the hand or deep structures including joints, bones and tendons. The management of bite wounds consists of intensive irrigation with large volumes of normal saline and a cautions debridement of devitalized tissues. Generally, it appears prudent to

leave the wounds open, however, in cases carrying a low risk of infection, a primary surgical closure might be appropriate. If a bite wound is infected, an antibiotic course with amoxycillin/clavulanic acid (first choice) or tetracyclines (second choice) for 10-14 days is recommended. In patients who present early after the injury, an antibiotic prophylaxis for 3-5 days is appropriate, particularly when the risk for the development of infection is high. Furthermore, a tetanus booster and in case of possible transfer of rabies, a rabies vaccination with immunoglobulins and inactivated virus preparation is recommended.Human bite wounds have long had a bad reputation for severe infection and frequent complication. However, recent data demonstrate that human bites occurring anywhere other than the hand present no more of a risk for infection than any other type of mammalian bite. The increased incidence of serious infections and complications associated with human bites to the hand warrants their consideration and management in three different categories: occlusional/simple, clenched fist injuries, and occlusional bites to the hand.

References: 1)Dog, cat, and human bitesSchweiz Rundsch Med Prax. 1998 May 20;87(21):716-8. 2)Dog, cat, and human bites: a review.J Am Acad Dermatol. 1995 Dec;33(6):1019-29. 3)Diagnosis and treatment of bites by cats, dogs and humansDtsch Med Wochenschr. 2003 May 9;128(19):1059-63. Imp... Tips On Diabetes and SurgeryFor non–insulin–dependent diabetes: perioperativeIntravenous solutions containing glucose should be avoided unless hypoglycemia is a risk.If the operation is major, treatment should be the same as for insulin–dependent diabetes until the stress response of surgery is finished . For insulin–dependent diabetes: preoperativePreoperative admission for stabilization may be needed to achieve a blood glucose concentration of 100–180 mg/dL.If control is good, the insulin regimen need not be changed until the day of surgery; if control is poor before a meal, short–acting insulin achieves rapid metabolic control.Blood glucose concentration must be monitored.The main side effect is hypoglycemia. For insulin–dependent diabetes: perioperativeShort–acting insulin, 50 units in 50mL normal saline solution with an infusion pump; 500mL 10% dextrose with 10mEq/L potassium chloride through a separate infusion pump infused 5–hourly.Blood glucose concentration must be maintained at 100–180 mg/dL; 1–4 units of insulin may be needed hourly via an infusion pump Alternatively, glucose–potassium–insulin infusion: 10% dextrose, 500 mL, with potassium chloride, 10 mEq/L, and short–acting insulin, 15 units infused as a mixture 5–hourly.Premixed insulins are unsuitable when insulin need changes rapidly.

If blood glucose is >200 mg/dL, 20 units insulin should be added; if blood glucose is <100 mg/dL, 10 units should be added; other parameters should not be changed.The insulin infusion must be continued until the patient's first meal, when s.c. insulin should be started 30 min prior to cessation of insulin infusion; insulin needs will probably be higher than usual.please explain "dicrotic pulse, pulsus bigeminus, pulsus alternans?"- Normal Arterial pulse: The normal arterial wave form has a smooth, fairly sharp upstroke, a momentarily sustained peak and a quick downstroke. - Dicrotic pulse: A dicrotic pulse is one in which two impulses are felt for each heartbeat; the second small upstroke appearing in the blood pressure wave during the diastolic phase. It literally means twice beating pulse. It is only clinically apparent in pyrexic patients, and some valvulopathies. - Pulsus Alternans: also called pressure alternans. This is the situation where the amplitude of a regular pulse is large and small alternately. Pressure Alternans is a sign of a failing ventricle. - Pulsus bigeminus consists of groups of two beats close together, followed by a longer pause. It is associated with every contraction and is an alternate contraction produced by the extra systolic PVC. - Pulsus paradoxus is caused by a greater than normal decrease in systolic pressure and pulse wave amplitude during inspiration. It is associated with circumstances in which respiration is labored and often accompanies such conditions as emphysema, pulmonary embolus, cardiac tamponade or lung cancer.Questions8yo child by stander with gun shot wound in the epigastric region..stable..next step..choices exploratory lap,usg,tetanus prophy, xray abdomen many qs on localisation of lesion... with some weakness in the body and where possibly could be the problem... copd abg compensated.. next step.. psychiatry- all diagnosis...direct sympotms.. skin - case of pityriasis rosea.. diagnosis.. classic presentation.. GI many qs on pain next best step.. details could not remember.. hep Bpositive pregnant women... what do you do??? HUS, ITP,. urology- many on all the membrano proliferative,all those conditions.. mix and match ... told me to read that well one day before the exam... sorry this is really hotch potch but this is the info i could gather so farAnswers: #1: Rule for gunshot wounds to the abdomen = Always exploratory lap. Not so much to exctract the bullet but to explore solid and hollow organs for any perforation. #2: First nebulizer= Treatment of choice anticholinergic lately has been coupled to beta-adrenergics (duoneb). Treat underlying infection if indicated. Mucolytics and pulmo-aid to help clear secretions. #3: Pityriasis Rosea = Herald patch then erythema in Christmas Tree distribution in the back. Differential with Syphilis. #4: Hep +ve pregnant. Baby should be vaccinated at birth with HepB IG, then vaccine at 1 month then at 6 months. Monitor LFT's.

#5: HUS. post-diarrheal episode in a child (E.Coli-Shigella). Develops thrombocytopenia, hemolytic anemia (shistocytes, Helmet cells), acute renal failure, hematuria. Treatment is supportive. May dialyse and/or transfuse. It is the same as TTP without the neurological symptoms and in a child rather than adult. Treatment to TTP: Plasmapheresis and NSAID's NO PLATELETS. #6: ITP: It could be child or adult. The trigger infection is viral, and it causes autoimmune (Antiplatelet antibodies) thrombocytopenia without hemolysis and kidney's not affected. Treatment is steroids and if it fails resots to splenectomy with subsequent H.Flu and Pneumovac Vaccine. Repeat Pneumovac after 65yo. #6: For completion: Henoch-Shonlein Purpura. In a child, after an URI. Purpura with hematuria, arthritis, melena and abdominal pain (if intususception). No thrombocytopenia. Treatment supportive. #7: Next post – NephropathiesNephropathies

Glomerular Pathology: - NEPHR"I"TIC SYNDROME: Hematuria, HTN, Oliguria, Azotemia+>Acute Post Streptococcal Glomerulonephritis - Lumpy Bumpy + Granular pattern on IF+>Crescentic Glomerulonephritis (i.e.:Rapidly progressive) - Crescent moon shape+>Good Pasture Syndrome - Linear pattern deposits+> Membranoproliferative Glomerulonephritis - Subendothelial humps + Tram Tracks+> IgA Nephropathy (Berger's Disease) - Mesandial deposits of IgA NEPHR"O"TIC SYNDROME: Massive proteinuria, hypoalbuminemia, generalized edema, hyperlipidemia. +> Membranous Glomerulonephritis - Spike and dome + Diffuse capillary and basement membrane thickening (MCC in adults)+> Minimal Change disease - Foot process Effacement (MCC in children)+> Focal Segmental Glomerular Sclerosis - Segmental Sclerosis and hyalinosis+> Diabetic Nephropathy - Kimmelstielwilson Disease+> Lupus Nephritis - 5 patterns: Wire-loop appearance with extensive subendothelial/Basement Mb granular deposits. ----------------------------------------------------------------------------------------------------------------------------------------------

- #1: MINIMAL CHANGE DISEASE = LIPID NEPHROSISMost common cuase of nephrotic syndrome in children. Associated with URI's, immunizations, hypersensitivity to drugs (NSAID's), or allergic reactions (Bee stings). May be paraneoplastic for Hodgkin. Treatment: Predisone induces remission in 90% children and 50% adults. Add Cyclophosphamide or Chlorambucil for relapses. Complications: Incrased susceptibility to bacterial infections, spontaneous bacterial peritonitis, thromboembolic events, svere hyperlipidemia, and protein malnutrition. Case: 10 YO BOY WITH 2 MONTH HISTORY OF LOWER EXT EDEMA AND PROGRESSIVE ABDOMINAL DISTENTION. PRIOT TO THAT, H/O "BAD COLD" FOR SEVERAL WEEKS. PE/ ABDOMEN DISTENDED WITH SHIFTING DULLNESS. PEDAL EDEMA 2+. U/A PROTEINURIA 3+, SERUM ALBUMIN AND

PROTEIN DECREASED. NEPHROTIC SD ON LABS. ANA -VE. RENAL BIOPSY: LIGHT MICROSCOPY UNREMARKABLE. IF: -VE FOR Ig. EM: fusion of epithelial foot processes. #2: MEMBRANOUS GLOMERULONEPHRITISIt's the most common cause of primary nephrotic syndrome in adults. Increased incidence of occult neoplasms: lung, stomach, and colon in patients >50yo. Associated with autoimmune disorders and use of medications (Gold, Penicillamine, and Captopril). Treat with Prednisone with ot without cytotoxic agents for 3 months. Complications: progressive loss of renal function over 3-10years in 10% of cases. CASE: 52YO WITH RHEUMATOID ARTHRITIS AND BORDERLINE DIABETES PRESENTS TO THE OUTPATIENT CLINIC COMPLAINING OF ABDOMINAL BLOATING AND BILATERAL ANKLE SWELLING OF SEVERAL MONTHS. PATIENT HAS BEEN USING GOLD THERAPYF FOR HIS RA FOR 2 YEARS NOW. ABDOMEN ASCITIC WITH 3+EDEMA. U/A 3+ PROTEIN AND NEPHROTIC RANGE PROTEINURIA. RENAL BIOPSY SHOWS THICKENED GLOMERULAR BASEMENT MEMBRANE AND "SPIKE AND DOME" PATTERN WITH SILVER STAIN. IF: GRANULAR DEPOSITS OF IgG AND C3. #3: DIABETIC NEPHROPATHY: Insidious proteinuria secondary to diabetic microangiopathy. Chronically progresses to glomerulosclerosis after about 10 years. asymptomatic at first, it then presents with HEMATURIA, FROTHY URINE (from protein), ANASARCA, AND HYPERTENSION. U/A = Protein and fatty casts. Renal Biopsy = KIMMELSTILL-WILSON = INCREASED MESANGIAL MATRIX AND THINKENED BASEMENT MEMBRANES. Treat with ACE-Inhibitors, Protein restriction, strict Glucose control with Insulin if necessary. Case: 48YO WHITE FEMALE ADMITTED BECAUSE OF WORSENING GENERALIZED EDEMA AND WEAKNESS ALONG WITH HYPERTENSION. H/O DM1. PE: HTN (160/110), PITTING EDEMA, NO EVIDENCE OF PLEURISY/ASCITES. NEITHER KIDNEY IS PALPABLE. FUNDOSCOPY: PROLIFERATIVE DIABETIC RETINOPATHY. BS: 234MG/DL. Hb A1C= 10%. BUN AND C ELEVATED, DECREASED ALBUMIN. U/A 3+ PROTEIN AND GLUCOSE, BROAD AND FATTY CASTS. #4: Iga NEPHROPATHY = BERGER'S DISEASEIdiopathic Glomerulonephritis associated with URI or GI infections. NO LATENCY PERIOD (AS OPPOSED TO POST-STREPTOCOCCAL). Presents with HEMATURIA, ELEVATED IgA, U/A= Protein and Red Cell Casts. Renal Biopsy = IgA DEPOSITS. CAN BE CRESCENTIC, FOCAL SEGMENTAL, OR MESANGIOPROLIFERATIVE. IT IS THE SAME GLOMERULAR PATHOLOGY SEEN IN HENOCH-SHONLEIN IN KIDS. IT IS ALSO SEEN IN CELIAC DISEASE AND LIVER DISEASE because of decreased IgA clearance. NO EFFECTIVE TREATMENT. 1/2 of patients progress to renal failure. CASE: 22YO WHITE MALE COMPLAINS OF RECURRENT EPISODES OF BLOODY URINE FOR SEVERAL DAYS IN CONJUNCTION WITH URI. NO PRIOR HISTORY. U/A RED CELL CASTS IN URINE, GROSS HEMATURIA. INCREASED IgA. RENAL BIOPSY: FOCAL GLOMERULONEPHRITIS INVOLVING ONLY SELECTED GLOMERULI WITH MESANGIAL PROLIFERATION AND

SEGMENTAL NECROSIS WITH CRESCENTS. IF: MESANGIAL IgA DEPOSITS WITH SOME IgM, IgG, AND C3. TREAT SUPPORTIVELY. EVEN AFTER RENAL TRANSPLANTS, IgA DEPOISTS REAPPEAR. #5: CRESCENTIC GLOMERULONEPHRITIS CASE: 36YO WHITE MALE COMPLAINS OF CHRONIC COUGH OF SEVERAL MONTHS WITH LIGHTHEADEDNESS, FATIGUE, AND MALAISE. YESTERDAY, HE COUGHED UP BLOOD. HE ALSO DESCRIBES EPISODES OF DARK ORANGE URINE. PATIENT IS A HEAVY SMOKER. CXR=BILAT ALV INFILTRATES. U/A OLIGURIA, HEMATURIA, PROTEINURIA. ABG'S: HYPOXEMIA. RENAL BIOPSY: KIDNEYS' ENLARGED AND PROLIFERATIVE NECROTIZING GLOMERULONEPHITIS IN CRESCENTS WITH ACCUMULATION OF NEUTROPHILS AND MACROPHAGES IN BOWMAN'S CAPSULE. THERE IS CHARACTERISTIC IgG LINEAR DEPOSITS ON BASEMENT MEMBRANE AND ALVEOLAR SEPTA ON IF. IN SERUM = ANTIGLOMERULAR BASEMENT MEMBRANE ANTIBODIES +VE. ASSOCIATED: NECROTOZING HEMORRHAGIC ALVEOLITIS ON LUNG BIOPSY = TYPE II HYPERSENSITIVITY.TREAT GOOD PASTURE'S SYNDROME WITH PLASMAPHERESIS, STEROIDS, AND IMMUNOSUPPRESSIVE THERAPY. #6: HYPERTENSIVE RENAL DISEASE: CASE: 45YO BLACK MALE PRESENTS WITH UNCONTROLLED HYPERTENSION, SEVERE OCCIPITAL HEADACHE, VISUAL BLURRING, AND OLIGURIA X24H. FUNDOSCOPY REVEALS PRESENCE OF PAPILLEDEMA WITH HYPERTENSIVE RETINOPATHY. U/A: PROTEINURIA, RED CELL CASTS. SERUN: ELEVATED BUN AND C. CBC: SHISTOCYTES, HEMOLYTIC ANEMIA. RENAL BIOPSY: FIBRINOID CHANGES OF ARTERIOLES (NECROTIZING ARTERIOLITIS), HYPERPLASTIC ARTERIOLOSCLEROSIS (ONION SKINNING), AND NECROTIZING GLOMERULITIS WITH THROMBOTIC MICROANGIOPATHY. TREAT BY REDUCING DIASTOLIC BLOOD PRESSURE TO AT LEAST 100 AND MAINTAIN URINE OUTPUT AT LEAST 20CC/HR. GIVE NITROPRUSSIDE BECAUSE IT DOESN'T IMPAIR MYOCARDIAL BLOOD FLOW. HOWEVER IT IS METABOLIZED TO CYANIDE AND THIOCYANITE SO MONITOR BLOOD LEVELS. #7: Lupus Nephritis:There are five patterns to Lupus Nephritis: - Class I: NORMAL in light/immunofluorescence microscopy. - Class II: MESANGIAL LUPUS GLOMERULONEPHRITIS. Minimal hematuria or proteinuria. - Class III: FOCAL PROLIFERATIVE GLOMERULONEPHRITIS. Recurrent hematuria, and mild renal insufficiency.- Class IV: by far the most common as in the folling case. DIFFUSE PROLIFERATIVE GLOMERULONEPHRITIS. - Class V: MEMBRANOUS GLOMERULONEPHRITIS. Nephrotic syndrome.

Case: 30YO BLACK WOMAN WITH PAIN IN BOTH KNEE JOINTS AND SMALL JOINTS OF HANDS TOGETHER WITH MILD FEVER, WEIGHT LOSS, AND LOSS OF HAIR. SHE ALSO HAS RECURRENT ORAL ULCERS AND PHOTOSENSITIVE SKIN RASH. SHE IS HYPERTENSIVE WITH BUTTERFLY RASH OVER MALAR AREA. LABS: NORMO/NORMO ANEMIA U/A MICROSCOPIC HEMATURIA WITH RBC CASTS IN ADDITION TO PROTEINURIA. ELEVATED BUN AND C. ANA +VE, ANTI-DS DNA +VE, VDRL +VE. RENAL BIOPSY: DIFFUSE PROLIFERATIVE GLOMERULONEPHRITIS WITH IMMUNE COMPLEX DEPOSITS TYPICALLY SUBENDOTHELIAL AND FORMING "WIRE-LOOPS". TREAT WITH STEROIDS AND CYCLOPHOSPHAMIDE/AZATHIOPRINE/CHLORAMBUCIAL. LONG-TERM HEMODIALYSIS OR TRANSPLANT. #7: MEMBRANOPROLIFERATIVE GLOMERULONEPHRITISIdiopathic. It may be associated with inherited diseases of complement components and partial lipodystrophy. TypeI MPGN: both classic and alternative complement pathways are activatedTypeII MPGN: Dense Deposit disease. Activation of only alternate complement pathway.

50% progress to renal failure. There is a high recurrence rate following renal transplantation. Case: 11YO WHITE FEMALE BROUGHT BECAUSE OF HEADACHE, CHEST PALPITATIONS, AND RINGING IN HER EARS TOGETHER WITH GENERALIZED EDEMA. SHE ALSO HAS HEMATURIA. HYPERTENSION 140/110. PERIORBITAL EDEMA, NO EVIDENCE OF PLEURAL EFFUSION/ASCITES. LABS: ELEVATED RUN/C, HYPOALBUMINEMIA, HYPOCOMPLEMENTEMIA, ANA-VE, NORMAL ASO TITERS. U/A FATTY CASTS AND OVAL BODIES ALONG WITH PROTEINS. RENAL BIOPSY: DIFFUSE GLOMERULAR INVOLVEMENT WITH THICKENED CAPILLARY WALLS AND LOBULAR MESANGIAL PROLIFERATION ON LIGHT MICROSCOPY. ON PAS OR SILVER: SPLITTING OF BASEMENT MEMBRANE CAUSING RAILRAOD TRACK APPEARANCE. IF: PROMINENT MESANGIAL AND SUBENDOTHELIAL DEPOSITS OF IMMUNE-COMPLEXES. TREAT WITH STEROIDS AND RENAL TRANSPLANT. #8: MEMBRANOUS GLOMERULONEPHRITISNephrotic Syndrome may be idiopathic or caused by membranous glomerulonephritis (the most common cause in adults), Minimal change disease (MCC in children), focal glomerulosclerosis, or membranoproliferative GNitis. Patients with nephrotic syndrome have hypercoagulability secondary to loss of ANTITHROMBIN III in the urine. Case: 47YO BLACK DIABETIC FEMALE COMPLAINS OF WEIGHT LOSS, PROGRESSIVE SOB, AND SWELLING OF BOTH LEGS AND ARMS. NO PAST MED H/O. PE: PERIORBITAL EDEMA, AND ASCITES. UA/ PROTEINURIA, HYPOALBUMENIMA, HYPERLIPIDEMIA. RENAL BIOPSY: THICKENED BASEMENT MEMBRANE, SUBENDOTHELIAL DEPOSITS OF IgG AND C3 ALONG BASEMENT MEMBRANE SEEN IN "SPIKE AND DOME" PATTERN ON SILVER STAIN. THE IMMUNE DEPOSITS ARE IN A "LUMPY-BUMPY" = DISCONTINUOUS PATTERN ON IMMUNOFLURESCENCE.

TREAT WITH STEROIDS, CYCLOPHOSPHAMIDE, ACE-I REDUCE PROTEIN LOSS. #9: POST-STREPTOCOCCAL GLOMERULONEPHRITIS: It's an immune complex disease that is usually caused by beta-hemolytic streptococcus type 12 and 49. it occurs 10-14 DAYS AFTER URI. Pathogenesis may be related to the deposition of strep antigen-antibody complexes in the glomeruli, followed by activation of complement leading to inflammation. It is the most common childhood nephritis and affects preschool and school-age children. Occurs after URI, skin infections, and scarlet fever. Treat with Penicillin for 10 days to prevant spread of nephrogenic strain (erythromycin if allergic). Diet high in carbohydrates and low in protein, sodium, potassium and water. Treat Renal and Cardiac Failure with peritoneal dialysis. Resolution may take 6-12 months. Complications: Cardiac Failure, Hypertensive Encephalopathy (vomiting, severe headache, convulsions, visual disturbances), uremia, acute pulmonary edema, and chronic glomerulonephritis. Case: 5YO MALE PRESENTS WITH MALAISE, PERIORBITAL EDEMA, SMOKY COLORED URINE (HEMATURIA), ABDOMINAL PAIN WITH VOMITING, AND MILD FEVER. H/O THROAT INFECTION TEN DAYS AGO FROM WHICH HE REOVERED UNEVENTFULLY. PE: HTN, TONSILS CRYPTIC BUT NO EXUDATE, ANKLE EDEMA. CBC, ANEMIA, LEUKOCYTOSIS. INCREASED ESR, AND BUN/C. C3 AND TOTAL COMPLEMENT CH50 LOW. INCREASED ASO TITER. ABG'S=METABOLIC ACIDOSIS. U/A RBC'S AND RBC CASTS, PROTEINURIA. ANA -VE, DNASE TITER HIGH. RENAL BIOPSY: ELECTRON-DENSE HUMPS ON THE EPITHELIAL SIDE OF THE GLOMERULAR BASEMENT MEMBRANE. IF: GRANULAR PATTERN OF Ig DEPOSITION. #10: PRIMARY AMYLOIDOSISPrimary Amyloidosis commonly presents with nephrotic syndrome. Amyloidosis may be primary (monoclonal Ig light chains), or secondary (Rheumatoid Arthritis, Tuberculosis, Multiple Myeloma, Pregnancy where it's beta2microglobulin).

CASE: 45YO WHITE FEMALE COMPLAINS OF PALPITATIONS AND SOB, MORNING SWELLING OF THE EYES, AND ARMS/LEGS, WITH NUMBENESS OF THE LOWER LEGS TOGETHER WITH WEIGHT LOSS AND FATIGUE. NO P.MED.H/O. PE: MILD CARDIOMEGALY, MACROGLOSSIA, PITTING EDEMA, ASCITES, AND CARDIAC ARRYTHMIA. EKG: LOW-VOLTAGE. LABS: HYPERLIPIDEMIA, HYPOPROTENEMIA. U/A: PROTEINURIA. CXR: BIVENTRICULAR CARDIAC ENLARGEMENT. RENAL BIOPSY: APPLE-GREEN BIREFRINGENCE IN POLARIZED LIGHT WHEN STAINED WITH CONGO RED, AMYLOID DEPOSITION IN MESANGIUM AND ENDOTHELIUM SURROUNDING HEPATIC SINUSOIDS AND IN SPLEEN; HYALINE THICKENING OF ARTERIOLAR WALLS, LEADING TO NARROWING OF LUMEN AND ISCHEMIA. #11: RENAL TUBULAR ACIDOSIS It results from:

- TypeI: Deficient H+ secretion in the distal tubule. Urine pH>5.4. Secondary Hyperaldosteronism and HYPOKALEMIA. There is Nephrocalcinosis and Nephrolithiasis. Can be secondary to auto-immune disease (e.g.: Sjogren) or drugs (Amphotericin, Lithium, Analgesics), or other (Nephrocalcinosis, Sickle CEll Disease, and Chronic Infection). DIAGNOSIS: ACID LOAD TEST: Give ammonium chloride, which should lower urine pH secondary to increase H+ production. If RTA I, Urine pH remains elevated. Serum Bicarb is 10. TREAT WITH ORAL BICARB (proximal tubule reabsorption still works)AND POTASSIUM REPLACEMENT. - Type II: Defective bicarbonate reabsorption in the proximal tubule. ETIOLOGY: Fanconi Syndrome, Wilson Disease, Amyloidosis Myeloma, VIT D DEFICIENCY, HEAVY METALS, CHRONIC HYPOCALCEMIA, CHRONIC HEPATITIS, AUTOIMMUNE DISEASE (SLE, Sjogren). - BONE LESIONS (OSTEOMALACIA and RICKETS), whereas Type I get Kidney Stones. Both are hypokalemic.- Diagnosis: Bicarb Loading Test - IV sodium bicarb = ph urine still basic. Normal individuals do not excrete bicarb until serum >24. These patients secrete it but do not reabsorb it. So they are also Acidemic. - Inability to absorb bicarb. Urine pH is basic HYPOKALEMIA, SERUM BICARB 18-20. - Treat with potassium. Mild volume depletion enhance proximal reabsorption of bicarb. Give thiazides and very large amounts of bicarbs. - Type III: Supressed renal generation of ammonia secondary to reduced GFR. - Type IV: Aldosterone Deficiency of any cause, or Adrenal Insensitivity to Angiotensin II, which normally stimulate Aldosterone secretion. This results in defective secretion of both potassium and H+ in the distal nephron. The resulting hyperkalemia decreases proximal tubule ammonia production and reduces H+ secretion, leading to inadequate excretion of the acid load. These patients produce acidic urine despite reduced H+ secretion because of inadequate ammonia to buffer the protons in the distal tubule. - Etiology: Diabetes, Addision, Sickel Cell, Renal Insufficiency. - Diagnosis: ORAL SALT RESTRICTION => CON'T HIGH URINE SODIUM.RTA is ALSO diagnosed by asymptomatic hyperkalemia, lab evidence of primary hyperchloremic hyperkalemic metabolic acidosis, normal anion gap, zero/positive urine anion gap, and exclusion of presence of diarrhea, calcium chloride, or other acids. Type IV is the most common, and it is seen in patients with renal insufficiency and chronic medical illnesses such as DM with nephropathy, tubular interstitial renal disease, HTNm and AIDS. Also with drugs: ACE-I, NSAID's and HEPARIN. Treat by restricting potassium, and D/C aldosterone antagonists drugs (NSAIDS, ACE-I, HEPARIN). Mineralocorticoid replacement with Fludrocortisone usually improves hyperkalemiaand acidosis but may worsen HTN. Patients may require higher doses because of resistance. Loop diuretics and exchange resins can also be used. Acidosis usually corrects as soon as hyperkalemia contributing to decreased ammonia production, is corrected. Complications: metabolic acidosis, hyperkalemia (confusion, weakness, paresthesias,

paralysis, arrythmias, and even cardiac arrest). CASE: 73YO MALE WITH ADULT ONSET DM FOR 30Y, HTN, AND RHEUMATOID ARTHRITIS PRESENTS WITH A DECLINE IN MENTAL STATUS AND DECREASED URINE OUTPUT. DM WAS POORLY CONTROLLED DESPITE INSULIN AND ORAL AGENTS. HE HAS HAD OCULAR DISEASE AND RENAL INSUFFICIENCY FOR >5Y. HE IS CURRENTLY ON ACE-I, THIAZIDE, CALICUM CHANNEL BLOCKER FOR HTN, AND TAKES NSAIDS FOR PERSISTENT RA. PE: HTN, LETHARGY.LABS: HYPERKALEMIA, HYPERCHLOREMIA, ELEVATED BUN/C. ABG: NON-ANION GAP METABOLIC ACIDOSIS. U/A URINE PH ACID<5.5, URINARY ANION GAP+VE. DIFFERENTIAL: DIABETIC NEPHROPATHY WHICH SHOULD BE TREATED WITH ACE-I AS OPPOED TO TUBULAR ACIDOSIS WHERE THEY HAVE TO BE D/C'ED. #12: HIV-ASSOCIATED NEPHROPATHYHIV associated Nephropathy Pearls (HIVAN) - Mostly African Americans (So and so, that in the absence of renal biopsy, and if the patient is white, think of other diagnoses) - Patient may present with Uremic Encephalopathy with confusion, new-onset seizure (Management of seizure with subsequent treatment using phenytoin), asterixis etc. - Labs: Uremia, massive proteinuria in the nephrotic range with little to ne edema - Order Ultrasound: which in contrast with usual shrunken small kidneys of End-Stage Renal Disease, in HIVAN, kidneys are ENLARGED!! - Confirm with Renal Biopsy: FOCAL SEGMENTAL GLOMERULAR SCLEROSIS is the most typical one. Management: Antiretroviral Therapy is the most important feature. It not only slows the progression but may in fact reverse it. Hemodialysis should also be started. Of note: Encephalopathy is a sign of rapidly changing nephro status either deteriorating ir improving (encephalopathy on first hemodialysis = Dialysis dysequilibrium Syndrome). ACE-I also have been used successfully to further slow progression. Corticosteroids have also been used but with caution because of the advanced stage of AIDS already for the patient. Differential: Hep B and Hep C associated nephropathy, Heroin Associated Nephropathy. REFERENCES:- UNDERGROUND CLINICAL VIGNETTES: Pediatrics, Pathophysiology, Internal Medicine- Kaplan notes- - Medscape (posted from the AIDS reader)New Onset Seizures as an Initial Presentation of End-Stage Renal Failure in Patients With HIV/AIDSToyin F. Olatinwo, MD; Ross G. Hewitt, MD8/2002- First Aid Step 1

Causes of Abdominal pain in Different QuadrantsCauses of abdominal pain in different quadrants:

For differential diagnosis purposes let us see the abdomen in 4 anterior quadrants and the left and right flanks. RUQ-perihepatic structures Liver- hepatitis, Budd chiari(esp if pt. has Nephrotic/paroxysmal nocturnal hemoglobinuria), ruptured hepatic adenoma(young female on oral contraceptives), hepatic trauma, Perihepatitis(eg Fitz-Hugh-Curtis syndrome-chlamydia or gonococcus). Gall bladder- calculous or non calculous(esp in pt in ICU or on cephalosporins) cholecystitis,Gallstone colic Duodenum-ulcer (on exams- think of Zolinger Ellison) Lung base- Lower lobe pneumonia, pleuritis Diaphragm-irritation due to blood in peritoneum(also pain in Right shoulder when patient is in head low position-esp seen in splenic rupture-inspite of spleen being left sided) RLQ: Structures around caecum and thse on right side of uterus. Appendix-Appendicitis-acute or chronic Caecum-Typhilitis esp in post chemotherapy patient - occurs with Pseudomonas and Clostridium septicum with approximately equal frequency. Ovary- cyst rupture or torsion, fibroid- red degeneration, ECTOPIC pregnancy, Tubo-ovarian abscess(PID) GIT-rarely diverticulitis(usually occurs on left side). LUQ: Spleen and structures around it. Stomach, Spleen, pancreas, colon-splenic flexure LLQ: Sigmoid colon- especially diverticulitis that is seen in elderly population and is treated with antibiotics, hydration-PO or IV and stool softening. Encourage fiber in diet. Uterine structures-as in LLQ area. Abdominal Us is almost always right answer is you;re looking for answer using the least invasive most cost effective procedure. ERCP when you want to investigate the Common Bile Duct rather than the gallbladder. It's beyond the scope to talk about each one regarding each diagnosis. Orient youself with the differential above and study each condition on CMDT . They're pretty good about this. Side effects of interferon therapy in Hep C:Is it depression or irritability or hallucinations or delirium.DEPRESSION Reference: Acute hepatitis C: response to treatment with interferon-alpha plus ribavirinGastroenterol Hepatol. 2002 Oct;25(8):483-6. Others:Anorexia, Fatigue, Somnolence, Confusion, Weight loss Reference:CMDTQuestionDiabetic for many years. Developed Autonomic neuropathy of gastroparesis. Given Meoclopramide. Developed Tardive dyskinesia despite reducing dosage. Switch dugs to which one?Erythromycin.

Improves gastric emptying by binding to motilin receptors in stomach. What if it was Diarrhea in Autonomic Neuropathy setting? Ans: Often self-limited but use broad-based Antibiotic therapy. If not responding => associated with impaired sphincter control and fecal incontinence : Use Loperamide QD or Combination Diphenoxylate/Atropine. What if it is constipation? Laxatives such as senna. Refernce: CMDTQuestionA 20-year-old woman has a history of repeated admissions for sudden abdominal pain. No known etiology has been found for this pain despite an intensive metabolic, infectious, and endocrine evaluation. The patient has also had repeated imaging studies of her abdomen and 1 exploratory laparotomy. She comes to the emergency department complaining of similar pain and is admitted to the hospital for management. A social work consult is called and reveals that the patient is a single mother with 2 small children who stay at her mother's house when she is hospitalized. Now on the inpatient floor, the patient is relaxed. Extensive imaging and laboratory studies are all normal. Which of the following is the most likely diagnosis?A. Body dysmorphic disorderB. Conversion disorderC. Factitious disorderD. HypochondriasisE. MalingeringFactitious disorder. History typical of hypochondriasis but what differentiates is the obvious secondary gain. It is of psyhological nature (20yo singe mother needs attention, or kids at mom's, or Demerol addiction). It would have been malingering if there hadn't been surgery involved as it would be less likely she would resort deliberately to surgery just to have the pain medication or her mother take the kinds in. Answers to some BIOSTATS MCQ'sthere were lot os interpretation qs of case control and cohort studies.. like the values were given... and then there were various options to interpretation..like the OR, RR attributable risk..PPV senstivity and specificity.. and the conclusion after the end of the study were asked... - IN A COHORT STUDY FOR SMOKING AND RISK OF LUNG CANCER: RELATIVE RISK = INCIDENCE IN EXPOSED GROUP/INCIDENCE IN UNEXPOSED GROUP => IT MEANS HOW MUCH MORE RISK WOULD A SMOKER HAVE TO DEVELOP LUNG CANCER COMPARED TO A NON SMOKER ATTRIBUTABLE RISK = INCIDENCE IN EXPOSED GROUP - INCIDENCE IN NON EXPOSED GROUP => MEANS HOW MANY MORE CASES WERE THERE OF LUNG CANCER IN SMOKERS COMPARED TO NON SMOKERS, OR HOW MANY MORE CASES WERE ATTRIBUTABLE TO SMOKING. - IN CASE-CONTROL STUDIES:ODDS RATIO = ODD OF EXPOSURE FOR CASES/ODDS OF EXPOSURE FOR CONTROLS = AD/BC => IF YOU HAVE LUNG CANCER, WHAT ARE THE ODDS

YOU HAVE BEEN EXPOSED TO SMOKING IN THE PAST? Q: A research study investigated the relationship btw mean blood values in pregestational diabetic women and major fetal malformations: Major fetal malf--------->130----------<130------Total Present------------------10---------------2-------350Absent-------------------50-------------300-------350total--------------------60-------------302-------362 What is the odds ratio? How would you put it in a sentence to interpret? Ans: Odds ratio = ad/bc = 10x300/50x2 = 30. It means if the baby has a feta; malformation, there is 30-fold likelihood the woman HAD a pre-gestational diabetes. It's in the past because the study is retrospective case-control. If it were prospective, then we would not use the Odds ratio, but the relative and attributale risk (see below). Q: Match with the correct value: ------------Disease------WellExposed------520----------175-----------695Non exposed---80----------225-----------305-------------600----------400----------1000 #1- Assume the table is a cohort study, what is the attributable risk? Interpret it in a sentence.#2- Assume the table represents a cross-sectional study, what is the relative risk? Interpret it in a sentence.#3- Assume the table represents a case-control study, what is the odds ratio? Interpret it in a sentence. Choices:A- 520/695B- 600/1000c- 520/600D- 695/1000E- 80/305F- (520/695)(80/305)G- (520x225)/(175x80)H- (520/695)-(80/305)I- Cannot be determined by this type of studyJ- (520/695)/(80/305) Ans: #1: H => Incidence of exposed - Incidence of nonexposed = approximately 0.5Incidence of exposed= New/Total = 520/695Incidence of non exposed = New/total = 80/305Of every thousand cases observed, the risk factor exposed to in this study accounted for 0.5 cases. #2: I. it cannot be determined. The attributable risk is happens in cohort studies not cross-sectional. With that said, Let's assume it's a cohort. The relative risk is = Incidence of Exposed/ Incidence of non exposed. = J = approximately 2.87

It means people exposed to the risk factor studied are 2.87 more likely to develop the disease IN THE FUTURE than non exposed people. #3: G - Odds ratio=AD/BC = (520x225)/(80x175)= approximately 8.36This means the odds of having a history of exposure IN THE PAST to the risk factor is 8.36 times greater in those who have the disease than in those who do not. IN DISEASE STATISTICS:- PREVALENCE = PERSONS WITH DISEASE/PERSON AT RISK => HOW MANY EXIST AT A GIVEN TIME- INCIDENCE = PERSONS WITH DISEASE ONSET/PERONS AT RISK => HOW MANY NEW CASES. Q: A recently dicovered treatment for leukemia extends the lifespan but does not prevent the diseas or lead to its cure. What happens to Incidence and Prevalence? Ans: Incidence is unchanged. Prevalence increases. IN SCREENING TESTS: #1: In general population screening where the prevalence is low: - SENSITIVITY = TRUE POSITIVES/ALL SICK (tp+fn) => How often is the test positive in patients who have the disease. - SPECIFICITY = TRUE NEGATIVES/ALL WELL (tn+fp) => How often the test is negative if people who do NOT have the disease. If a test has 100% sensitivity, that means it has no false negatives. If it is negative, it rules out the disease. However if it is positive, we still don't know if it is True Postive or False Positive. Therefore, sensitivity is for screening random asymptomatic people. If a test has a specificity of 100%, that means it has no false positives. If it is positive, it confirms the disease. But it still doesn;t say anything if it is negative, if it is a True Negative or False Negative. For both those reasons, we resort to positive and negative predictive value. #2: In clinical settings where prevalence is higher: - POSITIVE PREDICTIVE VALUE = TRUE POSITIVES/ALL POSITIVES => HOW POSITIVE IS A POSITIVE? When a test returns positive, the question to ask is is it a True positive or a False Positive? That's the PPV of a test. If the test has 100% specificity, that means, the Positive Predictive Value will always be 100% too because there is no False Positives. - NEGATIVE PREDICTIVE VALUE = TRUE NEGATIVES/ALL NEGATIVES. If a test comes back negative, you ask yourself is your negative a true negative or a false negative? Of course if it is a test with 100% sensitivity, then there are no false negatives and therefore NPV=100% too. Q: Test X for SLE is positive in 60 out of 100 patients with known SLE and is normal in 80 out a 100 controls. If test X retunrs positive in a person who is randomly selected in this population under study, what is the percent chance that the person has SLE? Ans: In other words, they're asking for the Positive predictive value. It would be easier to redistribute the information in a visual way. ------------------------------Test+ve------Test(-ve)Patients with SLE--------------TP60--------FN40

Control Population-------------FP20--------TN80Totals---------------------------80---------120 Now we see it clearly. It is a population with a 50% pervalence (100 controls and 100 sick with total 200), which is why we use the PPV. TP/TP+FP = 75% - ACCURACY = TRUE POSITIVES + TRUE NEGATIVES / ALL => HOW MUCH PERCENT IS THIS TEST ACCURATE? - ACCURACY VS PRECISION: ACCURACY (also called Validity) IS HOW CLOSE TO THE NORM. PRECISION (or reliability) IS HOW CONSISTENT AND CLOSE TOGETHER THE RESULTS OF THE TEST ON THE SAME PERSON IN THE SAME CONDITION WOULD BE. A TEST COULD BE PRECISE (REPRODUCIBLE) BUT NOT ACCURATE. A TEST COULD ALSO BE ACCURATE BUT NOT PRECISE. THE IDEAL WOULD BE IF A TEST COULD BE BOTH. also there was some q about interval analysis which my friend said that he did not understand at all.. WE HAD ALREADY DISCUSSED AT LENGTH THE PERCENTILES WITH REGARDS TO THE MEAN. IF IT IS AT 1SD, IT IS THE 84TH PERCENTILE. - STATISTICAL SIGNIFICANCE:RELATIVE RISK AND CONFIDENCE INTERVAL (THIS ONE I LEARNED IT LIKE THIS)=> IF THE RELATIVE RISK CONTAINS 1.0 VALUE IN ITS CONFIDENCE INTERVAL, THEN IT'S NOTSTATISTICALLY SIGNIFICANT. IF IT'S ABOVE 1.0, THERE IS INCREASED RISK. IF IT'S BELOW 1.0, THERE IS DECREASED RISK. - ANOTHER NOTION IS THE P-VALUE. IT IS SO CONFUSING COMTIMES BUT KEEP IN MIND THESE NOTIONS. P VALUE IS PART OF WHAT WE CALL INFERENTIAL STATISTICS. WE WOULD INSTEAD OF STUDYING THE ENTIRE POPULATION OF THE EARTH, CHOOSE A SAMPLE AND GET RESULTS OF A STUDY, THEN EXTRAPOLATE AND DRAW CONCLUSIONS ABOUT THE ENTIRE POPULATION OF THE PLANET. BUT THEN AGAIN, WE NEED TO KNOW IF WE ..CAN...EXTRAPOLATE OR IF OUR RSULTS WERE ONLY OBTAINED BY CHANCE, THATS WHY WE CHOOSE THE CONFIDENCE INTERVAL WHICH IS THE P VALUE. IT MEANS OUR RESULT IS ACCURATE AT A 95% CONFIDENCE IF P-VALUE IS 0.5 (5%). IT MEANS WE LEFT 5% CHANCE FOR PURE COINCIDENCE TO HAVE BEEN THE CAUSE OF OUR RESULTS. SO BEFORE WE START OUR STUDY, WE DECIDE WHAT IS OUR STATEMENT? E.G: SMOKING CAUSE CANCER. THAT'S THE HYPOTHESIS WE WANT TO TEST. WE WOULD CALL IT HYPOTHESIS ZERO OR NULL. WE WILL EITHER CONFIRM IT (ACCEPT IT) OR DENY IT (REJECT IT). HOWEVER, CONVENTIONALLY, SINCE IT IS IN REAL LIFE EASIER TO REJECT THAN TO ACCEPT, WE WILL MAKE THE START POINT HYPOTHESIS (NULL HYPOTHESIS) THE OPPOSITE OF WHAT WE WANT TO PROVE. IF WE REJECT IT, THEN WE HAVE PROVEN OUR GOAL. IF WE ACCEPT IT (IN THE STATISTICIANS LANGUAGE THEY SAY "FAIL TO REJEC" INSTEAD OF "ACCEPT"), THEN WE HAVE REJECTED OUR GOAL. SO FOR THE EXAMPLE OF "SMOKING CAUSES CANCER". oUR NULL HYPOTHESIS IS "SMOKING DOES NOT CAUSE CANCER". WE WILL DO A STUDY AND REJECT IT WITH A P VALUE OF 0.5 (ONLY 5% CHANCE OUR

RESULTS OF REJECTION OCCURED BY RANDOM COINCIDENCE). IN DOING SO, WE WILL ACCEPT OUR ALTERNATIVE HYPOTHESIS WHICH WAS WHAT WE WANTED IN THE VERY BEGGINING WHICH IS" SMOKING CAUSES CANCER". THIS ACCEPTANCE WOULD BE AT 95% CONFIDENCE (I.E.:P VALUE OF 0.5). SO:- P-VALUE IS A CRITERION FOR MAKING DECISIONS ABOUT THE NULL HYPOTHESIS (SEE ABOVE)= ACCEPT OR REJECT- QUANTIFIES CHANCES FOR ERROR (5% CHANCE IT'S AN ERROR = BY RANDOM)- TELLS STATISTICAL SIGNIFICANCE = STATISTICALLY SIGNIFICANT AT P VALUE=0.5 WHAT P VALUE DOESNT DO:- NO TELLING IF AN INDIVIDUAL WILL BENEFIT (SEE BELOW)- % OF PATIENTS WHO WILL BENEFIT- DEGREE OF BENEFIT EXPECTED. Q: The difference in mean diastolic pressure among 150 subjects in a low-salt diet group and 150 subjects in a no-added-salt group is 10mmHg. This difference is significant at a p value less than 0.05. Which of the following statements about the two groups is true? Which of the following is true?A- Blood pressure difference is clinically significantB- The chance that an individual would benefit from a low-salt diet is less than 0.05C- It is unlikely that random variation accounted for the difference in diastolic blood pressure between the two groupsD- Increasing the number of subjects would tend to change the p value from significant to non-significant. Ans: C. p value is part of inferential statistics and merely reflects if a result occured by chance or not. It can't tell if patient can benefit from salt diet or not and to what degree. Q; A cancer researcher conducted a medical experiment and failed to reject the null hypothesis although the experiment was successful. What type of error did the resreacher make? A- Type I errorB- Type II errorC- Type II errorD- An experimental errorE- An inferential error Ans:B- Type II error is failing to reject the null hypothesis when it is actually false. Beta = 1-PowerPower of a test is the capacity to detect a difference when it exists. To increase the power of a test, you need to increase the sample size. Type I error (Alpha) is rejecting the null hypothesis when it actually true. The chance to do that is set by the criterion alpha (with is the same as p-value). if Pvalue if 0.05 => Chance of Type I error is 5%. he had 5-6 qs on ethics which were standard...important to know about minors and when they can make decisions on there own and when they need parental consent...

ALREADY DISCUSSED PREVIOUSLY IN THE FORUM. REFER TO PREVIOUS QUESTIONS LABELED ETHICS. References:- Kaplan notes- NMSR Tests- High Yield BiostatisticsComplete Approach Thyroid NoduleFirst: TSH. If it is thyrotoxic, then radionuclide scan. If it is non-functioning, then it is FNAC. If you do the thyronuclide scan, and it is a "hot" nodule, then you treat with radioiodine or surgery. If it is "cold", you do a FNAC as well. If it is bening, repeat FNAC in 6 months. If it is indterminate or clinical suspicon, then thyroid lobectomy. If it is malignant, then Surgery with Radioiodine. Subtotal Thyroidectomy only in 2nd trimester pregnancy or in children. Obviously, you skin the Radionuclide scan because it is contraindicated in pregnancy. Reference: American Association of Family PhysiciansThyroid nodulesFeb 2003http://www.aafp.org/afp/20030201/559.htmlwhat is Purtscher's retinopathy? Barter's syndrome?Purtscher Retinopathy: is a hemorrhagic and vasoocclusive vasculopathy. Sundden Bilateral Painless blindness first associated with severe head or blunt thoracic trauma, but then described with a number of conditions such as acute pancreatitis, fat embolization, amniotic fluid embolization, and vasculitic diseases. In a child, it should warrant investigating a battered child syndrome with intracranial hemorrhage (Subdural), and chest compression. There is usually no provem treatment for it except that of the underlying condition (Pancreatitis, trauma, SLE, etc.). It appears on the fundoscopy as cotton-wool spots around the optic nerve (ie, retinal microinfarcts at the level of the nerve fiber layer), and intraretinal hemorrhage. Barter Syndrome: A syndrome characterised by deranged NaCl transport in the ascending thick limb of the loop of Henle and the distal tubule. This results in the follinwg: - K, Na, and Cl wasting contributing to the stimulation of renin release accompanied by juxtaglomerular cell hyperplasia. - Aldosterone levels are elevated. - K depletion is not eliminated by correction of the hyperaldosteronism. - Na wasting results in a chronically low plasma volume reflected by a NORMAL BP despite high renin and angiotensin levels and by an impaired pressor response to angiotensin infusion. - Metabolic alkalosis often develops. Platelet aggregation is inhibited. Hyperuricemia and hypomagnesemia may occur. The kinin-prostaglandin axis is stimulated, and urinary excretion of prostaglandins and kallikrein is increased. Clinical features present in infancy or childhood and include anorexia, failure to thrive, polydipsia, polyuria and muscle weakness. Mental retardation may be a feature. It usually occurs in children and growth retardation is frequently associated. Aetiology is unknown. Inheritance is autosomal recessive.

In adult patients, bulimia nervosa, vomiting, or surreptitious diuretic or laxative abuse must be excluded as a cause. In these conditions, the urinary chloride is usually low (< 20 mmol/L).K supplementation plus spironolactone, triamterene, amiloride, an ACE inhibitor, or indomethacin will correct most features, but no drug completely eliminates K wasting. Indomethacin 1 to 2 mg/kg/day usually maintains the plasma K level close to the lower limit of normal. In summary: Child with growth delay, who presents with normal BP, SEVERE HYPOKALEMIA, and Metabolic alkalosis. Think of Bartter syndrome. Postexposure Prophylaxis to HIV-Patient (Source: CDC)SUMMARY AND USMLE TAKE HOME MESSAGE:TESTING AND PEP ARES RECOMMENDED ASAP AFTER EXPOSURE. IF UNCLEAR WHAT TO START WITH ZDZ/3TC IS A GOOD START SINCE 3TC (lAMIVUDINE) IS GOOD FOR ZDV RESISTANCE. tHIRD AGENT CAN BE INDINAVIR AS PREVIOUSLY RECOMMENDED OR MORE RECENTLY NELFINAVIR. OTHER AGENTS ARE AVAILABLE. PREGNANCY SHOULD NOT PRECLUDE SAME RECOMMENDATIONS. NO DATA ABOUT TERATOGENICITY OF ANTIRETROVIRALS EXCEPT EFAVIRENZ IN ANIMALS. CHOICE OF ANTIRETROVIRALS DEPENDS ON THEIR TOXICITY AND UNDERLYING CONDITIONS OF PATIENT. FOLLOW-UP IS 6 WEEKS, 12 WEEKS AND 6 MONTHS. 12 MONTHS FOLLOW-UP IS ON AN AS-NEEDED BASIS. MORE ON THE SUBJECT: Risk:The average risk for HIV transmission after a percutaneous exposure to HIV-infected blood is approximately 0.3% and after a mucous membrane exposure is 0.09%. HIV seroconversion: the estimated median interval from exposure to seroconversion was 46 days (mean: 65 days); an estimated 95% seroconverted within 6 months after the exposur. Antiretrovirals in prophylaxis:Several antiretroviral agents from at least three classes of drugs are available for the treatment of HIV disease. These include the nucleoside analogue reverse transcriptase inhibitors (NRTIs), nonnuceloside reverse transcriptase inhibitors (NNRTIs), and protease inhibitors (PIs). #1: ZDV (an NRTI) is the only agent shown to prevent HIV transmission in humans.#2: NRTIs that can be considered for use with ZDV for PEP are lamivudine (3TC), didanosine (ddI), and zalcitabine, each of which has been included in recommended regimens that include ZDV.#3: The addition of a PI as a third drug for PEP following high-risk exposures : Previously, indinavir (IDV) was recommended as the PI for PEP because of its increased bioavailability when compared with saquinavir and its more favorable immediate toxicity profile compared with ritonavir. Since the 1996 PEP recommendations were published, nelfinavir (NEL) was approved for use by FDA and is now included in regimens recommended for the treatment of primary HIV infection. Antiretrovirals in pregnancy:ZDV appears safe and well tolerated in both women and their infants who have had a

follow-up period of several years. The use of PIs in HIV-infected persons has been associated with hyperglycemia; it is unknown whether the use of these agents during pregnancy will exacerbate the risk for pregnancy-associated hyperglycemia. Choice of agents: Most HIV exposures will warrant only a two-drug regimen, using two NRTIs, usually ZDV and 3TC. The addition of a third drug, usually a PI (i.e., IDV or NEL), should be considered for exposures that pose an increased risk for transmission or where resistance to the other drugs used for PEP is known or suspected. Postexposure follow-up:HCWs with occupational exposure to HIV should receive follow-up counseling, postexposure testing, and medical evaluation regardless of whether they receive PEP. HIV-antibody testing should be performed for at least 6 months postexposure (e.g., at 6 weeks, 12 weeks, and 6 months). Reference:http://www.cdc.gov/mmwr/preview/mmwrhtml/00052722.htmUpdated March 2003

"atropin psychosis” in is old men?A 72yr. old man brought to er and complained of both auditory & visual hallicination on ph/exa. his skin is dry &pupil dilated What is the drug a)amitriptylineb)atropinc)risperidoned)ImipramineAns: Tricyclics. They have antimuscarinic effects which would contribute to an atropine-like effect of psychosis. The one that has most antimuscarinic is imipramine which is why it is used for Urinary incontinence and Enuresis along with desipramine. So I would go for that onewhat is "sick euthyroid syndrome?"It's a much talked about syndrome still being investigated, but for purposes of USMLE: Setting = Chronic illness (very sick patient), with not many symptoms as far as hypothroidism, but TSH is slightly high, T3 and T4 total are low. Checking Free T3 and T4 (mostly T4) would show normal range. No treatment is necessary except for tht of underlying chronic illness, or tuning the dosage of medication (Lithium, Amiodarone etc.). epileptic woman want to have a baby, what to do?She is on phenytoin.a.stop phenytoin in first trimester of pregnanyb.change to Phenobarbital3.change to valproicacid4.advice abortionSWITCH TO PHENOBARBITAL. ON THE OTHER HAND, IF CHOICE IS GIVEN NOT TO STOP DRUG (STICK WITH PHENYTOIN), TAKE IT. IF CHOICE IS GIVEN TO STOP PHENYTOIN FIRST TRIMESTER (AND NO OTHER CHOICE LIKE IN THIS CASE TO SWITCH TO PHENOBARBITAL), I HAD VERIFIED IT,

TRIAL OF STOPPING THE DRUG ON THE FIRST TRIMESTER IS ATTEMPTED++Most commonly asked Infections: Focus on ManagementMost commonly asked infectious disease questions I. Meningitis:: Strep Pneumoniae most common in adults. Listeria, H. Flu and Strep B in neonatal. Neisseria Meningitides in adolescents. Staph Aureus in neurosurgery. Cryptococcus in HIV. CT of scan if Papilledema. Otherwise, LP. Give antibiotics prior to CT scan. Empirically: Ceftriaxone or cefotaxime. Ampicillin if Listeria suspected (Neonates, immunocompromised esp T cell, Elderly). If h/o of MRSA: Vanco. If Cryptococcus suspected: Amphotericin B followed by fluconazole therapy lifelong. If TB meningitis, treat with ERHZ or SRHZ for 2 months then RH for 7 months (total 9 months. If pregnant: ERH higher doses same period as PZA and Streptomycin are contraindicated). There is no treatment currently proven for viral/aseptic meningitis. II. Otitis Media:

Strep Pneumoniae most common, then Heamophilus Influenzae, and Moraxella Catarrhalis. Same as in bronchitis and sinusitis. Ds with physical examination (bulging tympanic membrane in setting of ear pain and fever and decreased hearing). Best initial therapy: AMOXICILLIN. If recent Amoxicillin use, or no response, then Amox/Clavulanate. If PCN allergy: azithromycin or clarithromycin. III. Pneumonia: 1- Typical: Strep Pneumoniae (rusty sputum), Heamophilus (smoker COPD), Moraxella2- Atypical: Legionella (older smoker, water source, Accompanying Diarrhea), Mycoplasma (Bullous myringitis, Cold Agglutinins), Chlamydia3- Viral4- Other: PCP (HIV, CD4<200, Bronchoalveolar Lavage => Bactrim. Pentamidine or Atovaquone if sulfa resistant. Dapsone for prophylaxis is sulfa resistant. Atovaquone for prophylaxis if G6PD deficient), Q Fever (Coxiella Burnetti, Meatpacking, Cattle, HSMG, Endocarditis, Hepatitis => Doxycycline), Klebsiella (Current Jelly, Alcoholics => 3rd Cephalo. ), Coccidiomycosis (Arizona, Southwest desert => Fluconazole or Itraconazole. If life threatening = Amphotericin B), Chlamydia Psittaci (Birds). FIRST STEP = CXR. Then Sputum if indicated. ADMIT IF: HYPOXIA (pO2<70, O2 Sat <94%, RR>20-24), COMORBIDITY, DHA.Outpatient: Azithromycin or Clarithromycin. Or new Fluoroquinolones.Inpatient: Fluoroquinolones alone if it’s CAP (Levo-Moxi-Gati) /or /3rd Cephalo. (Ceftriaxone or cefuroxime) + erythromycin or doxycycline. IF HOSPITAL ACQUIRED (after 5-7days inpatient) : GNB Resistant: 3rd Cephalo (Ceftazidime/Cefotaxime), carbapenems, or zosyn (Piperacillin/tazobactam)PNEUMOVAC if increased susceptibility. One dose before 65yo, and another one after. IV. Tuberculosis: - HIV negative: ERHZ or SRHZ for 2 months then RH for 4 months- Pregnant and lactating woman: PZA and Streptmycin CI. ERH (increased doses) for 4-8weeks, then RH twice Qweek x 7months. If resistance is a concern: ERH x 9months. Add Vit B6 with INH use. Breastfeeding is not contraindicated despite presence of small

amounts of meds in milk. - HIV Positive: Same treatment with additional considerations: longer duration, Rifampin interaction with anti-HIV meds, directly-observed TB therapy should be used for all HIV pts, VitB6 mandatory with INH to reduce side effects. - Drug-resistant patient: Resistance only to INH => RZ with E or S x 6months or E/R x 12 months. Others require expert intervention. - Extrapulmonary TB: 9months with same drugs if miliary, bone, meningeal, or joint TB. If bone: early surgical drainage and debridement of necrotic bone. Steriod therapy prevents cardiac constriction and neurologic complications from meningitis. - Latent TB: targeted testing to identify candidates, test for HIV, screen for prior TB Rx and current contraindications. Three regimens are considered: 1) Ideally: INH for 9months. Give Vit B6 if at risk to develop neuropathy (Pregnant, DM, HIV, alcoholic, uremia, Seizure disorder). 2) RZ x 2 months. 3) Rifampin x 4 months (Side Effect: Red Man Syndrome). Care must be taken if HIV patients on Nonnucleoside RT Inhibitors or Protease inhibitors. If Pregnant or lactating: INH QD or BID + Vit B6. - BCG Vaccine: recommended only on individual basis: e.g.: Healthcare worker with high % of multiresistant TB patients. CI if immune deficiency/impairement. - Steriod use in TB Rx: only in TB meningitis and TB pericarditis. V. Infectious Diarrhea: - Most common agent: CAMPYLOBACTER (Acute in healthy patient, bloody diarrhea).- Poultry/Eggs => Salmonella- Travler’s watery diarrhea => Enterotoxic E. Coli most common. If gas, steatorrhea, bloating, and contaminated water source, think Giardia. If bloody with fever and pain, think Enteroinvasive E. Coli or Shigella. If immunocompromised, along with RUQ pain and Jaundice, think Entamoeaba Histolytica. - Undercooked hamburger meat, hemolysis and uremia=> Enterohemorrhagic E. Coli 0157:H7- Friend rice => Bacillus Cereus- Camping trip => either Staph aureus if around 4hrs with UGI symptoms, or contaminated water source Giardia or Cryptosporidiosis. - Immunocompromised and bloody diarrhea => CMV- Immunocompromised (CD4<50), weight loss, high volume watery diarrhea => Cryptosporidium- Acute onset diarrhea with RUQ pain => Yersinia- Contaminated shellfish => Vibrio Parahaemolyticus or V. Vulnificus (if seawater contamination and underlying liver disease)- Ship diarrhea => Most common Norwalk virus. Rotavirus second. Also in children. - Antibiotic use => C. Difficile. Canned food => C. Botulium. Unrefrigerated meat => C. Perfringens. - Pruritic rash, Diarrhea, Vomiting and WHEEZING => Scrombroid (histamine reaction)FIRST STEP: Is there blood or not?- Blood => Enteroinvasive : Salmonella, Shigella (HUS), Yersinia, E. Coli (HUS), and Campylobacter (Guillain-barre). MC protozoan with blood = Entamoeba Histolytica- No blood => Giardia, Cryptosporidium, Protozoans, Cyclospora, Viruses. NEXT STEP: NO BLOOD => WBC in Stools (Methylene Blue Testing) => WBC+ = invasive

NEXT STEP: - Stool culture- Ova/parasites: Giardia and Cryptosporidia- C. Diff Toxin in stool TREATMENT: Oral fluid and electrolyte replacement (hypokalemic metabolic acidosis). IV Fluids if severe. - Campylobacter => Erythromycin- Giardia => Metronidazole- Cryptosporidium => Control of HIV with HAART- C. Diff => Metronidazole. - Scrombroid => Antihistamine - E. Coli => 3rd Cephalosporins VI. Hepatitis: - Chronic Hepatitis B: Interferon or Lamivudine 3TC. - Chronic Hepatitis C: Interferon combined with Ribavirin- Needlestick: HBIg and Hep B vaccine if Hep B. No PEP for Hep C VII. STD’s: A- Urethritis/Cervicitis: - Gonococcal: Gram –ve, diplococci coffeebean intracellular. Serology for Chlamydia or Ligase chain reaction test. Culture for Gonorrhea on Thayer-Martin. IM Ceftriaxone Single dose and PO doxycycline x7days or PO Azithromycin Single dose. (We add treatment for Chlamydia)- Non Gonococcal Urethritis: Chlamydia, Ureaplasma, Mycoplasma Hominis, Trichomonas, Herpes Simplex. B- PID: cervical motion tenderness. NEXT: US Pelvis (R/O Ovarian cyst or TOA). Laparoscopy is definitive test. Treat: Admit when high WBC or high fever• Inpatient – IV CEFOXITIN (or Cefotetan) + Doxycycline• Outpatient – IV CEFTRIAXONE single dose + Doxycycline x14d C- Syphilis: - Primary: Benzathine-Penicillin single dose IM 2.4 million- Secondary: Benzathine Penicillin Q week x 3 weeks same dose- Tertiary: IV Penicillin 10-20 million U/day x10 days- If allergy to penicillin: desensitize or give doxycycline. Desensitize first in Tertiary and Pregnancy D- Chancroid:Heamophilus Ducreyi = Gram –ve B. Diagnose clinically along with Gram stain, culture and PCR if needed. TREAT : Azithromycin single dose or IM rocephin single dose. Alternatives: Ciprofloxacin x 3 days or Erythromycin x 7 days. E- Lymphogranuloma Venereum:Chlamydia Trachomatis – transitory primary lesion with suppurative lymphangitis. Diagnose clinically and rising titers of complement fixing antibodies. Isolate organism in pus of buboes.TREAT: DOXYCYCLINE OR ERYTHROMYCIN F- Granuloma Inguinale:

Donovania Granulomatis – painless red nodule, then elevated granulomatous mass in perineal area. Healing and scar formation. Diagnose with Giemsa or Wright stain = Donovan bodies – Punch biopsy if necessaryTREAT: DOXYCYCLINE OR TMP/SMX. OR ERYTHROMYCIN G- Genital Herpes: Herpes Simples II – Tzanck test and cultureTREAT: ACYCLOVIR – famcyclovir – valacyclovir. C-section if active disease in pregnant. If mother has disease but no vesicles apparent (not active), vaginal delivery. VIII. Urinary Tract Infections: A- Cystitis: E. Coli, then Proteus, Klebsiella, Enterobacter, Enterococci, Staph Saprophyticus (Honeymoon cystitis). FIRST: U/A (nitrites = Gram –ve)TREAT: • Uncomplicated: 3 days of TMP/SMX or Quinolone• Diabetic: 7 days• Pregnant: Amoxicillin or Nitrofurantoin B- Pyelonephritis: FIRST: U/A and US to r/o obstructionTREAT: • Outpatient: 10-14 days fluoroquinolones, TMP/SMX• Inpatient: IV Quinolones, IV Ampicillin/Genta, 3rd cephalosporins IX. Impetigo/Erysipela: Group A BetaHemol. And S Aureus (Bullous impetigo).TREAT: If blood cultures +ve or facial erysipelas => IV ATBx - DOC = IV Peni 2 million Q4h - If allergic = IV Cefazolin if minor or IV Vanco if major allergyIf oral therapy only:- Amoxicillin twice daily for 7 -10 days- If allergic: Cephalexin, Clindamycin, or macrolides (Clarithromycin or Azithromycin) are alternatives: X. Osteomyelitis FIRST: X-Ray+++ Takes two to three weeks before we see signsNEXT: Technecium bone scan or MRINEXT: Blood Culture, BONE BIOPSY AND CULTURE if BCx are sterileTREAT: IV Semisynthetic Penicillin + Genta or 3rd Ceph until culture results obtained. IV ATBx x 6-12 weeks. If Chronic Osteomyelitis: IV x 12weeks then PO x 8-12 weeks. If allergic, IV VANCO. XI. Septic Joint: FIRST: TAP IT + CULTURE – XRAYTREAT: - Non gonococcal: IV Semi-synthetic Penicillin + Genta or 3rd Ceph. If non gonococcal. - Gonococcal: IV Ceftriaxone XII. Infective endocarditis: FIRST: Blood Cultures + Transesophageal EchocardiogramTREAT: IV Semi synthetic Penicillin + IV Ampicillin + Genta. Then alter according to

cultures x 4-6 weeks. 8 weeks if it’s prosthetic valve. IV VANCO IF PENI RESISTANT.

TAKE TO OR IF: CHF, systemic emboli, persistent bacteremia despite ATBx, Fungal etiology, Extravalvular infection, Prosthetic valve obstruction, Recurrence of infection despite ATBx. IF HACEK organisms (Hemophilus aphrophilus, Hemophilus parainfluenzae, Actinobacillus Actinomycetemocomitans, Cardiobacterium hominis, Eikenella Corrodens, Kingella Kingae): DOC = CEFTRIAXONE x 4 weeks. GIVE PROPHYLAXIS IF: - DENTAL PROCEDURES: AMOXICILLIN OR CLINDA IF ALLERGIC (azithromycin, Clarithromycin, or Cephalexin)- URINARY OR GI PROCEDURES: AMP/GENTA OR VANCO/GENTA IF ALLERGIC- Bronchoscopy rigid only- CARDIAC CONDITIONS: Mitral Valve prolapse (WITH REGURGITATION), Hypertrophic cardiomyopathy, Congenital Malformation except in ASD only Primum, H/O Bacterial Endocarditis. - NO PROPHYLAXIS IN CARDIAC CATH+++, MITRAL VALVE PROLAPSE W/O REGURGITATION, LABOR WITH ONLY MVP, ASD ostium secundum, Intubation, Flexible bronchoscopy, Transesophageal Echocardiography, C-section, Hysterectomy, circumcision. XIII. Lyme Disease: - Ixodes scapularis (dammini) tick. Needs 24h of attachment to transmit borrelia burgdorefri.- Bite often not remembered may be missing in the question stem- Symptoms 3-30days after bite.- ERYTHEMA MIGRANS resolving in a few weeks, Flu-like syndrome. - Weeks later: NEUROLOGIC SYMPTOMS – FACIAL PARALYSIS, MENINGOENCEPHALITIS, Headache, Memory- Other: MYOCARDITIS, PERICARDITIS- Months: JOINT INVOLVEMENT DIAGNOSE: Clinical (ERYTHEMA + ONE LATE MANIF) + LAB (ELISA +WB)TREAT MINOR SYMPTOMS WITH DOXYCYCLINE OR AMOXICILLIN. TREAT CARDIAC AND FACIAL PALSY WITH PO DOXYTREAT JOINTS WITH A MONTH OF ORAL DOXYTREAT MYOCARDITIS, and NEURO WITH IV CEFTRIAXONE

XIV. OTHER: 1) Infectious Mononucleosis: - Fever, malaise, sore throat, lymphadenopathy, SMG, and occasionally, maculopapular rash. - FIRST: Monospot (heterophil agglutination test) – Atypical lymphocytes on blood smear- 95% recover without intervention. If amp/amox given because of Cx+ve for Strep, may cause maculopapular rash. Do nothing. Self-limited. - Hepatitis, Mycoarditis, Encephalitis are managed symptomatically. Splenic rupture =>

Emergency Splenectomy 2) Rocky Mountain Spotted Fever:- Exposure to the woodtick Dermacentor Andersoni EASTERN USA- 2-14days: Flu-like, Pneumonitis (usual cause of death), FLUSHED FACE AND INJECTED CONJUNCTIVA- day 2-6: RASH WRITS AND ANKLES SPREADS CENTRALLY – PALMS AND SOLES+++- Complicated cases: ARDS – DELIRIUM – HSMG/JAUNDICE – MYOCARDITIS- Labs: Thrombocytopenia, High LFT’s- DIAGNOSE WITH SKIN BIOPSY OR SEROLOGY (not until 2 weeks later)- Differentiate from Meningococcemia - PROPHLAXY AFTER TICK BITE IS NOT CURRENTLY RECOMMENDED.- Prevent with protective clothing, tick-repellant, removal of ticks at frequent intervals. - TREAT WITH PO or IV DOXYCYCLINE++ - IF PREGNANT: CHLORAMPHENICOL. 3) KAWASAKI Syndrome: - MUCOCUTANEOUS LYMPH NODE SYNDROME- Asian Chilren at higher risk- FEVER AND 4 OF: ♣BILATERAL CONJUNCTIVITIS MUCOUS MEMBRANE CHANGES: STRAWBERRY♣ TONGUE desquamation, erythema)♣EXTREMITY CHANGES (edema,CERVICAL ♣LYMPHADENOPATHY- MAJOR COMPLICATION: CORONARY ARTERITIS = 1 OUT OF 4 => AMI- TREAT with ASIPIRIN AND IV Ig in high doses. Steroids in refractory disease. - COUMADIN FOR CORONARY ARTERY ANEURYSMS > 6.5mm 4) BOTULISM: Difference between adult and infant botulism: - Adult: ingestion of pre-formed toxin from canned food- Infant: ingestion of organisms with elaboration or toxin in vivo (e.g.: honey-related = floppy-baby syndrome). TREAT WITH BOTULINUS ANTITOXIN FROM AUTHORITIES AFTER REPORTINGTREAT RESPIRATORY FAILURE WITH INTUBATION AND MECHANICAL VENTILATION. 5) ANTHRAX: - Naturally occurring: exposure to sheep, horses, goats- Bioterrorism- B. Anthracis = Gram+ve Bacillus aerobic spore-forming. - Transmission: DIRECT INOCULATION OR INHALATION- Cutaneous Anthrax within 2 weeks of exposure provokes toxin which impairs neutrophils, and generates edema. The capsule allows host defense evasion. First, erythematous papule, then vesiculates, then ulcerates, then painless black eschar. Regional lymphadenopathy ++, headache, nausea, vomiting => self-limited or may spread with meningitis. - Inhalational Anthrax: 10 days after exposure. Flu-like symptoms, then mediastinitis.

Within hours, spores germinate in macrophages in lungs or multiply in lymphatics causing hemorrhagic lymphadenitis – overwhelming sepsis. CXR IS FIRST STEP (Mediastinal widening = hallmark).- GI anthrax: 2-5 days after ingestion. Fever, abdominal pain, rebound tenderness, coffee-ground emesis, even bowel perforation. Can be only oropharyngeal: regional lymphadenopathy, cervical edema, dysphagia, and URI. - DIAGNOSE WITH PLEURAL FLUID, CSF, GRAM STAIN CUTANEOUS LESION =>BOXCAR SHAPE CAPSULE ORGANISM – CULTURE – PCR - TREAT WITH: DOC = CIPROFLOXACIN X 7-10DAYS (IF NATURALLY OCCURING – 60 DAYS IF BIOTERRORISM). DOXYCYCLINE = FIRST-LINE ALTERNATIVE. - Add rifampin if inhalational/systemic X 2 WEEKS- PROPHYLAXIS AFTER EXPOSURE TO SPORES: SINGLE-DRUG THERAPY. 6) Whooping Cough:B. Pertussis. Catarrhal stage, paroxysmal stage (whoop cough), and convalescent stage. DIAGNOSE BY ISOLATING ORGANISM FROM NASOPHARYNGEAL CULTURE (Bordet-gengou agar). Imminzation available. TREAT WITH ERYTHROMYCIN x 10 days. 7) SARS = coronavirusDIAGNOSE: Fever 38deg, h/o travel or contact, rhinitis, sore throat, diarrhea. MANAGEMENT: - History of contact or travel- ADMIT and ISOLATE (Universal Precautions for now)- Throat Swab for SARS Test – CXR – CBC/Diff – GOT/GPT – LDH – CPK- Does CXR show Pneumonia?♣YES: Send to negative pressurized quarantine + report online to CDC♣NO: Lymphocyte < 1000 or Inc GOT/GPT or Inc LDH or CPK, or Platelet<150K YES: Respiratory distress? If yes => Suspect case of SARS, If no => Quarantine but no need for reverse isolation. NO: Respiratory distress? If yes => Suspect case of SARS, If no => Home rest with quarantine, recheck in 3 days. TREAT: No specific treatment recommendations can be made at this time. Empiric therapy should include coverage for organisms associated with any community-acquired pneumonia of unclear etiology, including agents with activity against both typical and atypical respiratory pathogens. Treatment choices may be influenced by severity of the illness. Infectious disease consultation is recommended.

References:- CMDT- Kaplan notes - CDC- WHOPUPIL SIZE, REACTIVITY, etc. ASSESS PUPIL SIZE, SHAPE, REACTIVITY, AND ACCOMMODATION. (The parasymp system controls constriction -dilatation is controlled by the symp system.

parasymp system is dominant because the iris sphincter is a stronger muscle.)Two most common pupillary problems: anisocoria and decreased pupillary constriction to light.A) Unequal Pupils - Anisocoria:20% of the population have perceptible anisocoria. more pronounced in darkness. Horner’s syndrome – occulo-sympathetic paralysis-- interruption of sympathetic innervation of eye (interruption of sympathetic pathways in the medulla, spinal cord, or peripheral sympathetic trunk) causing miosis, ptosis, ipsilateral anhidrosis may be secondary to lung cancer, etc. If longstanding--- heterochromia of the iris may occur (affected side being less pigmented). There may be transient dilation of conjunctival vessels and increased accommodation, preganglionic lesion is more serious; hydroxyampthetamine 1% will dilate a pre-ganglionic lesion, but fail to dilate a post-ganglionic lesion. topical cocaine which will fail to dilate the miotic pupil relative to the larger pupil. Causes are vertebral fractures, mediastinal tumors, aneurysms of the carotid or subclavian arteries, demyelinating disease, migraine variants and apical lobe bronchogenic carcinoma. CVA, Syringomyeliaif newly acquired in an adult --must be thoroughly investigated -- significant possibility that there is an underlying malignancy Adie’s tonic pupil- -- idiopathic benign internal ophthalmoplegia. Parasympathetic dysfunction at or distal to ciliary ganglion. relative mydriasis in bright illumination, poor to absent light reaction. a slow contraction to prolonged accommodation-- a slow re-dilatation after completion of accommodation (the near stimulus is removed). slow dilatation in prolonged darkness, slow constriction in prolonged light. Mostly in young females & unilateral .. pupil is hypersensitive to weak pilocarpine 0.125%. (in Pharmacological mydriasis, no response). There is a segmental palsy of the iris sphincter muscle. If associated with altered deep tendon reflexes (hypo- or areflexia) is called Holmes-Adie syndrome.

B) Equal Pupils:Argyl Robertson pupil –miosis, bilateral, poor responsiveness to light good response to accommodation; Pupil contracts with Physostigmine (agonist), but Atropine (antagonist) produces poor dilatation. Pupil must be small , Retina is light sensitiveCause lesions in Edinger Westphal nucleus associated with neurosyphilis, Lyme disease , diabetes (ischemia due to narrowing of small vessels that supply nerve), MS and midbrain tumors. Miosis can also be seen in the early stages of coma and Due to drugs like Morphine and Pilocarpine. dilated pupil is not indicative of pending herniation unless the patient is comatose. C) Relative Afferent Pupil DefectThe afferent pupil defect is the dilation of the pupil in the both eyes when the light is swung from the normal eye over to the defective one. This is due to poor transmission of the light to the brain via a damaged optic nerve. The brain perceives the same stimulus as having a decreased intensity and dilates both pupils. If both eyes are functioning poorly, there will not be an relative afferent pupillary defect. There has to be extensive retinal or

optic nerve damage for an RAPD to be demonstrable. Media opacities like cataracts or vitreous hemorrrhages alone would not be sufficient. May be caused by optic neuritis, ischemic optic neuropathy, chiasmal tumors, retinal artery or vein occlusion, retinal detachment, acute angle-closure glaucoma, etc.

Drugs. Mydriasis. Cyclopentolate: maximal dilatation at 25 to 75 minutes, lasting 6 to 24 hours;homatropine: maximal dilatation is rapid, must be used TID or QID to maintain mydriasis; scopolamine: dilatation at about 1 hour, must be used TID to QID. Miosis. Pilocarpine in 0.25%, 0.5%, and 1.0%. Generally needed only once per day. See section on acute glaucoma for exception. Anesthesia. Tetracaine 1%, proparacaine 0.5%. These cause corneal toxicity with repeated use--NEVER prescribe for home use. Apart from atropine, class of drugs which cause pupillary dilatation: Alpha adrenoceptor agonists (sympathomimetics) such as epinephrine Small Pupil: Cause: Meningitis, pilocarpine instillation, morphine, histamine Pinpoint Pupils: Pontine hemorrhage or infarct, lesions in the Subthalamus, Thalamus, interruption of ascending Sympathetic pupillodilator fibers

Large Pupil: Causes: Traumatic irridioplegia, unilateral mydriasis via Hippocampal herniation through the Tentorium with compression of CN III as it exits brainstem. temporal Arteritis, CVA.Patients with congenital achromatopsia and congenital stationary night blindness have been known to show a transient pupillary constriction to darkness.

OCULAR MOVEMENTS:If supranuclear control is lacking -- total gaze palsy (ie inability to move both eyes together in a certain direction), limitation of gaze excursion, deviation with coarse nystagmus or internuclear ophthalmoplegia can result. Frontal lobe lesions prevent conjugate movements to the opposite side on demand but normal movements on pursuit. Irritative lesions cause the eyes to deviate to the opposite side acutely. Destructive lesions result in ocular deviation to the same side but inability to turn to the opposite side. Occipital lesions cause impairment of the pursuit movement to the same side and a homonymous field defect to the opposite side.Brain stem pathology is associated with multiple cranial nerve palsies and contralateral hemiplegia.Oculogyric crises are spasms of upward gaze typical of post-encephalitic states, Parkinson’s disease and Phenothiazine toxicity.Downward deviations occur in patients with hydrocephalus (setting sun), coma, thalamic pathology and transiently in newborns.Vertical gaze paresis is classical for dorsal midbrain lesions, such as pinealomas (Parinaud’s), gliomas, AV malformations, MS and 3rd ventricle tumours.

Internuclear ophthalmoplegia results from lesions in the medial longitudinal fasciculus connecting the ipsilateral CN VI and the contralateral CN III nuclei. loss of the medial rectus adduction in the contralateral eye and a jerky nystagmus in the abducting eye, often indicative of MS, but can also be the result of head trauma, brain stem neoplasms, AV malformations or hydrocephalus. All ocular muscle palsies and ptosis may be mimicked by myasthenia gravis. Here a Tensilon test is diagnostic. In younger patients up to 25% have a thymoma.Try yourselves at this: Neuro Cases#1: A 26-year-old man was involved in a street brawl and received a knife wound of the right arm at about the midhumeral level. Motor loss consisted of paralysis of the pronator muscles of the forearm and the long flexor muscles of the wrist and fingers, with the exception of the flexor carpi ulnaris and the medial half of the flexor digitorum profundus. As a result of this, the right forearm was kept in the supine position; wrist flexion was weak and was accompanied by adduction. The latter deviation was due to the paralysis of the flexor carpi radialis and the strength of both the flexor carpi ulnaris and the medial half of the flexor digitorum profundus. No flexion was possible at the interphalangeal joints of the index and middle fingers, although weak flexion of the metacarpophalangeal joints of these fingers was attempted by the interossei. When the patient was asked to make a fist of his right hand, the index and, to a lesser extent, the middle fingers tended to remain straight, while the ring and little fingers flexed. The latter two fingers were weakened by the loss of the flexor digitorum superficialis. Flexion of the terminal phalanx of the thumb was lost due to paralysis of the flexor pollicis longus. The muscles of the thenar eminence were paralyzed and the right thumb was laterally rotated and adducted. Sensory loss of the skin of the right hand involved the lateral half of the palm and the palmar aspect of the lateral three and one-half fingers. There was also sensory loss of the skin of the distal parts of the dorsal surfaces of the lateral three and one-half fingers. What steps should the surgeon take in the repair of the wound? Is there a crucial time factor in repairing the nerve? How long would it take until function returns? What will be the first sign? Until recovery of function, how should the arm be treated?

#2: A 45-year-old man was recovering from a mild upper respiratory tract infection when he suddenly noticed weakness in both legs while walking up the stairs. He also developed a numb sensation over the lower part of both legs and the feet. Two days later, while shaving, he noticed a weakness of the muscles on the right side of his face. On physical examination the patient did not appear to be ill. He had no pyrexia. Examination of his leg muscles showed obvious signs of muscle weakness involving both legs, especially below the knees. Both ankle reflexes were absent and the right knee jerk was diminished. He had sensory deficits for touch and pain sensations in the distribution of the stocking area of both feet and lower legs, and a mild form of facial nerve palsy involving the right side of the face. Would you expect damage to be central (spinal cord, brain) or peripheral? Why?Would you request that an MRI be done?Would you hospitalize the patient?

#3: A successful 48-year old attorney was told he was hypertensive, but did not take his blood pressure medications. He was apparently well until 4 days after his birthday, when he developed several episodes of blurred vision, "like a shade coming down," involving his left eye. These attacks each lasted less than an hour. He was referred for neurologic evaluation but because of a busy schedule, canceled the appointment. Several weeks later, he complained to his wife of a left-sided headache. She found him 1/2 hour later, slumped in a chair apparently confused and paralyzed on the right side. Neurologic examination in the hospital revealed total paralysis of the right arm and severe weakness of the right face. The leg was only mildly affected. Deep tendon reflexes were initially depressed on the right side, but within several days, became hyperactive; there was Babinsky response on the right. The patient was globally aphasic; he was unable to produce any intelligible speech and appeared to understand only very simple phrases. What's the territory?Headache and Visual problems on the right. Motor problems on the left. How do you explain that?The CT scan showed that the infarct included the territory of the middle cerebral artery but the occlusion involved the internal carotid artery. Why was the territory of the anterior cerebral artery not infarcted?

#4: After a fight in a bar, a middle-aged lady was brought to the emergency room. She has a knife wound low on her right side in the back. She complains of numbness on her right leg and especially on her thigh. Upon examination you find the following. She has no sensation when stimulated, including pin pricks and touch, along a strip on her thigh on the right side. When her right toe is moved up or down, the patient cannot tell in which direction it is moved. Although she can feel that she is touched on the right leg (except where noted above), she cannot discriminate whether she is touched with two objects close together or just one object. She cannot feel when the tuning fork stops vibrating. She can tell the difference between the sharp point versus the dull point of the safety pin on the right leg. She has normal sensation above her pelvic region on both sides. She has a reflex response when her Achilles tendon is tapped (ankle reflex); she has no reflex response when the tendon of the quadriceps is tapped (knee jerk) on the right side. Her left side is totally normal. What type of lesion would result in absence of sensation of a narrow strip on her thigh?Which sensory tracts are damaged to explain the lack of proprioception, fine discrimination and vibration?There was no loss of pain and temperature. Which sensory tract was spared?Explain the results of the Achilles and quatricepes reflex tests.Where is the lesion? #5: Now it's the same case. But this time, the patient also complains that her left leg is numb and she has lost voluntary movements of her right leg. Her sensory findings for her right leg are the same as during the previous examination. On the left leg, the patient cannot tell the difference whether she is touched with the dull

part or the sharp part of the needle --> they both feel dull to her. However, on her left leg, she can accurately identify when the tunning fork stops and whether you move her toe up or down. She is unable to move her toes, foot and leg on the right side upon command. When stroking the bottom of her feet, the right one has the plantar extension. You test reflexes: the knee jerk is still absent on the right side; however, the Achilles tendon reflex is much more brisk on the right side than the left side. When quickly dorsoflexing her right foot, you can feel a rhythmic oscillation; there is also some resistance to movement of the foot and lower leg. Over the area of the right quadriceps, you note some twitching of the muscle fibers. What sensory tract is now affected to give the absence of pain sensation on the left side?What structure is included in the lesion to give the plantar extension (Babinsky sign)?Explain the results of the reflex tests and the resistance felt when dorsoflexing the foot.What does the twitching of the muscle fibers tell you about the area included in the lesion?Where is the lesion and which areas are included?#6:A 44-year-old woman was admitted after having a seizure. She was lethargic, with a right facial droop, right hemiparesis, and right hyperreflexia. She complained of headache and a painful neck. A few days later, she seemed slightly more alert and made purposeful movements with her left hand--but not her right hand. She was still unresponsive to spoken commands and had a rigid neck. Other findings included bilateral papilledema, a right pupil that was smaller than the left, incomplete extraocular movements on the left side, decreased right corneal reflex, and right nasolabial droop. The patient's right arm was hypertonic and paretic, but the other extremities were normal. All reflexes appeared within normal range. The right plantar extensor response was equivocal, but the left was normal. The blood pressure, pulse rate and temperature were in the normal range; the white blood count was 11,200 µL, and the erythrocyte sedimentation rate was 30 mm/h. A CT scan showed a high-density area in the cisterns, especially on the right side. There were right motor problems and problems with especially the left eye. What is this due to?Would you request a lumbar puncture? What might it show?What is the diagnosis?

#7: A 36-year old school teacher was evaluated for headaches and left arm weakness. The headaches began approximately 2 years previously, following an episode of severe coughing. Dull occipital pain had persisted on and off since that time. In addition, she had noted dull pain in the left hand during the past year. Six months ago she noted the onset of progressive numbness and weakness of the left hand and she reports that she is now unable to distinguish the temperature of bath water with her left hand. During the past month she also has found that her walking is "stiffer than usual." There have been no bowel or bladder abnormalities and her general health is otherwise well. Examination was remarkable for mild thoracic kyphoscoliosis. Cranial nerve testing revealed a pupillary asymmetry (left, 1 mm, right 3 mm). Both pupils were round and reactive. There was, in addition, a left ptosis; the left side of her face appeared drier than on the right. There was marked atrophy of all muscle groups in the left arm. There was

complete loss of pain and temperature perception from C5 to T1 on the left and patchy abnormalities of pain and temperature sensation in the right forearm. Position and vibration perception were normal throughout. She was areflexic in the left arm and hyperreflexic in the right arm and in both legs, with moderate spacticity of both legs. Gait was mildly stiff legged, but otherwise unremarkable. What could be the cause for the unequal size of pupils and ptosis?Why would one side of her face be drier?Which area in the spinal cord could be involved to give the above symptoms? #8: A 49-year old landscape artist, who had traveled to many countries, was admitted to the hospital because of a sudden onset of facial numbness, ataxia, vertigo, nausea, and vomiting. Examination revealed impaired sensation over the left half of the face. The arm and leg on the left side were clumsy, and there was an intention tremor on the left. A left-sided Horner's syndrome (miosis, ptosis, anhydrosis) was apparent. There was subjective numbness of the right arm, although no abnormalities could be detected on examination. Over the ensuing 12 hours, he developed difficulty swallowing and complained of intractable hiccups. Vibratory and position sense were now impaired in the left arm, the vocal cord was paralyzed, and the gag reflex was diminished. On the right side of the body, there was impaired pain and temperature sensibility. What's your diagnosis?Why was sensation impaired over the face? Why were the arm and leg on the left described as clumsy? There is also an intention tremor. Why was there a Horner's syndrome? The patient had difficulty in swallowing, had paralyzed vocal cords and a diminished gag reflex. Why? What caused the vertigo, nausea and vomiting? Vibration and position sense impaired on left arm. Why? Pain and temperature were impaired on the right side of the body. Why? #9: A 53-year old woman is referred to the neurologist. She complains about numbness on the right half of her face and loss of motor control also on the right side of the face. She is numb on the left side of her body. Upon examination the following are noted. When asked to smile, the patient can draw the lips back on the left side and she can wrinkle her forehead on that side; however, the right side shows little motility. When the right side of her face is touched with a sharp or a dull object, she cannot distinguish the sharp one from the dull one--both stimuli feel dull to her. When the left or right corneas are lightly touched, only the left one responds with an eye blink. Touching various parts of her body with dull and sharp objects, she makes accurate responses when touched on the right side; however, on the left side both stimuli appear dull to her (on her body and upper and lower limbs). She can accurately detect, on both sides of the body, when the tuning fork stops vibrating and if the toes are moved up or down. She can accurately localize where she is touched on her body and all limbs. She has no motor problems involving her body and upper or lower limbs. How would you explain the motor problems to her face?Would you expect that sounds appear louder on the right side? Why?How do you explain the lack of pain discrimination on the right side of her face versus

the left side of her body?How do you explain the results when testing the corneal reflex?Where would you expect the lesion to be? #10: A 67-year old man suddenly could no longer move his left arm and leg. His family took him to the emergency room; on the way there he also complained that he was seeing double. The neurological examination found the following. Testing the motor system, there were exaggerated biceps, triceps, knee and ankle reflexes on the left limbs but normal on the right. The plantar response was extensor on the left (Babinsky sign) and flexor on the right. On the left side of his face, he could not pull his lips back on command; however, he could wrinkle his forehead. Testing for eye movements: when asked to look to the left, the left eye moved laterally but the right eye looked straight ahead; at rest, the right eye tended to move laterally. Testing the sensory system, the only finding was that the right eye did not constrict in response to light. All other sensory systems were intact. Which structure was involved to give you the exaggerated reflexes on the left upper and lower limb?What caused the problems with the facial muscles?What did the tests for eye movements show? Is that consistent with the pupillary response?What would an MRI show? #11: A 59-year-old man was well until 2 months prior to admission, when he developed early morning headaches which would awaken him from sleep. Although the headaches typically disappeared shortly after arising from bed, the severity of the headaches was increasing with time. Two days earlier he developed progressive weakness of his left hand and an unstable gait. Past medical history was notable for a malignant melanoma which was discovered on his right arm 2 years previously and treated with local resection and a radical axillary lymph node dissection. Review of systems revealed weight loss of 15 pounds over the past month. On examination, the patient was alert, although confused and unable to give a coherent history. Fundoscopic examination revealed bilateral papilledema. There was nearly complete paralysis of the left arm and mild weakness of the proximal left leg. There was a left facial asymmetry. Reflexes were increased on the left and plantar responses were extensor bilaterally. Sensory examination was normal. The remainder of the cranial nerves were normal. Gait was stable, although the patient had a tendency to fall when turning rapidly. What is the most likely cause of the patient's symptoms?What is the significance of the papilledema and headaches (especially early morning ones)?What tests would you recommend? #12: A 55-year-old professional exhibiting signs of confusion was brought to the hospital. The history gathered from his landlady disclosed that he had been separated from his family because he drank too much. Although he was apparently in good health, his landlady had entered the apartment on the day of admission because he did not respond to her calls. She found him lying on the floor, incontinent of urine and appearing bewildered; he had also bitten his lip. The landlady remembered that 2 months earlier he had been involved in a fight in a bar; 3 weeks previously he had fractured his wrist falling

down stairs. On examination, the patient was unconcerned, disheveled, and dirty. Bruises on his head and legs were consistent with recent trauma from a fall. The patient appeared to fall asleep when left alone. Neurologic examination showed normal optic fundi, normal extraocular movements, and no abnormalities that would result from dysfunction of other cranial nerves. The reflexes were normal and symmetric, and there was a left sided plantar extensor response. Vital signs, complete blood count, and urinalysis were within normal limits. A lumbar puncture showed an opening pressure of 180 mm of water, xanthochromia, a protein level of 80 mg/dL, and a glucose level of 70mg/dL. Cell counts in all tubes showed red blood cells, 800/µL; lymphocytes, 20/µL, and polymorphonuclear neutrophils, 4/µL. A CT scan of the head was obtained. Over the next 36 hours, the patient became deeply obtunded and seemed to develop a left-sided hemiparesis. Could the patient's problems be due to recent trauma?What is the most likely diagnosis? What do the findings from the lumbar puncture indicate?ANSWERS: ------------------------------------------------------------------------------------------------------------------------------Answers:#1: Median nerve. The wound has to be cleaned and be free of infection. The nerve can be repaired up to 12 months post-trauma. How long will it takes for recovery and how will the patient know? 1.5 mm per day is the average rate of regeneration. Once the nerve has entered the distal segment, this segment becomes sensitive to mechanical stimulation (Tinel’s sign). Pain caused by deep pressure is the first sign of recovery; vasomotor control return at this time; touch and tactile sensation are the last to return. Sensation returns before voluntary movement. Until recovery of function, how should the arm be treated? Paralyzed muscles should be protected with a splint and joints exercised daily to maintain circulation and preserve motility. #2: Would you expect damage to be central (spinal cord, brain) or peripheral? Why?The damage would involve the peripheral nervous system. The reflexes were either diminished or absent-central problems would cause hyperreflexia; sensory and motor systems were equally affected on both legs-indicating most likely peripheral involvement

Would you request that an MRI be done?No. On what area would you have them focus the MRI? Would you hospitalize the patient? Yes. The peripheral nerves dealing with breathing and/or swallowing could become involved. Patient has polyneuropathy, probably Gullain-Barré syndrome. #3: A CT scan revealed an infarct in the territory of the middle cerebral artery of the left side. Angiography revealed occlusion of the internal carotid artery. The patient recovered only minimally. Motor problems on the left because the motor fibers crossed therefore the symptoms were opposite to the lesion.

The CT scan showed that the infarct included the territory of the middle cerebral artery but the occlusion involved the internal carotid artery. Why was the territory of the anterior cerebral artery not infarcted? There was collateral circulation via the anterior communicating artery, i.e. blood supply from the right internal carotid system supplied the blood to the left anterior cerebral artery. The visual problems are called "amaurosis fugax" = transient monocular blindness due to interruption of blood flow from the ophthalmic artery to the retinal artery. #4: To have absence of all sensation, anesthesia, the damage must occur either to the incoming afferents (i.e. dorsal root) or the posterolateral area in the spinal cord where the fibers enter (i.e. zone of Lissauer). In this case, the proximal portions of the dorsal root and zone of Lissauer was damaged.Damage to posterior column on the right side could explain the lack of proprioception, fine discrimination and vibration. The anterolateral system was spared. The Achilles reflex is normal therefore sensory and motor root segments of S1-2 are intact. The knee jerk is absent which means either the sensory or motor component is absent. In this case, the incoming sensory information (proprioception) is absent on the right side (L2-4).The lesion therefore is at the posterolateral quadrant of the spinal cord at level L2. #5: What sensory tract is now affected to give the absence of pain sensation on the left side?The anterolateral system is damaged on the right side. However, the posterior columns are intact on the left side, therefore there is no loss of discriminatory touch or vibration on that side. What structure is included in the lesion to give the plantar extension (Babinsky sign)?The corticospinal tract on the right side. Explain the results of the reflex tests and the resistance felt when dorsoflexing the foot.The Achilles tendon reflex is hyperreflexic. The spasticity and clonus, together with the hyperreflexia, indicate an upper motor problem = damage to the corticospinal tract. What does the twitching of the muscle fibers tell you about the area included in the lesion?The twitching indicates that a motor neuron has been damaged (lower motor neuron sign). This area as well as the level of anesthesia and other sensory losses, indicates that the lesion is at the L2 level.Where is the lesion and which areas are included?This is a lateral hemisection of the spinal cord (Brown-Sequard syndrome) at L2 level. # 6: The sudden onset suggests involvement of blood vessels; however, it does not appear to be due to hypertension. It could be an aneurysm.There were right motor problems and problems with especially the left eye. What is this due to?These alternating signs are due to damage to the motor tract carrying information for the opposite side of the body; the problem with the eye is caused by involvement of the cranial nerve III. Would you request a lumbar puncture? What might it show?The bilateral papilledema suggests increased intracranial pressure; a lumbar puncture is not advisable. If done, the finding would be blood in the CSF. What is the diagnosis?Subarachnoid hemorrhage on the left side of the midbrain, putting pressure on the medial

portion of the left cerebral peduncle (especially motor pathways to the arm and face; the sign for the latter are decreased corneal reflex and nasolabial droop) and the CN III (signs are small pupil, loss of some eye movements). Brainstem = Crossed signs: Ipsilateral Nerve Palsy (Left), and contralateral motor sings (right). #7: What could be the cause for the unequal size of pupils and ptosis?Left eye: miosis, ptosis and anhydrosis due to interruption of sympathetic innervation of the face including the eye; this is called Horner's syndrome. Why would one side of her face be drier?Interruption of sympathetic ANS; this is also part of Horner's syndrome. Which area in the spinal cord could be involved to give the above symptoms?Syringomyelia: cavitation within the spinal cord around the area of the central canal; usually in the cervical spinal cord. It often affects only the crossing pain and temperature fibers (these are second order fibers: cell bodies are in the dorsal horn and the axons are crossing near the central canal to join the contralateral anterolateral system (ALS). The initial symptoms are loss of pain and temperature bilaterally but only for arm and shoulder regions. As the cavity expands, the posterior columns might be involved (with loss of discriminatory touch) as well as the anterior horn (causing paralysis of muscles in shoulders, arms and hands).In this case, the intermediolateral cell column was affected giving the Horner's syndrome; there was greater involvement of the left portion of the spinal cord. #8: MRI demonstrated an abnormality, presumably infarction, in the lateral medulla on the left side, and a presumptive diagnosis of Wallenberg's syndrome (lateral medullary plate syndrome) on the basis of occlusion of the posterior inferior cerebellar artery was made. Arteriography, carried out on an emergency basis, revealed occlusion of the posterior inferior cerebellar artery with "beading" (evidence of inflammation) of vertebral arteries and anterior inferior cerebellar arteries. Lumbar puncture revealed 40 white blood cells (mostly lymphocytes) per cc of CSF. Serologic testing of the CSF and serum was positive for syphilis. The patient was treated with intravenous penicillin. Over the ensuing six months, many of his deficits resolved and he resumed his activities including his painting.

Why was sensation impaired over the face? The trigeminal nerve, tract or nuclei must be involved. Why were the arm and leg on the left described as clumsy? There is also an intention tremor. Most likely, this is ataxia. Together with the intention tremor, this indicates damage to tracts or areas associated with the cerebellum. In this case, most likely one of the cerebellar peduncles was involved. Why was there a Horner's syndrome? There is a descending tract dealing with sympathetic information going to the upper thoracic spinal cord; the tract is located in the lateral portion of the medulla and was damaged. The patient had difficulty in swallowing, had paralyzed vocal cords and a diminished gag reflex. Why? The lower motor neurons innervating the pharynx and larynx were damaged. Therefore the vagal nerve complex and/or nucleus ambiguus were damaged.

What caused the vertigo, nausea and vomiting? Damage to the vestibular area. Vibration and position sense impaired on left arm. Why? Only the gracilis nucleus/tract were damaged; cuneate nucleus is more rostral and was not damaged. Pain and temperature were impaired on the right side of the body. Why? ALS damage gives contralateral signs. #9: How would you explain the motor problems to her face?Since there is little motility in both the upper and lower portion of the right half of the face, the damage is either to the nerve or nucleus of CN VII on the right. Would you expect that sounds appear louder on the right side? Why?The sounds would be louder in the right ear. The stapedius muscle, which dampens incoming sounds, is innervated by CN VII. How do you explain the lack of pain discrimination on the right side of her face versus the left side of her body?Loss of pain discrimination on the face is due to damage of the spinal trigeminal nerve; the information has not yet crossed. However, the loss of pain on the body is due to damage of the ALS, which has crossed. How do you explain the results when testing the corneal reflex?CN VII is responsible for the motor component of the corneal reflex. It is therefore absent on the right. Note: CN V is responsible for the sensory component of this reflex. CN V, the spinal component dealing with pain and temperature, on the right was also damaged. The innervation of the cornea does consist primarily of pain fibers but there is sufficient touch innervation to get the corneal response, although it may be sluggish. Where would you expect the lesion to be?At the level of the pons but the caudal portion; only the lateral tegmental portion is involved, sparing the medial lemniscus and the corticospinal tract. #10: Which structure was involved to give you the exaggerated reflexes on the left upper and lower limb?This is hyperreflexia; this is an upper-motor-neuron sign and the corticospinal tract (or motor cortex) is damaged. The plantar extension on the left is also indicative of this type of damage. What caused the problems with the facial muscles?The problem was only with movement of the lower, left face. This suggests damage to the contralateral (right) corticobulbar tract.Note: The corticobulbar tract also involves other cranial nerves; however, the innervation of cranial nerve motor nucleus is bilateral (except for the motor neurons innervating the lower portion of the face) and damage to one corticobulbar tract gives minimal signs if any. What did the tests for eye movements show? Is that consistent with the pupillary response?The right eye could not move medially and at rest tended to move laterally--these are sign of CNIII damage; the medial rectus is required to move the eye medially and oppose the lateral rectus muscle. What would an MRI show?

An infarct to the right ventromedial quadrant of the mesencephalon. Compromised were the CNIII and portions of the cerebral peduncle involving the corticospinal and corticobulbar tracts. The medial portion of the cerebral peduncle contains corticopontine fibers which do not give a symptom. #11: What is the most likely cause of the patient's symptoms?Metastic tumor in the right frontal lobe; spread via blood supply. What is the significance of the papilledema and headaches (especially early morning ones)?Both signs are due to increased intracranial pressure What tests would you recommend?CT: would expect to find multiple areas of increased density in both hemispheres = gliomas. Cerebral metastes usually occur in the setting of widely disseminated cancer therefore one should investigate the extent of cancer spread; in this patient chest X-ray revealed metastic lesions of lung and bone.Prognosis is poor. The majority of patients with brain metastasis die of complications from systemic cancer rather than from direct effects of the brain tumor. #12: Could the patient's problems be due to recent trauma?Yes. The fight in the bar could have caused arachnoid tearing which was aggravated by his fall. What do the findings from the lumbar puncture indicate?The findings indicate subdural hemorrhage. [xanthochromia = old and fresh blood; increased pressure; increased protein and glucose.] What is the most likely diagnosis?Right-sided subdural hemorrhage confirmed by CT. Treatment is surgical removal of blood and closure of bleeding veins.His level of consciousness had deteriorated and he seemed to have had a seizure (incontinence and a bitten lip), both findings suggesting cerebral involvement. The worsening of the patient's condition was caused by imminent herniation of the brain, triggered by the blood mass, the drop in CSF pressure associated with lumbar puncture or both. References: http://www.indiana.edu/~m555/cases/cases.htmlOLD REQUEST: Next step in of Cholesterol/LDL labsBasically, think of it this way. Patient has less than 2 risk factors or more than 2. FIRST OF ALL: RISK FACTORS: - AGE MEN>45 WOMEN>55 (If you give male gender as a risk factor don't add age and vice versa)- FAM/H FIRST DEGREE RELATIVES CAD<55 MEN <65WOMEN- CURRENT SMOKING- HTN>140/90- DM- low HDL<35 Note: TG are NOT A RISK FACTOR YET. HDL>60 IS PROTECTIVE AND NEGATES ONE RISK FACTOR. TYPE A PERSONALITY IS NOT RISK FACTOR. Now: after assessing your patient: - Less than 2RF:

Cholesterol<200 => remeasure in 5 yearsCholesterol 200-240 => remeasure in 1-2 yearsChoesterol >400 => MOVE TO NEXT STEP =>LDL LDL<160 => Remeasure in 1 yearLDL 160-190 => DIET 3 months step1 then 3 months step2 before moving to meds. LDL >190 => START MEDICATION. Usually HMG CoA is first line in high cholesterol, but according to CTB, NIACIN AND GEMFIBROZIL ARE FIRST LINE UNLESS CHOLESTEROL IS REAL HIGH. - More than 2 risk factors: Cholesterol<200 : Remeasure in 5 yearsCholesterol >200 : => LDL LDL<130 : Remeasure in 1 yearLDL 130-160 : DIET (same as above)LDL >160: Medication Cases: 72yo woman presents with Type 2 diabetes history, CAD S/P Inf MI 8 years ago, and hypothyroidism. She is a lifelong NONsmoker. She is on Glyburide, Atenolol, ASpirin, and thyroxine. She denies any sympptoms. Cholesterol: 240HDL 46LDL 167TG 135HBA1C 7%Glucose 128Next step in managing her lipid panel?Introduce medication HMG CoA Reductase with goal LDL<100. Niacin would increase her BS. Playing with her blood sugar medications in hope to reduce the TG will not do much since she is already borederline. A 51 yo male comes to ur office for yearly visit. BP 170/100, body mass index 31. His uncle had an MI at 58, and brother at 55yo. Lipid Panel:Total Chol=288TG= 262HDL 37LDL=199 This guy also has more than 2 risk factors. He never had an MI, so primary prevention has a goal for LDL of < 100. Chol >200 and LDL>160. Introduce Diet and Medications HMG CoA Reductase. Reference:-CTB-Swanson- Blueprints medicineRupture Thoracic AortaSuspect with Deceleration injury of Fx of hard to break bone (Scapula, Sternum). CXR is First stepIf Wide Mediastinum = NONINVASIVE TESTS: Spiral CT or Transesophageal Echo. IF non-conclusive THEN AORTOGRAM.

THEN Surgical RepairBistats QA 55 yr old man visits his physician with a complaint of urinary infrequency.Examination finds a 1cm nodule in his prostate.PSA is ordered .commonly PSA greater than 4 is considered abnormal.using this standard this test has a sensitivity of 80 and specificity of 90.the prevelance of prostate ca in this age group is 10.the pt's test result is 7.what is ur best estimate that this man is actually having ca prostate. the answer is 47%. can anyone plz explain how u arrive at this.The prevalence in this population is 10%. Means if we assume a number of 1000 people, 100 will have the disease and 900 wil be well. - The sensitivity is 80%. Means out of the 100 people sick, 80 are positive AND sick (True Positives), and 20 are negative AND sick (FAlse Positives) - The specificity is 90%. Means out of the 900 well population , 90% (810)are NEGATIVE AND WELL (True negatives). And 90 are NEGATIVE AND SICK (False Negatives). Let's draw that in a table shall we? -------POSITIVE-----NEGATIVESICK---80(TP)--------20(FN)WELL---90(FP)--------810(TN) We use the numbers of positive predictive and negative predictive value = Meaning How positive is positive and how negative is a negative. The best estimate that the patient is ACTUALLY HAVING CA PROSTATE = Positive Predictive Value. PPV = True positive/all positives = 80/80+90 x100 = 47%. Random Q & A from exam1>in UC, when shud we start colonoscopy/sigmoidoscopy?START COLONOSCOPY 8-10 YEARS AFTER DIAGNOSIS. Frequency: every 1-2 years – multiple biopsies each time (@least 32).If low-grade dysplasia => every 3-6 months to confirm presence of dysplasia before performing colectomyRisk of colon carcinoma after 10 years of disease = increases 0.5-1%/year Other: - familial polyposis = genetic screen at 10yo. If not possible, yearly sigmoidoscopy at 12 yo. If diag +ve = surgery before 20yo. - HNPCC = suspect after colonic adenoma => Genetic screening. If +ve => Family colonoscopy q1-2y starting from 25yo or 5 years younger than the earliest diagnosed in the family. Endometrial cancer screening (endom aspiration or transvaginal US) at 25-35yo for women. 2> at what age is FOBT recommended?AVERAGE RISK:- FOBT annually >50yo in average risk individuals- FOBT + Flex Sig Q 5 y- Colonoscopy Q 10years- Barium Enema Q5-10 years FAM/H OR 1ST DEGREE RELATIVE WITH COLORECTAL CA:

Begin at 40yo or 10 years younger than diagnosis of youngest relative => Colonoscopy q3-5y

3>what is the first step in cushing diseaseLOW-DOSE OVERNIGHT DEXAMETHASONE SUPPRESSION TEST (excludes Cushing in 98% cases). 4>gold standard test for aortic dissection?AORTOGRAM

5> first step in aortic dissection if the pt is stable => CXRSuspect with Deceleration injury of Fx of hard to break bone (Scapula, Sternum). CXR is First stepIf Wide Mediastinum = NONINVASIVE TESTS: Spiral CT or Transesophageal Echo. IF non-conclusive THEN AORTOGRAM. THEN Surgical Repair 6>what is the clinical association for ADHD genetically?TOURETTE SYNDROME, OCD. CLINICAL ASSOCIATION. GENETICALLY REALTED = STILL UNDER INVESTIGATION. 7> IF A ASTHAMAA AND HTN IN A PT GIVE CA CHANEEL BLOCK SAME IS WITH ACHALSIA ND HYPERTENTION. 8>drug of choice for VF and AFV-FIB IMMEDIATE DEFIBRILLATION A-FIB SYNCHRONIZED CARDIOVERSION IF UNSTABLE, IV VERAPAMIL, DIGOXIN, BETA-BLOCKER, OR DILTIAZEM. CAREFUL IF THAT’S WPW. AVOID VERAPAMIL/DIG…USE PROCAINAMIDE/QUINIDINE 9>diarrhea in mexico with no blood and wbc in stool..most common cause: NO BLOOD/ WBC +VE => INVASIVE. TRAVELER = E. COLI. (Montezuma’s revenge). If no history of travel, most common = campylobacter. Other invasive: Shigella, Salmonella, Yersinia. 10>drug of first choice in depressionSSRI = FLUOXETINE 11>what the adult form of conduct disorderANTISOCIAL PD = Long criminal record, tortured animals when kids, and set fires. Liars with no remorse, strong association with alcoholism, drug abuse and somatization disorder. 12>can we diagnose a pt of 15 yrs with personality disorderYES, EXCEPT ANTISOCIAL WHICH BEFORE 18YO WOULD BE CONDUCT DISORDER. 13>drug of choice for eclampsiaMAGNESIUM SULFATE 14>drug for narcolepsyFORCED NAPS AT REGULAR TIMES IS TREAMENT OF CHOICE. DRUG: PSYCHOSTIMULANTSIF CATAPLEXY PRESENT (sudden loss of muscle tone precipitated by loud noise or emotion = pathognomonic. But remains awake if short episode) = TRICYCLIC ANTIDEPRESSANTS

15>pt with wt loss, purging, fasting, exercise, good academics,female,15 yrs:diagnosis?MAIN QUESTION is it Anorexia or Bulimia? Both have fasting, exercise, purging etc. USUALLY - BN = RELATIVE GOOD WEIGHT FOR AGE – BORDERLINE PERSONALITY DISORDER (body image, poor impulse control). - AN= WEIGHT LOSS – SPORTS – AMENORRHEA – MAJOR DEPRESSIVE DISORDER with atypical features (leaden paralysis, overeating). 16>most common case of suicide?UNTREATED DEPRESSION 17>which drug to administer first to a pt of thyroid storm?PROPRANOLOLOTHER: PTU (=THIOUREA DRUG) – POTASSIUM IODIDE (1H AFTER FIRST ANTITHROID DRUG) – Ipodate sodium (1h after PTU) – DXM. Definitive treatment with radioactive iodine or surgery is delayed until euthyroid. 18>black male,30s,…..HTN which drug to administerTHIAZIDE DIURETIC 19>smoking cessation.effect on lung changesFEV1 IMPROVES FIRST YEAR, THEN STARTS DECLINING SEMI-PHYSIOLOGICALLY MORE THAN NON-SMOKERS, BUT LESS THAN CURRENT SMOKERS. RATIO SHOULD NOT BE DECLINING SINCE FEV1 AND FVC ARE DECLINING “PHYSIOLOGICALLY” AT SAME RATE. RISK OF LUNG CANCER DECREASES BY 80-90% AFTER 15YEARS BUT NEVER REACHES LEVELS OF NON-SMOKERS. OTHER: RISK OF HEART ATTACK IS DECREASED BY 50% WITHIN 24H OF QUITTING. Joins level of non-smokers after 2 years. Risk of having a stroke and brain aneurysm declines by 30-50%. 20>effects of malignany on psychological state I CAN’T FIGURE OUT THIS QUESTION. NEEDS MORE EXPLICIT. 21> IN A PATIENT WITH ACUTE LUNG INJURY BREATHINGSPONTANEOUSLY, INTUBATION AND THE APPLICATION OF BOTH AN ENRICHED FiO2 AND PEEP SUFFICIENT TO RECRUIT COLLAPSE ALVEOLAR UNITS SHOULD DO WHAT TO PULMONARYVASCULAR RESISTANCE?Decrease it by reversing hypoxic pulmonary vasoconstriction. how do you treat Cyclosporine induced HTN?ACCORDING TO DR SAKALA HE SAID CYCLOSPORIN IS MUST FOR POST TRANSPLANT PT BCZ WE DONT WANT TH E PT TO REJECT KIDNEY BUT WE HAVE TWOMAIN PROB ..HTN AND NEPHROTOXICITY...... HE ALSO SAID NEVER D/C CYCLOSPORIN BUT JUST DEC TH EDOSE AND GIVE PT ACE INHIBITERS.....HE ALSO SAID THAT FOR EXAM POINT OF VIEW THEY WILL ASK U THAT AFTER 7 DAYS OF KIDNEY TRANSPALNT PTS CRETINE WENT SKY HIGH ...WEATHER ITS BCZ KIDNEY IS BEING REJECTED BY THE PT OR ITS DUE TO CYCLOSPORIN U HAV ETO DO BIOPSY....BUT B4 THAT U HAVE TO C WEATHER THERE IS ANY TENDERNESS AT TH E GRAFT SITE OR FEVER IS

HIGH BCZ INFECTIONIS MOST COMON CAUSE OF DEATH IN ALL TRANSPLANT PTS JUST LIKE AMI IS TH EMOST C C IN EVERY VASCULAR SURGERY THAT THERE IS....

I REMMBER IN MY ACTUAL TEST THEY ASK THAT THIS PT IS ON LITHIUM FROM LAST 3 YRS BCZ OF BIPOLAR SHE IS OKAY NOW S/S FREE FROMLAST SIX MONTHS AND NOW SHE IS SO DEP AND FEEL WEAK AND GAINING WEIGHT AND HAIR R CANGING IN TEXTURE BARDYCARDIAC , SLOW DEEP TENDON REFLEXES WITHDRWAL..WHAT U WILL DO.....CHECK LITHIUMMM LEVEL ......? START HER ON SYNTHYROID.....D/C LITHIUM AND START HER ON SYNTHYAROID ...SO POINT THAT I AM MAKING HERE IS THEY WILL ALWAYS MAKE U FOOOL THAT PT IS SYMPTOM FREE SO SINCE U KNOW THAT LITHIUM CAUSE HYPOTHYROIDSM U WILL THINK PT IS OKAYS LETS D/C MEDICATION SO PT WILLL B OKAY...NEVER EVER DO TAHT THEY TEST U WITH AMIDIARON AND OTHER MEDS IN SAME WAY BCZ IF U WILL D/C PRIMARY PT WILL RELAPS AND THEN U WILL HAV ETWO PROBLEMS INSTAED OF I.....JUST CHECK TH E LEVEL AND DEC THE DOSE AND START PT ON THE MEDICATION THEN SLOWLY INC THE MEDCIATION IN A THERAPETIC RANGE... SO THANKS PHARMAA GOOOD Q..Unique Ethics case: Minor patient doesn’t want to abort but mother wants her toIt's depending on the states, but generally:the parents of a pregnant minor cannot legally force their child to have an abortion. On the other hand, if the minor chooses to give birth and not place the baby with adoptive parents, the minor's parents are NOT legally obligated to support the baby. If the minor and the baby's father and any other willing relatives cannot or will not support or care for the baby, the government may step in. This could result in the baby becoming Dependent. Some Dependent children live with birth parents or other relatives but are subject to government monitoring. Other Dependent babies are placed in foster care.

Multiple Sclerosis and PregnancyIn women with Multiple Sclerosis, the rate of relapse declines during pregnancy, especially in the third trimester, and increases during the first three months post partum before returning to the prepregnancy rate.However, the lifetime risk rate does not appear to change because of pregnancy, and on the basis of current retrospective studies, long-term disability is not higher in pregnant women or even women experiencing relapses during the pregnancy year. MS has little or no effect on the course of pregnancy or delivery, although patients with severe MS may have difficulty fully caring for their newborns. Postpartum IVIg treatment is beneficial in preventing acute childbirth-associated exacerbations in patients with Relapsing/Remitting Multiple Sclerosis. Reference:http://www.albany.net/~tjc/pregnancy.htmlMy review of the examI had one of those days. And it's over. I took a break at the end of every block. That's when the CD of Kaplan was useful. It had given me the opportunity to take a whole day

exam. And when I did it, I made observations when was my score coming down (After lunch, last block). So I made a point to take longer breaks on those times. Before I went this morning, I made my "break plan". 5 min after each block. 10 min for "lunch" which was a turkey sandwich. 10 min on the block after lunch. Because I would be feeling sleep y by then. Ten minutes between the 7th and 8th one. Because, you get a little impatient and you tend to want to do it without a break, but two hours answering questions is not good, coz you're cutting yourself short on the questions at the end of the last block. It started with "easy" questions. I had definitely just about 1% of pathophysiology, and 90% of management with about 40% treatment. X-rays of Fractures, Obvious. And 2 graphs for Arrest of descent. It was pretty easy. The questions were NOT tricky as much as I expected. I had learned som much from doing questions, and explaining the methods to my students that it was SO easy to pick on the clues. There is ALWAYS something to look for. I had time to read the Question stem twice. I always finished ontime. READ THE QUESTION STEM LIKE IF YOUR LIFE DEPENDED IN IT. Beleive me, you miss the clues easily if you don't read it that way. I helped myself with my finger following each and every word, and the other hand hiding the choices. I used the scratch laminated paper to SUMMARIZE the questions stem. When reading the questions, their strategy is to put the Cheif complaint, then a piece of history which may be relevant but it distracts you, then putting again something relevant to the present, then something about family history. If your mind is not trained to filter the parasites, USE YOUR HANDS ON THE SCREEN MONITOR. And reconstruc the scenario in your mind. A 30 TO WITH CHEST PAIN AND EKG CHANGES IN THE SETTING OF 2 RISK FACTORS AMONG WHICH FAMILY HISTORY ..doesn't read the same as 30YO PRESENTS TO THE CLINIC WITH BURNING SENSATION IN LEFT SHOULDER AND UPPER LEFT CHEST. HE STATES HIS FATHER DIED OF HEART ATTACK AT 55YO. THE CHEST PAIN EXACERBATES ON EXERTION. TWO YEARS AGO, PATIENTS UnDERWENT APPENDECTOMY. AT THAT TIME, HE REFUSED TO CHECK HIS BLOOD CHOLESTEROL. PATIENT'S LABS ARE....Blah blah blah. You have to rearrange it in your mind. Now about the topics: The usual:- I had about 4 ABG Questions. The examples I had posted before SHOULD BE MORE THAN ENOUGH. Type on the search button ABG and it'll take you to them. - I had the usual ATHMA treatment changes - OB-GYN usual OB complications: Arrest of Descent - Placenta Previa - Risk factors for Breast Cancer - Hirsutism - - Gynecomastia in male with infertility- EKG: VERY EASY ones: V Fib, MI- LOTS OF ETHICS, VERY EASY ONES. You'd know the answer right off the bat. Power of attorney = Treat like if it's patient talking to you. - HTN + Hypokalemia = Think Hyperaldosteronism even if positive family history of essential hypertension- Mechanism of alcohol-induced hypoglycemia (Inhibition of gluconeogenesis).- Derma: the mundane ones. 2 Q. - Multiple Sclerosis and Pregnancy. Effect of the latter on the first.

- Fractures of children = Treatment: Cast, Splint, ?- Transplantation: Very easy: Increase the steroids if acute rejection at 30 days for ex. - 2 Q on Mechanical Ventilation: Recognize indication for weaning (Type Mechanical Ventilation notes #2: don't read everything. Only the modes of ventilation, and when to wean patient off). Also Complications of MEch Ventilation: Barotrauma if pressure-controlled ventilator. Know about permissive hypercapnia in ARDS. - Didn't have much of the nephropathies. - Neuro: again very easy ones. Usual stroke. MS, Meningomyelocele. - Prevention of Rheumatic Fever: What antibiotic and for how long. - Zenkel Diverticulum: 2 Q. but tricky. They don't name it Zenkel. - Achilles Tendon rupture is also called Hell Cord rupture hehehe- Down and Alzheimer. - Alzheimer and genetic testing for iff-spring possible? What is it? What does it predict?- CSF findings in Antisocial PD. Finally:I will be available for any questions. I don't know if I will get it. But I feel releived I took the test. For the colleague who asked about not sholwing up to the test today please follow with us and tell us what happened. I specifically asked. The guy said at the end of each day they send the data to ECFMG including NO SHOWS, which will have to go to reprocessing and paying for the test again with ECFMG. So let us know your experience. I am available for questions or more details. I am bound not to say much about the specific questions as Dr Carl reminded us again that this site is being monitored which is why I posted mostly topics, as they would appear on the Step2 outline, with a touch of care from me:) THANKS TO EVERYBODY WHO SUPPORTED ME ALL THROUGHOUT. I CAN'T NAME PEOPLE BUT I'M SURE THOSE WHO I HAVE BEEN IN CONTACT WITH THROUGH THE WEBSITE OR EMAIL KNOW ME AND KNOW HOW WEL THEY CONTRIBUTED TO WHERE I AM NOW. I WISH AND HOPE IT'LL BE THE LAST WITH STEP2. I SHALL REMAIN HELPING FOR WHOMEVER NEEDS HELP. GUIDANCE OR SPECIFIC MEDICAL QUESTIONS. USE ME PLEASE. I KNOW AND HAVE BEEN THROUGH A LOT. IF I DON'T KNOW I WILL POINT YOU TO THE RIGHT RESOURCE. SINCERELYHASSAN